Двач.hk не отвечает.
Вы видите копию треда, сохраненную позавчера в 11:00.

Скачать тред: только с превью, с превью и прикрепленными файлами.
Второй вариант может долго скачиваться. Файлы будут только в живых или недавно утонувших тредах. Подробнее

Если вам полезен архив М.Двача, пожертвуйте на оплату сервера.
наукич.jpg239 Кб, 1299x870
Тред вопросов и ответов 584346 В конец треда | Веб
Перед тем как перейти к тредам, убедитесь, что вы ознакомились с правилами: https://2ch.hk/static/rules.html#sci (М)

Это прикрепленный тред, где интересующийся анон может задать свои вопросы (и, возможно, даже получить ответ). Здесь можно общаться, а также обратиться к модератору.

Однако, помните, что /sci/ - собирательный раздел для всех наук, но если вы глубоко заинтересованы обсуждением политики - вам в /po/, математики - /math/, философии - /ph/, психологии и психиатрии - /psy/, медицины - /me/, космоса и астрономии - /spc/.
2 584349
https://mir24.tv/news/16467844/detenyshi-pterozavrov-okazalis-gotovy-k-poletu-s-rozhdeniya
Как птерозавры могли летать сразу после вылупления? У птиц и рукокрылых полёт — приобретаемый навык, а не врождённый + требует регулярной тренировки. Птиц, родившийся в неволе, выдохнется через пару минут полёта, тупо умения и стамины не хватит. А хладнокровный (мезотермический?) рептилоид рождался уже с достаточной мышечной массой, выносливостью и умением использовать воздушные потоки / маневрировать / приземляться, не сломав себе лапы?
3 584352
>>84346 (OP)
Вот как сейчас можно читать WOS или Science? На сайхабе новых статей нет. Готов умеренно потратиться
4 584356
Верните прошлый закреп, и да, если я заинтересован в физике, куда мне идти? Нахуй?
5 584358
>>84356
Оставаться здесь. В закрепленном посте указаны доски для тех наук, которые вышли за пределы /sci/ в свете своей наибольшей востребованности. В любом случае, их можно обсуждать и здесь, но будьте готовы к тому, что вам мало кто ответит по поводу математики или философии, ибо для этого есть свои разделы. Физику же можно обсуждать в закрепленном треде или создать свой собственный, - ограничений на это нет.
6 584361
Двач
ogog146608467323883122.jpg689 Кб, 1200x628
7 584363
Сколько теоретически может существовать изолированная экосистема в ограниченном объёме? Проще говоря, в банке.
Мох, водоросли, какие-то растения, мелкие почвенные насекомые. На практике это всё может расти и зеленеть в закупоренной банке год, два. Дольше просто не проверял. А тысячу лет сможет? Миллион? Что там будет через 50 миллионов лет, при условии, что банка не разрушится и не вмёрзнет в лёд?
8 584364
>>84363
Очень мало, так как в такой экосистеме все висит на балансе освещенности и количестве веществ в круговороте. Обычно это прокисает на 10-20 год, если экосистема на 500-1000 литров. И причина, обычно, в выводе веществ в говно и невозможности его быстрого расщепления.
Экосистема планетарного биома имеет показатель вещества в сотни раз выше к жизни, и именно этим и успевает обеспечивать и поддержание жизни и прирост ее.
В замкнутом пространстве такое не реализовать. Это, кстати, одно из доказательств системного архитектора, лол.
9 584370
Термодинамика говорит, что все добро переработается в говно.
Но это не точно.
Может быть там смогут выжить какие-нибудь экстремофилы, которые просто впадут в анабиоз до момента разрушения банки.
10 584397
Можно ли использовать обратный пьезоэффект тобишь электрострикцию для передачи механического усилия?
Я имею ввиду, вот есть кристалл кварца, который при давлении меняет потанцевалы.
Но если сделать нить из SiO2, сделать жгут из сотни таких и пропускать заряды противоположных значений по каждой из них, получится ли добиться значительного момента силы? Скажем, сдвинуть многотонный кусок железа.
11 584404
>>84397
Понадобится уйма энергии, но да, теоретически это возможно.
12 584409
>>84346 (OP)
Мочухан, докажи-ка антуану истинность точного определения термина "наука" - или тупой пиздабол.
13 584414
>>84409
Не хочу.
Можете закрывать тред.
14 584416
А электрослабое состояние вещества всё ещё прозрачно для нейтрино или они там активно переизлучаются подобно фотонам во внутренностях солнца? И соответственно при таких энергиях и концентрациях вещества нейтрино становятся полноценными реагентами, а не просто инертным квантовым пеплом? А есть ещё более инертные частицы, которые начинают жечь только при суперобъединении?
15 584422
>>84416

>электрослабое состояние вещества


Это как?

Сечение взаимодействия нейтрино с электронами или кварками можно повысить не таких уж многими способами.
-Первое это очень сильно нагрев вещество, такие температуры возможно разве что в пике взрыва сверхновых или аккреции вещества на релятивистские объекты.
-Другой способ это повышение количество частиц вещества в объеме - короче повысить плотность. Плотность тут должна быть на порядок выше ядерной. Такое даже у нейтронных звезд редко бывает.
-Запихнуть вещество в очень глубокую потенциальную яму, глубже его энергии покоя. Только у нейтронных звезд, которые близкие в коллапсу в черные дыры.
-Или наоборот сильно охладить вещество, чтоб полезли квантовые эффекты, которые подавляются декогеренции. На отдельных частицах или системе частиц работает, но на макро телах не очень.

>А есть ещё более инертные частицы, которые начинают жечь только при суперобъединении?


Полно! На любой вкус найдутся расширения стандартной модели. Суперсимметрия дает целый зоопарк неуловимых джо.
16 584430
>>84409
определения придумывают люди, они не даны свыше
об определениях договариваются
следовательно истинное определение — то, о котором есть договорённость в рамках дискуссии
айда договариваться

наука — это область человеческой деятельности, знания в которой приобретаются научным методом

соглашайся с моим определением или предлагай своё
17 584447
А где Артемий, что случилось?
Неужели раздел может перестать быть говном?
18 584449
>>84447
А почему он "говно"? И что (в твоем понимании) нужно сделать для того, чтобы он перестал "им" быть?
19 584450
>>84449
Хуй знает может ли что либо помочь этому гадюшнику. Но закрепленный Татьяныч это такой пиздец кринжовый был.
20 584452
>>84346 (OP)
Наконец эта ублюдская рожа исчезла, спасибо.
21 584453
>>84346 (OP)
Каким образом в электрической цепи при увеличенном напряжении, но при той же силе тока, можно передавать больше электроэнергии, если сила тока по определению это количество электричества проходящее через проводник за единицу времени?
22 584454
>>84453
Напряжение распространяется со скоростью света и заставляет электроны в конце цепи двигаться. Успешность их борьбы с сопротивлением в средине цепи не так важна.
23 584455
В аналогии с водой, напрежение это давление, а сила тока это сколько воды протекает. Вот и думай
24 584456
>>84454
скорость света в материале
конец цепи там где питаемое устройство, средина всё что между ним и источником энергии
25 584459
Как происходит редактирование генов с CRISPR/Cas на практике? Ну вот я знаю, что есть такие-то нуклеазы с наноножницами, они по рофлу режут гены и все такое. Но как от этой теории прийти к практическому применению? Как сделать светящуюся гигамышь убийцу?
26 584464
>>84459
Берешь гены гигантизма, светимости и неукротимой жажды человекоубийства и вставляешь их в ДНК мышки.
27 584468
>>84464
Как их брать и как вставлять? Общую теорию на уровне научпопа то я знаю.
28 584470
>>84459
Берешь клетку, растворяешь у нее мембрану так, чтобы ДНК не пострадало.
Берешь бактерию, вытаскиваешь у нее специальные белки, которые аккуратно режет и превращает ДНК в РНК без каких либо превращений.
Выискиваешь из вырезанных РНК нужную.
Берешь бактерию с кассетами и впрыскиваешь в нее фрагмент нужной РНК. Вуаля фрагмент РНК запихнут в ДНК.
Разбираешь бактерию на части, получаешь кассеты.
Выбираешь нужную.
Эти кассеты пускаешь в клетку, а они уже сами себя врежут в ДНК.
29 584471
>>84470
Да меня не идея интересует, а как там это все в лаборатории устроено. Что в пробирках и каким образом получается генная модификация. Как пришли от идеи, к инъекции чудо жидкости с переносчиками ГМО.
30 584475
>>84471
А хули тут не понятого?
Выращивают нужные клетки или бактерии, потом их поливают всякой химий, получившийся рассольчик запихивают в центрифугу, где все разделяется.
ДНК и РНК это кислоты и они легко обматываются вокруг ионов. Поэтому есть целая куча реактивов для их связывания.
С белками сложнее, но те что нужны, можно вытащить вместе с ДНК и РНК, ибо они связанны с друг другом.
Получившиеся вещества уже дополнительно фильтруют, поливают другой химией и эту гадость вкалывают клетках или бактериям под микроскопом.
31 584480
На нашу жизнь влияют как-то квантовые случайности, которые противоречили бы принципу причинности? Я имею ввиду не кот Шрёдингера, а допустим, согласно закону причинности человек должен выбрать а, но квантовая неопределённость говорит б. Или это работает только на оооочень больших дистанциях?
32 584482
Рандомайзеры действуют по какому-то алгоритму, или там в основе тоже квантовая неопределенность?
33 584485
Уверен, что ответа не получу, поскольку здесь отвечают только на стандартные вопросы по теориям относительности. Но попробовать стоит.

Есть жиробас 100кг, который прыгает с метрового стола на весы. Какой вес покажет дисплей? Упругость ног, весов и пола, а также сопротивление воздуха и прочий кринж игнорируем.
34 584488
>>84454
>>84455
Ну так каким именно образом большее напряжение позволяет передать больше ватт энергии, при прочих равных, если за скорость и объем передачи электричества отвечает сила тока? Откуда берётся эта энергия? Как это выглядит на атомном уровне?
35 584495
>>84488
Напряжение передают фотоны, от электронов требуется только не мешать.
36 584498
>>84485
170 кг
37 584499
>>84498
почему не 400?
38 584516
>>84453
ток — количество зарядов
напряжение — их скорость
при сохранении количества зарядов можно передавать больше электроэнергии, увеличив их скорость
39 584527
>>84516
а еще есть такая штука как сопротивление
40 584529
>>84527
есть, да
двигаясь внутри проводника заряды иногда сталкиваются с атомами, из которых состоит проводник, отдавая им часть своей кинетической энергии
атомы проводника от этого начинают дёргаться сильнее обычного (=повышение температуры)

чем больше зарядов — тем чаще происходят столкновения
чем выше скорость зарядов — тем больше энергии они отдают
41 584538
>>84485
Бамп
42 584542
>>84485
>>84538
не найдёшь ты весов, которые в описанных условиях покажут эффекты теории относительности

ускорение свободного падения 9.78м/с/с
то есть пролетев метр, он будет двигаться со скоростью 9.78м/с
c=1, тогда скорость в момент приземления в единицах скорости:
9.78 / 299792458 = 3.262256851037927e-8
релятивистская масса = масса / лоренц-фактор
лоренц-фактор = 1 / (1 - v^2)
масса чела = 100 / ((1 - ((3.262256851037927e-8)^2))^0.5) = 100.00000000000006

твои выдуманные весы покажут 100.00000000000006кг
43 584543
>>84542
Шизик, таблетки прими.
44 584544
>>84542
быстрофикс
лоренц-фактор = 1 / ((1 - v^2)^0.5)
45 584545
>>84542
опечатался, быстрофикс:
лоренц-фактор = 1 / ((1 - v^2)^0.5)
остальное правильно
46 584546
>>84545
Любые весы покажут 150+, иди нахуй шизоид.
47 584547
>>84546
вопрос был про теорию относительности
и про несуществующие весы, для которых можно пренебречь упругостью
48 584548
>>84547
Нет, иди перечитывай вопрос пока не прозреешь.
49 584549
>>84548
ну и ладно
50 584558
>>84480
Да. Пример локализация электрона в атоме, с классической точке зрения он там движется выше скорости света.
Квантово вырожденное вещество - всякие сверхтекучести и сверхпроводимости, в них образуется вихри, которые ебут классическую казулаьность.
Ну общий пример это квантовые вычисления как таковые.
51 584563
>>84529
Если напряжение равно произведению тока и сопротивления, то. Получается что на напряжение ни ток, ни сопротивление не влияет.
...что странно. Зачем тогда провода нужны?
52 584567
>>84563

>Зачем тогда провода нужны?


R = p * l / S, где
p - удельное сопротивления материала
l - длина отрезка проводника
S - площадь поперечного сечения (при условии, что по всей длине оно одинаковое)
53 584587
>>84563

>Зачем тогда провода нужны?


Провод нужен для проведения поля. Обычные материалы вокруг нас не являются металлами и вообще плохо проводят ток, или проводят не так, как нам надо (ток связанных зарядов, смещения, etc).
54 584599
Тупой, наверное, вопрос ответ на который я видимо сам и напишу.
Короче, в рабочей машине адово ебашит статическим электричеством.
Вопрос, если купить антистатическую обувь то вопрос ведь не решится? Ибо бьет током по голым рукам ведь.
Правильно думаю?
55 584603
>>84599
У тебя спецовка с мехом? Просто элекризуешься от одежды, как эбонитовая палочка на уроке физики.
56 584607
>>84349
Ну можно то же самое по хождение и млекопитающих сказать. Многие могут стоять и ходить с рождения, но человек не умеет даже голову держать, начинает ползать только в полгода, а ходить в год-полтора.
Видимо, у птиц есть какие-то важные плюсы, которые исключают полёт с рождения, а у птерозавров их не было.

Можно потеоретизировать.
У птиц кости полые, более хрупкие, зато проще летать во взрослом возрасте, например.
57 584612
>>84599
Поставь цепь если у тебя грузовик и антистатический хлястик (или как он там называется) если легковушка.
58 584616
>>84599
Я просто чехлы заземлил. Взял ленту-скотч для кондейщиков и по бокам чехлов нанес тонкую линию к самому тоннелю, прямо за ручником. И все. Плюшевые чехлы тоже генерят много статики из-за синтетики и меха.
Безымянный.jpg63 Кб, 800x600
59 584621
Привет аноны.
Будет ли в теории работать схема пикрил? Вопрос не в экономической целесообразности, потому-что она очевидна никакая, а именно в технической возможности.
Есть Озеро на стометровом холме. Если подвести от него трубу к дому у подножья, то из трубы по закону сообщающихся сосудов будет постоянно литься вода. А если эту воду будут греть земные недра, то она будет двигаться в том же направлении и обогревать дом?
60 584622
>>84621
Мне кажется, она не будет у тебя успевать нагреваться.
61 584625
Почему селекционеры не могут вывести более плодоносный шафран? Типа сильно упадёт качество специи, потому что природа тупо не сможет её столько синтезировать?
62 584626
>>84625

>шафран


При селекции ирисовых у них возникают проблемы с количеством и качеством эндосперма или клеток зародыша. Гибридизация ирисовых требует десятилетий. А сейчас - тяп-ляп и в продакшн. Деньги делай или сделает Какой-нить Глоценко\Дрочун\Писюнчук.
63 584627
Почему официальная новука отрицает существование гавваха?
Ацтеки видимо в своих пирамидах просто по приколу устраивали массовые жертвоприношения?
64 584628
>>84627
Почему отрицает? Наука ничего не отрицает, она лишь говорит "у нас нет пруфов существования"
1-1-1.jpg51 Кб, 555x370
65 584629
А что если использовать перегретый пар в ледоколах, но топить льды не сверху, чтобы пар не рассеивался в атмосфере, а снизу, чтобы струя пара разделяла воду и лед прослойкой, смещала центр тяжести льда выше точки опоры и тот бы сам ломался под своим весом. Взлетит?
66 584631
>>84621
https://youtu.be/lstoJ_juDe4?si=ot_XwFlQreZTGH92
можно без озера на стометровом холме
67 584634
>>84629
Ледокол давит лед ВЕСОМ. Он не греет лед.
68 584635
>>84629
А еще - центр тяжести льдин, как и у айсбергов - в нижней части льдин. И его не сместить.
69 584636
>>84627
Ну хорошо. Вот я ученый, я начал вникать в твое сверхъестественное. С чего начнем исследование?
70 584642
>>84636
Исследовать принцип выделения гавваха, на примере той же СВО и зиккурата на Красной площади. Понять, как перестать кормить тварей.
71 584644
>>84628
Вот есть такой пруф, как НЛО, есть документально подтвержденные похищения и садистские опыты над людьми гуманоидами. Но официальные ученые это не признают, уфология у них проходит по разделу креационизма и плоской Земли.
72 584645
>>84644

>есть документально подтвержденные похищения


Да, я в документалке "4 вид" видел видосы с камер. Там даже видосы с видеорегистраторов копов есть.
73 584646
>>84645
Хуй с ними – с копами. Чего скажешь про три известных видео: go fast, gimbal и flir?
74 584647
>>84642
Выдели предмет исследований. С научной точки зрения ты предлагаешь найти душу, выделить ее эманации, изучить ее спектр частот? Какой научный инвентарь необходим?
75 584648
>>84647

>Какой научный инвентарь необходим?


Котел - 1шт
Шар хрустальный - 1шт
Палочка волшебная - 1шт
Очаг с адским пламенем - 1шт

Расходники: Глаза жаб, крылья летучих мышей, кровь девственницы, змеиные языки - по необходимости.
76 584649
>>84648
Не, я начну с электрографа Кирлиан. Вышеуказанные препы и оборудование - это прямо высшая магия.
77 584651
>>84647
Компьютер, мозги, что-то чем можно их сканировать.
78 584654
>>84651
Я вижу, ты еще повидлодаван и не можешь в джедайские речи. Зачем ты тут, отрок? Тебе стоит сесть да диету воздержания и повысить уровень мидихлорианов в себе, чтобы не задавать червеимперские вопросы славному республиканцу...
NSRWAirPump1.png131 Кб, 480x528
79 584699
Как работают эти клапаны ебаные? Обясните
80 584700
>>84699
Как дверь которая может отрываться только в одну сторону.
81 584704
>>84700
Как жопа, внутрь огурцы впускает, обратно не выпускает
82 584705
>>84704
За этим наверняка стоит интересная история.
Бильярд-кун, ты ли это?
imgstouch (6).png86 Кб, 720x720
83 584719
Допустим я хочу знать всю ныне существующую математику, абсолютно все разделы до нынешнего времени, сколько примерно часов у меня уйдет на это при среднестатистической обучаемости?
84 584720
>>84719
Есть брать чистую математику, только только абстракции, только общие случаи и обобщения, только хардкор, то это примерно 20+ лет по академическим программам.
Если подключать сюда прикладные вещи и теорию алгоритмов, то это растянется на тыщи лет.
85 584722
>>84719
Если у тебя полный начальный математический базис, и есть алгоритмы обучения, то 400 часов проанализированного материала. И примерно 200 часов на закрепление с проверкой.
Только зачем тебе это?
86 584732
Физики, когда квантовая гравитация? Накидаю за щеку если не завтра.
87 584749
Почему в природе 3 поколения частиц? Так боженька захотел?
9-fmafEXtGs.jpg40 Кб, 361x480
88 584751
Почему если дрочить или ебаться с похмелья, то оргазм в разы круче, чем в обычном состоянии и если в обычном состоянии меня хватает на 2-3 раза, то после похмелья нормой будет 7-10 раз за сутки? Спрашивал на медаче, там не ответили, но сказали, что подобная ситуация при отходняках с амфы.
89 584754
солнечный полюс, земной полюс, 2 реки, 3 облака нижнего уровня, 0 облаков верхнего уровня, 1 ледяная гора, одна лавовая гора и 3 фонтана и на луне тоже самое, а солнце, меркурий и венеру нахуй.
90 584755
>>84749

>3 поколения частиц


Поколения есть у лептонов, а не у частиц. Частицы лишь делят на три группы.
91 584756
с какой вероятностью помрёшь, выпив литр напополам масла и водки с противорвотным?
92 584757
93 584759
>>84756
У тебя от поллитра масла придёт пизда поджелудочной, особенно если очень холодной водой запить. Тут даже водка не нужна.
94 584762
всем привет, всю жизнь интересовался гуманитарными дисциплинами, но испытывал небольшие комплексы по поводу того, что в точных науках я всегда был опущ, иногда появляется интерес влезть в математику\физику\химию, но я не знаю что именно выбрать. С другой стороны, не хочется решать кучу абстрактных задач и формул из разряда "рассчитайте в каком очке встретятся два атома, выпущенные из анальной пушки". Ну так вот, реквест такой: что можно изучить, чтоб была практическая польза\можно было бы проводить всякие эксперименты в реал лифе или на компутере (если возможно)? ну вот химики реально сидят дома со своими пробирками и кайфуют. Но это дорого и опасно, что ещё можно придумать?
95 584764
>>84755
Поколения есть как у лептонов, так и у кварков. Причем есть прямое соответствие по квантовым числам. В природе прям три копии одного говна.
Всякие бозоны порождаются калибровочными симметриями над ними.
96 584772
>>84645
Мишаня, ты?

21 октября 2022 г. NASA объявило состав группы по изучению неопознанных аномальных явлений. Это исследование будет сосредоточено на несекретных данных.

23 декабря 2022 г. президент Байден подписал закон Джеймса М. Инхофа о полномочиях в области национальной обороны на 2023 финансовый год. В этот закон были включены специальные меры защиты для разоблачителей UAP (спасибо u/joeyisnotmyname за подборку этих страниц). Я повторю это еще раз: в NDAA на 2023 год были включены меры защиты информаторов КОНКРЕТНО в отношении UAP.

1 июня 2023 г. НАСА раскрывает информацию о наблюдениях НЛО на Ближнем Востоке на первой в истории публичной встрече, посвященной наблюдениям НЛО.

6 июня 2023 г., всего через полгода после подписания Байденом закона NDAA 2023 г., офицер ВВС США и бывший сотрудник разведки Дэвид Груш становится разоблачителем НЛО, утверждая, что в США существует сверхсекретная программа по поиску разбившихся НЛО.

9 июня 2023 г. Томас А. Монхейм, генеральный инспектор разведывательного сообщества, считает заявления Груша "срочными и заслуживающими доверия", что открывает путь к проведению слушаний в конгрессе.

14 июля 2023 г. лидер сенатского большинства Чак Шумер включил в NDAA в качестве поправки свой новый закон о раскрытии информации об UAP от 2023 г. В тот же день NDAA был принят. Я настоятельно рекомендую прочитать как можно больше текста Закона о раскрытии информации об UAP, если у вас есть на это время, - формулировки очень точные и убедительные.

18 июля 2023 г. координатор и пресс-секретарь Совета национальной безопасности Джон Кирби признал, что НЛО и UAP начинают создавать проблемы для ВВС США.

26 июля 2023 г., всего через полтора месяца после того, как Груш сообщил об этом, состоялись двухпартийные слушания в Конгрессе по UAP. Конгрессу предоставляется 177-страничная хронология, которая начинается с 1947 года. На этих слушаниях дают показания разоблачитель Дэвид Груш, Дэвид Фравор и Райан Грейвс. В течение почти двух часов их допрашивали о личном опыте, имеющихся у них доказательствах, а также о том, как следует поступать в дальнейшем. В связи с тем, что Груш пользуется официальными каналами защиты информаторов, он не может разглашать сведения, которые остаются конфиденциальными, но предлагает имена как враждебных, так и сотрудничающих свидетелей, а также конкретные места нахождения найденных материалов, если это будет делаться по соответствующим каналам на закрытом заседании, т.е. на S.C.I.F. К сожалению, на слушаниях не было времени для заключительных слов, но вы можете прочитать заключительное слово Груша здесь (а также его вступительное слово). Из его заключительного выступления мне особенно запомнились следующие цитаты: "Действительно, будущее нашей цивилизации и наше понимание места человечества на Земле и в космосе зависит от успеха именно этого процесса" и "Продвигаясь по этому пути, мы сможем обеспечить необычайный технологический прогресс в будущем, когда наша цивилизация превзойдет нынешние достижения в области двигателей, материаловедения, производства и хранения энергии".

27 июля 2023 г. Сенат принял МНОГОЧИСЛЕННЫЕ меры UAP, включая презумпцию немедленного раскрытия информации, дополнительные меры защиты информаторов и др.

13 Сентября 2023 В Мексиканском конгрессе выставили на обозрение трупы "инопланетян"

14 сентября 2023 НАСА выкатило отчет главный вывод которого «Главный вывод из исследования заключается в том, что еще многое предстоит узнать»

18 октября 2023 года Годовой отчет о неопознанных аномальных явлениях за 2023 год был представлен Конгрессу Министерством обороны и Управлением директора национальной разведки (ODNI) в среду, согласно пресс-релизу Пентагона, выпущенному в тот же вечер.
96 584772
>>84645
Мишаня, ты?

21 октября 2022 г. NASA объявило состав группы по изучению неопознанных аномальных явлений. Это исследование будет сосредоточено на несекретных данных.

23 декабря 2022 г. президент Байден подписал закон Джеймса М. Инхофа о полномочиях в области национальной обороны на 2023 финансовый год. В этот закон были включены специальные меры защиты для разоблачителей UAP (спасибо u/joeyisnotmyname за подборку этих страниц). Я повторю это еще раз: в NDAA на 2023 год были включены меры защиты информаторов КОНКРЕТНО в отношении UAP.

1 июня 2023 г. НАСА раскрывает информацию о наблюдениях НЛО на Ближнем Востоке на первой в истории публичной встрече, посвященной наблюдениям НЛО.

6 июня 2023 г., всего через полгода после подписания Байденом закона NDAA 2023 г., офицер ВВС США и бывший сотрудник разведки Дэвид Груш становится разоблачителем НЛО, утверждая, что в США существует сверхсекретная программа по поиску разбившихся НЛО.

9 июня 2023 г. Томас А. Монхейм, генеральный инспектор разведывательного сообщества, считает заявления Груша "срочными и заслуживающими доверия", что открывает путь к проведению слушаний в конгрессе.

14 июля 2023 г. лидер сенатского большинства Чак Шумер включил в NDAA в качестве поправки свой новый закон о раскрытии информации об UAP от 2023 г. В тот же день NDAA был принят. Я настоятельно рекомендую прочитать как можно больше текста Закона о раскрытии информации об UAP, если у вас есть на это время, - формулировки очень точные и убедительные.

18 июля 2023 г. координатор и пресс-секретарь Совета национальной безопасности Джон Кирби признал, что НЛО и UAP начинают создавать проблемы для ВВС США.

26 июля 2023 г., всего через полтора месяца после того, как Груш сообщил об этом, состоялись двухпартийные слушания в Конгрессе по UAP. Конгрессу предоставляется 177-страничная хронология, которая начинается с 1947 года. На этих слушаниях дают показания разоблачитель Дэвид Груш, Дэвид Фравор и Райан Грейвс. В течение почти двух часов их допрашивали о личном опыте, имеющихся у них доказательствах, а также о том, как следует поступать в дальнейшем. В связи с тем, что Груш пользуется официальными каналами защиты информаторов, он не может разглашать сведения, которые остаются конфиденциальными, но предлагает имена как враждебных, так и сотрудничающих свидетелей, а также конкретные места нахождения найденных материалов, если это будет делаться по соответствующим каналам на закрытом заседании, т.е. на S.C.I.F. К сожалению, на слушаниях не было времени для заключительных слов, но вы можете прочитать заключительное слово Груша здесь (а также его вступительное слово). Из его заключительного выступления мне особенно запомнились следующие цитаты: "Действительно, будущее нашей цивилизации и наше понимание места человечества на Земле и в космосе зависит от успеха именно этого процесса" и "Продвигаясь по этому пути, мы сможем обеспечить необычайный технологический прогресс в будущем, когда наша цивилизация превзойдет нынешние достижения в области двигателей, материаловедения, производства и хранения энергии".

27 июля 2023 г. Сенат принял МНОГОЧИСЛЕННЫЕ меры UAP, включая презумпцию немедленного раскрытия информации, дополнительные меры защиты информаторов и др.

13 Сентября 2023 В Мексиканском конгрессе выставили на обозрение трупы "инопланетян"

14 сентября 2023 НАСА выкатило отчет главный вывод которого «Главный вывод из исследования заключается в том, что еще многое предстоит узнать»

18 октября 2023 года Годовой отчет о неопознанных аномальных явлениях за 2023 год был представлен Конгрессу Министерством обороны и Управлением директора национальной разведки (ODNI) в среду, согласно пресс-релизу Пентагона, выпущенному в тот же вечер.
97 584776
>>84762
Физика электромагнитных волн. Можешь всякие приёмники-передатчики конструировать, гаусс-пушку собрать, вкатиться в радиолюбительство или клепать на ардуино какие-нибудь тестеры. Всё это относительно дёшево и безопасно, плюс находится на стыке физики, математики, инженерии, а в случае ардуино — ещё и программирования. Научишься работать руками и писать скрипты.
98 584786
Пространство не гнется. У него нечему гнутся. Тот, кто выдает изменение кривую траекторию за кривое пространство - мошенник или дурак.
Пространство реального мира 3х-мерно. Мерность пространства - это выбор. Люди для удобства выбрали угол 90, три линии под углом 90 перекрывают все пространство. Нет возможности провести четвертую. Кто пытается рассказать про 4х-мерное, 5-ти,23, 65 мерное пространство является или мошенником, или дураком.
Пространство не может быть двумерным и одномерным. Это подмена терминов. Кто называет площадь и прямую пространством является мошенником или дураком.
Мерности пространства нельзя сложить с временем и получить пространственно-временной континуум. Как нельзя сложить пространственные мерности с ветром и получить пространственно-временной континуум. Нет, ну сложить то конечно можно, но это мир гарри поттера или дурки, а не науки. Любой второклашка знает, что складывая 3 апельсина с 1 тапкой не получить ни 4 апельсина, ни 4 тапки, потому что у этих предметов разные свойства. Любой, кто так делает в науке - либо мошенник, либо дурак.
У пространства нет конца. У пространства нет начала. У пространства нет середины. У пространства нет свойств, кроме возможности вмещать и давать свободно перемещаться в любом направлении материи. Из этого следует строгий факт, что пространство бесконечно в любом из направлений. Кто рассказывает про конец пространства, его раздутие, или зависимость пространства от времени является или мошеником или дураком.
Я могу долго продолжать, но не вижу смысла.

Любая теория, акисоматика которой начинается с подобных подтасовок, является или мошеннической, или тупой. Нет смысла обсуждать что-то в этой теории, если ее аксоматика построена на подлоге. Это не наука, это фентези, бред, мошенничество, обман, развод.

Если кто-то хочет что возразить - включаете камеру на вашем мобильном телефоне и покажите треду как вы гнете пространство или засовываете руку в 4е измерение. Ждем простейший эксперимент, а не фентезийное словоблудие.
98 584786
Пространство не гнется. У него нечему гнутся. Тот, кто выдает изменение кривую траекторию за кривое пространство - мошенник или дурак.
Пространство реального мира 3х-мерно. Мерность пространства - это выбор. Люди для удобства выбрали угол 90, три линии под углом 90 перекрывают все пространство. Нет возможности провести четвертую. Кто пытается рассказать про 4х-мерное, 5-ти,23, 65 мерное пространство является или мошенником, или дураком.
Пространство не может быть двумерным и одномерным. Это подмена терминов. Кто называет площадь и прямую пространством является мошенником или дураком.
Мерности пространства нельзя сложить с временем и получить пространственно-временной континуум. Как нельзя сложить пространственные мерности с ветром и получить пространственно-временной континуум. Нет, ну сложить то конечно можно, но это мир гарри поттера или дурки, а не науки. Любой второклашка знает, что складывая 3 апельсина с 1 тапкой не получить ни 4 апельсина, ни 4 тапки, потому что у этих предметов разные свойства. Любой, кто так делает в науке - либо мошенник, либо дурак.
У пространства нет конца. У пространства нет начала. У пространства нет середины. У пространства нет свойств, кроме возможности вмещать и давать свободно перемещаться в любом направлении материи. Из этого следует строгий факт, что пространство бесконечно в любом из направлений. Кто рассказывает про конец пространства, его раздутие, или зависимость пространства от времени является или мошеником или дураком.
Я могу долго продолжать, но не вижу смысла.

Любая теория, акисоматика которой начинается с подобных подтасовок, является или мошеннической, или тупой. Нет смысла обсуждать что-то в этой теории, если ее аксоматика построена на подлоге. Это не наука, это фентези, бред, мошенничество, обман, развод.

Если кто-то хочет что возразить - включаете камеру на вашем мобильном телефоне и покажите треду как вы гнете пространство или засовываете руку в 4е измерение. Ждем простейший эксперимент, а не фентезийное словоблудие.
99 584787
>>84786
Откуда дурная паста? С ютуба воруешь смешных чуваков?
100 584791
>>84786

>Нет возможности провести четвертую.


Время же.

>5-ти


Тут сложнее.
101 584793
>>84787
написал за 3 минуты.
дурная? данный тезис является бредом потому что " твой аргумент".

>>84791

>Время


не имеет отношения к пространству, не является его мерностью.
описывает скорость протекания процессов уже внутри пространства.
102 584795
>>84793
Каждый раз когда мы добавляем новую ось. Мы видим, что каждой координате новой оси, соответствует определенное состояние
описываемого предыдущими осями пространства. Время работает точно так же как и еще одна добавленная к пространству ось.
103 584797
>>84795
когда мы к осям 1,2,3 добавляем ось 4 под углом в 90 градсуом, мы получаем ось, которая соответствует оси 1, когда добавляем 5 ось, то она соответствует оси 2, когда добавляем 6 ось, то она = оси 3, когда седьмую ось, то она соответствует оси 1. мы может проделать экспериментально 2354656843453454234534413123123124345656867342342322334345465676787 итераций и каждая новая ось будет равна 1,2 или 3.

если это утверждение неверно, пожауйста возьмите мобилу, уголок под 90 и покажите миру феномен несоответсвия.

и причем тут время?
23657689-4.jpg71 Кб, 640x426
104 584798
Есть тут тред про чудеса и тайны?

Нашел сегодня случайно такое, пещера/шахта из мегалитов с прямыми углами
https://ru.m.wikipedia.org/wiki/Харахора
Учёные, ответьте, что это? Вход в шамбалу, город атлантов или база рептилоидов?

Единственное гуглится что какой-то чел сказал что это якобы естественное образование. Но если так должны же быть аналоги, а они есть? Как например базальт трескается на бревна, это дивно, но это в тысячах мест такое, и миллион фоток гуглится. Кто нибудь знает про прямоугольные дырки в горах?

Ещё там странная небольшая пещерка есть, но явно рукотворная https://viktorkotl.livejournal.com/115863.html

И свастика видимо сделанная диверсантами или тайными нацистскими исследователями во время войны дидов 1942. Хотя пещера обнаружена только в 2011 после обвала.
105 584799
>>84797
Ты хорошо знаешь линейную алгебру? Десятимерное пространство как-нибудь представляешь?
106 584800
>>84799
если примат что-то может представить или не может, это не говорит о том существует это или нет.

10 мерное пространство не существует, но шиз из больнички степанова-скворцова или из больнички кащенко может его представить. а шиз-математик даже описать наукообразными циферками. но то, и то являются формами глупости.
107 584801
>>84800
Математические модели не описывают реальность, они лишь дают инструмент для предсказания. Трехмерное пространство настолько же нереально, насколько и четерехмерное
Т.е. ты дурачок для нас не потому что, внезапно обнаружил, что четырехмерное пространство-время это выдумка, а потому что считаешь, что трехмерное пространство существует не как абстракция математическая
108 584804
>>84800
Это уровень Лысенко какой-то. Стыдно должно быть такую хуйню нести.
109 584806
>>84797

>когда добавляем 5 ось, то она соответствует оси 2


Нет, не соответствует. Она соответствует всей предыдущей размерности разом.
110 584807
>>84798
Вполне возможно что что-то какую-то пещеру естественного происхождения допилили фашисты и местные коллаборационисты, чтобы прятаться в этой хуйне. 100-200 человек имея примитивную технику не только такую хуйню могут сделать и выпилить.
111 584809
Науч... научач... наукач .. в общем объясни почему можно до дыма нагреть влажные листья в микроволновке, но сухим будет абсолютно похуй. Типа вода так разогревается что ... поджигает листья?...........
112 584810
>>84809
Микроволновка разогревает воду которыми пропитана влажные листья.
113 584811
>>84453
Это невозможно. Изменив напряжение изменится и сила тока
114 584812
Какой кинетической энергией обладает электрон на первой орбите в атоме водорода?
115 584813
>>84809
В микроволновке стоит лампа-генератор (магнетрон), которая создает волны такой частоты, что атомы водорода начинают резонировать и нагреваться. Именно потому можно разогревать вещества, в которых есть вода, сахар или свободный водород.
А также сушить СВЧ всякие вещи, от белья и до древесины и камня.
Отсюда, кстати, все мифы про разваливание еды на атомы. Это правда.
изображение.png4 Кб, 164x56
116 584814
117 584815
>>84813
Не водород, а колебательные уровни связей -OH
Молекулярный водород ты так хуй нагреешь

И чтобы разломать молекулу возбуждая колебательные уровни, это надо чтобы у тебя молекула разъебывалась даже от простого нагревания на плите
118 584816
>>84815
Нет, гидроксо-ион имеет другую частоту резонанса. Именно атомарный водород. Тупо, как в МРТ, только с другим физическим входом.
Попробуй нагреть метан, ради интереса.
>>84815

>И чтобы разломать молекулу возбуждая колебательные уровни, это надо чтобы у тебя молекула разъебывалась даже от простого нагревания на плите


Тут без комм. Мне не интересно спорить, успехов вам, здоровья, пенсий и т. д.
119 584818
Почему человек (да и большинство животных в принципе) не может есть (полноценно усваивать) траву, листья, кору, корешки, древесину? Разве это бы не добавляло ахуенных эволюционных преимуществ и живучести?
120 584819
>>84818
Выживание просчитавшихся и не сумевших достать нормальной еды лузеров такое себе эволюционное преимущество.
Ну и травку придется жевать круглосуточно чтоб наесться никакой тебе науки и ремесел. И аппарат для жевания постоянно поддерживать даже когда не пользуешься.
121 584820
>>84804
математика появилась когда примат начал описывать количественные отношения и геометрические формы реального мира.

>инструмент для предсказания


форм и количественных отношений реального мира

>четерехмерное


если примат начинает описывать математикой мир нереальный, то он либо шиз, либо мошенник, либо писатель. 16 лапые кошки с 4 хвостами в 10 мерном пространстве существуют только у шизов, мошенников, писателей. это не имеет отношения к реальности и математике.

>трехмерное пространство существует не как абстракция математическая


т.е. ты живешь в абстракции? и сейчас из нее капчуешь?

>>84804
фееричная критика. в детском саду даже дети могут выдавить конструкцию это "глупость потому что ... ". ты не можешь.

>>84806
т.е. ты когда последовательно расскладываешь яблоки по 3м корзинам. 1 яблоко в 1, 2 яблоко во вторую, 3е в 3, 4 яблоко в 1, то 5 яблоко у тебя попадает сразу во все три? лол...
122 584821
>>84810
>>84813
он спросил про поджигание. вы ему рассказали про кипячение воды. НОУКА
123 584822
>>84816
В МРТ используется ядерный магнитный резонанс, он вообще не при чём тут

>Тут без комм. Мне не интересно спорить, успехов вам, здоровья, пенсий и т. д.


О... слился... ну после твоего примера неполярной молекулы водорода, понятно почему слился, думал тут школьники сидят только, отгадай почему пример с H2 нихуя не работает, ответ начинается на дип... и заканчивается ...ент
124 584823
125 584824
>>84820
Я живу в реальном мире, но осей x y и z нигде тут не нарисовано

Если такой умный, запиши мне пожалуйста видео на телефон, чтобы я увидел три декартовые оси и где ты к ним подходишь и руками трогаешь эти оси
126 584825
>>84824
а я где то утверждал, что оси существуют как материальные объекты?
или ты создал соломенное чучело и стал с ним воевать?
127 584826
>>84821
Вода поглощает электромагнитные волны излучаемые микроволновой антенной. Поглощая эти электромагнитные волны вода нагревается, повышается его температура это энергия переходит листьям и они загораются.
128 584827
>>84814
Это я понимаю. Но где доказательство, что электрон имеет какую-то кинетическую энергию?
129 584828
>>84820

>т.е. ты когда последовательно расскладываешь яблоки по 3м корзинам


Яблоки и корзины это не размерности пространства.
130 584829
>>84828
это пример последовательности счета.
ты сказал что >Нет, не соответствует. Она соответствует всей предыдущей размерности разом.

рандомно поставленный треугольник в пространстве образует первую плоскость
поставленный к нему второй треугольник под углом в 90 образует вторую плоскость
третий треугольник поставленный углом в 90 к первому и второму треугольнику образует третью плоскость

поставь пожалуйста 4й треугольник под углом 90 к 3 предыдущим так, чтобы он >соответствовал всей предыдущей размерности разом, а не одной из трех
поставь пожалуйста 5й треугольник под углом 90 к 3 предыдущим так, чтобы он >соответствовал всей предыдущей размерности разом, а не одной из трех
поставь пожалуйста 6й треугольник под углом 90 к 3 предыдущим так, чтобы он >соответствовал всей предыдущей размерности разом, а не одной из трех

сними это на видео.
131 584830
>>84826
от чая в микроволновке можно прикурить сигарету?
132 584831
>>84830
Просто возьми кусочек фольги и положи на сигу в СВЧ.
133 584832
>>84827
Еще раз на формулу посмотри, скудоумец.
134 584834
>>84831
но листья не являются металлом
135 584835
>>84829

>это пример последовательности счета.


Да, но пример последовательности счёта это не размерность пространства.
136 584836
>>84834
В случай листьев вода в них испаряется (переход из жидкого состояния в газ) и выходит, но передаёт часть энергии сухим листьям вот они и загораются.
137 584837
>>84835
пример счета количества не пример счета количества. понял
138 584845
>>84837
Нет, не правильно. Пример пространства это не пример счёта.
139 584847
>>84845
пример счета пространственных мерностей - это не пример счета. ясн
140 584848
>>84825
Ну если что "нематериальное" то это манямирочек и обман
141 584849
>>84848
пространство материально?
142 584850
>>84849
В нашем ограниченном восприятии - не материально.
А вообще - материально.
143 584851
>>84836
Чо у тебя за сиги такие, что мокрый кончик загорается?
144 584852
>>84850
что у пространства является материальным? где это найти? покажешь?
145 584857
>>84852

>что у пространства является материальным?


Гомеоморфность его топологии. И вообще, это не моя тематика, обратись к псиъху номер 2, уж он точно все знает, раз уж метрику углами меряет.
146 584858
>>84857
эта гомеоморфность с тобой в комнате? можешь показать ее?

> раз уж метрику углами меряет.


а чем ее надо мерить? хвостами котов?
147 584860
>>84858

>эта гомеоморфность с тобой в комнате? можешь показать ее?


Я же не Перельман. Это он ее находит везде и доказывает. Мне достаточно и пятимерной конструкции. А уж ее наличие, как подтверждение теории метрики, я вижу в материальном проявлении всего вокруг себя и во всем перцепционном многообразии.
148 584861
>>84858

>а чем ее надо мерить? хвостами котов?


Языками лахтовиков.
149 584863
>>84860

>Это он ее находит


>Мне достаточно и пятимерной конструкции. я вижу в материальном проявлении всего вокруг себя



т.е. шиза одного чела тебя не устроила, поэтому ты придумал свою.

>во всем перцепционном многообразии.



я тебя научил как видеть 3. это высота, ширина, длина. берешь прямой угол и проверяешь строгость данного утверждения. научишь как видеть 4 и 5 измерение без помощи санитаров?
150 584867
>>84863

>т.е. шиза одного чела тебя не устроила, поэтому ты придумал свою.


Аму-Няму! Покарай его!
>>84863

>научишь как видеть 4 и 5 измерение без помощи санитаров?


Для этого нужно собрать пять мужиков и диван, чел. Ты готов пройти в наш сороковой рад-град-градус? РГГ ждет тебя, сахарилло!
151 584868
>>84807
Фашисты вроде туда не дошли. Кроме может диверсантов.
152 584869
>>84826
Если вода нагреется до температуры возгорания она ебанет так примерно как тротил.
153 584870
>>84836
Блэт, при испарении листья охлаждаются.
154 584871
>>84759
так на видео мужик 3 литра масла выпил
155 584873
>>84871
Наеб. Мужик мочу пил.
156 584874
>>84836
>>84869
Водяной пар в незакрытой системе может поджечь целлюлозу?
157 584876
Если бы состав воздуха был не как сейчас (кислород 21, азот 78, остальное 1%), а резко изменился, например, кислород 30, азот 69, или кислород 40, азот 59, смогли бы люди дышать при таком составе атмосферы? Животные и растения были бы такими же или другими?
158 584879
>>84876
А тебе зачем, пидорас? Атмосферу засрать хочешь?
159 584895
>>84820

>фееричная критика. в детском саду даже дети могут выдавить конструкцию это "глупость потому что ... ". ты не можешь


Ты свои посты читал? У тебя куча сильных утверждений без доказательств, причём видно, что знаний по теме у тебя нет.
Например, мерность пространства это не выбор, это его свойство. Например, это количество векторов в базисе. Они не обязательно должны быть ортогональны друг другу, как ты писал, достаточно, чтобы они были линейно независимы. Так, как бы ты не выбирал базис для нашего пространства, в нём всегда будет три вектора, поэтому пространство трёхмерное.
Время прибавляют как четвёртое измерение, потому что оно хорошо вписывается в уравнения протекания процессов как четвёртый базисный вектор с множителем c (скорость света в вакууме).
Искривление пространства - это интерпретация тех же уравнений протекающих процессов. Причём, никакой другой интерпретации не существует. Например, LIGO пару лет назад засёк гравитационные волны - а это ни что иное как сжатие/разжатие пространства.
И так далее.
Я поэтому и назвал твои посты уровнем Лысенко - он так же нихуя не знал, подгонял действительное под желаемое, использовал ненаучные аргументы в спорах.
160 584900
Поцаны, а как же быть с тем, что пространство-время это вторичная структура, которая порождается алгебраическими свойствами комплексных пространств?
161 584901
>>84874
Нет, точно. Если его сжечь и успеть получить плазмоид в свч потоке - да. Но это нестандартные условия и свч обычно ломается при обратной связи.
162 584902
>>84876
Дышать смогли бы. Но выжили бы не все, точно.
Дело в том, что человек имеет зависимость - парциальное давление газов/давление жидкости внутри организма. И пока она изменится и подстроится под внешку - помрет 9/10. В принципе, я тебе описал происходящее в предыдущем вымирании.
163 584903
>>84900
В абстракции? Да пох.
164 584932
>>84901
откуда там дым тогда?
165 584934
>>84895

> сильных утверждений без доказательств,


приведи любое

>видно, что знаний по теме у тебя нет.


а мне видно, что у тебя лапша на ушах

> мерность пространства это не выбор, это его свойство


фееричное заявление, я выбрал угол 60 вместо 90. у нас изменилось количество мерностей. их стало 5.

покажи в каком месте это свойство в рельности изменилось с 3 до 5. на видео запиши пожалуйста трасформацию "свойства".

>как бы ты не выбирал базис для нашего пространства, в нём всегда будет три вектора,


я только что показал дал тебе иной выбор в 5 векторов. хочешь заниматься бесмысленным математическим дрочиловом - выбирай рандомный угол, дели 360 на него, вычитай 1 и вуаля.

пространство девятимерно, потому что мы выбрали 36.
пространство восьмимерно, потому что мы выбрали 40.
пространство семимерно, потому что мы выбрали 45.
пространство шестимерно, потому что мы выбрали иррациональное 51,428....
пространство пятимерно, потому что мы выбрали 60.
пространство четырехмерно, потому что мы выбрали 72
пространство трехмерно потому что мы выбрали 90

опровергай. любая выбранная мерность конвертируется в другую без нарушений и равна целому.
неважно на сколько кусков ты выберешь порезать апельсин, хоть на 3, хоть на 45, хоть 456, он соберется обратно в целый объект.
хочешь страдать - намути 456 мерное пространство и считай в нем.

>Время прибавляют как четвёртое измерение,


ты не можешь прибавить секунды к метрам. у тебя не может быть один вектор в секундах, а второй в метрах. ты не можешь работать в этой системе. это является клиническим бредом, либо полнейшим непониманием скаляров, либо мошенничеством.

единственное как ты можешь получить четырехмерное пространство - это если ты выберешь другой базис. но времени там не будет.

если ты не согласен - пожалуйста включи телефон и продемонстрируй как ты двигаешь своей рукой. сначала вверх-вниз, потом влево вправо, потом назад-вперед, а потом вперед во времени - назад во времени.

если ты это не продемонстрируешь, то ты либо ошибался всю жизнь, и тебя обманули, либо ты все понимаешь, но пытаешься обмануть меня, приравнивая свойства скалярной величины измеряющей расстояния к величине, измеряющей скорость протекания процессов. ты не можешь приравнивать одно к другому и называть это все мерностями пространства..

ты не можешь приравнять улитку к носорогу и назвать их улитками.
ты не можешь приравнять пчелу к гравитации и назвать их пчелами.
в пятый раз для самых глупых и очень медленно - ты не можешь приравнять выбранную лично тобой мерность пространства с базисом в 90' в метрах и приравнять ее к времени в секундах, назвав тоже мерностью пространства.
это действие является нарушающим закон тождества, а следовательно наука тут заканчивается и начинается гарри-поттер.

>посты уровнем Лысенко


лысенко тут ты. над кантором все смеялись. над минковским тоже в 19 веке. все нормальные посоны делали химию, физику, радио, лампочку, аккумы, движки, паровые, двс, телеграф, телефон.

что миру дал твой минковский, эйнштейн и кантор? где результаты вашей шизы?

>ненаучные аргументы


сказал фанат ото и сто, построенных на постулатах. мило.
165 584934
>>84895

> сильных утверждений без доказательств,


приведи любое

>видно, что знаний по теме у тебя нет.


а мне видно, что у тебя лапша на ушах

> мерность пространства это не выбор, это его свойство


фееричное заявление, я выбрал угол 60 вместо 90. у нас изменилось количество мерностей. их стало 5.

покажи в каком месте это свойство в рельности изменилось с 3 до 5. на видео запиши пожалуйста трасформацию "свойства".

>как бы ты не выбирал базис для нашего пространства, в нём всегда будет три вектора,


я только что показал дал тебе иной выбор в 5 векторов. хочешь заниматься бесмысленным математическим дрочиловом - выбирай рандомный угол, дели 360 на него, вычитай 1 и вуаля.

пространство девятимерно, потому что мы выбрали 36.
пространство восьмимерно, потому что мы выбрали 40.
пространство семимерно, потому что мы выбрали 45.
пространство шестимерно, потому что мы выбрали иррациональное 51,428....
пространство пятимерно, потому что мы выбрали 60.
пространство четырехмерно, потому что мы выбрали 72
пространство трехмерно потому что мы выбрали 90

опровергай. любая выбранная мерность конвертируется в другую без нарушений и равна целому.
неважно на сколько кусков ты выберешь порезать апельсин, хоть на 3, хоть на 45, хоть 456, он соберется обратно в целый объект.
хочешь страдать - намути 456 мерное пространство и считай в нем.

>Время прибавляют как четвёртое измерение,


ты не можешь прибавить секунды к метрам. у тебя не может быть один вектор в секундах, а второй в метрах. ты не можешь работать в этой системе. это является клиническим бредом, либо полнейшим непониманием скаляров, либо мошенничеством.

единственное как ты можешь получить четырехмерное пространство - это если ты выберешь другой базис. но времени там не будет.

если ты не согласен - пожалуйста включи телефон и продемонстрируй как ты двигаешь своей рукой. сначала вверх-вниз, потом влево вправо, потом назад-вперед, а потом вперед во времени - назад во времени.

если ты это не продемонстрируешь, то ты либо ошибался всю жизнь, и тебя обманули, либо ты все понимаешь, но пытаешься обмануть меня, приравнивая свойства скалярной величины измеряющей расстояния к величине, измеряющей скорость протекания процессов. ты не можешь приравнивать одно к другому и называть это все мерностями пространства..

ты не можешь приравнять улитку к носорогу и назвать их улитками.
ты не можешь приравнять пчелу к гравитации и назвать их пчелами.
в пятый раз для самых глупых и очень медленно - ты не можешь приравнять выбранную лично тобой мерность пространства с базисом в 90' в метрах и приравнять ее к времени в секундах, назвав тоже мерностью пространства.
это действие является нарушающим закон тождества, а следовательно наука тут заканчивается и начинается гарри-поттер.

>посты уровнем Лысенко


лысенко тут ты. над кантором все смеялись. над минковским тоже в 19 веке. все нормальные посоны делали химию, физику, радио, лампочку, аккумы, движки, паровые, двс, телеграф, телефон.

что миру дал твой минковский, эйнштейн и кантор? где результаты вашей шизы?

>ненаучные аргументы


сказал фанат ото и сто, построенных на постулатах. мило.
166 584935
>>84934
Так легко же, просто держишь руку некоторое время неподвижно, вот она и подвинулась на c*t, и даже размерность в метрах
167 584937
>>84935

>потом вперед во времени - назад во времени.


>держишь руку некоторое время неподвижно, вот она и подвинулась



обратное движение во времени продемонстрируй.

> на c*t, и даже размерность в метрах


этот ментальный пиздец я пока комментировать не буду. жду пруфа обратного движения во времени, там и поговорим
168 584938
>>84937

>обратное движение во времени продемонстрируй.


Для времени наше субъективное восприятие ограничено только в одну сторону, как храповик не дает проворачивать что-то обратно, так и наше физическое тело устроено так, чтобы мы не могли двигаться в обратную сторону
169 584939
>>84938
ты сейчас занимаешься демагогией.
ты либо возвращаешь свою руку во времени и меняешь свой пост, на который я сейчас отвечаю, крепя к нему фото например кабана, либо ты не возвращаешься во времени, потому что не можешь.
все очень просто.

если ты не возвращаешься назад во времени, но продолжаешь утверждать что время является пространственной мерностью, то ты либо безумен, либо глуп, либо лгунишка.

> мы не могли двигаться в обратную сторону


т.е. оснований считать что это пространственная мерность никаких, кроме постулатов из мира фентези и нарушений тождества?
170 584940
>>84939
У тебя время входит в физические законы буквально симметрично пространственным координатам
171 584941
>>84940
плевать, куда оно входит или не входит.

мы сейчас занимаемся подсчетом пространственных мерностей под углом в 90 друг к другу.
три насчитали. свойства определили. можно взад-вперед гонять по любой из трех, ничто не мешает.
ты говоришь что четвертая - это время. я прошу пруфнуть, как ты гоняешь по ней взад-вперед. ты говоришь, что тебе уже что то мешает. так является время пространственной мерностью, или ты пытаешься натянуть сову на глобус?

дальше будет смешной разговор, где ты будешь мне доказывать, как ты определил угол в 90' между высотой и временем, между шириной и временем, между глубиной и временем.

если ты не понимаешь что такое время, то это твоя проблема, а не проблема времени.
172 584942
>>84932

>откуда там дым тогда?


Пар перегретый?
173 584943
>>84942
они загораются буквально. а не парят.
174 584949
драсте

а может ли так оказаться что за всей физикой всевозможной нашей вселенной стоит пустота?
ну что на самом деле ничего нет ничего было
а то что якобы есть это какое-то мнгновение флуктуации которо тут же исчезает бесследно как будто и не было

и что результат один: все если и существует то мгновенно и бесследно и без всякой причины и следствия исчезает в абсолютное небытие

слышал чето подобное про сумму всей энергии или как там что она равна нулю

короче может ли такое быть что мы в пустоте и она схлопнется и мы исчезнем как будто и никогда и не было
175 584951
>>84949
Ты сначала определись что такое пустота.

Нулевой баланс энергии это про то, что в эпоху инфляции отрицательная энергия инфлатонного поля, которое породило инфляцию, равна энергии всей нарожденной материи(излучение + вещество).
Тут не никакой пустоты, наоборот есть стартовые параметры.
Уважаемые моды 176 584958
Хочу, чтобы по этому треду можно было ракету построить, а не узнать как относится к трансгендерам или что есть сознание, с философской позиции.
Сойбои увлечены психологиями и философиями, а также другими науками связанные с обществом.
Задал здесь, однажды, биологический вопрос, но получил опять, ебучий и связанный с обществом.
Доску можно переименовывать в "Человек и мир"
177 584961
>>84941

>мы сейчас занимаемся подсчетом пространственных мерностей под углом в 90 друг к другу.


А зачем мы этим занимаемся? А какое определение 90 градусов ты используешь? Мерность пространства не связана с углами, держу в курсе

>так является время пространственной мерностью, или ты пытаешься натянуть сову на глобус?


Так обычные x y z это тоже натягивание совы на глобус, их нигде нет, это абстракция удобная

Люди раньше думали что земля плоская, а потом обнаружили, что нет. Так и с пространством-временем, ты же плоскоземельщик почти буквально "иди посмотри на горизонт, он же плоский..."
178 584962
>>84961

> какое определение 90 градусов


половина развернутого угла

> Мерность пространства не связана с углами


правда? а с чем?
тогда мерности пространства - это ЦыWd, петух, 4 макаронины, оля, запятая, запятая. согласен? готов работать в таких терминах? ноука!!

> их нигде нет,


круто. я гле то говорил, что есть?
я по-моему 10 раз повторил, что мерности и их количество - это выбор. мы выбрали мерности которые лежат друг к другу под углом в 90, получилось 3. они заполнили все пространство без остатка.

я тебе уже предлагал другие варианты, ты проигнорил.

пространство девятимерно, потому что мы выбрали 36.
пространство восьмимерно, потому что мы выбрали 40.
пространство семимерно, потому что мы выбрали 45.
пространство шестимерно, потому что мы выбрали иррациональное 51,428....
пространство пятимерно, потому что мы выбрали 60.
пространство четырехмерно, потому что мы выбрали 72
пространство трехмерно, потому что мы выбрали 90

в любом из этих вариантов мы заполняем все пространство без остатка. это экспериментально доказуемо.

но ты говоришь что тут еще будет где то время. я все жду где ты пруфнешь угол. и свободу перемещения. чтобы твоя абстракция была корректной.

>Люди раньше думали что земля плоская


не ебет что там раньше приматы думали, дорогой демагог. воюй с ветряными мельницами в другом месте. можешь доту скачать.

> ты же плоскоземельщик почти буквально "иди посмотри на горизонт, он же плоский..."


уводишь тему хуй пойми на что, имея ноль предпосылок. как блядь в здравом рассудке можно назвать человека плоскоземельщиком, когда тот ни говорил
а) что он плоскоземельщик
б) не аппелировал к плоской земле или чему-то подобному
в) считал мерности пространства

у тебя логика поломалась? из этого не следует дедуктивный вывод, что я плоскоземельщик. просто твоя позиция слаба, у тебя закончились аргументы? и поэтому вместо чтобы со сной согласиться, тебя начало трясти.

но раз тебе можно сделать неебацца логичный вывод из данного диалога, что я якобы топлю за плоскую землю, то почему мне нельзя? такой же логичный вывод - ты сегодня ебал немецкую овчарку. это следует из количества букв Н в твоем сообщении.
178 584962
>>84961

> какое определение 90 градусов


половина развернутого угла

> Мерность пространства не связана с углами


правда? а с чем?
тогда мерности пространства - это ЦыWd, петух, 4 макаронины, оля, запятая, запятая. согласен? готов работать в таких терминах? ноука!!

> их нигде нет,


круто. я гле то говорил, что есть?
я по-моему 10 раз повторил, что мерности и их количество - это выбор. мы выбрали мерности которые лежат друг к другу под углом в 90, получилось 3. они заполнили все пространство без остатка.

я тебе уже предлагал другие варианты, ты проигнорил.

пространство девятимерно, потому что мы выбрали 36.
пространство восьмимерно, потому что мы выбрали 40.
пространство семимерно, потому что мы выбрали 45.
пространство шестимерно, потому что мы выбрали иррациональное 51,428....
пространство пятимерно, потому что мы выбрали 60.
пространство четырехмерно, потому что мы выбрали 72
пространство трехмерно, потому что мы выбрали 90

в любом из этих вариантов мы заполняем все пространство без остатка. это экспериментально доказуемо.

но ты говоришь что тут еще будет где то время. я все жду где ты пруфнешь угол. и свободу перемещения. чтобы твоя абстракция была корректной.

>Люди раньше думали что земля плоская


не ебет что там раньше приматы думали, дорогой демагог. воюй с ветряными мельницами в другом месте. можешь доту скачать.

> ты же плоскоземельщик почти буквально "иди посмотри на горизонт, он же плоский..."


уводишь тему хуй пойми на что, имея ноль предпосылок. как блядь в здравом рассудке можно назвать человека плоскоземельщиком, когда тот ни говорил
а) что он плоскоземельщик
б) не аппелировал к плоской земле или чему-то подобному
в) считал мерности пространства

у тебя логика поломалась? из этого не следует дедуктивный вывод, что я плоскоземельщик. просто твоя позиция слаба, у тебя закончились аргументы? и поэтому вместо чтобы со сной согласиться, тебя начало трясти.

но раз тебе можно сделать неебацца логичный вывод из данного диалога, что я якобы топлю за плоскую землю, то почему мне нельзя? такой же логичный вывод - ты сегодня ебал немецкую овчарку. это следует из количества букв Н в твоем сообщении.
179 584968
>>84962
Ну ладно, давай сначала начнем рассуждать о действии в механике и интегралах движения
180 584970
>>84962

>пространство девятимерно, потому что мы выбрали 36.


Нормальные люди чертят оси так, чтобы точек с невозможными координатами не было. Для этого нужно чтобы движение вдоль одной оси никак не влияло на координаты точки на других осях.
181 584971
>>84970
и что немозможного в осях под 36 градусов?

>>84968
давай. а потом обсудим усталость тормозного диска при растяжении на разрыв. какое это имеет отношение к подсчету координатных прямых?
182 584972
>>84971

>и что немозможного в осях под 36 градусов?


Точку (1,0,0,0,0,0,0,0,0) в своем девятимерном пространстве найди.
183 584973
>>84972
в смысле найди?

ты не понимаешь что если разрезать торт из центра не 3мя линиями под углом в 90, а 9ю под углом в 36, то ничего не поменяется? вообще ничего? кроме сложности рассчета.

она будет в (1,0,0), если перевести в трехмерное. нашел. что дальше? причем тут время?
184 584974
>>84973

>она будет в (1,0,0)


Не будет. Точка (1,0,0,0,0,0,0,0,0) невозможна.
У точки (1,0,0), при переходе в твою систему координат, вместо нулей обязаны быть разные значения.
185 584975
>>84974

> Точка (1,0,0,0,0,0,0,0,0) невозможна.


лол.
таааак. давай по порядку. ты можешь поставить точку отсчета и прикрепить к ней 10 палок так, что угол межу каждой палкой в точке отсчета будет 36?
186 584976
>>84975
>>84974
я переформлирую. а то доебешься до слов.

ты можешь поставить точку отсчета и прикрепить к ней 10 палок так, что угол межу двумя любыми ближайшими палками в точке отсчета был 36?
да или нет?
187 584977
>>84976
Сделай это сам. А потом попробуй поставить точку (1,0,0,0,0,0,0,0,0). И осознай что сотворил.
2023-11-1419.25.56 (Unsaved)-Blender4.0.mp46,4 Мб, mp4,
1364x740, 0:29
188 584979
189 584980
>>84979
...а теперь спроецируй ее на все остальные оси
190 584982
>>84980
хорошо. уболтал, я конечно могу полезть в залупу, как лобачевский или эйнштейн, с постулатами запрета такой проекции, но мне лень.

хорошо. вот опытным от обратного путем мы нашли количество палок координат. их 3.
не больше.
и что дальше?
191 584984
Не могу кое что понять, гравитационное поле как известно искревление пространства-времени которое массой создается, распространяется сферически во все стороны по закону обратных квадратов, на землю действует сила гравитации со стороны солнца и перпендекулярно ей сила движения, почему земля не падает на солнце когда перед ним проходит, солнце ведь тоже движется и не улетает от него когда за ним, пространство то искривленное вместе с солнцем движется?
193 584987
>>84986
у пространства нечему гнуться. модель некорректна. автор даун или мошенник.
но если ему можно нарушать тождество, то почему мне нельзя? я могу покатать шары по столу и сказать что пространство не гнется.
а если на столе сделать рельсы и покатать шарики, то можно сказать что у пространства есть невидимые рельсы

этот видос рассчитан либо на зомбирование пубертатных детей, либо на совсем патау с капающей слюной.
194 584988
>>84984

>солнце ведь тоже движется


в смысле вокруг ядра галактики?
двигаясь вокруг ядра галактики Солнце тащит Землю вместе с собой
Земля вокруг ядра галактики движется с той же скоростью, что и Солнце
195 584989
>>84988
Да я вроде сам понял, вообщем похоже когда земля перед солнцем движется если за перед солнца считать направление в котором оно движется земля увеличивает скорость и центробежная сила увеличивается, она компенсирует то что земля в более напряженном поле находится, а когда ща ним наоборот уменьшается.
196 584992
>>84987

> у пространства нечему гнуться. модель некорректна. автор даун или мошенник.


Он вначале говорит, что это модель. Модель работает, имеет предсказательную силу, значит моделью пользуются. Но можно быть долбаебом как ты, кричать что это неправда и поэтому моделью пользоваться нельзя, и вместо того чтобы делать по этой модели расчеты движения планет и звёзд и летать к Луне и на Марс надо строить суперкомпьютер для расчёта гравитационного взаимодействия и остановить все развитие на сотни лет. Какой компьютер нужно построить для полёта на Луну по правильной модели? Его на земле хотябы построили?
197 584993
>>84992
это неверная модель. мне повторить в 35 раз? у пространства нечему гнуться.
гнутая траектория не значит пространство погнулось.
гнутая траектория - это просто гнутая траектория.
футбольный мяч летящий по баллистической траектории не гнет пространство. пространству похуй как мяч летит. прямолиненйно или криволинейно.
пространству абсолютно похуй.

> вместо того чтобы делать по этой модели расчеты движения планет и звёзд и летать к Луне и на Марс надо строить суперкомпьютер для расчёта гравитационного взаимодействия и остановить все развитие на сотни лет. Какой компьютер нужно построить для полёта на Луну по правильной модели? Его на земле хотябы построили?



сформулируй мысль. я не понял что тут было сказано. идея такова: "без релятивистских поправок не долететь до луны?" я все верно понял?

так я спешу тебя расстроить что матан наших полетов, что американских есть публичном доступе. там нет никаких релятивистских поправок, многомерного пространства, лямбда сидиэм моделей и прочей срани. ее даже в жпс нет, вся документация в паблик домене.

к чему ты это тогда написал?
Стикер575 Кб, 160x150
198 584996
>>84992
1. Своими глазами видит, что все шарики падают на большой шар
2. Знает, что Земля не падает на Солнце
3. Говорит с ебалом эксперта: >Модель работает

Что ты такое?
199 584998
А разве частицы движутся не по градиенту напряженности гравитационного поля, которое есть искривление пространства-времени?
3456757798.png229 Кб, 798x766
200 585001
Так шо, мы стоим на закате (очередного) золотого века навуки?
В общем то и без всяких навучных изысканий это очевидная истина. За 30+ лет своей жизни я не наблюдаю никаких прорывов в технологиях. Ну кроме разве что гаджетов, хотя и они основаны на довольно старых технологиях, которые довели до ума. В остальном все теже машины, самолеты, ракеты, дома и т.д, что и 50-100 лет назад.
Компухтеры вроде резво начали, но в итоге тоже сдулись. Еще лет 5-10 и технология дойдет до своего пердела. При этом даже на горизонте ничего нового не видно, даже в очень отдаленной перспективе.
201 585002
>>85001
навука кончилась в 1920е-30е. с тех пор просто шла инженераи имплементация в железо того, что "успели" открыть до релятивистов и квантователей. ничего существенного в фундаментальной науке с тех пор более не происходило. никаких существенных феноменов не было открыто или нормальных моделей построено, потому что теоретические построения исходят из ложной аксиоматики с гнутыми пространствами и сказками про вакуум из ничего рождающий что-то.
вообще чудо, что инженеры рожают иногда до сих пор вопреки моченым, которые занимаются откровенным бредом. вся натурфилософия превратилась в инфоцыганщину с шарлатанами-мошенниками, имеющими академические степени, на полном серьезе рассказывающими, что гнутая прямая - это тоже прямая, а не кривая. а то и вообще вводящие неопределяемые понятия. им место в тюрьме или в дурке.
перспективы грустные. пиздец короче.
202 585003
>>85001
Ты просто не замечаешь, как и люди в 50-x не замечали, ибо между разработкой в лаборатории и попаданием тебе на глаза в магазине огромный лаг в 20-30 лет
Биологи много всякого говна наоткрывали за последние годы, генная терапия для медицины это же буквально как холодный термоядерный синтез для физиков.

А по поводу твоей картинки, объемы научных статей просто стали больше (поэтому на одну статью меньше влияния) так еще и сама наука стала тяжеловесной, даже открыв невероятные феномены, ты лишь создашь еще одну ветку, а не полностью перевернешь всю науку.

А вообще жди третью мировую, вот там попрет так попрет
203 585004
>>84934

>> мерность пространства это не выбор, это его свойство


>фееричное заявление, я выбрал угол 60 вместо 90. у нас изменилось количество мерностей. их стало 5.


Я не просто так сказал, что ты нихуя не знаешь.
Базис по определению линейно независимый, а твои векторы будут линейно зависимые. Можно выбрать три вектора в пространстве, такие, что углы между ними попарно будут 60 градусов, и они составят базис трёхмерного пространства, но как только ты добавишь любой вектор к этим трём, это перестанет быть базисом, потому что этот четвёртый вектор можно будет выразить через первые три.
Так вот, это первый семестр физ и мат факультетов, а ты дальше рассуждаешь о вещах, которые требуют старших курсов. Ты не просто не обладаешь нужными знаниями, ты даже не знаешь, что такие знания существуют.
Твоё требование про телефон это опять Лысенковщина. Типа, если электрон не щупал руками, значит его нет. Я не виноват, что твой мозг воспринимает время именно последовательно. Можешь видео поперематывать туда-сюда, вот тебе и движение во времени вперёд-назад.
ОТО подтверждается опытами. Не существует ни одного эксперимента, который бы опровергал ОТО.
204 585005
>>85001
Я сосачую вот этого >>85003, но хочу добавить, что наука реально замедлилась.
Дело в том, что всё простое уже открыто. Понимание СТО доступно многим, ОТО уже сложнее. Квантмех в разных его частях может быть понятен от мало кому до долей процентов от всех людей.
Индустриализация высвободила много человеческих ресурсов для брейншторма реальности, научный метод позволил сделать поиск более качественным, но это просто дало линейное ускорение логарифмическому или вовсе асимптотическому процессу познания.
Это примерно как взять историю мировых рекордов по, например, прыжкам в высоту. Мы сначала увидим быстрый рост, который будет всё замедляться и замедляться. Так и в науке. У нас были ебейшие открытия в первой половине XX века, неплохой прогресс во второй половине, но теперь уже ничего крышесносного не происходит. Да, что-то открывают потихоньку, но гораздо реже и менее значительное.
205 585007
>>84996

>Знает, что Земля не падает на Солнце


Но ведь Земля падает на Солнце...
206 585009
>>85002
Ну все же до 60-70ых что то да придумывали, а потом уже начался явный спад.
>>85003
Ой да иди ты нахуй. Все я замечаю, ибо прожил больше четверти века. Ты знаешь сколько всего менялось за этот срок в 19 веке или в начале 20 века? От паруса до парохода, от семафора до телеграфа, от дирижабля до самолета и т.д. Армиям приходилось каждые 15-20 лет проводить полное перевооружение, ибо появлялась куча нового оружия (а сейчас из нового только игрушечные дроны). Так что пиздуй нахуй со своими потешными робопылесосами и гейфонами.
Какая нахуй биология мать твою? Корону победили средневековыми методами, карантином и прививками. Спидораки как были так и есть, вон Джобса даже его миллионы не спасли. Термояд это пустышка для прогрева гоев, уже как 50 лет там никаких сдвигов. Коколайдер вообще ничего нового не открыл, хотя это детище всего мира и ничего подобного ближайшие лет 50 уже не создадут. Про кокосмос я вообще молчу, он сдох и превратился в мавзолей куда залетают только по великим праздникам. ИИ вообще смехатура для фотожопа
207 585010
>>84996

> 1. Своими глазами видит, что все шарики падают на большой шар



Еблан, в примере на видео шарики падают на большой шар из-за потерь на трение-качение, а в космосе их нет.
208 585011
>>85005
Да нихуя индустриализация не дала. Людей стало в 8 раз больше, а открытий в 10 раз меньше. А крупных открытий вообще по нулям. Тупо перевели людей из одной сферы труда в другую, все равно что поменяли шило на мыло.
Принципиальная проблема в том любые технологии имеют конечную точку развития, и бесконечно паразитировать на них не получится. А чтобы открыть что то новое, как показала практика, бабла и кучи краснодипломных долбоебов не достаточно. Для этого нужно буквально божественное озарение.
Кто то даже предсказывал технологическую пустыню в ближайшие десятилетия или даже столетия. К чему в общем то все и идет. Так что футуристы коколонизаторы соснули хуйца
209 585013
>>85001

>мы стоим на закате (очередного) золотого века навуки?


Не очередного, а последнего.

>Еще лет 5-10 и технология дойдет до своего пердела


Не, не так скоро.

>на горизонте ничего нового не видно


Ну как сказать...
- Темная материя/энергия толсто так намекает, что есть куда расти. Руки опускать пока рановато.
- Гравитационно-волновая астрономия началась как наблюдательная наука только в 2015 году.
- Ни много ни мало, а в 2025 году вполне может случиться самая настоящая долгожданная новая физика:
9 июля 2023 года коллаборация Muon g-2 завершила наработку экспериментальных данных, продолжавшуюся шесть лет. Первые результаты, полученные после обработки данных первого года работы, были опубликованы 7 апреля 2021 года[12]. Учёные сообщили, что результаты исследований мюонов бросают вызов Стандартной модели и, соответственно, могут потребовать пересмотра существующей модели элементарных частиц. Результаты первых трёх лет сбора данных коллаборация опубликовала в августе 2023 года. Ожидается, что окончательные результаты, основанные на статистике за полные шесть лет измерений, будут представлены в 2025 году.

https://ru.wikipedia.org/wiki/Muon_g-2

Тем не менее, никакую гипотетическую новую физику в около/бытовые технологии превратить не удастся. Это даже не фантастика, это сказки.
https://www.youtube.com/watch?v=navIPCYhSss
210 585018
>>85013

>Не очередного, а последнего.


С чего бы? Ты плохо знаешь историю. Таких прорывов была целая куча. И еще будут, но видимо не сейчас

>Не, не так скоро.


Уже почти дошли до предела. Если 20 лет назад каждое поколение процов выдавало в десятки раз большую производительность, то сейчас прибавка лишь в 2-3 раза, да и то не факт. Ну и дальнейшее уменьшение транзисторов физически невозможно.

>Темная материя/энергия толсто так намекает, что есть куда расти. Руки опускать пока рановато.


А причем тут технологии?

>Гравитационно-волновая астрономия началась как наблюдательная наука только в 2015 году


Очередная хуйня без задач

>Ни много ни мало, а в 2025 году вполне может случиться самая настоящая долгожданная новая физика


Как уже заебали эти громкие заголовки... А вот как обстоят дела в реальности https://www.youtube.com/watch?v=BW7xaKua71o
211 585019
>>85018

>Уже почти дошли до предела


5-10 лет - это роадмап какого-нибудь Интела или Н-Видии. И через 10 лет они закрываться не собираются.
Как минимум будут пилить аппаратную поддержку нейронок.

>Темная материя/энергия


>А причем тут технологии?


Технологии тут не удел. Но доска все-таки посвящена науке и для науки это имеет фундаментальное значение.

>Гравитационно-волновая астрономия


>Очередная хуйня без задач


Нонешними средствами мы не можем заглянуть в прошлое Вселенной раньше, чем 300 000 лет от начала расширения, т. к. до этого времени Вселенная была непрозрачна.
Гравитационным волнам на прозрачность пофигу, поэтому с их помощью можно заглянуть вплоть до самого начала. В частности, это позволит экспериментально доказать (или опровергнуть) живем ли в Мультивселенной или наша Вселенная единственная.

>эти громкие заголовки


Это не громкие заголовки, это многолетний эксперимент, движущийся к завершению. Обоснованно ожидается, что будет открыто пятое взаимодействие.

> в реальности


Как же заебали эти наивные реалисты
https://ru.wikipedia.org/wiki/Наивный_реализм
212 585020
>>85001

> Еще лет 5-10 и технология дойдет до своего пердела. При этом даже на горизонте ничего нового не видно, даже в очень отдаленной перспективе.


Ебать дурачок ты малой.
На горизонте еще столько всего, что на твои 10 000 жизней не хватит работать.
213 585021
>>85018

>Очередная хуйня без задач


У гравитации до хрена задач.
214 585025
>>85004
налил водички, красиво.

>только ты добавишь любой вектор к этим трём, это перестанет быть базисом, потому что этот четвёртый вектор можно будет выразить через первые три.



а знаешь почему так? потому что пространство трехмерно.
теперь сманяврируй как ты добавляешь сюда четвертое измерение под названием время и не видишь противоречий.

И Я ВСЕ ЖДУ ВИДЕО, ГДЕ ТЫ ДОСТАНЕШЬ КАМЕРУ И ПОКАЖЕШЬ МНЕ СВОЙСТВО ОБРАТНОГО ПЕРЕМЕЩЕНИЯ ВО ВРЕМЕНИ ЭКСПЕРИМЕНТАЛЬНО.

НЕ ЗАБУДЬ ЗАОДНО ПОГНУТЬ ПРОСТРАНСТВО.

>Твоё требование про телефон это опять Лысенковщина.


мое требование называется эксперимент. ты отрицаешь это действие, поздравляю ты шарлатан и крутишь жопой.

>Можешь видео поперематывать туда-сюда


больше демагогии. больше, мошенник.

>ОТО подтверждается опытами.


ой, а что такое. т.е. эксперимент, чтобы пруфнуть примитивную аксиоматику ты провести не можешь и крутишь жопой. а вот якобы эксперимент где в 100000 больше матана и составляющих ты проводишь и все подтверждается. так ничего не подтверждается, вы просто мошенники, которые выдаете желаемое за действительное и обманываете всех вокруг.
gps не работает на ОТО или поправках СТО. документация в доступе - приноси пруф.

> твой мозг воспринимает время именно последовательно.


твой мозг воспринимает, что ты сейчас не сосешь у немецкого шепарда. на деле ты сосешь. НОУКА постановила, что лично ты зоофил!11

>>85007
на твоей модели все падает за 15 секунд. нам осталось жить 15 секунд. проведем эксперимент. ждем.

ты наебал. твоя модель не сработала

>>85010
а еще знаешь чего нету в космосе? ГНУТОГО ПРОСТРАНСТВА АЗАЗА
214 585025
>>85004
налил водички, красиво.

>только ты добавишь любой вектор к этим трём, это перестанет быть базисом, потому что этот четвёртый вектор можно будет выразить через первые три.



а знаешь почему так? потому что пространство трехмерно.
теперь сманяврируй как ты добавляешь сюда четвертое измерение под названием время и не видишь противоречий.

И Я ВСЕ ЖДУ ВИДЕО, ГДЕ ТЫ ДОСТАНЕШЬ КАМЕРУ И ПОКАЖЕШЬ МНЕ СВОЙСТВО ОБРАТНОГО ПЕРЕМЕЩЕНИЯ ВО ВРЕМЕНИ ЭКСПЕРИМЕНТАЛЬНО.

НЕ ЗАБУДЬ ЗАОДНО ПОГНУТЬ ПРОСТРАНСТВО.

>Твоё требование про телефон это опять Лысенковщина.


мое требование называется эксперимент. ты отрицаешь это действие, поздравляю ты шарлатан и крутишь жопой.

>Можешь видео поперематывать туда-сюда


больше демагогии. больше, мошенник.

>ОТО подтверждается опытами.


ой, а что такое. т.е. эксперимент, чтобы пруфнуть примитивную аксиоматику ты провести не можешь и крутишь жопой. а вот якобы эксперимент где в 100000 больше матана и составляющих ты проводишь и все подтверждается. так ничего не подтверждается, вы просто мошенники, которые выдаете желаемое за действительное и обманываете всех вокруг.
gps не работает на ОТО или поправках СТО. документация в доступе - приноси пруф.

> твой мозг воспринимает время именно последовательно.


твой мозг воспринимает, что ты сейчас не сосешь у немецкого шепарда. на деле ты сосешь. НОУКА постановила, что лично ты зоофил!11

>>85007
на твоей модели все падает за 15 секунд. нам осталось жить 15 секунд. проведем эксперимент. ждем.

ты наебал. твоя модель не сработала

>>85010
а еще знаешь чего нету в космосе? ГНУТОГО ПРОСТРАНСТВА АЗАЗА
215 585027
>>85004

>Можешь видео поперематывать туда-сюда, вот тебе и движение во времени вперёд-назад.


Итак эксперимент, где мы будем пруфать одинаковые свойства у пространственных мерностей и времени.

Запускаем эксперимент.
Все это должно двигаться единомоментно. Я понимаю что у вас не 10 рук, но вы справитесь. Записываем на пленку:
1) шарик летит вниз,
2) другой шарик летит вверх.
3) шарик катится влево
4) другой шарик катится вправо
5) шарик катится от нас
6) другой шарик катится к нам
7) песочные часы ссыпают песок вниз
8) другие песочные часы ссыпают тоже вниз. ой, а что такое. время работает только в одном направлении?

но нам мошенник посоветовал мошенническое действие.

>Можешь видео поперематывать туда-сюда,


воспользуемся. воспроизводим видео в обратном направлении:

1) шарик летит вверх
2) другой шарик летит вниз.
3) шарик катится вправо
4) другой шарик катится влево
5) шарик катится к нам
6) другой шарик катится от нас
7) песочные часы ссыпают песок вверх
8) другие песочные часы ссыпают тоже вверх. ой, ДА ЧИВО ТАКОЕ ТО....НЕПРИЯТНА

экспериментально доказано - время не является мерностью пространства, не дает свободу перемещения, эйнштейн мошенник, ты - манька-религиозный верун в авторитеты.

не плакай.
215 585027
>>85004

>Можешь видео поперематывать туда-сюда, вот тебе и движение во времени вперёд-назад.


Итак эксперимент, где мы будем пруфать одинаковые свойства у пространственных мерностей и времени.

Запускаем эксперимент.
Все это должно двигаться единомоментно. Я понимаю что у вас не 10 рук, но вы справитесь. Записываем на пленку:
1) шарик летит вниз,
2) другой шарик летит вверх.
3) шарик катится влево
4) другой шарик катится вправо
5) шарик катится от нас
6) другой шарик катится к нам
7) песочные часы ссыпают песок вниз
8) другие песочные часы ссыпают тоже вниз. ой, а что такое. время работает только в одном направлении?

но нам мошенник посоветовал мошенническое действие.

>Можешь видео поперематывать туда-сюда,


воспользуемся. воспроизводим видео в обратном направлении:

1) шарик летит вверх
2) другой шарик летит вниз.
3) шарик катится вправо
4) другой шарик катится влево
5) шарик катится к нам
6) другой шарик катится от нас
7) песочные часы ссыпают песок вверх
8) другие песочные часы ссыпают тоже вверх. ой, ДА ЧИВО ТАКОЕ ТО....НЕПРИЯТНА

экспериментально доказано - время не является мерностью пространства, не дает свободу перемещения, эйнштейн мошенник, ты - манька-религиозный верун в авторитеты.

не плакай.
216 585030
>>85019
Короче будет как с коколайдером, много шума, мало толку
Но ты главное ВЕРЬ
>>85020
Всем насрать на эти теории, речь о практическом применении.
И на горизонте нет никакой альтернативы транзисторам
Уже лет так 30 визжат про кококвантовые компухтеры, а на деле они уступают даже не самому новому айфону
>>85021
Ну подтвердят одну из доминирующих теорий и че? Впрочем не факт что подтвердят.
Такое ощущение у местных шизов какая то форма дислексии. Им говоришь о замедлении развития, они в ответ начинают кидать какие то громкие заголовки, что вот вот ухх!! Надо только подождать!! Только раньше не ждали, а делали
217 585033
>>85027
Ты предлагаешь измерять время с помощью органов чувств, но наше чувство времени это просто сломанная рулетка, которая умеет только разматываться, но умеет сматываться, наше тело просто ебануто устроено, а ты пытаешься реальность натянуть вокруг восприятия, ну это немного тупо. Пока люди не начинают вмешиваться в процессы, все частицы не различают разницы между туда и обратно (ну.. там с оговорками).
218 585034
>>85033

>предлагаешь измерять время с помощью органов чувств,


это ты предлагаешь, я такого нигде не говорил

>наше чувство времени это просто сломанная рулетка


наше чувство времени идеально. иначе бы я бы не смог играть ритмично на гитаре.

> но умеет сматываться,


доказательства будут? я пока слышу только голословные утверждения

> наше тело просто ебануто устроено,


в данном случае банут твой мозг/
который не понимает чем метр отличается от секунды.
что это разные сущности.
первая измеряет расстояние. вторая измеряет скорость протекания конкретного физического процесса в конкретных физических условиях. метры не зависят от секунд. секунды не зависят от времени.

дегенерат эйнштейн привязал метр к секунде через свет. основание для этого действия было только Я СКОЗАЛ. или по другому постулат. постулаты не являются наукой. потому что постулировать можно любой бред. это работает в фентези и рпг, но не работает в реальности. ей наплевать что там напостулировал дегенерат эйнштейн.

метры все также не зависят от секунд. а моя фраза что ты сосал у собаки, также врядли имеет отношение к реальности.

> не начинают вмешиваться в процессы,


парадокс наблюдателя появляется только у дегенератов, которые напридумывали себе постулатов. а потом реальность взяла и нассала на ролеплей данжнмастера. потому что реальности похуй на то какой ее видит данжн мастер. виновата не реальность, а дегенерат данжн мастер, которые не может ее засунуть в корректную модель.

в реальности никаких парадоксов не существует. там все примитивно тупо и скучно. но вы своими фентезийными постулатами ломаете мозг и себе и окружающим.
219 585035
>>85034

> метры не зависят от секунд. секунды не зависят от времени.


секунды не зависят от метров.* фикс
220 585036
>>85034

>скорость протекания конкретного физического процесса в конкретных физических условиях


Что такое скорость?
221 585037
>>85034

>чем метр отличается от секунды


метр отличается от секунды размером
1 секунда = 299792458 метров

>скорость протекания конкретного физического процесса в конкретных физических условиях


расстояние, на которое конкретный физический процесс в конкретных физических условиях успевает переместиться во времени за время своего протекания

мимодругой
222 585038
>>84943
бамп
223 585039
>>85038
температура самовоспламенения
224 585040
>>85036
пройденное расстояние в пространстве за эталон времени, который определен, как скорость протекания конкретного физического процесса в конкретных физических условиях

>>85037
красиво. >секунды равны метрам.
но идиот почему то я, а не ваша манятеория с вашим гуру-шарлатаном

ананасы равны карликовым слонам.
сигареты равны стальным трубам
а нефть равна кофе

> процесс в конкретных физических условиях успевает переместиться во времени за время своего протекания


конкретный физический процесс, который мы выбрали эталоном времени, мой будильник перемещается вокруг центра земли, вокруг солнца, в рукаве галактике, а вся эта срань куда то двигается.
где релятивистские поправки? ах, да, это другое.
225 585041
>>85040
Быстро летящие частицы дольше живут.
image.png25 Кб, 300x100
226 585042
Почему эта машинка не поедет? Или поедет?
227 585043
>>85040
Т.е. у тебя скорость определяется через время, а время через скорость? Окей, хули
228 585044
>>85039
что её достигает? пар?
229 585045
>>85042
когда магнит тянет машину, машина тянет магнит
230 585046
>>85044

>пар?


что-то что с ним контактирует
231 585047
>>85040

>ананасы равны карликовым слонам


не, не равны
если ананас заменить на кролика (или наоборот), результат поменяется

от замены секунд на соответствующее количество метров (или наоборот) не поменяется ничего
232 585048
>>85046
ну лист контактирует. он загорается. как?
233 585049
>>85048
Вода греется и греет лист. У листа температура самовозгорания ниже чем у воду. Вот он и самовозгорает.
234 585051
>>85043
господи. ты можешь различить скорость перемещения в пространстве и скорость закипания кастрюли воды?
скорость перемещения в пространстве и скорость изменения цвета хамелеона?
скорость перемещения в пространстве и скорость сгорания листа бумаги?

ни кастрюля с водой, ни хамелеон, ни лист бумаги никуда не перемещаются. но скорость процесса есть.

у тебя ментальный ступор после эйнштейна?

>>85047
ананас = метру
секунда = карлику
метр=секунде
ананас=карлику

опровергай.
235 585052
>>85051

>ни хамелеон, ни лист бумаги никуда не перемещаются


их части перемещаются
236 585053
>>85049
то есть пар зажигает лист?
237 585054
>>85052
куда у тебя перемещается вскипающая кастрюля? ты в палате номер 6?
238 585056
>>85051
Что такое скорость?
239 585057
>>85056
скорость чего?
240 585062
>>85057
Скорость перемещения
241 585064
>>85062
ты глупый?
>>85040
242 585068
>>85064
Что такое

>скорость протекания конкретного физического процесса в конкретных физических условиях


???
243 585069
>>85068
напишу сказку. так уж и быть. а то вы настолько промыты что не понимаете, чотакое время...

представим ты разумное теплокровное существо. у тебя нет часов. ни стрелочных, ни песочных, ни атомных, никаких. даже солнечных, потому что ты живешь допустим в пещере подо льдом антарктиды в 5 км от поверхности, где темно. снизу камень и мерзлая почва, по бокам тонны льда, сотнекилометровая сеть пещер

итак ты разумное теплокровное существо из подо льда антарктиды. ты не слышал ни про эйнштена, ни в курсе про аристотеля, то что ты летаешь вокруг звезды, ты не знаешь про космос, ты нихуя не знаешь.

ты просто разумное теплокровное существо из подземелья. и вот ты решил заняться наукой. как ты заимеешь концепт времени?

предлагаю рандомный вариант.

ты знаешь что лед тает, если ты долго будешь греть его свои пузом. откалываешь кусок льда объемом примерно в свою залупу и кладешь себе на пузо, он естественно тает.
ты называешь это 1 залупа.
2 таких куска последовательно тают за 2 залупы.
3 куска последовательно тают за 3 залупы.
полкуска тает за ползалупы.
треть куска тает примерно за треть залупы.

поздравляю ты освоил время и примитивные часы (скорость протекания конкретного физического процесса в конкретных физических условиях) .

далее ты измерил длину своего хвоста. и назвал это 1 хвост. поздравляю, ты изобрел расстояние.

ты измерил свою пещеру от стены до стены, получилось 160 хвостов.

ты пригласил кореша из соседней пещеры помощь тебе, и попросил пройти твою пещеру от стены до стены. кореш пошел, а ты последовательно клал себе куски льда на пузо.

кореш прошел расстояние в 160 хвостов за 4 залупы. скорость движения кореша = 40 хвостов в залупу.

ты все еще ничего не знаешь ни про вселенную, ни про солнце, ни про скорость света, ни про эйнштейна, ни про вращение планеты, но у тебя уже есть величины для измерения времени и расстояния. причем довольно точные.

через 20000 залуп развития этой цивилизации появится местный эйнштейн-дегенерат, будет рассказывать, что расстояние относительно таяния льда, что 1 залупа на самом деле = 1 хвосту. и что время неотделимо ото льда и зависит от скорости его таяния. появится местный дегенеративный релятивизм. будут рассказывать что если использовать 3 пуза, чтобы погреть лед, то расстояние блядь сокращается. а когда проверят, что нихуя не сокращается, то будут вилять жопой и рассказывать про эффект наблюдателя. и тд тд.

сказка-аналогия закончена.
243 585069
>>85068
напишу сказку. так уж и быть. а то вы настолько промыты что не понимаете, чотакое время...

представим ты разумное теплокровное существо. у тебя нет часов. ни стрелочных, ни песочных, ни атомных, никаких. даже солнечных, потому что ты живешь допустим в пещере подо льдом антарктиды в 5 км от поверхности, где темно. снизу камень и мерзлая почва, по бокам тонны льда, сотнекилометровая сеть пещер

итак ты разумное теплокровное существо из подо льда антарктиды. ты не слышал ни про эйнштена, ни в курсе про аристотеля, то что ты летаешь вокруг звезды, ты не знаешь про космос, ты нихуя не знаешь.

ты просто разумное теплокровное существо из подземелья. и вот ты решил заняться наукой. как ты заимеешь концепт времени?

предлагаю рандомный вариант.

ты знаешь что лед тает, если ты долго будешь греть его свои пузом. откалываешь кусок льда объемом примерно в свою залупу и кладешь себе на пузо, он естественно тает.
ты называешь это 1 залупа.
2 таких куска последовательно тают за 2 залупы.
3 куска последовательно тают за 3 залупы.
полкуска тает за ползалупы.
треть куска тает примерно за треть залупы.

поздравляю ты освоил время и примитивные часы (скорость протекания конкретного физического процесса в конкретных физических условиях) .

далее ты измерил длину своего хвоста. и назвал это 1 хвост. поздравляю, ты изобрел расстояние.

ты измерил свою пещеру от стены до стены, получилось 160 хвостов.

ты пригласил кореша из соседней пещеры помощь тебе, и попросил пройти твою пещеру от стены до стены. кореш пошел, а ты последовательно клал себе куски льда на пузо.

кореш прошел расстояние в 160 хвостов за 4 залупы. скорость движения кореша = 40 хвостов в залупу.

ты все еще ничего не знаешь ни про вселенную, ни про солнце, ни про скорость света, ни про эйнштейна, ни про вращение планеты, но у тебя уже есть величины для измерения времени и расстояния. причем довольно точные.

через 20000 залуп развития этой цивилизации появится местный эйнштейн-дегенерат, будет рассказывать, что расстояние относительно таяния льда, что 1 залупа на самом деле = 1 хвосту. и что время неотделимо ото льда и зависит от скорости его таяния. появится местный дегенеративный релятивизм. будут рассказывать что если использовать 3 пуза, чтобы погреть лед, то расстояние блядь сокращается. а когда проверят, что нихуя не сокращается, то будут вилять жопой и рассказывать про эффект наблюдателя. и тд тд.

сказка-аналогия закончена.
244 585070
>>84980
>>84974
Я тут подумал 5 минут, а ведь у этой хуйни есть простое элегантное решение, без прибегания к постулатам.

Ты говоришь, что такая точка невозможна, и держишь в голове угол проекции на другую ось естественно в 90 градусов. Ну так в модели пространства, где у тебя угол между осями 36 градусов - тебе внезапно надо проецировать под углом 36, а не под 90. Там где 40, проецировать под углом 40, а не под 90 и тд. Вот и вся аксиоматика.
(1,0,0,0,0,0....) возможна без нарушения чего либо. Можешь взять линейку и потратить полминуты и понять, что все проецируется.

Там кстати полезут интересные строгие математические формулы при проекции на дальние оси, назовем их осями второго, третьего и далее порядков, типа фрактальных прогрессий, не знаю как это корректно обозвать. Думаю это дерьмо уже где-то описано у любителей дрочить шизомногомерия просто в других терминах. Но пока лень про эт думать и писать.
245 585071
>>85070
Не надо думать, Анон. Надо проецировать.
246 585072
>>85054
Частицы кастрюли двигаются быстрее - кастрюля греется.
247 585074
>>85069
А есть какое-то строгое определение, а не графомания?
248 585075
>>85074
ладно, ты потроллить решил.

Время – сравнительная количественная характеристика физических процессов, происходящих в пространстве, привязанная к
длительности физического процесса, взятого за эталон.

скорость = расстояние в пространстве / время.

>>85072
я рад за частицы. вскипевшая кастрюля = 1 единица времени. 2 вскипевших кастрюли= 2 единицы времени. и тд.
в эталонной кастрле может быть литр воды и все, хоть политра, хоть поллитра и курица, хоть кокакола и свиная голова. какое мне дело что там происходит с частицами свиной головы и частицами курицы и воды, когда мне важен факт вскипания, чтобы взять ео за эталон?

>>85071
я спроецировал. все сошлось.
количество мерностей пространства - это выбор. единицы времени - это выбор. единицы расстояния - это выбор.
249 585076
>>85075

>я спроецировал. все сошлось.


у тебя не должно сходиться. ни под каким углом
250 585077
>>85075

>происходящих в пространстве


Т.е. процесс происходит, если что-то движется в пространстве? Не чувствуешь какой-то наеб?
251 585086
>>85076
возьми нарисуй и спроецируй. все сходится

>>85077
а ты можешь чем то двигать вне пространства? жду видео.
252 585087
>>85086

>возьми нарисуй и спроецируй. все сходится


...а теперь черти линии от точки до каждой оси под одним и тем же углом.

>а ты можешь чем то двигать вне пространства? жду видео.


Мораль в том что количество движения в пространстве влияет на количество движения во времени. Отсюда и связь между метрами и секундами.
253 585088
>>85087

>.а теперь черти линии от точки до каждой оси под одним и тем же углом.


они идут в ноль, в начало координат.

>Отсюда и связь между метрами и секундами.


нет. просто кринж и какой то паралич мозга.

если ты берешь эталоном метра длину банана и эталоном часа за сколько вода вытекает из дырявого ведра, то ты считаешь скорость в длинах банана/опустошенные ведра.

из этого не следует что длина банана фундаментально связана с опустошением ведра.

как мне еще изъебнуться и показать бредовость всех потуг найти связь между рандомно взятыми эталонами?
254 585089
>>85088

>они идут в ноль


нет не идут, по такой системе каждая точка вне оси будет нулем на всех осях
255 585090
>>85088
идея в том что пройденное любым объектом за единицу времени расстояние всегда одинаково
и проходится оно частично в пространстве, частично во времени
256 585091
>>85088
метр и секунда — это абстракции
их объективно не существует
объективно существуют расстояния между объектами в пространстве и во времени
метр и секунда — это два рандомных эталона, которые придумали люди
эталон можно "приложить" вдоль любого направления, включая временнóе
257 585098
>>85089
нет, с чего вдруг? оно будет работать по паттернам
(1,1,1,0,0,0,0,0,0)
(0,0,0,1,1,1,0,0,0)
(0,0,0,0,0,0,1,1,1)
если примитивно.

а если дрочить обмазываясь калом, как любят математики, то появится доп вариант оперировать не точками, отрезками и площадями, а любыми фигурами, например четырехугольниками, или пятиугольниками. надо что бы количество мерностей было больше или равно, чем кол-во углов основания фигуры.
кому нехуй делать и любит дрочить в позе раком левой пяткой - может поинтегрировать какое-то дерьмо в такой системе, находя площади многоугольных фигур.

есть путь дальше - вывести аксиоматику и матаппарат интегрирования площади внутри объема в доп координатах внутри допкоординат.

только нахуя это все надо. там можно голову сломать. я вот нарисовал в блендере пространство деленное под 60, покрутил и понял, что этот монстр работает, все там проецируется. но там на каждой итерации можно сдохнуть запутаться и обмякнуть.

и даже если я пиздец как неправ, то я все еще не вижу причем тут время.

>>85090
оно не проходится ни в каком времени. потому что времени не существует объективно. а вот расстояние объективно и существует.

>>85091

>метр и секунда — это два рандомных эталона, которые придумали люди


спасибо что повторил ровно то что я тут талдычу.

>объективно существуют расстояния между объектами в пространстве и во времени


нет батенька, тут вы занимаетесь подменой терминов. расстояние между пиком ленина и эльбрусом не имеет в себе никакого времени.

>эталон можно "приложить" вдоль любого направления, включая временнóе


у времени нет направления. у пространства есть расстояние и есть направление движение по нему материального объекта. у времени нет направления. если ты считаешь, что есть, то я просто в 24 раз требую, если ты не согласен - продемонстрировать движение во времени назад.
движение вбок по времени.
если ты не можешь, то это объективно. а раз это объективно, то время и расстояние - разные сущности.
одно является свойством самого пространства, второе является свойством материи.
257 585098
>>85089
нет, с чего вдруг? оно будет работать по паттернам
(1,1,1,0,0,0,0,0,0)
(0,0,0,1,1,1,0,0,0)
(0,0,0,0,0,0,1,1,1)
если примитивно.

а если дрочить обмазываясь калом, как любят математики, то появится доп вариант оперировать не точками, отрезками и площадями, а любыми фигурами, например четырехугольниками, или пятиугольниками. надо что бы количество мерностей было больше или равно, чем кол-во углов основания фигуры.
кому нехуй делать и любит дрочить в позе раком левой пяткой - может поинтегрировать какое-то дерьмо в такой системе, находя площади многоугольных фигур.

есть путь дальше - вывести аксиоматику и матаппарат интегрирования площади внутри объема в доп координатах внутри допкоординат.

только нахуя это все надо. там можно голову сломать. я вот нарисовал в блендере пространство деленное под 60, покрутил и понял, что этот монстр работает, все там проецируется. но там на каждой итерации можно сдохнуть запутаться и обмякнуть.

и даже если я пиздец как неправ, то я все еще не вижу причем тут время.

>>85090
оно не проходится ни в каком времени. потому что времени не существует объективно. а вот расстояние объективно и существует.

>>85091

>метр и секунда — это два рандомных эталона, которые придумали люди


спасибо что повторил ровно то что я тут талдычу.

>объективно существуют расстояния между объектами в пространстве и во времени


нет батенька, тут вы занимаетесь подменой терминов. расстояние между пиком ленина и эльбрусом не имеет в себе никакого времени.

>эталон можно "приложить" вдоль любого направления, включая временнóе


у времени нет направления. у пространства есть расстояние и есть направление движение по нему материального объекта. у времени нет направления. если ты считаешь, что есть, то я просто в 24 раз требую, если ты не согласен - продемонстрировать движение во времени назад.
движение вбок по времени.
если ты не можешь, то это объективно. а раз это объективно, то время и расстояние - разные сущности.
одно является свойством самого пространства, второе является свойством материи.
258 585099
>>85098

>оно не проходится ни в каком времени. потому что времени не существует объективно. а вот расстояние объективно и существует.


Угу... расстояние то реально существует, не выдуманная концепция а РЕАЛЬНОСТЬ
259 585100
>>85099
расстояние между пиком ленина и эльбрусом реально?
260 585105
>>85100

>расстояние между пиком ленина и эльбрусом реально?


Только для рожденного ползать.
Рожденный ползать сворачивать пространство своё не может от того и ползает. Но далеко не уползёт. У него вилка. Эволюция или вымирание.
261 585109
>>85098

>нет батенька, тут вы занимаетесь подменой терминов. расстояние между пиком ленина и эльбрусом не имеет в себе никакого времени.


нет батенька, тут вы ошибаетесь
между пиком ленина и эльбрусом есть расстояние в пространстве и есть расстояние во времени
с точки зрения пика ленина эльбрус находится немножко в прошлом
с точки зрения эльбруса пик ленина находится немножко в прошлом
нельзя отдалиться от объекта в пространстве, не отдалившись при этом и во времени
262 585111
Никто сможет ответить, что произойдет с водой, если ее сжимать при огромных давлениях как в центре планеты, например. Учоные еще не определились как работают жидкости))))
263 585113
>>85100
Время за которое свет летит от пика ленина до эльбруса реально?
264 585114
>>85111
Ты пробовал задавать вопрос в гугл?
265 585115
>>85111
Никто не сможет ответить сколько у твой мамки было половых партнеров )))))
266 585116
>>85113
вы еврей? ответьте на мой вопрос.
на ваш отвечаю - нет. есть только расстояние.

>>85109

>расстояние во времени


нет, его нет. есть только расстояние в пространстве. время за которое кто-то проходит это расстояние - это действие. что-то может совершать действие, а может и не совершать.
между эльбрусом и пиком ленина можно преодолевать расстояние, воспроизводя репчик. значит есть расстояние в репчике?

>с точки зрения пика ленина эльбрус находится немножко в прошлом


>с точки зрения эльбруса пик ленина находится немножко в прошлом


нигде они не находятся. это шиза относительности. проистекающая из того что мы все делаем относительно скорости света, запостулировав свет как инвариант.

если мы запостулируем репчик по воздуху, как инвариант, то что, получится, что горы относительно друг друга находятся побольше в прошлом?

>нельзя отдалиться от объекта в пространстве, не отдалившись при этом и во времени


постулирую почту как инвариант. я живу в квартире 44, и шлю письмо в квартиру 45. кидаю его в почтовый ящик на стене. приходит почтальон, вынимает письма из ящика, везет в на почту, там сортируют, почтальон выезжает раскидывает, приходит житель 45 квартиры и вынимает письмо. прошло 3 дня.

выводы - выходит между квартирой 44 и 45 расстояние во времени 3 дня? и пространство погнуто?
только не крути носом. это буквально так вы применяете ОТО
267 585117
Чому гейство это плохо?
268 585118
>>85116

>на ваш отвечаю - нет. есть только расстояние


А как вы это поняли? Мы можем измерить и одно и другое... в чём разница?
269 585119
>>85118
между вершинами 2х горе есть расстояние? есть
между вершинами 2х гор есть время? нет

из-за того что примат начал измерять время полета чайки, звука, света, брошенного бычка время в пространстве не появилось. а вот расстояние мужду ними как было так и останется, хоть ты его замеряй, или не замеряй. хоть замеряй неправильно. расстояние в пространстве- объективно существует.
за сколько бычок, чайка или шмеля пролетят это расстояние - пространству поебать. могут вообще никуда не лететь. ему похуй.
270 585120
>>85116

>нет, его нет


а вот говорят, между Землёй и Солнцем есть расстояние во времени
дескать мы видим Солнце каким оно было ~8 минут назад
врут?

и ещё про какое-то аномальное смещение перигелия Меркурия
нет аномального смещения?
271 585121
>>85119

>между вершинами 2х горе есть расстояние? есть


Т.е. просто "зуб даю есть"? Что считаешь критерием существования?

>за сколько бычок, чайка или шмеля пролетят это расстояние - пространству поебать. могут вообще никуда не лететь. ему похуй


В том дело что не похуй, в разных системах отсчета расстояние разное будет
272 585122
>>85120

>, между Землёй и Солнцем есть расстояние во времени


т.е. ты мне доказываешь существование времени тезисами, выходящими из парадигмы ОТО?
что дальше? будешь доказывать непорочное зачатие парадигмой из библии? а гиперлуп парадигмой из звездных войн лукаса?

>дескать мы видим Солнце каким оно было ~8 минут назад


и что дальше? мне почтальон из 45 в мою 44 квартиру привез письмо спустя 3 дня после его отправки. это значит что между квартирами в полуметре расстояние в 3 дня?

хватит блядь называть задержку сигнала расстоянием.

> аномальное смещение перигелия Меркурия


представляешь размер половины листика а4?
273 585123
>>85121

> разных системах отсчета расстояние разное будет


не будет. оно будет везде одинаковым. у тебя мозг окончательно сломался?
из-за того что ты запускаешь чайку, цессну, су27, калибр, свет между двумя вершинами - расстоянию между вершинами абсолютно похуй.

> Что считаешь критерием существования?


между твоим носом и пяткоой есть расстояние? если оно есть, то это объективно. если ты между носом и пяткой пустишь муравья или бросишь камень, то пройдет время. ты сделаешь субъективный выбор, считать свою задержку сигнала по скорости муравья или по полету камня. это блядь субъективный выбор, потому что времени нет. ты черрипикаешь процесс как эталон
274 585124
>>85123

>не будет. оно будет везде одинаковым. у тебя мозг окончательно сломался?


Ты споришь с результатами экспериментов? Ладно

>если оно есть, то это объективно


Мои глаза его измеряют с помощью фотонов, прикинь?

>ты черрипикаешь процесс как эталон


Предложи другой процесс, но такой, который не меняет расстояние в разных системах отсчета. Есть какой-то магический способ измерять расстояние без действия?
275 585125
Заметил, что многие ученые довольно поверхностные, например, недавно прочитал неплохую книгу одного американского доктора биологии и его жены 2020 года, в целом мне очень понравилось, но автор на мой взгляд сделал ошибку в одном суждении: они подумали, что все люди одомашнены в примерно равной степени и поэтому когда он попытался объяснить наличие жестокости у людей, то делал это уже имея большое заблуждение насчет людей. А так-то понятно, что какие-то люди более одомашнены, чем другие, которые могут не сильно отличатся от наших диких, вымерших родственников, таких как гомо эректусы и гомо сапиенс неандертальцы и вот от таких людей и происходит жестокость, часть из них - психопаты.

В общем на мой взгляд это ошибка авторов книги в следствии недостаточно глубокого изучения вопроса.
276 585126

>т.е. ты мне доказываешь


нет, я ж вопрос задал, а ты не ответил
врут ли все, кто говорят, что Солнце с земли видно с задержкой в 8 мннут?

>это значит что между квартирами в полуметре расстояние в 3 дня?


почтальон из 45 в мою 44 квартиру привез письмо спустя 3 дня после его отправки. это значит что между квартирами в полуметре расстояние в 3 дня?
между квартирами 0.5 метра
между событиями начала и конца доставки 77706205113600 метров

>представляешь размер половины листика а4?


представляю
а ты опять не ответил
есть аномальное или нет?
277 585128
>>85124

>Ты споришь с результатами экспериментов? Ладно


пример эксперимента.

>Мои глаза его измеряют с помощью фотонов, прикинь?


т.е. когда ты закрываешь глаза, расстояние между пяткой и носом нет?

> который не меняет расстояние в разных системах отсчета.


он меняет его только в вашей болезной теории, где вы постулируете постоянство скорости света и невозможность ее превысить. без экспеиментом, без нихуя, без доказательств, просто на основании я скозал.

>Есть какой-то магический способ измерять расстояние без действия?


время без попытки его измерить не существует.
расстояние без попытки его измерить существует.

тигр ничего не знает про метры, ни про секунды. но до жопы лани есть объективное расстояние.
278 585129
>>85126

>врут ли все, кто говорят, что Солнце с земли видно с задержкой в 8 мннут?


не, врут. и что из этого следует?
ты вызвал два такси на двух телефонах, стоишь на шоссе.
ты видишь эти такси на карте, они на равном расстоянии от тебя.
приложение на 1м телефоне говорит что такси приедет через 1 минуту со скоростью 100кмч.
приложение на 1м телефоне говорит что другое такси приедет через 2 минуты со скоростью 50 кмч.
какой фундаментальный вывод из этого следует? (никакого)

>между событиями начала и конца доставки 77706205113600 метров


круто, пространство погнуто. шуе ппш

>есть аномальное или нет?


объясняю. хорошо что представил.
аномалия есть. но объяснение ОТО притянуто за уши.

рассказываю как был сделан "ноучный" эксперимент якобы подтвердивший ОТО с перигелием меркурия.

1) вот у тебя половинка листа А4. только век назад это была не бумажка, а фотопластинка такого же размера. ебнули эмульсию, серебра. сфоткали. проявили

2) прошел год - взяли ДРУГУЮ фотопластинку с ДРУГОЙ эмульсией сфоткали в ДРУГОЕ ВРЕМЯ отличающееся на несколько секунд с ДРУГИМ состоянием атмофсеры с ДРУГОЙ окружающей тепературой с ДРУГИМ атмофсерным давлением, сфоткали, проявили ДРУГИМ составом, залили ДРУГИМ закрепителем, получили ДРУГОЕ расположение зерен серебра и тд

сравнили 1) и 2) и нашли что одно зерно серебра уехало как якобы предсказывает ОТО = ОТО доказано.
это буквально сука так было сделано. буквально блядь. я не шучу.


это пиздец, а не наука.
279 585130
>>85129

>какой фундаментальный вывод из этого следует?


если мы доверяем приложениям, то можно сделать вывод, что от таксопарка до тебя выбирай (разные названия одного и того же):
1.66 километра
1.03 сухопутной мили
0.92 морской мили
0.0000055 секунды

>пиздец, а не наука


я правильно понимаю, что человечество до сих пор не знает, из-за чего прецессирует орбита Меркурия?
280 585131
>>85130
опять 25.

метры = секундам.
негры = меду
лабутены = кочерге

>из-за чего прецессирует орбита Меркурия?


да
edMezon.mp413,7 Мб, mp4,
640x360, 5:56
281 585132
>>85131
это ж и почему мезоны долетают — тоже не ясно, получается
282 585133
>>85132
все что говорится под портретом эйшнтейна не является наукой, потому что у людей там сломан мозг. саентологическая секта.
я комментировать это не буду.

потому что пространство не гнется, время не является пространственной мерностью, постулат про инвариант скорости света неверен, постулат что нет нечего быстрее скорости света нарушается самим же светом который в плазме хуярит быстрее, а в плазме бод углом еще быстрее, и прочая и прочая. это не наука. это марксизм-ленинизм с циферками.
Time-dilation-002-mod.svg.png55 Кб, 1920x678
283 585134
>>85133
чем объясняется увеличение длины траектории фотона в движущейся системе зеркал по сравнению со стационарной?
284 585135
>>85134
а, скорость света не инвариант, тупанул, сорри
285 585136
>>85134
ничем не объясняется, потому что это вялый эксперимент, а если прямо - нерелевантное криво поставленное говно, где все цифры притянуты за уши релятивистской аджендой, чтобы получить грантик.

чтобы просто хоть как-то измерить скорость света, хоть сколько нибудь точно - тебе понадобятся километровые установки, потому что она очень большая.
чтобы измерить с минимальными знаками до запятой - тебе понадобится стерильная многокилометровая установка
а чтобы померить то, что у тебя на пике тебе понадобится едущая со значительной скоростью многокилометровая стерильная установка , где все внешние воздействия устранены.

как ты понимаешь ничего подобного нигде не делалось. это все маняфантазии.

как говорится если эксперимент не поставлен в реальности, то тем хуже для реальности (c) ото и группа шизов математиков
286 585137
>>85136

> чтобы просто хоть как-то измерить скорость света, хоть сколько нибудь точно - тебе понадобятся километровые установки, потому что она очень большая.


Эм?

The Laser Interferometer Gravitational-Wave Observatory (LIGO) типа просто макет?
287 585138
>>85137
на ligo проводят другой маняэксперимент, а не тот, который у тебя на картинке. я хуй знает к чему ты это запостил.
288 585148
Вот щас щас очко человечеству ии порвет.
Как можно в это верить
289 585153
>>85148
Человечество уже в рабстве ИИ, лучшие умы человечества пущены для того, чтоб делать ИИ лучше. Считай принесли самый дорогой ресурс человечества в жертву кремневому богу.
8c4134909ce756ed9e521929fa184880.gif6,2 Мб, 800x450
290 585161
Все машины времени в массовой культуре, что я видел, перемещают человека не только во времени, но и в пространстве (хотя этот аспект никогда не подчеркивается и не осмысливается).

Я правильно понимаю, что в силу движения Солнечной системы в пространстве необходимо еще и с огромной точностью телепортировать человека куда-то на поверхность Земли, иначе он в 99,9999% случае отправится в другое время в космос? Где-то разбирались подобные вопросы?
291 585165
>>85161

>Где-то разбирались подобные вопросы?


да, в этом треде.
перемещения во времени невозможны, потому что время не является мерностью, в нем невозможно перемещаться. конец сказке.
292 585166
>>85165
мерность простраснтсва*
293 585167
Я патлач. Постоянно вычёсываю у себя дохуя волос, за время их накапливается целая куча просто.
Почему в таком случае у меня 99.9% волос всё ещё одной длины, и мне не лезут в глаза волосы которые выросли на месте вычесанных?
294 585168
Почему радиосвязь при своем появлении не вытеснила полностью проводные телефоны (как беспроводной телеграф полностью вытеснил обычный)?
Почему даже к моменту появления компьютеров люди продолжали пользоваться проводными телефонами?
Почему на основе радио не создали интернет в 1950-х годах и на первых персоналках (конец 1970-х) не было радиомодема, а для интернета приходилось подключаться по dial-up даже в начале нулевых?
Только сейчас обычные телефоны начинают уходить в историю, вытесняясь мобильной связью с ее высокими скоростями и нетребовательной инфраструктурой (достаточно лишь построить вышку, в жилых домах никакого оборудования проводить не нужно).
В чем секрет долгоживучести проводных телефонов и в чем их преимущество над радиоволнами?
295 585169
>>85165
Время это вообще какая-то НЕХ, наука не может сказать что это такое. Поэтому машина времени находится за пределами понимания науки, мы можем лишь сказать, что в природе нет ни единого факта передачи энергии или материальных объектов из будущего в прошлое. Плюс это нарушило бы закон сохранения энергии и позволило бы создать вечный двигатель. В современной научной картине мире вечные двигатели невозможны.

На бытовом уровне можно описать перемещение во времени как процесс изменения объектов. Разные объекты могут изменяться с разной скоростью, как в парадоксе близнецов. Также время как-то связано с гравитацией, еще одной плохо понятной областью для современной науки (см. темную материю и энергию).

В математических моделях есть шкала времени и четырехмерное пространство, но это именно математическая модель, а не реальность.

Ну и в черных дырах со временем и массой происходят сплошные аномалии, которые и представить нельзя.
296 585175
>>85168
Стабильность. Радио сигнал постоянно помехи для стабильности нужны хорошо экранированные провода.
Удобство. Мобильные телефоны настолько удобней что стабильность уже не тянет.
297 585176
>>85169
черных дыр не существует. это тоже математическая маняфантазия.
в теории существуют черные звезды, где вторая космическая меньше гравитационной постоянной, поэтому свет не может просто улететь. и то врядли такие объекты возможны. нужна очень большая масса.
но ко всем маняфантазиям релятивистов о черных дырах, гнутых пространствах и замедленных временах эти объекты не имеют отношения.

>парадоксе близнецов.


нет никакого парадокса близнецов
298 585178
>>84809
чё всё? лучшие умы доски ничего не смогли ответить внятного?
299 585179
>>85053
Да. Что не так?
300 585181
>>85179
помаши листиком бумаги над кастрюлей с пельменями
изображение.png945 Кб, 1024x730
301 585185
>>85181
Во первых, махать нужно не листиком бумаги, а сушёными листьями. Во вторых, влаги над кастрюлей с пельменями несравненно больше, чем в сушёных листьях. В третьих, эта влага идёт извне, а не находится внутри, как в случае с сушёнными листьями которые зажигает микроволновка. Так что пикрил.
image.png8,2 Мб, 3000x2000
302 585186
>>85185
давай так. какой химический процесс в пробирке мне загуглить, где из вещества каким то образом абсорбируется вода, и вещество из-за этого само загорается?
303 585187
>>85125

>ошибка авторов книги в следствии недостаточно глубокого изучения вопроса.


Ты не прав, ошибка тут вот в этом:

>американского доктора биологии


Американский биолог с докторской степенью не может принять простую вещь - на планете обитает 1 вид людей с разным генетическим разнообразием, которое делит людей на типы. Признав такое нетолерантное свойство, этот биолог отправится кормить голубей на улицу и потеряет степень.
304 585188
>>85176
Адекват в теме.
305 585190
https://www.youtube.com/watch?v=RFh3akPPwBE

Как вкатиться в кибернетику как на видео?
Чтобы изучать эффективные менеджерские схемы с точки зрения кибернетики?
306 585210
>>85148
Это ебнутая секта футуристов
>>85153
Ага, а еще в рабстве у солнца, воздуха, воды, нефти, газа, огня, зерна, коровы и т.д. Таблетки прими, раб
307 585226
есть ли люди, решающие задачу 3+тел в уме?
308 585246
>>85226
Да, если у них развитый звуковой вектор.
309 585248
Посоветуйте легких учебников, чтобы хотя бы чуть-чуть понимать разные науки.
310 585256
>>85248
Легкие учебники слишком скучные, сложные учебники слишком сложные. Всем не угодишь.
311 585259
>>85248
Поварнин. Как читать книги
Сасскинд, Грабовски. Теоретический минимум
Семихатов. Всё, что движется
Никитин. Происхождение жизни
312 585286
>>84346 (OP)
Можно ли изобрести "чудо печку", чтобы бытовой мусор "превращать в чистый кислород"?
313 585287
>>85259
Благодарю.
>>85256
Ага. Вот бы кто-нибудь взял и написал легкий учебник, который интересно изучать.
Или сложный учебник, который легко понимать.
314 585291
>>85167
Бамп
315 585296
>>85286
Можно, но она будет жрать дохуя энергии, причем больше чем выделиться при сжигание топлива для получения оной.
316 585300
к абортосрачу Верно ли, что вытащить из бабы живую зиготу и поддерживать её жизнедеятельность за копейки нет никаких трудностей?
317 585301
>>85300
Зигота, зигота перейди на Федота.
318 585302
>>85301
зигота стримит доту
319 585306
>>85300
Трудности начинаются, когда зигота начала дифференциацию клеток и стала вырабатывать сигнальные вещества, которые включают целую гормональную программы в тушке матери, замыкания тем самым петлю обратной связи. Разрыв этой связи несет очень серьезные последствия как для зиготы, так и для матери.
320 585309
Есть тут жизнь какая-нибудь?

Посоветуйте годных учебников по ядерной физике/химии, с упором на теоретическую базу, где будет рассказываться, какие бывают изотопы и с чем их едят, какие продукты распада у разных веществ, и всё с этим связанное. Какие (Приблизительно) процессы происходят в атоме, базовые формулы, термины необходимые для вката во всё это.

Учебники по физике и химии для школы - сразу нахуй. Во-первых, там всё очень поверхностно, с кучей условностей и упрощений, которые только сбивают с толку, а во вторых, это я уже читал
321 585314
>>85309
Матан, линал, КЭД знаешь?
322 585315
>>85306
А заморозить зиготу в одноклеточном состоянии (чтоб либо она была в анабиозе и в любой момент можно оживить, либо просто жила, но не делилась) возможно? Речи о дальнейшем делении и формировании личинуса не идёт. Типа чтоб сказать: вот, мы не убили человека, живёт себе. Ну да, немножко недоразвитый, но не убили же, да?
323 585317
Я понимаю что это фильм, что там все натянуто.
Но он позиционирует себя как псевдонаучный.
Но даже тут у меня вопрос.
Это для тех кто фильм смотрел.

Понятно что раздел про науку, но все же.

Посмотрел фильм Прибытие 2016 года, я его и раньше смотрел.
Там инопланетяне изменили разум Луизы подобно своему, а они находятся сразу во всех точках временной линии одновременно. То есть для них понятие времени, оно не такое как у людей.
И всегда возникает вопрос, как Луиза узнала номер генерала?
Если её сознание находиться сразу во всех точках, но во всех точках определенной временной линии, линии где человеки напали на инопланетян. И в этой временной линии нет того банкета, за все хорошее где генерал лично прилетел к Луизе и сообщил свой номер.
Это сюжетная дыра, или я что то не понимаю?
Ну то есть она видит будущее, грубо говоря. Будущее где человеки напали на инопланетян. Но берет информацию из будущего где генерал дал её номер в будущем, где в прошлом она позвонила ему и остановила нападение.
324 585326
Интеллект во взрослом возрасте можно повысить? Не в смысле общую эрудированность или профессиональные навыки, а конкретно когнитивные способности головного мозга. Или если генетически тебе суждено быть 98iq дуралеем, то ты им и останешься?
325 585327
>>85314
Частично. Мои знания ограничиваются первым курсом вузика
326 585328
>>85326
Можно. Но в молодости это сделать проще.
https://habr.com/ru/articles/395965/
Мозг настолько нейропластичен, что может относительно нормально функционировать даже без 90% своих тканей, по крайней мере в единичных случаях и при постепенной потере этих тканей, чтобы было время на адаптацию.
Что уж говорить о возможности развития интеллекта. Но тебя на харкаче могут начать убеждать в обратном, вера в полный генетический детерменизм и/или в то что развиваться можно только в молодости здесь очень распространена из-за того что тут каждый второй самооправдывающийся лентяй и/или нытик.

>>85186

>где из вещества каким то образом абсорбируется вода


Она не абсорбируется.
Она нагревается до такой степени, что начинает нагревать другие составляющие сушёного листа, от чего лист возгорается. Не вижу ничего непонятного в этом. Лист же может загореться от зажигалки, независимо от того что в нём есть вода? Может. Почему он не может загореться от очень-очень горячей воды, которой настолько мало, чтобы перекрыть листу доступ к кислороду, и она настолько горяча, что не может забрать у листа энергию, которая нужна ему для возгорания, а может только наоборот, отдавать?
327 585331
>>85328

>может относительно нормально функционировать даже без 90% своих тканей


>Что уж говорить о возможности развития интеллекта.


Так ты ничего и не сказал этой ссылкой. Возможность адаптации мозга к деградации никак не доказывает возможности мозга к прогрессиированию во взрослом возрасте.
328 585332
>>85331
Отчасти это, конечно, справедливо, но как деградировавший мозг мог бы функционировать на описанном в статье уровне без его развития? Адаптация к уменьшению размера подразумевает, что среднее "качество" мозга на кубический сантиметр должно быть увеличено.
В любом случае, этой статьёй я просто подкрепил мои утверждения, которые основываются на личном опыте, это не железобетонный пруф, да.
329 585339
Как на практике осуществима генная модификация при помощи crispr-cas? В интернете одна хуйня научпопная, не нашёл ни как получить белки криспр и кас9 из бактерий, ни как заложить в кас9 нужную последовательность аминокислот. Если первое, могу предположить, получают, используя хроматограф, то со 2 вообще не понятно нихуя.
Что конкретно надо с чем смешивать, начиная с сырья в виде культуры бактерий и химических реактивов с небольшой молекулярной массой, чтобы геном редактировать у, например, червей?
330 585340
>>84346 (OP)

Вопрос по физике. Начитался всякой херни о квантовой запутанности, и словил несварение мозга. Анон поясни - есть две запутанные частицы. Измение состояния одной меняет состояние другой. Причём мгновенно. Что будет, если одну из них подвергнуть релятивистким эффектам? Они как-то согласуют системы отчета, или у них, у каждой будет своё мгновенно?
331 585341
>>85317
Там нет временных линий есть один сценарий детермизма, или даже фатализма.
Как герою фильма сказать что ониумрет в конце сюжет останется тот же
332 585342
>>85340
Две или более запутанные частицы на самом деле одна квантовая система и происходит одно измерение, но двумя детекторами.
Вся запутанность между детекторами, а не частицами, которые измеряют. И чтоб запутанность пронаблюдать, нужно приготовить нужное состояние для частицы и синхронизировать детекторы. Вот тут и причиность ебет всю мгновенность.
333 585343
>>85340

>Измение состояния одной меняет состояние другой. Причём мгновенно.


Нет.
У тебя есть красный и синий шарик. Ты, не глядя, кладешь их в две коробки. Потом одну из них открываешь и волшебным образом узнаешь что лежит во второй.
334 585352
>>85343
Иди читай матчасть. Твои шарики образуют статистическую смесь, это не выпуклая комбинация элементов гильбертова пространства. Тогда как для суперпозиции внедиагональные элементы матрицы плотности ненулевые, откуда и возникает интерференция и запутанность.
335 585381
>>85352
Метод окраски шариков сути процесса не меняет.
Факт в том, что если один из них перекрасить, то со вторым не случиться вообще ничего. Связь между ними существует только в твоем воображении.
336 585384
>>85381
Где я хоть что-то сказал про покраску?
Ещё раз, открой учебник, хуже не будет.
337 585385
>>85384

>открой учебник, хуже не будет.


Еблан, если ты не можешь простыми словами донести мысль до другого, то не факт, что ты эту мысль сам понимаешь
338 585386
вот этот >>85352 правильно говорит
суперпозиция состояний это линейная комбинация состояний как векторов, то есть есть одно единое состояние, выражаемое как сумма
а то о чем пишет >>85343 это вероятностная смесь, то есть когда веса (вероятности) просто отражают степень неопределенности
эти понятия различали почти столетие назад, в нормальных учебниках или даже статьях википедии будет все написано
советую книжку чанга про квантовые вычисления для вката
это >>85381 вообще какой-то нерелевантный бред
339 585387
>>85386
Суть в том, что если бы частицы действительно были связаны, то можно было бы с легкостью организовать мгновенную связь через любые расстояния. А ее всё нет и нет.
И все эксперименты состоят из тупого открывания одной коробки с последующими попытками представить знание о содержимом другой коробки результатом телепортации.
Но это всё просто болтовня.
340 585389
>>85387
Ты просто не шаришь за теорию измерений, поэтому не понимаешь в чём не прав. А тратить время на пояснения всем лень, т.к. ты наверное даже математику не знаешь
341 585391
>>85315

>А заморозить зиготу в одноклеточном состоянии (чтоб либо она была в анабиозе и в любой момент можно оживить, либо просто жила, но не делилась) возможно?


Конечно и это давно распространенная практика. И эмбрионы замораживают (на стадии нескольких клеток), рекорд с родами 20 лет вроде.
342 585396
>>85389

> Я сам не понимаю, потому обьяснить не могу


Понятно
Trinity Test Latest HD Restoration.mp4167 Кб, mp4,
840x412, 0:04
343 585405
Наукач поясни что происходит в центре на шебм. На шебм первые секунды первого в мире испытания технологии ядерного оружия. Я вообще не шарю, я так понимаю это плазма, но почему плазма делает "пуньк" сама в себя, ну как будто какой то портал открывается и туда плазма вбирается, как в кине епта научно фантастическом. Эта херь обьяснена уже? Че за эффект тогда)
344 585406
>>85405
В плазме скорость звука значительно выше чем в окружающей среде, отчего часть ударных волн на границе раздела фаз сами себя отражают и сжимают.
Так же при расширении плазменный шар охлаждается, превращается в газ или пыль.
В сочетании этих факторов плазма сжимается собственной ударной волной и одновременно худеет из-за охлаждения.
345 585408
>>85406
Нихуя себе, спасибо анонче
346 585439
Вселенная расширяется потому что из большего числа мест до нас начинает долетать свет или потому что тёмная энергия? Со временем мы будем видеть звёзд больше или меньше?
347 585441
Можно ли скрестив гены псины и хряка получить свинособаку!!?
348 585442
>>85439

>Со временем мы будем видеть звёзд больше или меньше?


Меньше. В какой-то момент вообще вся Вселенная станет тёмной. Галактики-то ещё будут, но где-то там настолько быстро удаляющиеся, что свет их никогда нас не достигнет. Будущие обитатели галактик будут думать что их галактика и есть вся Вселенная, как мы до Хаббла считали. А может и мы тоже чего-то важного для понимания уже не видим
349 585445
>>85439
Потому что темная энергия.
И наоборот, нового места становится больше, а наполнение этого места тоже самое.
350 585479
Есть какие-то современные виды птиц, уходящие эволюционными корнями в меловой период? Или все современные птицы происходят от одного предка, жившего уже после мелового вымирания?
351 585480
>>85479

> Или все современные птицы происходят от одного предка, жившего уже после мелового вымирания?


Предка которого инопланетяне завезли? Птицы прямые потомки динозавров.
352 585481
>>85480
Общий предок-птица современных птиц появился до или после мел-палеогенового вымирания?
353 585482
>>85480
Общий предок-птица современных птиц появился до или после мел-палеогенового вымирания? >>85480
354 585484
1. Почему в русскоязычной среде принято отрицать/приуменьшать значение IQ? В англоязычных странах это делают только по политическим мотивам. У нас же IQ отрицает и народ, и публика с претензией. Это так в вузах преподают? Откуда такое отношение?
2. Почему в РФ настолько распространено отрицание потепления?
355 585485
>>85484

>1. Почему в русскоязычной среде принято отрицать/приуменьшать значение IQ? В англоязычных странах это делают только по политическим мотивам. У нас же IQ отрицает и народ, и публика с претензией. Это так в вузах преподают? Откуда такое отношение?


IQ показывает исключительно надроченность на логических задачках. Решал задачки — показываешь высокие результаты. Не решал — не показываешь. С таким же успехом "уровень интеллекта в вакууме" можно было бы измерять способностью распознавать цвета (точнее, знать названия всяких серобуромалиновых оттенков) или скоростью решения кубика-рубика. Тупо приобретаемый навык.

>2. Почему в РФ настолько распространено отрицание потепления?


Потому что ещё в лохматые годы учёные говорили о климатических циклах и о том, что через десятилетия в Москве будут бананы расти. Т.е, никакого антропогенного потепления от того, что коровы пукают, нет. Периоды потеплений-похолоданий были всегда, на протяжении всей истории человечества.
356 585486
>>85485

>IQ показывает


Твое неверное мнение это очень интересно, конечно, но я спрашивал о другом.

>Потому что ещё в лохматые годы учёные говорили о климатических циклах


У тебя не больше 90, я думаю. Понимание текста даётся с трудом.
357 585487
>>85486

> но я спрашивал о другом.


На западе давно признано, что важность IQ равна 20%, а на 80% рулит эмоциональный интеллект.
358 585488
>>85487
80? Ты как, шнурки легко себе завязываешь, проблем нет? Держись там.
359 585489
>>85484

>1. Почему в русскоязычной среде принято отрицать/приуменьшать значение IQ? В англоязычных странах это делают только по политическим мотивам.


Так у нас тоже политические. Вон как сейчас топят за IQ, мол, господин Барин выиграл в генетическую лотерею и получил приз - высокое айсикью и надел рабов с тянками, ему положено, генетические детерминизм, блекпилл, всё такое. Какая-то правая либерал-шиза прямиком с лахтоцентра.

> У нас же IQ отрицает и народ, и публика с претензией. Это так в вузах преподают? Откуда такое отношение?


Так всё правильно же. В России сохранилось ещё материалистическое мышление, путём нехитрых рассуждений всегда можно прийти к выводу что айкью нихуя не влияет, а идеология и обучение влияют в сотни и тысячи раз больше. Логично, хуле.

>2. Почему в РФ настолько распространено отрицание потепления?


Уже нет. Два лета выдались жаркими шопиздец, дождей нет, зимы нет. Теперь каждый говорит "а про потепление-то есть, кажись они правы были".
360 585491
>>85489

>Вон как сейчас топят за IQ


Кто?

>Какая-то правая либерал-шиза прямиком с лахтоцентра.


што

>путём нехитрых рассуждений всегда можно прийти к выводу что айкью нихуя не влияет


Аахахахахах. И ты даже сам не понял, что сказал, это прекрасно.

>Два лета выдались жаркими шопиздец, дождей нет, зимы нет.


Ноучная доска, ясно.
361 585492
>>85491

> Кто?


> што


> Аахахахахах. И ты даже сам не понял, что сказал, это прекрасно.


> Ноучная доска, ясно.


Ты это, хотя бы предложение пиши чуть более развёрнуто, не знаю, какие-то утверждения в них делай, аргументы, какие-то опровержения. Например "я опровергаю тебя и привожу это как доказательства". Понимаешь что я?
362 585493
>>85492
Я тебя услышал: оказывается, всё наоборот, и все топят за IQ, потому что государство проплатило IQ-ботов с целью продвижения правого либерализма. Понял, принял.
Спорить с тобой про валидность IQ я не собираюсь.
363 585503
>>85493

>потому что государство проплатило


Только не государство, а ужратые жиды, и наверняка проплатили пижженными бабками. Потому что, никаких государств - не существует в принципе, без государственного суверенитета, на глобальном-то рынке, лол.
364 585504
>>85484
Потому что результаты айкю зависят от тренировки и мотивации, то есть не являются объективным показателем интеллекта. А на западе люди туповатые и склонные к депрессии, поэтому им нужно чем то компенсировать свои недостатки
Мне 38 годиков, помню 20 лет назад зимы были -25 по всей центральной РФ. Сейчас дай Бог -10, иногда -15 бывает. Так что потепление отрицают только тупые зумеры, которые жизни не видели
>>85485
Верно. Это как умение играть в шахматы или покер. Никто же не говорит что калькулятор умнее человека, только по тому что он задрочен на быстрое вычисление определённых задачек
Потепление уже как лет 20 заметно невооруженным взглядом
>>85488
Еще 100 лет назад средний айкю в США был 70 баллов. И что? За 100 лет надрочились выдавать на 30 баллов больше, только и всего
Я понимаю что айкю-дрочерам неприятно это осознавать, но такова реальность
365 585506
>>85489
Самое важное это социальные навыки
image.png190 Кб, 597x593
366 585509
>>85493

>Я тебя услышал: оказывается, всё наоборот, и все топят за IQ, потому что государство проплатило IQ-ботов с целью продвижения правого либерализма. Понял, принял.


Ну а что не так? Это не совсем правый либерализм, это просто правая шизофрения ради оправдания правления барина. Там как мартышек тренируют: Почему барин? Потому что айкью и гены! Что делать? Принимать блекпилл и не качать лодку!

Ты же не думал что все эти треды c всякими редпилами и iq, которые регулярно появляются, это всё просто так шизики срут годами? Не думал же что проправительственные блогеры, форсящие эту хуйню, форсят бесплатно?

>Спорить с тобой про валидность IQ я не собираюсь.


Тут и спорить невозможно, с чем ты собрался спорить-то, с тем что IQ может заменить хорошее образование? Или с тем что IQ постоянно росло? Что там, генетика поменялась у людей за 50 лет? Это же звучит как шизоидная хуета, бред просто, шизофрения порвачков, таблетки где, виу-виу-виу дурка едет нахуй блядь

Вся эти хуйня с IQ это просто форс ради барина. Если раньше барина оправдывала церковь, то сейчас форсят IQ и превосходство в генах. Это изначально форсилось как политическая хуета, ею остается и поныне.

>>85506
Неа. Идеология. Сначала идеология потом всё остальное, включая образование.
367 585510
>>85509

>с тем что IQ не может заменить хорошее образование


слоуфикс
368 585511
>>85509

>Что там, генетика поменялась у людей за 50 лет?


стандартное оправдание детям стало хватать еды
369 585512
>>85511
Тогда получается в США и Европе все голодными были целых сто лет, негоже такое говорить. Это уже нестандартное оправдание.
370 585514
>>85512

>Это уже нестандартное оправдание


Нестандартное для кого? Это буквально мэйнстрим исторической науки, принимаемый всеми. Ты всех своим скудным необразованным умишкой не наделяй
371 585515
>>85514
Ты бредишь. По твоему 100 лет назад люди были даунами? Но историческая наука сей тезис не подтверждает.
Дело банально в тренировке. Людей поколение за поколением прогоняли через шкалы, где они тренировали способности решать различные тесты. Это тупо приспособленность к определенной среде
Тащемто, уже давно доказано, что даже пару дней тренировок перед тестами повышают результаты на 10 баллов. А мотивация в виде деняг, еще на 20 баллов
372 585516
>>85504

>зависят от тренировки и мотивации, то есть не являются объективным показателем


охуенно
373 585517
>>85516
Да охуенно. Это показатель мотивации и тренировки тестов, а не интеллекта
374 585518
почему при механическом взаимодействии двух электрически нейтральных тел в процессе трения заряды переходят от одного тела к другому?
375 585519
>>85515

>По твоему 100 лет назад люди были даунами


Даунами они, конечно, не были, но по-моему очевидно, что 100 лет назад средний европейский рабочий питался хуже (особенно в детстве), чем сегодняшний зумер, но даже в этот период уровень жизни уже был лучше уровня городских рабочих в 19 веке

>Людей поколение за поколением прогоняли через шкалы, где они тренировали способности решать различные тесты. Это тупо приспособленность к определенной среде


Это правда, действительно, если хорошо кормить ребёнка, а также заставлять его заниматься умственным трудом, он, вероятно, будет умнее. Проблема заключается в том, что на разные человеческие популяции эффект Флинна тоже работает и голодные суб-сахарские негры отстают от своих упитанных афро-американских братьев, но последние всё равно катастрофически уступают белым, восточным азиатам, евреям, высшим индийским кастам, несмотря на то, что все эти люди живут в США уже несколько поколений и существуют в одной пиндосской системе образования, но почему то натренированные поколениями решать тесты негры всё равно отстают от таких же натренированных решать тесты белых и азиатов. И даже если сказать, что жесткое расовое угнетение насрало чёрным в статистику, то почему расовые ограничения не насрали в статистику азиатскому населению США?
376 585521
>>85518
Гугли трибоэлектрический эффект.
Это комплексное явление, которое в одном посте не объяснишь.
377 585522
>>85514
Речь про IQ, а не про историческую науку, о чем ты? Связь между доедание и айсикью вообще нихуя не доказана, кстати, тупо корреляция ради корреляции.

Нестандартная она потому что все эти тесты имеют политическую основу - каждый шиз который рассказывает про тесты будет рассказывать про негров, про то какие они тупые, про ещё какую-то порвачью хуйню, но никто в своей пропаганде не будет рассказывать что Европа и США недоедали последние 100 лет, или имеют огромные проблемы с образованием, это очень неприятно.

>>85519

> Это правда, действительно, если хорошо кормить ребёнка, а также заставлять его заниматься умственным трудом, он, вероятно, будет умнее.


@

> Проблема заключается в том, что на разные человеческие популяции эффект Флинна тоже работает и голодные суб-сахарские негры отстают от своих упитанных афро-американских братьев, но последние всё равно катастрофически уступают белым, восточным азиатам, евреям, высшим индийским кастам, несмотря на то, что все эти люди живут в США уже несколько поколений и существуют в одной пиндосской системе образования, но почему то натренированные поколениями решать тесты негры всё равно отстают от таких же натренированных решать тесты белых и азиатов.



Ну и что это? Ты так ничего и не понял?

Тебе сначала придётся доказать что они действительно доедают и действительно обучаются, ведь оказывается что "эффект Флинна" может по-разному на разные человеческие группы влиять, что вполне очевидно.

Но доказать ты это не можешь потому что таких исследований не проводится, а значит можешь смело свои порвачьи шизосказки выкидывать нахуй - они просто не имеют доказательств, зато имеют огромнейшую проблему в виде роста IQ на 30 балов за 100 лет.
378 585523
>>85522
iq не может расти
image.png190 Кб, 597x593
379 585524
>>85523
А это что тогда? Врёти?
380 585528
>>85522

>придётся доказать что они действительно доедают


Доказываю, чёрные - группа населения с самым большим ожирением, а значит они доедают

>и действительно обучаются


Среднее общее в США обязательно, значит, обучаются. В школе. Да-да и тесты там тоже есть.
Есть возражения?
381 585529
>>85528

>Есть возражения?


У тебя вместо доказательств пуки в лужу, о чём ты? Нужны научные исследования по большим когортам.
382 585530
>>85521

>Всеобъемлющая теория электризации пока не построена


ну ясно
383 585531
>>85529

>Ты не можешь доказать, что они действительно доедают


>Доказываешь


>Ваши доказательства - не доказательства

384 585532
>>85531
Порвак, плиз. То что ты пукнул какой-то хуйней является доказательством только в среде порвачков. Доказательства так не рабоатют.
385 585533
>>85532
Маня, ты написал >Тебе сначала придётся доказать что они действительно доедают и действительно обучаются
И оказалось, что они и доедают и обучаются
386 585534
>>85519
И че с того? Количество неравно качество

>но почему то натренированные поколениями решать тесты негры всё равно отстают от таких же натренированных решать тесты белых и азиатов.


Кто сказал? Майк Тайсон например жил в гетто, как и большая часть нигеров. Так что ни о каких равных условиях тут речи не идет.
387 585535
>>85485
Обоссываем тупую скотину за два шага:
1. Если iq негенетический, а механически надрачиваемый, как так получается, что у близнецов выросших в разных семья он коррелирует, несмотря на разные условия жизни (пик 1), а у людей разного происхождения выросших в одной семье во взрослом возрасте никакой корреляции нет ? (пик 2)
2. В случае, если iq генетический, но отражает ТОЛЬКО способность решать тесты в школе, каким образом получается, что iq коррелирует с агрессивностью (пик3), или уровнем дохода(пик 4)? Как способность решать тесты в школе может быть связана с шансом сесть в тюрьму и вообще с любой интеллектуальной деятельностью, которая у этих людей получается лучше, хотя они всего лишь "поколение за поколением прогоняли через шкалы, где они тренировали способности решать различные тесты"?
388 585536
>>85190
Кибернетика это лженаука, придуманная фашистами.
389 585537
>>85534

>Кто сказал? Майк Тайсон например жил в гетто, как и большая часть нигеров. Так что ни о каких равных условиях тут речи не идет.


Так это же пример против твоего же мнения, если черные доступа к образованию не имеют, но iq у них всё равно растёт, значит образование и iq не связаны, а значит растёт он у них по другим причинам
390 585539
>>85535

>iq коррелирует с агрессивностью (пик3)


А что имеется в виду? Хитрый социопат с высоким IQ будет умело манипулировать окружающими и строить на публике доброго няшу, будучи на самом деле тираном. А вот тупой дебил, нажравшись, устроит пьяный дебош. Это отнюдь не значит, что у второго уровень агрессии выше, чем у первого.

Покажите исследования, где IQ будет коррелировать со склонностью к психопатии, социопатии, маккиавелизмом, нарциссизмом, абьюзом и т.д. Нет таковых, конечно. Можно иметь сверхвысокий IQ и полностью нулевую эмпатию. У чиновников и депутатов, хитрых шифрующихся педофилов и многих других психопатов довольно высокий IQ, что и позволяет им умело маскироваться под нормальных людей и управлять последними в своих целях. Только некоторые шизики видят их истинную сущность и создают теории о рептилоидах.

>или уровнем дохода(пик 4)


Здесь, думаю, очевидно. Успешно сдаешь тесты вроде ЕГЭ - получаешь лучшее образование. Хорошее образование определяет уровень дохода.
391 585540
>>85539

>что имеется в виду


Блядь, ну там же подпись есть под картинкой даже, какой вопрос задали опрашиваемым

>Это отнюдь не значит, что у второго уровень агрессии выше, чем у первого


Буквально это и значит, ты просто слово "агрессивность" странно как то понимаешь, что в манипуляциях окружающими агрессивного? (Но на самом деле это просто дело определения, конечно)
392 585541
>>85533
Ты тупой, блядь, что ли, порвак? Доказательство это не когда ты пукаешь какой-то хуйней.

Доказательства это несколько научных независимых исследований.
image.png190 Кб, 597x593
393 585542
>>85535
Давай я тебя обоссу за 1 шаг.

1. IQ за 100 лет повысился на 40 баллов.
394 585543
>>85509
Зачем ты пытаешься критиковать тему, в которой не разбираешься?
Ну если тебе не нравится IQ мы можем напрямую пойти в генетику и изучать как наследуются гены, которые ассоциируются с интеллектом или с обучаемостью, или с заработком, кстати гены "богатства" и гены интеллекта не полностью совпадают (по очевидным причинам). И хули? Будешь сейчас рассказывать, что люди от рождения одинаковые рождаются? Или что у всех одинаковый потенциал?
395 585544
>>85542
А как то, что его можно надрочить противоречит тому, что успешность его надрачивания зависит от генов? Втф?
396 585561
>>85542

>В случае, если iq генетический, но отражает ТОЛЬКО способность решать тесты в школе, каким образом получается, что iq коррелирует с агрессивностью (пик3), или уровнем дохода(пик 4)? Как способность решать тесты в школе может быть связана с шансом сесть в тюрьму и вообще с любой интеллектуальной деятельностью, которая у этих людей получается лучше, хотя они всего лишь "поколение за поколением прогоняли через шкалы, где они тренировали способности решать различные тесты"?

397 585562
>>85537

>черные доступа к образованию не имеют, но iq у них всё равно растёт, значит образование и iq не связаны, а значит растёт он у них по другим причинам


1. Программы благоустройства хоть и медленно, но работают, а значит, всё больше негров оканчивают школы и даже вузы.
2. IQ и образованность зависят ещё и от культуры. Даже имея доступ к бесплатной вышке, условный нигга (мага, вася) будет забивать хуй на саморазвитие, читать рэпчик и ходить на турнички — потому что пацанская жизнь для него имеет бóльшую ценность, чем лоховское ботанство.
Нужны глубокие переломы в менталитете и смена поколений, чтобы увидеть какой-то заметный прогресс.
мимо
398 585564
>>85543
Нихуя маневры начались.

Ты мне ответь, почему IQ увеличился на 35 баллов, если он от генома зависит?

>>85544
Т.е. теперь уже у тебя не IQ, а какое-то надрачивание? Интересный манёвр.

Покажешь мне научные работы по этому надрачиванию? Его кто-то измерял? Что это, ехех, что, надрачивание-то, а?

>>585552


Найс порвак порвался. Понимаю, очень неприятно. Тебя годами тренировали что IQ это врождённое свойство барина, а тут нестыковочка, бух, пух, порвак порвался!

>>85561
Ты давай без манёвров, вопрос-то предельно простой.

Как так вышло что IQ выросло на 35 баллов за 100 лет? Гены сменились? Чо, как вообще в целом? Инополнетяне облучали и сменили геном? Леваки правым в шатны насрали? Кто говно подложил-то? Кто?
399 585566
>>85564

> Как так вышло что IQ выросло на 35 баллов за 100 лет?


Культурное влияние.
Смотри, в ДНК заложен темперамент, который в равных культурных условиях даст разное IQ. IQ это логические закономерности и, соответственно, у логика он будет выше, при прочих равных условиях.
Но есть и культура, т.е. в каких условиях воспитывался индивид, если 100лет назад это было молится Богу и пасти коров, то сейчас, сначала через распространение техники, а потом и компьютеров, которые суть логические системы, человек воспитывается в логической среде, поэтому IQ на тестах и растёт, т.к. даже не предрасположенные природно к логике интуиты начинают эту логику понимать, а природные логики понимать еще глубже. Вот и все.
image.png241 Кб, 640x480
400 585568
>>85566

> Культурное влияние.


Получается все те у кого низкое айсикью и нет физиологических отклонений - просто не подвержены культуре, на них это не повлияло. Загадка решена!

> IQ это логические закономерности


Не, логические закономерности не IQ, с чего ты это взял? Ты слово "логика" не встретишь рядом с "IQ" ни в одной научной работе.

> у логика он будет выше, при прочих равных условиях.


Неее, таких данных не существует. И что за логик ещё? Твой протык? Барин твой, ты так его уважительно называешь?

> человек воспитывается в логической среде, поэтому IQ на тестах и растёт,


Ну т.е. все у кого низкое айкью просто росли в нелогичной среде? Загадка решена!

> природные логики


Кто, блять? Что это за магические звери такие? Хоть кто-то их определил?

Тут это, понимаешь что произошло. Тут тест на IQ от "логической среды" зависит, откуда я тут знаю кто логик, а кто нет, я его определить не могу.
401 585569
>>85568

>Получается все те у кого низкое айсикью и нет физиологических отклонений - просто не подвержены культуре, на них это не повлияло. Загадка решена!


Еблан, я же написал, что не одна культура, а и

>в ДНК заложен темперамент, который в равных культурных условиях даст разное IQ



>Не, логические закономерности не IQ, с чего ты это взял? Ты слово "логика" не встретишь рядом с "IQ" ни в одной научной работе.


Открой википедию, шизик.

>Каждый тест состоит из множества различных заданий нарастающей сложности. Среди них тестовые задания на логическое и пространственное мышление, а также задания других типов — в тесты обычно входят логические и арифметические задания, ориентировка в практических ситуациях — умение самостоятельно сопоставлять, обобщать известные факты (творческий подход, в том числе нестандартное мышление — допускается неоднозначный ответ, формулировка нескольких гипотез, разная аргументация), проверка оперативной памяти и т. п.

402 585573
>>85569

> Еблан, я же написал, что не одна культура, а и


И как ты собрался отделять культуру от некультуры? Магическим компасом?

> в ДНК заложен темперамент, который в равных культурных условиях даст разное IQ


Шизофантазии которые ничем не подтверждены.

> Открой википедию, шизик.


Когда википедия стала научной статьёй? Или ты читать разучился, порвак?

Речь была про научную статью, ты осознаешь написанное мною или ты уже огородился в манямирок и не воспринимаешь реальность?

Притом на английской вики такого даже нет, только русская вики изобилует подобными шизофантазиями. Наверняка какой-то лахтёнок переписывал всю статью чтобы барину подлизнуть.
403 585577
>>85535
Ты совсем дурачок? Айкю это комплексное явление, которое зависит от кучи факторов, в том числе и генетических.
Все просто, с дипломом проще устроится на работку. Что тут не понятного? Давно известно, что без бумажки ты хуй простой. Хотя исключений тоже хватает. Например Джобс даже шарагу не закончил. Да и по жизни был полным ублюдком. Что впрочем не помешало ему стать богатеем
>>85537
В школу то они ходят, да в шараги всякие
>>85566
Скорее уж наоборот. Это раньше люди решали логические задачи, как построить дом, выследить зверя, привести инструмент в рабочее состояние и т.д. А сейчас в школах 90% материала это абстракция и теория, которая имеет мало общего с практикой и прикладной логикой. В большинстве случаев долбоеб с красным дипломом, даже розетку дома починить не может.
404 585578
>>85536
Соционика это лженаука, придуманная коммунистами.
405 585585
>>85573

>Притом на английской вики такого даже нет, только русская вики изобилует подобными шизофантазиями. Наверняка какой-то лахтёнок переписывал всю статью чтобы барину подлизнуть.


Таблетки прими, шиз, везде тебе заговоры видятся - паранойя называется
image.png623 Кб, 1024x576
406 585586
>>84349
Птерозавры вообще охуевшими были. Они и бегать могли, и летать, и прыгать и всё это делали в стаях. Универсальные твари, прямо из какого-то фантастического романа. Им бы немного мозгов побольше, плодовитости и всё, пизда всему живому, приземляется такая хуйня и сразу же бежит на тебя отрывая нахуй голову.
407 585589
>>85585
Какой это заговор? Ты что, не в курсе что википедию регулярно проправительственные организации атакуют и переписывают? Лол, да на эту тему даже статья из вики есть.

Это не какой-то заговор, это просто какая-то лахтохуйня по методичке шизу хуярит. Ну либо те кто начитался правых лахтосказок и спешат открыть миру правду через правки в вики.

Посмотри только, ты тоже нахавался этой хуйни и рассказываешь шизоидные сказки про IQ которые буквально звучат как бред. В ДНК темперамент, в ДНК культура, ДНК-надрачивание, просто какие-то бредовые фантазии. Кто только этим дерьмом срёт повсюду.
408 585592
>>85586
Почему я не могу уместить в своей голове научность этой эволюционной хуйни? Что типа оно само вот так вот просто методом тыка, мутаций и отбора выстраивается, а потом ВНЕЗАПНО появляется возможность летать. Это же пиздец, когда организм должен выживать каждую секунду времени МАССОВО претерпевает такие изменение и переходы поведенчески и генетически. И отрастить крылья мало, нужны ещё перья, а ещё и лёгкие скелет, который может оказаться уязвимым для привычных нагрузок при охоте. Ну и просто крылья сами по себе не могут закрепиться никак на ровном месте, тем более без наличия все остальных особенностей для возможности полёта и активного их использования.
409 585594
>>85589

> Ты что, не в курсе что википедию регулярно проправительственные организации атакуют и переписывают?


И чо, и чо? Физику уже переписали? Естествознание, научный метод, социогумманитарные науки и т.д.? Наверное это Дугинцы в темноту веков Википедию переписывают, чтоб на Руси все хороводы водили и радовались Богу солнца, чтоб извели морскую нечисть долларовую и единым порывом создали муравейник Хартланд счастливого общества людей без деняк. Что в этом плохого? Кроме газенвагенов и дахау для морской нечисти на переходной к счастью период. Цель оправдывает жертвы?
410 585596
>>85594

>Цель оправдывает жертвы?


А что нет? На этом вся эволюция строится. Захотелось богу людей вместо динозавров - ебанул астероид. Захотелось новое человечество - утопил всех кроме горстки в ковчеге. Захотелось коммунизма миллионы расстрелянных и сгноенных в гулагах. Ядерная бомба туда же. В чём проблема?
411 585600
>>85594

> И чо, и чо? Физику уже переписали? Естествознание, научный метод, социогумманитарные науки и т.д.?


Не, физику и естествознание переписывают капиталисты и их маркетологи. Несколько лет назад там ведро описывали как работающий двигатель, работающая гомеопатия, лекарства без доказанной эффективности лечат все болезни. Сейчас этой хуйни ещё больше стало. Соцгум почти весь в правильных интерпретациях, история тоже вся, включая исторические мифы поданные как правда или откровенная пропаганда.

> Что в этом плохого?


Что плохого в том что тебя кормят антинаучным бредом про IQ и летающие вёдра? Новые тёмные века довольно плохо, да. Поколение дегенератов которые не понимают что такое интеллект это очень плохо - дегенераты не способны ничего нормального создать.
412 585602
>>85600

> Поколение дегенератов которые не понимают что такое интеллект это очень плохо - дегенераты не способны ничего нормального создать.


И что делать? Может шапочку из фольги одевать? Поможет?
413 585609
>>85602
Создавать ресурсы на которых статьи проходят проверку научным советом двачеров, очевидно же. ща штаны подтяну и создам
Но ты, наверное, ничего не можешь, у тебя жопа горит.
414 585610
>>85592
А вот атеизнутые материалисты уже более 1000 лет верят в самозарождение жизни. По их вере вселенная появилась случайно. Жизнь самозародилась случайно. Разум возник случайно. Охотник пошел в лес и случайно встретил добычу, а потом случайно попал из ружья
415 585616
>>85592
Потому что это происходит не "внезапно"
416 585617
>>85564

>Ты мне ответь, почему IQ увеличился на 35 баллов, если он от генома зависит?


Почему рост увеличился на 15 см, если он от генома зависит?
1699772411074277.jpg2,6 Мб, 3664x2844
417 585621
>>85592

>ВНЕЗАПНО появляется возможность летать


Вода и воздух одно и тоже, нет разницы между летать и плавать.
418 585622
>>85617
Почему ты сравниваешь рост и интеллектуальную деятельность? По-твоему это корректное сравнение? Как это можно сравнить вообще? Или ты просто решил высрать аргумент из методички "как защитить айсикью барина для дебилов" не напрягаясь?
419 585624
>>85622
Если твой аргумент работает только для интеллекта (но не для роста), то тебе нужны дополнительные пояснения, чтобы было понятно, почему для роста эта логика не работает
420 585625
>>85624
Погоди, ты привёл аргумент про то что рост и айсикью это одно и то же, а я должен за тебя этот аргумент доказывать?

Нет, анон, это так не работает. Ты привёл аргумент - ты его и доказываешь. Не можешь доказать - значит твой аргумент инвалид умственного труда.
421 585626
>>85625
И рост и ийсикью зависят одновременно от среды и от генов, собственно на этом всё.
422 585629
>>85626
И как ты собрался влияние генов на айсикью измерить-то?

Среда может добавлять +/- 40 баллов айсикью, что ты измерять собрался? Входит ли значение айсикью в доверительный интервал? Ну входит, дальше что?
423 585634
>>85629
Ты знаком с таким явлением как статистика? Ну там учёт среды и т.д.
424 585635
>>85634
Ты вопроса не понял что ли? Как ты собрался корректировать на среду, или вычислять влияние генов, если +/- 40 баллов айкью подходят к границам доверительного интервала?

Не знаю даже, как упростить вопрос, чтобы понял даже верующий в айку.
425 585638
>>85635
Мне надо тебе лекцию по математической статистике и теории вероятности сейчас бесплатно читать? У тебя математическое ожидание по выборке из нескольких тысяч человек тоже +- 40?
426 585646
>>85638
Речь не про конкретный результат выборки, что ты несёшь-то, блядь? Какое матожидание? Матожидание применяются к одной выборке.

Речь про разные выборки. В разные временные промежутки (влияние среды, очевидно) выборки дают разные результаты. Айсикью сдвигается на две сигмы нахуй.

У тебя какой-то ментальный блок стоит на это или что?
427 585647
>>85646
Т.е. найти людей из разной среды, и сравнить их средний IQ это слишком тяжело, да?
428 585648
>>85647
Так тесты откалиброваны на другой выборке.
429 585653
>>84346 (OP)
Живу в МКД, слышно маргиналов соседей через вентиляцию. + ощущение что фонят стены. Решил рассверлить стену в комнате, и обнаружил, что в стенах присутствуют полости (либо каналы, не уверен точно, т.к. пока не сильно рассверлил стену), по ощущениям гладкие но с порами, примерно 2,5 см в ширину. Включил на кухне вытяжку - и из тех дырок которые я насверлил подул холодный ветерок.
Вопрос: каналы/полости 100% "запитаны" от вент каналов моего дома? Чем это обосновано, что это такое вообще в принципе?
И если нет, то как объяснить попадание воздуха из вент канала кухни в стену комнаты в которой так же есть вент канал
430 585654
>>85653
Алсо, поджигал спички и водил ими вокруг отверстий и засовывал прямо в них - в наиболее крупном спички затухали даже если их 2 и т.д., т.е. поток воздуха оттуда точно есть.
431 585661
>>85653

> Вопрос: каналы/полости 100% "запитаны" от вент каналов моего дома? Чем это обосновано, что это такое вообще в принципе?


Вытяжка тянет в вентиляцию, а из дырок через щели тянет с улицы, от соседей, от куда получится. Ты вытяжкой через комнату, через проделанные дырки, через полости откуда-то тянешь воздух.
432 585664
Это реально? Из фильма 1970 года.

>об открытии жизнеспособных микроорганизмов Николаем Чудиновым в 1954 г. возрастом не менее 250 млн.лет


https://ru.m.wikipedia.org/wiki/Узники_Пермского_моря
IMG20231203014432BURST000COVER.jpg1,2 Мб, 4080x3072
433 585666
Не могу повторить действия в нужном порядке написанные на бумажке. Что это может быть?
434 585667
>>85664
Всякое бывает:

> Turritopsis dohrnii (лат.) — вид биологически бессмертных медуз, обитающих во всём мире, в водах умеренного и тропического климата.

435 585680
>>85621
С той лишь одной малюсенькой разницей, что при первой же ошибке в полёте летающее существо убивается ап землю, тогда как плавание можно оттачивать абсолютно безопасно во всех стилях.
436 585681
>>85678 (Del)
С этим в /math/
437 585685
>>85678 (Del)
Промывка тупой школоты. Бабы запилили себе уютненький матриархат с феминизмом. Баба с ребёнком это вообще неприкасаемое существо, у которого прав больше чем у тебя. И живут они дольше тебя лет на 10. Стоимость писечки вообще улетела в небеса. Сейчас баба может лутать доллары даже не вставая с дивана, ибо куколды всегда найдется.
438 585688
>>85678 (Del)
Корреляция не значит каузация.
Если X коррелирует с Y, и Y коррелирует с Z, то это не означает что X и Z будут с друг другом коррелировать.
439 585699
>>85661
Потоки воздуха из дырки это всос вытяжки? Что-то бредом отдает. Даже при закрытой двери на кухню тянет ведь.
440 585700
>>85689 (Del)
Да. И более того. Выявление корреляций не достаточно, нужно еще смотреть на распределение величин и смотреть как они отклонятся от статистической модели. И даже тут есть ненулевая вероятность случайного совпадения.
441 585701
>>85688

>Если X коррелирует с Y, и Y коррелирует с Z, то это не означает что X и Z будут с друг другом коррелировать.


Статистически то скорее всего будет, надо просто проверить как ковариация XZ связана с cov(XY) и сov(YZ) (мне лень проверять). Но вполне себе связи какой-то причинно-следственной между ними вообще не окажется, это ты прав.
442 585703
>>84346 (OP)

>Однако, помните, что /sci/ - собирательный раздел для всех наук, но если вы глубоко заинтересованы обсуждением политики - вам в /po/, математики - /math/, философии - /ph/, психологии и психиатрии - /psy/, медицины - /me/, космоса и астрономии - /spc/.


Праина понимаю, что тут скрыт панч, что история не наука?
443 585705
>>85678 (Del)
Как уже сказано выше, с этим в матх. Тут такое местным не по зубам.
О корреляции можно думать, как о косинусе угла между векторами (центрированных) случайных величин, поскольку ковариацией можно ввести скалярное произведение на векторном пространстве. Простой геометрический рисунок показывает, что, зная углы между X и Z, и между Y и Z, угол между X и Y однозначно определить нельзя. Если с. в. не центрированы, то смысл не меняется (т. н. косинусовое подобие). Однако - точно так же, как с геометрическими векторами - можно дать оценку наименьшему/наибольшему возможному значению корреляции (расчёт использует т. н. частичную корреляцию). Но пользы от этого чаще всего мало, поскольку границы скорее всего будут очень широкие.

Это про твой теоретический вопрос. Что касается статистики, то такое последовательное "продвижение" корреляции может иметь смысл, потому что коэффициенты корреляции связаны с коэффициентами линейной регрессии (многоступенчатая оценка) и более сложных моделей вроде структурных. Но это имеет смысл только при разумных моделях - корреляции в общем случае не "транзитивны".

>>85688
Про причинность анон вообще ничего не говорил. Вот уж не понимаешь, о чём речь - сиди себе тихо.

>>85689 (Del)
Зависит от конкретного применения.

>>85700
Корреляция - это понятие из теорвера, а не из матстата. "Ненулевая вероятность случайного совпадения" это какой-то нерелевантный бред. Корреляция это просто характеристика распределения. Здесь настолько же уместно говорить о "случайности" "совпадения", как и "случайности" "совпадения" углов между произвольными парами векторов.
444 585714
445 585717
>>85685
Им же хуже, тупеют, не развиваются. А могли бы уже в квантовой физике нобелевки брать, если бы письками не зарабатывали и проблемы не решали.
446 585726
>>85703
Исправим.
16950341722310.jpg34 Кб, 300x300
447 585727
448 585764
Какова вероятность того что у двух голубоглазых родителей родится ребенок с темными глазами?
449 585768
>>85764
Цвет глаз определяется в первую очередь экспрессией генов и не привязаны к конкретным генам.
Вообще говоря, голубой цвет глаз это нарушение нормальной покраски. Гены в не той последовательности включаются. А почему это они делают может завит от множества факторов, в том числе не наследуемых.
Если брать самый распространённый вариант голубых глаз, то за них ответственны мутации в пяти разных генах, и достаточно выбить их комбинацию, а не иметь гомозиготные аллели.
В том случае вероятность заиметь карие глаза у голубоглазых родителей в районе 11% процентов.
450 585780
>>85717
Бабы прекрасно себя чувствуют, в отличие от тебя, инцела) Ну можешь показать свою нобелевку, посмотрим к чему привело твое "саморазвитие"
451 585795
1. Бывает ли "быстрый" и "медленный" метаболизм?
2. Можно ли "подстегнуть метаболизм" хотя бы в теории, например чтобы это было таблеткой из хард-сайфай.
Гормональные болезни, генетически аномалии, старость - это всё исключаем.
452 585796
>>85785 (Del)
Рекомендую поставить свечу в ближайшем Храме Господнем и приобрести Святой Образ в дом. Всех благ, молодой человек.
453 585798
>>85795

>1. Бывает ли "быстрый" и "медленный" метаболизм?


Бывает, но отклонение от нормы всё равно небольшое (скажем, 10%). Так что если ты колобок, то это не от медленного метаболизма, нужно просто жрать меньше. И наоборот.
454 585799
>>85798
Не 10% а в разы, в разы буквально больше жрут и вообще не толстеют.
455 585803
>>85786 (Del)
>>85785 (Del)
Сходи к врачу, пока что не так всё плохо, но выглядит так, что может развиться в шизу
456 585804
>>85798
Если людям гормоны колоть, даже стройных можно колобками легко сделать. Просто, если у тебя поломанный метаболизм, то жрать меньше просто тяжело, т.к. организм постоянно будет тебя ебать адовым чувством голода
457 585812
>>85678 (Del)

>это нормально?


Ты можешь сколько угодно получать кореляций. Количество пиратов корелируют с потеплением, например, что дальше?

>т.е. приходят к тому, что мужчины умнее женщин,


Шизофрения. Тут никто не понимает что такое ум и интеллект, а ты говоришь про то что кто-то умнее или не умнее. Может и умнее. Но доказать ты это не сможешь.
458 585813
Чем лазер отличается от мазера?
459 585814
>>84346 (OP)
В чём суть претензий к Савельеву и оснований для его статуса фрика в сообществе очкариков? Кому не сложно, введите в курс дела, если вы в смежной с ним сфере трудитесь и разбираетесь в её аспектах.
460 585817
>>85813
Лазер светит в оптическом диапазоне (условно еще светом называют ИК и УФ, что близкие к видимому диапазону)
Мазер светить в микроволновом диапазоне, у него длина волны значительно больше чем и света.
461 585818
>>85817
хм, то есть мазерами можно давить спутники Машка?
462 585819
>>85795

>Можно ли "подстегнуть метаболизм" хотя бы в теории


Чем пиндосы и знаимались, подсаживая на таблетки для похудения из амфетамина
463 585823
>>85818
Можно, то на таких длин волн сильна дифракция.
464 585824
>>85819
Подобные таблетки влияют на поведение (подавляют аппетит), а не на метаболизм.
>>85798

>Бывает, но отклонение от нормы всё равно небольшое (скажем, 10%).


Давай уточним: есть два здоровых человека одинакового веса, оба ведут себя идентично, находится в идентичной среде, питаются одинаково, допустим избыточно. Через год у одного +10 кг, а у другого +11 кг? Как это возможно? За счёт чего?
465 585825
>>84346 (OP)
Как называются явления, сам концепт которых является проявлением сложности. И не имеет никакого смысла при декомпозиции, подобно стулу, сколоченному из грубых досок - ни в одной из досок по отдельности невозможно найти стул, более того невозможно понять когда они все вместе становятся стулом (стул, который нельзя собрать, стул, который нельзя определить, стул на котором невозможно сидеть и так далее). Не зацикливайтесь на стуле, то же самое можно сказать про пару мозг->сознание.
466 585826
>>85825

>ни в одной из досок по отдельности невозможно найти стул


На доске тоже можно сидеть. Все свойства стула происходят из свойств досок. С сознанием это не работает.
467 585828
>>85799
Нет, это миф. Лично знал двух таких человек - худющие, и действительно вроде как жрут кучу всего и что попало. А потом пересчитываешь всё на калории - и оказывается, что они в сутки в среднем хорошо если 1600 набирают, а иногда и на 1200 сидят.
468 585829
>>85825

> Как называются явления, сам концепт которых является проявлением сложности. И не имеет никакого смысла при декомпозиции, подобно стулу, сколоченному из грубых досок - ни в одной из досок по отдельности невозможно найти стул,


Хуй знает что ты имеешь ввиду, может https://ru.wikipedia.org/wiki/Эмерджентность это?

>ни в одной из досок по отдельности невозможно найти стул


Зато можно найти скамейку и диван.
image.png1,6 Мб, 1600x800
469 585839
>>85826
Почему у грузовых кораблей кбина сзади, а не спереди? Так же хуего видно куда плывешь.
470 585840
>>85610

>Охотник пошел в лес и случайно встретил добычу


Нет, конечно. Не может охотник случайно встретить добычу - это слишком невероятно. Очевидно, что кто-то дергает охотника и добычу за ниточки.
471 585841
>>85592
Мутации не могут быть случайны, но эволюции это не отменяет.
472 585855
>>85814
Далёк от этой темы, но слышал что он просто токсичное хуйло
473 585880
>>85840

> кто-то дергает охотника и добычу за ниточки.


Детерминизм большого взрыва
474 585882
Все знают, что происходит когда тело движется со скоростью звука и выше, например сверхзвуковой истребитель, пилот не будет слышать никаких звуков от источников позади себя, но что будет, когда тело будет двигаться со скоростью света? Представьте что вы летите со скоростью света головой вперёд опустим моменты с тем, что вас растянет и разорвёт и т.п., правда ли что вы не будете видеть даже свои ноги? Что будет позади? Просто чернота, застывшая картинка?
475 585884
погасит ли взрыв одной атомной бомбы энергию взрыва другой атомной бомбы, если их моща равна? какое будет давление на месте встречи?
мне для рассказа надо, я гуманитарий
476 585885
1. Существует ли научное объяснение самоубийств у людей? Не общие слова и статистические корреляции, а именно общепринятая и работающая теория, описывающая механизмы явления.
2. Если нет, то почему психиатры, психологи, социологи и прочие шарлатны кукарекают, как будто что-то понимают?
477 585888
>>85882

>Все знают, что происходит когда тело движется со скоростью звука


Ну ты вот не знаешь.

>сверхзвуковой истребитель, пилот не будет слышать никаких звуков от источников позади себя


Он все слышет, потому что звук передается внутри самолёта, а пилот относительно самолёта неподвижен. Ты что, на Конкордах не летал, портдж?
478 585896
>>85885
1. Есть только разные гипотезы. Наиболее правдоподобное это слишком сильный перехват контроля полушарий над лимбической системой, отчего та начинает сопротивляться и уводит мозг в разнос на химической уровне. И рано или поздно полушарии и лимбическая система находят радикальный вариант разрешения конфликта.
2. Они как прошаренные нейросетки успешно угадывают и на этом строят свои теории. На самом деле это нормальная практика в научной среде.
479 585899
>>85896

>1.


Ну фактически, это тоже просто общие слова. Типа как в физике 17 века уже знали про вакуум, но еще не существовало молекулярно-кинетической теории, поэтому вместо реальных причин для объяснения проведение жидкостей и газов ввели принцип, что "природа не терпит пустоты", который на самом деле по сути ничего не объясняет, а просто выражает наблюдаемые явления.
Или как в средневековой медицине, не имеющей представления о реальных причинах заболеваний, болезни объяснили какими-то циркуляциями желчи, крови и слизи.
allinone.jpg201 Кб, 1360x768
480 585903
Здравствуйте 🙂 посоветуйте пожалуйста качественную научно популярную книгу (но не без формул) по теории информации? Клод Шеноно и вот это вот всё. Чтобы был срединный путь между сжатием информации для не специалиста и подробностями для профессионалов.
37.png615 Кб, 2399x1262
481 585905
Если представить огромные лезвия должны превышать длину рычага во много раз, картинка не в масштабе раскрытые ножницы в космосе, которые начать "закрывать" , то можно логически предположить, что скорость смыкания лезвий этих ножниц ближе к концу превысит скорость света.
а) как это будет выглядеть для стороннего наблюдателя?
б) если между двумя лезвиями поместить какой-нибудь объект, который будет выталкиваться лезвиями вперёд, сможет ли этот объект превысить скорость света?
в) почему это невозможно на практике?

Если не совсем понятно, то можно представить "ножницы", чьи лезвия будут проходить расстояние от Земли до Луны, что равняется примерно 1 световой секунде с копейками. На Земле мы смыкаем эти "ножницы" менее чем за секунду, что в итоге приведёт к тому, что на Луне заметят смыкание ножниц даже быстрее, чем пока об этом дойдёт сообщение с Земли, или чем это можно было бы разглядеть через телескоп.
482 585906
>>85839
Ты был в океане? Там кругом вода. Просто сплошная вода. Там не надо никуда смотреть.
483 585907
>>85905
Сто раз уже было.
Ножницы не будут мгновенно закрываться, они будут изгибаться дугой, и вот эта волна изгибания будет идти медленней, чем скорость света.
484 585913
>>85907

>Сто раз уже было.


Ну, еп...

>они будут изгибаться дугой


А почему? В дело вступят какие-то фундаментальные законы, не позволяющие разогнаться выше скорости света?
485 585914
>>85785 (Del)
Не парься, этот кот - постфактум рационализация элементов сонного паралича, который бывает чуть ли ни у каждого первого человека. Бывает моменты когды ты вроде еще спишь, но чуть чуть проснулся, переходное состояние, но область мозга, которая отвечает за блокировку тела (чтобы во сне не сильно навредил себе) еще работает, а пошевелиться тело не может из за этого у тела возникает страх, который выдумывает у тебя кота, у кого то чертей сидящих на груди, тут фантазия обусловленна культурой. Но по факту это нормальная физиология, небольшой сбой который бывает почти у всех.

Я раньше думал я ебанулся из за похожих симптомов, уже думал о всякой религиознутой хуяте, а потом, как то на лециии описали физиологию сонного параличи и спросили поднять руку тех, у кого был он - почти вся аудитория подняла.
486 585915
>>85914
UPDATE: ничего не делал, само прошло лол
Y-OLHSdzSc.jpg170 Кб, 1115x1576
487 585917
HIanon. Нужна помощь в виде капли кристально чистой логики и структурированной информации для решения проблемы:

Где мне искать girlfrend IRL?

Не буду писать конкретную инфу о себе, мне не советы нужны, а интересует в принципе научный подход к решению этой проблемы.
489 585922
>>85917
В клубасике.
490 585933
>>85913

>А почему? В дело вступят какие-то фундаментальные законы, не позволяющие разогнаться выше скорости света?


Отнюдь. У тебя ножницы не цельные, а состоят из молекул. Если представить себе лезвие просто из цепочки молекул, то смыкание будет выглядеть так: сдвигается одна молекула, потом благодаря электромагнитному взаимодействию между молекулами сдвигается вторая, и т.д.
Электромагнитное взаимодействие распространяется со скоростью не выше c, поэтому и сдвиг ножниц будет передаваться не быстрее скорости света. А в реальности значительно медленнее.
16850087163990 (1).jpg804 Кб, 1440x1440
491 585934
>>84346 (OP)
Может ли быть у планеты больше двух магнитных полюсов?
492 585935
>>85922

>В клубасике.



Ты видишь размер моей головы на фотке? Какой клубасик? Клубасик из одного человека им.Гриши Перельмана?
493 585936
>>85934
Да.
Пример Уран. у него почти квадрупольное магнитное поле.
494 585937
>>85936
А шо если в его ядро шо-то въебётся, и квадруполь рассыпется в диполь, какие будут изменения по всей планете?
495 585938
>>84346 (OP)
Чем отличается лоббизм от коррупции? Для самых маленьких, если можно.
496 585939
>>85936
чем-то кол-во полюсов ограничено? обязательно должно быть четное количество
за тупые вопросы не бей. лучше обоссы - я гуманитарий
497 585944
>>85938
Государство строиться из различных институтов.
Институты это набор устойчивых практик, которые не должны зависеть от людей, которые эти практики осуществляют. Коррупция это порча институтов или когда осуществление практики начинает зависеть от персоналий. Поскольку институты в государстве взаимосвязаны, то порча одного института автоматом портит другие.

Лоббизм это сам по себе институт, он позволяет некоторой группе продвигать свои интересы и консолидировать с другими в определенном поле, не шатая при этом другие институты.
498 585946
>>85938
Тем, что лоббисты обязаны действовать открыто. В теории, действующего во вред избирателям политика снимут и не переизберут. Но на практике это, конечно, не работает.
Ну и работают лоббисты не только через взятки но и через пропаганду.
499 585953
>>85935
Тогда на вписочке.
500 585958
Почему
ЧЕЛОЧЕЧЕСТВО 20 ТЫСЯЧ ЛЕТ ОХОТИЛИСЬ
@
7 ТЫСЯЧ ЛЕТ В ЗЕМЛЕ КОПАЛОСЬ
@
ТОЛЬКО СЕЙЧАС НАЧАЛСЯ ГЕЙМПЛЕЙ А НЕ ЖИВОТНФЙ УРОВЕНЬ ЖИЗНИ
501 585959
>>85958
В Северной Америке всё ещё охотятся, а в бСССР всё ещё копаются, если ты не в курсе.
502 585969
как вообще началось развитие интеллекта у людей? типа может наши предки были слабыми да удалыми и решили пойти в командную работу и это дало толчок?
907b71267ba5c61fe2de68d24cb46968.jpg262 Кб, 1080x810
503 585971
504 585972
>>85971
Гриб сказал ему отправиться в путешествие где он и обрел способность адаптировался к любой среде, сверхгибкий разум.
505 585973
>>85969
Жили были древесные обезьяны(Проконсулы), они как мартышки были ловкими и подвижными, что давало некоторый буст к общему интеллекту.
Однако в один момент леса начали редеть и превращаться в саванну. Бывшим древолазам пришлось менять рацион с плодов на мясную пищу, которую они крали у хищников. Переход на мясную пищу позволил им уменьшить челюстной аппарат и отодвинуть предел роста мозга.
Воровство еды у хищников довольно сложная задача, которая решалась коллективно. Каждому обезьяну требовалось прогнозировать поведение хищника и своих товарищей. Получался специфический групповой отбор, где выживали только самые сплоченные и большие группы.
Эпоха сменялась, саванны отступили и появились снова леса. Пришлось переходить на всеядный образ жизни, да еще обманывать новых хищников(леопардов). Тут опять усилился отбор на социализацию.
506 585974
>>85969
Сначала нужны были условия для роста мозга и развития интеллекта - это прямохождение, что позволило расти голове без нагрузки на шею и освободило руки для орудийной деятельности (оперирование объектами необходимо для интеллекта). Дальше просто эволюционный отбор, в первую очередь внутривидовой - доминирование и выпиливание себе подобных, т.к. сверххищником человек стал очень быстро и давно. Но есть версия о скачкообразном росте количества нейронов из-за мутации гена и такого же отбора.
Если не рассматривать версию с мутацией гена, то какого-то начала или скачка нет. Почти вся история живого это эволюционное развитие нервной системы (=интеллекта).
4e49f369710981f72182e243c241092c.jpg8,2 Мб, 6566x8215
507 585975
>>85972

>Гриб сказал ему отправиться в путешествие



Грибы были в ахуе от того, где им приходится жить. Они посовещались (все 109 гендеров) и решили развить интеллект обезьян, чтобы создать цивилизацию, построить космические корабли и съебаться нахуй с этой конченной планеты.
508 585976
>>85824
Метаболизм даже у одного человека может меняться ОЧЕНЬ сильно. Несколько лет ежедневных занятий спортом и уже совсем другой метаболизм.
509 585977
>>85969

>как вообще началось развитие интеллекта у людей?


развитие интеллекта началось до людей
510 585979
>>85976

>Метаболизм даже у одного человека может меняться ОЧЕНЬ сильно. Несколько лет ежедневных занятий спортом и уже совсем другой метаболизм.


Нет. Когда ты занимаешься спортом, то и ешь больше, потому что тратишь больше энергии. Метаболизм меняется незначительно.
511 585980
>>85979
Сразу видно человека, который не занимается спортом.

Для остальных: гуглите про количество и КПД митохондрий и как эти показатели менятся в зависимости от типа тренировок. Например долгий бег с постоянной скоростью увеличивает количество митохондрий, а спринт увеличивает их эффективрность. Поэтому та же пробежка должна следовать правилу 80/20. Например 8 км бежишь в постоянном темпе, на увеличение количества митохондрий и 2 км с максимальной скоростью, на увеличение КПД митохондрий.

Например первое что выдал гугл:
https://openlongevity.org/mitochondria_medicine_2#:~:text=%D0%A4%D0%B8%D0%B7%D0%B8%D1%87%D0%B5%D1%81%D0%BA%D0%B0%D1%8F%20%D0%B0%D0%BA%D1%82%D0%B8%D0%B2%D0%BD%D0%BE%D1%81%D1%82%D1%8C%20%D0%B4%D0%BE%20%D1%81%D0%B8%D1%85%20%D0%BF%D0%BE%D1%80,%D0%BE%D0%B1%D1%8A%D0%B5%D0%BC%D0%BD%D1%83%D1%8E%20%D0%BF%D0%BB%D0%BE%D1%82%D0%BD%D0%BE%D1%81%D1%82%D1%8C%20%E2%80%94%20%D0%BD%D0%B0%2040%25.
45756785688.jpg213 Кб, 571x914
512 585981
>>85958
Потому что ты долбоеб
423543535.png69 Кб, 713x409
513 585982
>>85840
Очевидно что охотник применяет свои навыки для выслеживания добычи, а не прет в лес надеясь на удачу. Это только атеизнутые материалисты верят в удачу
image.png233 Кб, 908x801
514 585992
>>85919
так, блэт, требую пояснения
515 585998
>>85992
#meToo
chromemOmR6kt0zL.jpg53 Кб, 648x641
517 586000
>>85999
Thanks!

Залип и влюбился со второй страницы:
<-
518 586001
На Земле могут закончиться металлы? Когда?
chromeNWkFvz3wOf.jpg426 Кб, 2498x1353
519 586002
>>86001
Кратко: да. Для разных металлов по разному.

Подробно: конечно металлы не покидают эту планету. Но это как с электрическим обогревателем - формально энергия электричества никуда не исчезла, она перешла в тепло, но по факту использать это тепло продуктивно проблематично.

С металлами, на примере меди: столетие назад разрабатывали месторожения около 15% меди в руде. Эти месторождения уже закончились, сегодня разрабатывают уже месторожения с парой процентво меди в руде, даже меньше. Но по мере уменьшения содержания полезного вещества в руде количество затрат и объемы руды, которую нужно переработать, растет почти по экспоненте. Да технологии совершенствуются, но тонна грунта, это всё еще тонна, и количество энергии на её перемещение не меняется. Поэтому на примере меди, довольно скоро (может быть меньше сотни лет) наступит момент, когда по мере введения в разработку всё более бедных месторожений, затраты (в том числе не прямые, типа экологии) превысят стоимость меди. Опять же, нельзя всё сводить только к деньгам.

В общем мысль понятно надеюсь.
gynueb8nt4oy-3952461229.png789 Кб, 1280x533
520 586006
>>86001
Важный момент, что даже бесконечный источник энергии не решит эту проблему, потому, что даже при бесконечной энергии будет накапливаться загрязнение и разрушение окружающей среды от производства и добычи, со временем превысив допустимый предел. И это относится не только к металлам.

Предел этот конечно разный для выживания миллиардов и миллионов людей. Первыми под нож пойдут достижения благополучной жизни - демократия и права человека. Но быдлу конечно дадут возможность восстановить справедливость, и почувствовать себя героем - вымещать бессильную злобу в игрушках виртуальной реальности, стреляя по elysium bugatti. LOL
¯\_(ツ)_/¯
521 586007
Аноны кто разбирается в юриспруденции объясните почему так

>УК РФ Статья 59. Смертная казнь



1. Смертная казнь как исключительная мера наказания может быть установлена только за особо тяжкие преступления, посягающие на жизнь.

2. Смертная казнь не назначается женщинам, а также лицам, совершившим преступления в возрасте до восемнадцати лет, и мужчинам, достигшим к моменту вынесения судом приговора шестидесятипятилетнего возраста.

>Статья 19



1. Все равны перед законом и судом.

2. Государство гарантирует равенство прав и свобод человека и гражданина независимо от пола, расы, национальности, языка, происхождения, имущественного и должностного положения, места жительства, отношения к религии, убеждений, принадлежности к общественным объединениям, а также других обстоятельств.
522 586008
>>86006
Имея

>бесконечный источник энергии


можно будет фильтровать что угодно на уровне атом вправо - атом влево.

Сейчас Машк понаделает ракет и можно будет металлы притягивать прямо из космоса. У каждого бомжа будет палладиевый унитаз.

Плюс вот-вот напридумывают всяких хитрых сплавов или еще каких материалов с самыми невероятными свойствами. Я уверен в светлом будущем.

Но быдло действительно лучше бы подсократить, ну до миллиарда хотя бы, чтобы дышалось полегче, толку от него все равно никакого.
(JPEG Image, 187 × 270 pixels).jpg6 Кб, 187x270
523 586009
>>86007
Все животные равны, но некоторые животные равнее других.

Это же писечки - ценный товар для кого надо, пынемать надо.
524 586017
>>84346 (OP)
Правильно понимаю, что электродвигатель на сверхпроводнике не дает значимого прироста в эффективности?
https://youtu.be/fpkAh858nso?si=Dyiw5_TMfytoCa6s&t=1317
525 586018
Почему время в любом девайсе, не подключенном к интернету и, соответственно, не получаещем апдейт с "правильным" временем всегда начинает спешить? Поискал в интернете но нашел только применимое к обычным кварцевым часам мням шмям на кварцы влияют внешние факторы. Какое время вообще настоящее? Или я просто шиз, если так то извиняюсь.
526 586030
>>86018

> всегда начинает спешить?


Точные атомные часы штука громоздка и дорогая, все обычные часы идут с приличной, разной степени погрешностью. Если это не твоя шиза, про то что все часы спешат (я такой херней не занимался и первый раз об этом слышу), то можно только предположить, что учитывая априорную погрешность часов, их подкручивают на опережение что бы люди не опаздывали.

> Какое время вообще настоящее?


Как люди договорятся, то время и будет настоящим. Гугли: время по Гринвичу.
527 586031
>>86017

>Сказки о кисейных берегах


>Рашка


Проиграно. Нет не дает. Но (в теории) из сверхпроводника можно сделать хранитель энергии.

>>86030

>Если это не твоя шиза, про то что все часы спешат


Нда... поколение пориджей в жизни не видевшие часов уже подросло и всюду лезет.
528 586033
>>86031

> поколение пориджей в жизни не видевшие часов


Чел, я просто подвожу часы по необходимости, а отстали они или нет мне вообще не интересно, мне просто надо чтобы часы показывали правильное время.
Это у тебя шиза обострилась, во всякой ерунде связи искать, таблетки выпей

> Письмо на Балабановскую спичечную фабрику: «Я 11 лет считаю спички у вас в коробках - их то 59, то 60, а иногда и 58. Вы там ебанутые что ли все???»

529 586035
>>86033
Поридж ты вообще с этой планеты? Кто то выпустил жпт? Охуительные истории часы он не глядя подводит, какой то бред про спички.
IMG20230826082222.jpg799 Кб, 3264x2448
530 586036
>>86033
этот прав

>>86035
>>86031
этот из:

> Письмо на Балабановскую спичечную фабрику: «Я 11 лет считаю спички у вас в коробках - их то 59, то 60, а иногда и 58. Вы там ебанутые что ли все???»


_
За полвека жизни, даже до появления телефонов, ни разу не обращал внимания спешат часы или отстают когда я их подвожу. Это же пиздец какой жалкой жизнью нужно жить, чтобы тратить время на такую хуету как в какую сторону часы сбиваются.
531 586037
>>86036
Я думал речь идет про рассинхрон вообще. Снимаю тогда возражения.
532 586038
>>86037
И кстати я тоже могу быть неправ, возможно нужно глубже копать. Что если все часы действительно сбиваются в одну сторону? инкто не обращал внимания, а ты обратил. Извини что обосрал. Что это? Тайный заговор мирового правительства мальчиков копателей из литиевых шахт в Африке или новый, неизвестный закон физики расширяющий общую теорию относительности?
533 586039
>>86038
Я оп вопроса, имел в виду именно часы в девайсах - телефоны,планшеты. Например, у меня есть старый айпад, который я не подключал к интернету год и за это время он стал спешить ровно на 9 минут, при этом ни разу не выключался (стоит на зарядке как фоторамка). Механические часы понятно почему могут спешить или оставать, но почему компьютер отсчитывает неправильно, при условии что сам он работает без нареканий. Варианта два: или проблема в самой технике, или официальное время постоянно отводят немного назад, но второе это уже шиза, лол.
534 586040
>>86031

>Нет не дает


Почему? Насколько я понимаю, чем больше электронов можно прогнать по проводу с учетом физ. свойств, тем мощнее будет электродвигатель. Или это не так?

>хранитель энергии


Какие там теоретические пределы емкости?
535 586041
>>86040
У современных электродвигателей потери на механическое трение больше, чем омические потери в обмотках. Так же есть значительные потери при перемагничивание магнитопровода. Чтоб сделать йоба двигатели, нужно делать йоба подшипники и йоба железо.

Все упирается в размеры и материалов которые достаточно прочны. Чем больше контур тем лучше. Можно хоть вокруг планеты обмотать или на орбите развернуть и получить батарейку на десятые порядки джоулей.
536 586043
>>86039

>официальное время постоянно отводят немного назад


Так и есть, чтобы никто не замечал неправильного движения солнца из-за того, что земля на самом деле не шар.
537 586047
Почему никто ещё не додумался испытать на практике теоритическую модель автономного марсианского поселения, но не на Марсе, а в Антарктиде? Ведь Антарктида самое приближенное к условиям Марса место на Земле.
538 586048
>>86047
Потому что Марс нинужная хуйня. Весь хайп был для того, чтобы ты на фильм сходил.
539 586050
>>86048
А ведь анону который спросил - ему нужен Марс, так что получается что это ТЫ - никому не нужная хуйня.
540 586052
>>86050

>шизоид спросил шизоидный вопрос


>ряяя вы фсе нинужные яскозал


понятно
541 586056
>>86040

>Какие там теоретические пределы емкости?


На данный момент
https://en.wikipedia.org/wiki/Superconducting_magnetic_energy_storage
1–10 W·h/kg
Что не густо мягко говоря, на пару порядков меньше чем у аккумов, у которых в свою очередь на пару порядков меньше топлива.

Но в теории емкость ограничена критическим полем, которое ограничено.. неизвестно чем, потому что нет удовлетворительной теории высокотемпературных сверхпроводников. Так что i want to believe.
542 586065
>>84346 (OP)
Кто-нибудь, скажите:
Современная физика позволяет предсказать свойства вещества по его химическому составу, такие как электропроводность или теплоемкость? Как это работает?
543 586077
>>86065
Нет, не позволяет. не хватает вычислительной мощи. И этим не занимается физика.
Электропроводность по сути эмерджентное явление, и нет до сих пор внятной модели, только экспериментальные данные.
544 586078
>>86077

>Электропроводность по сути эмерджентное явление


Что эмерджентного ты нашёл в движении электронов? Нет тут эмерджентности.
545 586079
>>86078
Если брать металлы или полупроводники, то в них сами электроны никуда не двигаются и сычуют рядом с ядрами или в узлах решетки. Однако электроны могут участвовать в формирование обобщенной коллективной связи, которая размазана на весь кристалл. Вот у этой связи уже возникают эмерджентные свойства, которые не могут быть у электронов, состоящие их. Как пример перенос заряд через дырки или появление частиц, переносящие только спин.
546 586080
>>84346 (OP)
Как разделять разные типы нанотрубок? Такой способ существует?
https://youtu.be/ocKKYqOohwc?si=CGpmA4iNzEYMuuij&t=642
547 586081
>>86077
Чел, мне ноутбука хватит с мобильным чипом, чтобы посчитать теплоемкость какого-нибудь простого кристалла с +- 10% точностью, а если речь про газы, там вообще за 20 минут точность 0.1% можно добиться

Ну мы же не в 1995 году
548 586082
>>86081
Давай считай параметры решетки железа исходя только из атомного веса и номера.
549 586083
>>86082
Хватит только номера
550 586089
>>86083
Ну как считается?
У железе минимум четыре кристаллическое решетки, а еще есть куча метастабильных.
551 586098
>>86089
Лол, оно самую стабильную будет брать, чтобы найти всякие метастабильные или иные уже придется подрубать кластеры для расчетов, но опять же, банально взял 3,5,9 атомов, посмотрел какие там получаются расстояния и экстраполировал, получилось типа 2,85A

И это я брал самые простые и тупые методы, если захочется, можно заморочиться со спиновыми состояниями и взять методы посерьезнее, и там результат будет еще ближе к реальности

Ответ на вопрос:

>Современная физика позволяет предсказать свойства вещества по его химическому составу


Звучит как "зависит от сложности строения вещества, в простых случаях позволяет, в тяжелых нужно много вычислительной мощности, порой чересчур много"

А не как ты написал:

>Нет, не позволяет. не хватает вычислительной мощи.



Ну это же пиздеж, какую-нибудь NaCl и её теплоемкость считать можно в домашних условиях, c электропроводностью сложнее, но если структуру зон посчитать, то и электропроводность можно неплохо оценить. Если речь не про сложные системы со сверхпроводимостью или какие-нибудь суперзамудренные спиновые системы.
552 586103
>>86079

> Если брать металлы или полупроводники, то в них сами электроны никуда не двигаются и сычуют рядом с ядрами или в узлах решетки.


А что делать со свободным электронами, они же электроны проводимости? Которые как раз и прыгают между атомами?
553 586110
>>84346 (OP)
Интересуют обзорные книги по природе и сути власти, подчинения, авторитетов, идолов, кумиров, рабства. В первую очередь антропология, социология, а потом уже всё остальное.
Делитесь тем, что считаете достойным внимания, что нельзя обойти стороной.
554 586111
>>86103
Электроны проводимости это квазичастицы, которые порождаются коллективной связью, и они не являются электронами сами по себе, тем более свободными.
Они не могут быть привязаны к конкретному атому или узлу решетки, они даже не "чувствуют" их, вместо этого они сычуют в разрешенной зоне, которую не могут покинуть и рассеваются на других квазичастицах, которые являются такими же порождением коллективной связи всех атомов в куске материала.
555 586143
1. В СССР реально продавали в аптеках дерьмо летучих мышей (мумиё)? Это был дефицит?
2. Почему при первой же среднеазиатских "академиков" ввести это в официальную медицинскую практику им не постучали хуем по головам из Москвы? И АН и вся советская медицина это централизованная структура. В чём проблема? Брежнев любил обмазывался говном, и безопасности ради решили перейти с человеческого на летучемышиное? Как это объяснить?
Screenshot.png660 Кб, 1080x2400
556 586146
>>86143

>В СССР реально продавали в аптеках дерьмо летучих мышей (мумиё)


Поридж, спок.
557 586149
Российская наука в этом году официально признала существование Рептилоидов.

http://fonduniver.ru/wp-content/uploads/2023/12/un_2023_9_pb.pdf

Ваше мнение по этому поводу?
558 586150
>>86149
Фонд, ассоциация, журнал … причём здесь наука? Знакомое слово в названии увидел?
559 586151
>>86143
1. Мумиё (также известное как мумие) действительно продавалось в аптеках СССР. Это считалось естественным средством, имеющим различные преимущества для здоровья. Дефицита мумиё в аптеках не было.

2. Внедрение мумиё в официальную медицинскую практику в СССР, вероятно, было обусловлено несколькими факторами. Академики Центральной Азии, возможно, провели исследования и обнаружили потенциальные лечебные свойства мумиё. Кроме того, Академия наук и советская медицина не были полностью централизованными структурами, и могла существовать определенная степень автономии в принятии решений. Личные предпочтения политических лидеров, таких как Брежнев, обычно не являются единственной основой медицинской практики. Решение использовать мумиё, вероятно, было связано с научной оценкой и рассмотрением его потенциальных преимуществ.
560 586152
>>86150

> Журнал аккредитован в Российском Индексе Научного Цитирования (РИНЦ).


В общем, в российской науке Рептилоиды теперь база.
561 586162
>>86149
А там написано как реаптилойдов отпиздить или это за гранью науки? Если так то я тогда тогда в религию.
Безымянный.jpg76 Кб, 564x564
562 586181
Смотрел сейчас гуглкарты и не понял момент. Есть город Волгоград, есть волгоградский водоканал и его очистные сооружения.
Я так понимаю там очищают канализационную воду со всего Волгограда?
Но эти очистные сооружения находятся на острове в 2,5 км от Волгограда. Как туда доставляют канализационные стоки то? Их же не перевозят на танкерах, они что под Волгой на дне проложили гигантский говнопровод? И качают говно говняными насосами 24/7чтобы потом слить в ту же реку, но это же неэффективно как-то, нет?
Или водоканал там что-то другое очищает, типо питьевую воду для Волгограда?
563 586184
>>86181
Не всю.
Вообще-то в каждом районе должна быть маленькая говночистка в виде нескольких больших подземных емкостей, туда сливают говно со всей округе, оно отстаивается и идет дальше по говно проводу к очистным сооружениям, там воду приводят в нормальный вид и отправляют в водохранилища, из которых уже забирают воду для водопровода.
564 586185
>>86184
А спасибо, теперь понятно, там вода, просто дурно пахнущая. Я уж думал если прорвет на дне будет пиздец, всю Волгу засрут буквально.
565 586187
>>86185
Очистительные сооружение это не глубокие пруды, главное тут площадь, а не объем.
Там нечему особо прорывается, чтоб хлынуло потоком. Хотя конечно есть вероятность протечки и заражения вод, но Волга и так не очень чистая, особой разницы не заметят.
566 586200
>>84346 (OP)

У меня ПЛАТИНОВЫЙ вопрос - который наверное сюда.

А почему все Хомо вымерли? Логичная теория есть только про неандертальцев, которые фактически экстремофилы на пределе возможностей по жизни и рождению, чуток климат провернулся - чуток африканцы с копьеметалками пришли, и все.

Но почему работящий-прямоходящий и прочие вымерли? У них ареал остается. Их эволюционное преимущество над большими человекообразными обезьянами очевидно. Почему шимпанзе-орангутаны сегодня есть, а няши с пиков вымерли??? Как так?
567 586203
>>86200
Потому что древние и современные хомо в одной экологической нише и, соответственно, вытесняли-выживали друг друга, а обезьяны остались на деревьях - в другой экологической нише. Шипманзе-орангутанги, кстати, тоже эволюционировали, и так же вытесняли своих предков. У нас с обезьянами один предок, от которого разошлись по экологическим нишам и стали разными, а сами предки, что людей, что обезьян - умерли.
568 586205
>>86203

То есть на пустой Земле не было места для пары-тройки сверхприспосбленных видов?

Ну вот не верю, что прямоходящая обезьяна с зазубренной палкой и отщепом, которая может есть травку, фрукты, корешки, мясо, рыбу - не смогли выжить. Они уж точно лучше приспособлены чем обезьяны и хищники.
569 586207
>>86205

> Ну вот не верю


Правильно, не верь, человека создал Бог, по образу и подобию своему.
Ебанько, зачем ты задаешь вопрос в науке, но не веришь научным объяснениям? Иди в раздел религии или эзотерики и там спрашивай, видимо тебе нужно магическое объяснение появления человека
570 586208
>>86207

Всем известно, что рептилоида изнасиловал анунак = и так появились люди.

Я вполне научно задался вопросом:
- Есть вид, который всеяден, умеет в базовые орудия труда и стадность.
- Есть виды - которые не умеют всего этого, и тоже обитают в довольно узких нишах.
- Земля (африка) вполне достаточна для обитания ряда видов в одной нише, особенно столь продвинутых.

Условно говоря многие виды тех же хишников разделялись и эволюционировали а некоторые вообще крокодилы и им похуй на эволюцию параллельно и сейчас есть несколько видов в одних нишах.
571 586209
>>86208

> - Земля (африка) вполне достаточна для обитания ряда видов в одной нише, особенно столь продвинутых.


В одной экологической нише не могут сосуществовать длительное время два одинаковых вида, более сильный вид вытеснит более слабого, слабый либо вымирает, либо уходит в другую экологическую нишу.
572 586225
острие науки

Есть что-нибудь по самым передовым достижениям уровня фантастики? Типа: кристаллы времени, квантовые жидкости, конденсаты, компьютеры?
image.png626 Кб, 861x620
573 586238
Почему бегемоты считаются парнокопытными если у них по 4 пальца на ноге?
574 586252
>>86225
Какие научные прорывы, с 1960-х ничего нового не изобретено.

Ученые-дармоеды в XXI веке не могут сделать ни телепорта, ни кошкодевочек. Только попиливают гранты и борются с конкурентами за свои теплые местечки ("Ученые против мифов", Комиссия по борьбе с лженаукой и прочая). Сейчас наука превратилась в средневековую церковь с догматами, инквизицией и лицемерными попами-педофилами.

Вот раньше, когда Королев в шарашке когда отлынивал, так получал пиздюлей от Лаврения Палыча, в итоге и запустил ракету в космос (что раньше считалось фантастикой, совершенно невозможным). Этих грантоедов бы в шарашки, у нас бы сейчас появился не только телепорт, но и звездолет до Альфы Центавра.
575 586263
Почему поиск повторов в ДНК это важная задача?
576 586264
>>86263
Вдруг боженька оставил комментарии в ДНК в стиле "КТО ПРОЧИТАЛ ТОТ ПИДАР"?
577 586265
В фильме Оппенгеймер бытовала мысль, что во время испытания атомной бомбы может возникнуть цепная реакция и воспламенение атмосферы впоследствии, Энрико Ферми шутил якобы на эту тему. Как это вообще устроено и возможно ли? Или всё же режиссёр просто вешал лапшу для пущего эффекта и впечатления зрителя?
578 586274
>>86263

>поиск повторов в ДНК


А если тебе текст дать на незнакомом языке без пробелов и расшифровывать заставить, ты это как делать будешь?
579 586276
Почему ртуть запретили?
580 586283
>>86276
Никто ничего не запрещал.
Нашли менее опасные для жизни аналоги.
Там где не нашли продолжают использовать.
581 586293
>>86265
Собравшимся, впрочем, они говорили еще кое о чем, на что Теллер сам быстро указал. Ученые смотрели на каракули на доске, ошарашенные догадкой. При такой высокой температуре, более высокой, чем в центре Солнца, должны сливаться не только атомы водорода. При ней кулоновский барьер должны преодолевать не только атомы водорода в воде, но и атомы азота в воздухе. Она должна практически мгновенно зажечь водород в океанах и превратить в пламя воздух вокруг земного шара. Земля должна вспыхнуть меньше чем за секунду и навечно превратиться в безжизненную глыбу.

Никто из ученых, собравшихся в Беркли, не сомневался в теоретической осуществимости ядерного взрыва. Проблемы, которые, возможно, и не удастся преодолеть вовремя для практического использования во Второй мировой войне, были чисто техническими: например, никто не знал, можно ли удержать массу достаточно долго, чтобы цепная реакция привела к полномасштабному взрыву. Теперь же оказалось, что технические сложности создания бомбы – не главная проблема. Их преодоление могло обернуться не самой хорошей идеей.

Они начали проверять расчеты Теллера. Ошибку нашли довольно быстро. Он упустил критически важный фактор: передачу тепла атмосфере. Однако внесенные исправления не устранили возможности возникновения реакции, вызвавшей опасения.

Среди присутствовавших на презентации находился Ханс Бете, самый проницательный теоретик в группе, который впоследствии получил Нобелевскую премию за работу по термоядерным реакциям на Солнце. Опираясь на интуицию, он считал, что такой результат «невозможен».
https://military.wikireading.ru/hx6y14a27W
582 586295
>>86274
Можно и без аналогий. То есть есть надежда, что отдельные последовательности отвечают за какие то конкретные свойства?
583 586296
>>86265
Так или иначе, видимо, единственным итогом существоваших обсуждений является отчёт 1946 года «Ignition of the Atmosphere with Nuclear Bombs» за авторством Конопинского, Марвина и Теллера, в котором приводятся оценки с выводом о том, что как минимум тогдашних энергий для «поджига» атмосферы недостаточно, но что в районе температур от десятков то сотен мегаэлектронвольт (1011...1012 К) «зазор до поджига» становится небольшим и может быть перекрыт неточностями оценок.
https://certus.livejournal.com/54517.html
584 586305
>>84346 (OP)
Почему существуют фиолетовые деревья с фотосинтезом? (Вот прямо в парке краснодара растут) Если такие есть, то зеленый цвет не преграда и можно запилить фотосинтез телесного цвета?
и ЭТО ПОДВОДИТ К МОЕМУ ВТОРОМУ ВОПРОСУ: Алла Пугачева и Мадонна питаются фотосинтезом?
585 586306
Как поле Хиггса создаст новую вселенную? Протоны и нейтроны распадутся, кварки будут превращены в излучение чёрными дырами и разлетятся в пустоту.
Получается материи не останется? Так как "спонтанное нарушение электрослабого взаимодействия" создаст что-то из ничего?
586 586309
>>86305
Фиолетовый или пурпурный цвет возникает из-за большего количества пигментов, которые перекрывает зеленый хлорофилл.
Эти пигменты производные карантинов и служат этакими аккумуляторами "электронов".
Обычно растения это делают, когда внутри клетках нужно пускать много энергию мимо основной системы фотосинтеза. А это уже делают от довольно большего количества причин.
16740661411371.png1,1 Мб, 1280x720
587 586310
Поделитесь ссылкой на адекватный калькулятор для высчитывания возможных вариантов комбинаций из заданный чисел. В скотском интернетике для залётных пикабулей-маткапов из кали-юги по тарифу ростелекома насрато кликбейтом для их учащихся пиздюков-выблядков.

Я максимально не желаю думать самостоятельно: и мне поебать на раздел матана по комбинаторике, и то, насколько это просто (эта хуйня понадобилась мне лишь единожды — сейчас, почти в трицатник годиков, и не понадобится больше ни-ког-дааа). Мне лишь нужно узнать точное число вариантов написания чисел "0-9", если использовать "11" знаков.
588 586311
Тут есть люди, которые интересуются изменением климата и читали статьи про "aerosol masking effect"? Очень хочется поговорить с интересующимся и разбирающимся человеком.
589 586312
>>86310
Знаете... Похуй. Я просто сгенерю набор символов на компе и посчитаю варики в текстовом редакторе.
590 586313
>>86310
Да ты охуел школьную программу не знать.
Хотя бы википедию открыл и узнал о перестановках, размещение и сочетание, чтоб узнать в каких случаях их использовать.
591 586314
>>86312
Я охуел, потому что для генерации этого файла (с параметром разрешенных повторов чисел и их расположений как угодно) нужно ждать годы и иметь хард из 2050 года на охулиарды гигов. Без разрешения повторов — хуйня ,всего 100миллиардов
592 586317
>>86313
Зачем ему знать все это? Для такой задачи достаточно немного подумать головой, тут не надо ничего учить.
593 586323
Это правда, что обнаружили антигравитацию? На какой-то лекции про это сказали, но гугление ответов не дало.
594 586325
>>86323
В период инфляции во вселенной инфлатонное поле действовало как антигравитация.
В первом приближении Темную Энергию можно считать антигравитацией.
595 586326
>>86325
Я думал, что теория инфляции это довольно "зыбкая" теория.

> темную материю можно считать антигравитацией


Почему? Разве она наоборот не добавляет "массы" в пространство, чтобы формулы и наблюдения за гравитацией сходились?
596 586327
>>86326
Теория инфляции стандартная гипотеза, лучше нее пока ничего толкового не придумали.

Так я про темную энергию говорил, а не материю.
597 586328
>>86327
А, мозг подменил слова. Спасибо, значит это имел ввиду лектор

> Сущность тёмной энергии является предметом споров. Известно, что она очень равномерно распределена в пространстве[7], испытывает гравитационное отталкивание вместо гравитационного притяжения[7]

2059381132c9ec64c13bc3c7d9687273.jpg102 Кб, 811x1023
598 586360
1. Как прикладные знания могут быть заложены в инстинктах? Например, бобры строят сложные плотины с учётом кучи факторов чисто инстинктивно, никто их этому не учит. Птицы аналогично строят гнёзда, термиты возводят свои термитники, муравьи занимаются сельским хозяйством, выращивая грибы и тлю и т.д.
2. Как далеко это может зайти? Возможно инстинктивное понимание инженерии, металлургии, химии? Типа орков, которые, будучи полными дегенератами, могут собирать космические корабли из говна и палок.
Том.jpg90 Кб, 400x400
## Mod ## 599 586361
Друзья, вот и подходит к концу 2023 год. Совсем скоро мы преодолеем этот рубеж, и в очередной раз, надеясь на новогоднее чудо, перенесемся из прошлого в будущее. Наступили тяжелые времена: они, как никогда ранее, сплотили нас и научили многому. И я верю, что мы продолжим твердо и последовательно преодолевать трудности, возникающие на нашем пути. А собираться в уютный вечер за чашкой чая, чтобы подискутировать на разного рода интересующие нас темы здесь... Старый добрый /sci всегда открыт для каждого исследователя.

Вам же и вашим близким я желаю здоровья. Его никогда не бывает много и оно очень хрупко по отношению к другим имеющимся у нас ценностям. Берегите здоровье свое и своих близких: не огорчайте любимых вам людей и проводите с ними больше времени, уделяя заботу и внимание. Будьте сильны и идите с гордо поднятой головой вперед всем невзгодам и неудачам. Старайтесь терпеливо выдержать любое испытание, уготованное жизнью и преподнесенное вам. И помните о том, что жизнь прекрасна и заключается в мелочах. Живите каждым мигом и не думайте о плохом.

С праздником вас! Желаю множества научных открытий в наступающем Новом 2024 Году!

Модератор-кот-ученый
600 586363
>>86361
Спасибо, тебя тоже.

Ты гораздо лучше клоуна-модератора из /pr/, который включил автозамену самых употребительных терминов, и теперь там каждый второй пост невозможно прочитать.
601 586365
>>86361
Спасибо! С Новым Годом!
602 586369
Скажите, а извержение вулкана на Камчатке и землетрясение в Японии в один день как-то связано?
603 586371
Электрон квант какого поля?
IMG20240101180050727.jpg128 Кб, 854x1280
605 586374
>>86372
Бампает в тематике
606 586375
>>86369
Да. Там одна и та же плита подпирает и создаёт напряжения, при сбрасывание которых собственно возникают землетрясения.
607 586376
>>86371
Электронного.
IMG20231229185438124.jpg80 Кб, 640x480
608 586378
>>86376
Отлично. Спасибо. Где почитать подробности кроме учебника?

Эта капча заставляет меня ощущать себя умным
609 586384
>>86378
Да нигде особо.
Можешь попытаться почитать лекции Дирака, они легче воспринимаются, но мало отличается от других учебников.
610 586392
>>86360
Бампулька вопросику.
611 586397
>>86360
Инстинкты (отдергивание руки когда прикоснулся к раскаленной поверхности) это всего лишь предобученные нейронные сети. Буквально спиной мозг, это предобученная нейронная сеть, которая умеет управлять кучей всего без участия сознания. Задняя часть мозга различает прямые линии даже у людей слепых от рождения (определяли по электронной активности).

Жизнь представлена на земле от муравьев до китов, от медуз до кротов, так что возможности для создания разной живой материи в том числе и инстинктов обширны.

Другое дело, что у предобученных нейронных сетей есть проблема того, что они обучены не на свежих данных, а на старых. Возможно очень старых. Домашние коты скребут паркет дома пытаясь "закопать" экскременты. При этом я даже готов поверить что сами коты понимают бессмысленность этого занятия, но оно им кажется правильным и естественным как для человека поднимать брови когда он удивлен и улыбаться когда услышал смешную шутку.

В общем, теоретически можно на инстинктах и ракеты строить, но практически это потребует создание второго мозга в теле, который будет заниматься постройкой ракеты пока твоё сознание удивлённо будет смотреть на схемы и оборудование (или не удивлено, ведь как может быть иначе для такого существа, оно просто будет знать, что вот эта схема хорошая, а вот эта плохая, это же очевидно как для нас 2х2=4). Также это потребует как-то обновлять эти предобученные нейронки (вернее ДНК), чтобы строить не одно и тоже, а что-то новое.
1280px-ESTCube-1.jpg155 Кб, 1280x960
612 586401
Вопрос по космическим двигателям без рабочих тел.
Есть солнечный парус, использующий давление света.
Есть электрический парус, использующий давление протонов.

Почему нет активного электромагнитного паруса? Грубо говоря, сверхпроводниковой АФАР, нацеленной лучами на космические источники электромагнитного излучения, притягиваясь к одним и отталкиваясь от других силой Ампера?
613 586402
>>86360
Если я правильно помню, инстинктивно бобрам только хочется копать при звуках воды. Строить плотины они учатся в детстве.
Птицы тоже многому учатся у родителей и других другие птиц, у них даже есть подобие разных культур.
Если сделать человека которому очень нравятся сложные устройства, то он с высокой вероятностью станет хорошим инженером.
Это как, когда поток частиц в мозгу напоминает нужное явление, мозг жмет на кнопку удовольствия. И человеку хочется работать с этим явлением. Или наоборот.
614 586403
>>86401

>космические источники электромагнитного излучения


Это излучение же фотоны переносят.
615 586404
>>86403
Ну квазары, например, вполне радиоволны генерируют
616 586405
>>86360

> 1. Как прикладные знания могут быть заложены в инстинктах? Например, бобры строят сложные плотины с учётом кучи факторов чисто инстинктивно, никто их этому не учит.


В инстинктах заложены поведенческие, мотивационные паттерны. Это нельзя называть знанием (совокупность сведений, информации). У бобра нет чертежа в голове. Бобер действует как бы интуитивно, решая возникающие проблемы по месту, а удачные решения закрепляются условным рефлексом. Плюс наверняка они зеркалят взрослых и чему-то научаются у них. Т.е. научение в некоторой степени у них имеется: отзеркаливание/повторение и условные рефлексы.

> 2. Как далеко это может зайти? Возможно инстинктивное понимание инженерии, металлургии, химии?


У них нет понимания, знания в человеческом смысле. Развитие идёт методом рандома и отбора поведенческих паттернов. На отборе инстинктов и рефлексов не дойти до уровня человеческого понимания, это качественно другой уровень.

> Типа орков, которые, будучи полными дегенератами, могут собирать космические корабли из говна и палок.


Нет. Можно только гипотетически представить, что их кто-то научит, произведёт искусственный отбор на нужные поведенческие паттерны и т.п. В общем, утрируя, человек и есть такой орк, собирающий ракету слесарь просто делает несложную операцию, а знание как летают ракеты ему и не нужно.
617 586406
>>84346 (OP)
Интересно, возможен ли гипотетически и практически запуск цепной реакции синтеза без помощи ядерного взрыва? Чисто термоядерная бомба, без ступени деления.
618 586407
>>86406
В экспериментальных энергетических установках лазером зажигают. Но для взрыва фокусирующие оболочки (или как там их называют) все равно понадобятся.
619 586408
>>86407
Мне кажется лазер для бомбы это тупиковый путь. Во всех этих экспериментальных установках выход энергии меньше чем затрачено, цепная реакция гаснет, а сами установки имеют нереальные размеры.

Проблема в огромной плотности мощности, требуемой для удержания нагретой плазмы в компактной кучке, пока идет синтез. Интересно нельзя ли тут как-то использовать кинетическую энергию, в ней ведь можно запасти потенциально неограниченное количество энергии, можно даже не за один раз. Что-то типа боеголовки космического базирования на высокоэллиптической орбите, или вообще на ретроградной околосолнечной. Потом в процессе входа в атмосферу на нескольких десятках километров в секунду кинетика конвертируется в тепло, которое обжимает капсулу с горючим.
620 586412
>>86384
А чому в википедии не пишут про это?
621 586413
Объясните на пигмейском, из чего состоит материя?
Вот есть атом. Атом — это такая хуёвина с ядром и летающими вокруг него на световой скорости электронами. В ядре протоны и нейтроны. Что дальше? Почему там уже начинаются какие-то математические абстракции и гипотетические квазичастицы? Типа оно настолько мелкое, что уже ни увидеть, ни описать нельзя — только теории строить? Или как?
622 586414
>>86413
Из энергии.
623 586417
>>86405

>их кто-то научит


А кто научил бобров плотины строить? Могли бы грызть себе кору и в ус не дуть. Прочие грызуны так и делают — и ничего, процветают. Птицы аналогично. Зачем ткачу целая хижина на дереве, если голубю пары веток хватает?
Я к тому, что такие паттерны поведения, видимо, возникают рандомно — а дав какое-то преимущество, закрепляются и совершенствуются. Пять миллионов лет назад бобр строил хатку в случайной куче веток, а ещё через пять миллионов лет начнёт таскать в свою плотину камни для укрепления фундамента. Через десять научится углублять дно водоёма и изменять рельеф берега, а через двадцать освоит кирпичи из сырой глины. А там уже до освоения огня и начала медного века всего ничего — пара сотен миллионов — остаётся.
Грубо говоря, конечно.
624 586418
>>86413

> Типа оно настолько мелкое, что уже ни увидеть, ни описать нельзя — только теории строить?


Типа да. Настолько мелкое, что у нас нет инструмента для наблюдения, не оказывая влияние на него. Любое наблюдение это взаимодействие. Чтобы узнать положение или скорость фотона надо в этот фотон пульнуть другим фотоном, в результате мы видим результат взаимодействия измеряемого фотона (за которым пытаемся наблюдать) с фотоном от измерителя, а не измеряемого фотона самого по себе. Для измерения без влияния нужен измеритель, который воздействовал бы на измеряемое значительно более мелкой частицей, а такого пока нет и не предвидится.
625 586419
>>86413

> увидеть


Как увидеть то, что не отражает свет?

> описать


Описывают на основе наблюдений. Наблюдать за частицами можно посредством их взаимодействия с другими частицами.

> только теории строить


Если теория успешно предсказывает, то в чем проблема?

> из чего состоит материя


Правильный вопрос: чем является материя. Материя является формой энергии. Электрон как бы тоже не "шарик", а именно энергия в какой-то точке пространства.
626 586420
>>86417

> Через десять научится углублять дно водоёма и изменять рельеф берега, а через двадцать освоит кирпичи из сырой глины.


Такое можно гипотетически представить. Но чтоб бобры начали понимать металлургию, химию и построили ракету, такое методом перебора инстинктивных действий не получить, тут нужно абстрактное мышление, накопление информации (язык, письменность), специализация труда и прочее.
627 586424
>>86412

> А чому в википедии не пишут про это?


Напиши сам.
(кто? Я?)
628 586426
>>86412
Вообще-то пишут, например в статье про уравнение Дирака например.
629 586427
>>86414

> Из энергии.


* E0=m•c2 недвусмысленно об этом говорит, если ты ещё не понял. Именно в этом её "фишка".
630 586429
>>84346 (OP)
Анонэс! Доброй ночи. А конкретно- анаоэс-физики. Обращаюсь к вашим светлым( и темным) мозгам, потому что мои мозги уже потрескивают и поплыли. Сказу прямо и честно-я тупо гуманитарий. Закончил в свое время истфак(правда историком нихуя никогда нигде не работал). Короче я к тому, что я нихуя не физик, а скорее все же лирик. И вот с годами я начал в ютубчике посматривать и в интернетиках почитывать всякую интересную инфу про квантовые флуктуации, природу материи, сингулярность и всякие там бозоны Хиггса( благо этой хуйни там счас навалом). Ясен хуй, теоретической базы у меня никакой, из физики я помню ток mc квадрат, поэтому до конча понять этих прекрасных людей со светлыми очами мне трудновато. И все же я сам для себя начал делать кой-какие выводы. Вот хочу у вас уточнить кой-чаво.
Вот допустим для меня открытием стало что мы нихуя не тратим никакие энергии, которые используем. Они просто переходят из одного состояния в другие( кинетические в статические, энергия в массу), но в принципе все энергии сохраняются в своем первоначальном виде. Так это что получается, аноны? Вся жизнь на планете земля существует за счет перехода эпергий и масс из одного состояния в другое? насколько я понял вумных дядек, вся энергия во вселенной пытается перейти из концентрированного состояния в рассеянное( простите, если некорректно термины употребляю) и в принципе все это называется энтропия. То есть по сути мы существуем за счет энтропии получается?
631 586433
>>86429
Да. То, что обыватели называют использованием энергии, является получение нужного результата за счёт увеличения энтропии.
Но вообще закон сохранения энергии это середина школы. Как ты это пропустил?
632 586434
>>86429
Это с точки зрения вселенной. Земля не является замкнутый системой, можешь расслабиться.
633 586436
>>86434

> Это с точки зрения вселенной.


Вселенная замкнутая система?
634 586439
>>86436
Предположительно, да.
635 586443
>>86433
А как энтропия связана с течением времени? Я так понял, что сам факт течения времени- это следствие повышения уровня энтропии. Или, наоборот, повышение энтропии происходит за счет течения времени, Или мухи отдельно, котлеты отдельно?
636 586444
>>86418

>Для измерения без влияния нужен измеритель, который воздействовал бы на измеряемое значительно более мелкой частицей, а такого пока нет и не предвидится.



Проблема не в этом, если я тебе в классической физике ограничу способы измерения параметров какой-нибудь системы только теми, которые эту систему хуярят сильно, то ты всё-равно сможешь косвенно и с помощью математики извлечь в конце концов все нужные физические наблюдаемые с точностью, которую тебе приборы позволят. Потому что сами параметры и физические величины имеют свои значения, независимо от того измерял ты их или нет

В квантовой же механике уже из самого мат.аппарата вытекает, что физические наблюдаемые не всегда имеют значение, а измерение (сколько бы слабым ты его не делал, хоть косвенным вообще) и сколько статистики одинаковых систем не собирал, не дает тебе определить однозначную физическую наблюдаемую. Это в своё время вызвало споры о том, что квантовая механика лютый пиздеж и её надо напильником дорабатывать, но потом поставили эксперименты, которые четко указали, что дело не в измерении.

"Измерение которое влияет на систему, поэтому измерить точно нельзя" это научпоп-хуита для обывал, которым кто-то это скормил. Не надо так думать.
637 586445
>>86443
Вещи плохо связанные. Какой-нибудь маятник с колебаниями свою энтропию не меняет, но ведь время всё-равно течет. Энтропия это про другое, это про потерю информации о системе, т.е. чисто фишка, которая возникает из мысли "ну мы неточно знаем начальные условия, а еще окружающий мир хоть немного но влияет на систему всегда, поэтому информация утечет".
>>86436
Предположительно да, но не сохраняет энергию, так что не парься
638 586446
>>86436
Неизвестно. Когда мы говорим вселенная чаще всего имеется наблюдаемая вселенная. Что там дальше за наблюдаемой вселенной непонятно.
>>86444
То есть у частицы нет местоположения или все же местоположение неоднозначно в каком-то ограниченном пространстве? Вроде же второе, то что да мы не знаем и не можем знать где находиться частица, но с вероятностью такой-то она находиться вот здесь, а с другой вероятностью вот здесь.
639 586447
>>86439

> Предположительно, да.


>>86445

> Предположительно да


>>86446

> Неизвестно.



Поэтому меня всегда бомбит, когда понятие «энтропия» применяют ко вселенной и начинают из этого делать какие-то выводы и предсказания. Скажите тогда что вселенная это организм и он умрет или поделится на несколько вселенных, как клетка, это точно также «предположительно» как и энтропия.
640 586448
>>86447
Точные ответы только в библии. В физике всё что не проверили сотню раз в контролируемых условиях "предположительно".
641 586450
>>86448

> В физике всё что не проверили сотню раз в контролируемых условиях "предположительно".


«Предположительно» в науке это гипотеза, а чтобы считаться научной гипотеза должна быть фальсифицируемой. Закрытость вселенной (и соответственно энтропия) не может считаться научной гипотезой, она не фальсифицируема, ты не можешь вылететь за пределы вселенной и проверить закрытость вселенной. Так что энтропия вселенной это не наука, а диванные домыслы, метафизика если философски.
642 586451
>>86447
Ты в ближайшие сто лет умрёшь с гарантией. Очень вероятно, человечество не осилит даже следующие 20 тысяч и вымрет. Конечно, если вдруг научимся терроформациям и межзвездным перелётам, возможно, пару миллиардов протянем (не совсем уже мы), но это крайне оптимистичные предположения. Из разряда чудес.
643 586452
>>86446

>у частицы нет местоположения или все же местоположение неоднозначно в каком-то ограниченном пространстве?


Само понятие местоположения мало смысла имеет просто. Это что-то из нашего мира, что-то статистическое.
644 586453
>>86452
Не могу понять. Есть камень, он состоит из атомов, они из частиц. Где частицы? Там где камень. Вроде всё логично.
645 586454
>>86446

>То есть у частицы нет местоположения или все же местоположение неоднозначно в каком-то ограниченном пространстве?


Третье. Тело частицы бесконечно расширяется. Пока она не решит провзаимодействовать с другой пересекшейся с ней частицей. После чего исчезает. И новые частицы начинают расширяться уже из точки взаимодействия.
646 586455
>>86454
Понял, спасибо
442A7F15-D29C-44EC-8AE8-053E064BBF19.jpeg174 Кб, 1530x675
647 586456
>>85484

>Почему в русскоязычной среде принято отрицать/приуменьшать значение IQ?


Потому что пикрил. Корея — хз, но Китай, где обезьяны срут на улицах и живьём жарят собак на говняном масле? Иран, который буквально воплощение всех стереотипов о муслимских парашах пятого мира? Для полноты картины там только какого-нибудь Чада или Конго в пятёрке лидеров не хватает.
648 586457
>>86456
>>85484
Потому что IQ изначально придуман как тест показывающий недостаточный интеллект. Он не говорит, что один человек умнее другого, он говорит, что один человек имеет проблемы с интеллектом (IQ < 85), а другой нет (IQ > 85).
649 586459
>>86453
Ну ты когда говоришь "камень" ты довольно большой объем описываешь, при этом частиц там тоже много, поэтому среднее значение нахождения этих частиц отлично совпадает с твоими наблюдениями.

А если возьмешь какой-нибудь атом водорода, то электрон этот ебучий не будет уже находиться где-то конкретно, у него будет своё состояние квантовое, уже из которого ты можешь найти распределение его нахождения в этом атоме.

При этом это не классическая вероятность, где мы просто не знаем, там в принципе природа как-бы не в курсе где он.
650 586460
>>86459
То есть при большом скоплении частиц они становятся устойчивыми? Хотя нет, не так. Большое (интересно сколько) скопление частиц обладает свойством устойчивости в пространстве. Малое скопление частиц (до одной) не имеют свойства вообще находиться в конкретном месте.

Получается "местоположение" есть результат взаимодействия многих частиц друг с другом. То есть сосредоточенная энергия (а что есть частица как не энергия) превращается в массу, материальный объект, в то время как распределённая энергия в пространстве представляет собой некое энергетическое поле теряющие свойства привычной материи.
651 586461
>>86459
А правда, после какого момента концентрации частиц у них как у скопления появляются привычные нам координаты в пространстве?
652 586462

>Большое (интересно сколько) скопление частиц обладает свойством устойчивости в пространстве.


Там всё сложнее, смотря как ты определяешь устойчивость. Просто когда ты берешь камень, тебе немного похуй на погрешности в 10^-10 м, т.к. это 10^-10 %. А если у тебя объект размером с 10^-9, то на подобные штуки глаза уже не закрыть.

Плюс еще вопрос в том, как частицы взаимодействуют, свободная частица имеет тенденцию "расплываться", а если связанная, то распределение вероятностей не меняется либо циклично меняется.

>То есть сосредоточенная энергия (а что есть частица как не энергия) превращается в массу, материальный объект, в то время как распределённая энергия в пространстве представляет собой некое энергетическое поле теряющие свойства привычной материи.



Массу и энергию не разделяют особо на разные категории, но туда лучше не лезть, это уже философия.
653 586464
>>86459
Электрон это понятно, а кварки что?
654 586476
>>86462
То есть весь мир по сути нестабилен с точки зрения местоположения? Мы просто слишком большие, чтобы заметить это "дрожание" вселенной. Воодушевляет на самом деле. Столько ещё можно осознать.
fb.png1,1 Мб, 1200x630
655 586477
>>86476
Ещё осознай, что всё есть пустота с некими всплесками энергии
656 586478
>>86476

> То есть весь мир по сути нестабилен с точки зрения местоположения?


Не только положения, но и по куче других параметров. На микроуровне все "плавает", вернее колеблется.
657 586484
>>86476
Если начать немного копаться, внезапно появляется явление энтропии из этого.
658 586495
Не лучше ли говорить, что пространство-время не 4мерно, а 3.5мерно, потому что на данный момент ни наука, ни даже философия ничего не могут сказать о путешествиях в прошлое, т.е. пока что можем говорить только о движении вперёд. Причём путешествие в прошлое чаще всего представляют как скачок, телепорт,- а с плавным движением назад всё ещё запущенней. Что будет видеть путешественник и как будут видеть его? К чему заморачиваться - ну а типа вдруг для описания движения назад во времени неизбежно придётся вводить новые координаты, потому что стрела времени может быть прямой только в будущее, а в прошлое неизбежно пойдёт загогулинами дробной размерности, так что лучше сразу подчеркнуть, что врпмя может быть не 1-,2- и тд целомерными, а именно нецелой размерности. ладно, вы думойте, а мне спать пора
659 586499
>>86495

> наука ничего не может сказать про перемещение во времени в прошлое


Альберт Эйнштейн всё сказал уже про это. Ищи червоточину, создавай отрицательную гравитацию, чтобы компенсировать своё влияние гравитации и попадешь в прошлое. Нашли правда пока что ноль этих червоточин, но рано или поздно найдут или сами сделают.
660 586501
>>86495
Если ты начнешь время назад отматывать, не заметишь разницы, разбитая ваза не соберется, ты не в своё прошлое попадать начнешь, а в самое вероятное
662 586509
>>86499

>создавай отрицательную гравитацию


Это не поможет. Вселенная расширяется на из-за положительного знака гравитации.
663 586510
>>86499

> Ищи червоточину, создавай отрицательную гравитацию, чтобы компенсировать своё влияние гравитации и попадешь в прошлое.


только это уже не физика, а математика, не затрагивающая проблему причинности типа парадокса дедушек или как его. А физика понятия не имеет, как поставить соответствующий эксперимент для разрешения парадокса, ни про существование отрицательной/мнимой энергии для этого и как добыть её. Т.е. как я представляю хорошее продвижение в этом направлении: пишут статью - ставят соотв эксперимент, закрепились, повторили и тд маленькими шажками. А по факту тут с экспериментами туговато, городят пирамиду спекуляций одна на другой. А матаном можно отрицательную, мнимую, кватернионную и какую хочешь гравитацию ввести и телепортироваться по вселенной как хочеш
664 586511
>>86510

> парадокс дедушки


Что-то физики спокойно живут с парадоксом телепортации частиц в пространстве в места где их не могло быть по всем законам логики.

> с экспериментами туговато


> спекуляции


Всё так, но какой вопрос такой ответ. Лучше всё равно ничего нет кроме очевидного: пока никто не путешествовал, а значит мы не знаем возможно это или нет.
665 586512
>>86511

>> парадокс дедушки


>Что-то физики спокойно живут с парадоксом телепортации частиц в пространстве в места где их не могло быть по всем законам логики.


Неправда, никаких наблюдаемых парадоксов не существует.
666 586513
>>86512
Объяснишь как материальный объект телепортируется?
667 586514
>>86513
Я не физик, я математик.
Я имею в виду, что ты неправильно понимаешь, что такое парадокс. Парадокс, это когда из А можно вывести Б и не-Б одновременно. Это говорит о том, что А ложно, и на этом строится доказательство от противного.
В реальном мире нет и быть не может фактов, приводящих к парадоксам, потому что законы логики выполняются неукоснительно.
Если тебе кажется, что телепортирующиеся частицы нарушают какие-то законы логики, то нет, это не так. Просто твои представления о том, как эти частицы должны себя вести неверны.
668 586515
>>86514
Понял, но как это объясняет невозможность ситуации с дедушкой? Убил дедушку, как я родился в этом случае? А какая разница, родился и родился. Телепортировалась сквозь непроницаемую стенку? А как это возможно? А какая разница, возможно и возможно.

Первое это парадокс, второе это квантовая физика. Я понимаю, что второе квантовая физика это потому что мы наблюдали телепортацию. А первое парадокс потому что гипотетическая ситуация.

Ладно, в принципе я сам ответил на свой вопрос. Спасибо за внимание.
Силаев П. К.  -  Квантовая теория  -  Динамическая схема квантовой механики  (Лекция 6).webm19,8 Мб, webm,
1280x720, 0:50
669 586518
>>86513
Ты сам то знаешь, что называют квантовой телепортацией?
670 586519
>>86518
Нет, расскажи
671 586522
>>84346 (OP)
Когда железо или сталь калят, что за чешуя сыпется с них?
672 586524
>>86522
Окалина или оксид железа.
673 586528
>>86515
Ничего ты не понял.
Существование частицы никак не противоречит её телепортации за стенку, потому что частица не шарик, а поле, которое стенка не экранирует.

А вот мёртвый дедушка твоему существованию противоречит. Разве что ты "тунеллировал" вместо нормального рождения. Но это как раз одно из решений этого парадокса через многомировую интерпретацию: ты попадаешь в не свой мир.
674 586532
Если вселенная бесконечна, значит после моей смерти рано или поздно все молекулы разложившегося меня вновь соберутся обратно и я снова стану живым. В чём я не прав?
675 586535
>>86532
Что такое "я"?
676 586536
>>86535
Ну моё тело и моё сознание в любой момент моей текущей жизни епта
677 586538
>>86536
Ладно, что такое сознание?
678 586541
>>86538
Ты мне зубы то не заговаривай. По факту мне напиши в чём я не прав или извинись в случае моей правоты.
679 586551
>>86541
Вселенная конечна и имеет начало. Земля конечна и имеет конкретные границы. Ты сдохнешь навсегда.
680 586557
>>86541
Ты не определил понятия, а значит непонятно о чем говорить. Да и вселенная не бесконечна, это в точь так же как считать комнату бесконечной после того как сделал один шаг в темноте.
681 586558
>>86532
Школьник прочитал Ницше и прибежал искать научное подтверждение вечному возвращению? Не будет его. Начни с того, что ты секунду назад был другой, а организм полностью обновляется за 5-7 лет, в нем не остаётся ни одного атома прежнего тебя. Кто должен восстановится в будущем, если этого стабильного «кто» в принципе не существует?
682 586560
>>86557

>Да и вселенная не бесконечна


Школьник опечатался, для вечного возвращения вселенная должна быть конечной, если вселенная бесконечная, то ни каких повторений форм не будет.
683 586562
>>86560
Плевал я на бредни Ницше. Пока нет доказательств бесконечности вселенной. Есть лишь понимание, что если мы не изобретаем устройство способное на телепортацию, то никогда (предположительно) не поспеем до её края из-за скорости её расширения.
684 586563
685 586566
>>86560
Это не важно.
Если вселенная возникла, то ее возникновение возможно. И она возникнет вновь.
Через достаточное количество повторений, ребенок с памятью Анона может сформироваться случайно. Даже если это просто клон.
686 586568
>>86566

>ребенок с памятью Анона может сформироваться случайно


Дальше что? Анон воскреснет из мертвых? Душа Анона спустится с небес и начнёт опять жить земную жизнь в теле-копии?
687 586569
>>86568

>Душа Анона спустится с небес и начнёт опять жить земную жизнь в теле-копии?


Это не важно. Душа в теле-копии будет искренне верить что она и есть душа Анона.
Анон же, это не тело с его молекулами. И не сознание с его квалиа. Анон это личность и память.
688 586570
Душа и тело Анона - всего лишь среда в которой Анон существует.
689 586571
>>86569

> Анон это личность и память.


Для наблюдателя со стороны, с перспективы третьего лица. А от первого лица ты умрешь и от того, что когда-то появится твоя копия, ни чего для тебя не изменится, ты не начнёшь жить заново, не воскреснешь или что ты сам себе выдумываешь.
690 586572
Да нет никакого анона. Это всё самообман. Буквально. Самообман. То чего нет, но во что выгодно верить. Я не более чем смешное поле частиц и моя "смерть" значит июне больше чем пустяк.
691 586573
>>86572

> но во что выгодно верить.


Старикам выгодно, чтобы смерти не боятся, как самоуспокоение. И выгодно пролам насаждать такое мышление, чтоб работали и не роптали и в окопы смело шли умирать. А в молодом и зрелом возрасте это деструктивно, лучше жить с пониманием что жизнь у тебя одна и относится к ней с соответствующей ответственностью.
692 586578
>>86576 (Del)
Не будет, скорость света всегда скорость для наблюдателя. Как угодно изворачивайся, а выше этого скорость невозможна. Не баг, а фича. Константа в программном коде.
693 586584
>>86566
Нет. Это неопределенность бесконечность к бесконечности
694 586586
>>86574 (Del)
Смысл в проживании здесь и сейчас - Дазайн.
695 586591
>>86584
Без принятия на веру таких недоказуемых вещей как реальность прошлого, существование других людей и наличие причин у событий мышление становится бессмысленным и обсуждать просто нечего.
696 586603
>>84346 (OP)
Анчоус, что за еврейские фокусы? Это на каждом сухом мороженном, плюс-минус. На НГ изьебнулся, но получилось только сто грамм добавить, а тут обещают из ста смеси и трехсот миллилитров шестьсот продукта. Откуда двести то беретcя? Родители пиздели и Дед Мороз существует? Может Шваб в пизду со своими жуками идет, мы тут на самогенеряшимся мороженом протянем?
697 586606
Прошу пояснить причину тряски касательно проблемы измерения в КМ, и особенно разгона про СОЗНАНИЕ.

Если измерение это тупо встреча фотона инструмента с испытуемым электроном, то чем это отличается от встречи электрона с левым фотоном, который просто мимо летел? В обоих случаях схлопнется волновая функция. И причём тут тогда наблюдатель с СОЗАНИНИЕМ? У хомо сапиенс монополия на испускание фотонов? Смешно
698 586608
>>86606
Тут проблема в том, в КМ постулируются два типа систем. Классические - у них строго детерминированы динамические переменные. Они образуют прибор/наблюдатель/мамашу-шлюху/наблюдаемую вселенную.
И квантовые - они живут в своем манямирке, который не имеет физического смысла, пока не испытает возмущение со стороны классической системы.
Однако по идеи классические системы являются продолжением квантовых систем, но у них не из чему взяться "классичности" кроме как другой классической системы. А конечном итоге все сводиться к тому, что изначальной классичностью обладает некая магическая хуита, которая интерпретирует измерения. От сюда и лезет шиза с сознанием.
699 586609
>>86603
Ты путаешь массу и объём.
700 586610
>>86606

>В обоих случаях схлопнется волновая функция.


Там кек в том, что схлопывания в уравнении нет. Т.е. если у тебя электрон ебучий в суперпозиции находится, и на него фотон налетает, то фотон тоже становится в суперпозиции, и если ортодоксально подходить к квантовой мехнике, то когда потом этот фотон в детектор залетает, сам детектор тоже в суперпозиции, и ты потом смотришь в него и тоже в суперпозиции. И если подходить так, только в этот момент ты заметишь какую-то хуйню в результатах.

Причем если я не ошибаюсь, там объекты довольно классического размера точно в квантовое состояния загоняли уже в экспериментах.
701 586612
>>86609
Когда инструкцию читал, тоже об этом думал, хитрожопый маркетолог аккуратно жонглирует. Но я же сделал его, взвесив все этапы на весах. В моем понимаем масса в простом процессе не может появится, мы не меняем кардинально вещество, мы буквально делаем смешение, однородность и меняем агрегатное. Добавилось сто грам - было 397 грамм из смеси и сливок, а стало 501 грамм. Возможно я рукожоп или маркетолог смело припизднул еще сто грамм, не принципиально, но сотка то добавилась. Откуда?
702 586613
>>86612
У тебя в инструкции написано 600 мл, а не г.
А сотка могла легко взяться из воздуха.
703 586614
>>86495

> Не лучше ли говорить, что пространство-время не 4мерно, а 3.5мерно, потому что на данный момент ни наука, ни даже философия ничего не могут сказать о путешествиях в прошлое, т.е. пока что можем говорить только о движении вперёд.


Никакой связи мерности и путешествий в прошлое нет. Мерность возникает совсем из других соображений.
704 586615
>>86576 (Del)

> Если из другой галактики создать световой пучок маятник который будет шатать со световой скоростью то световое пятно на поверхности другой галактики будет двигаться выше скорости света! Думойти!


Это известный факт. И он никак не противоречит физике, тк никакая материя или энергия тут не движется быстрее скорости света.
Аналог этой ситуации - сверхбольшие ножницы, у которых точка соприкосновения лезвий может двигаться со сверх световой скоростью. Или можно просто выставить ряд лампочек и зажечь их все почти одновременно так, что "точка зажигания лампочек" движется со сколь угодно большой скоростью.
Все эти примеры не позволяют передать энергию или информацию со скоростью большей скорости света.
705 586616
>>86607 (Del)

> инфополе вселенной


Что это такое?
706 586617
>>86607 (Del)

> инфополе вселенной


Что это?
707 586619
>>86613

>из воздуха.


Наверное да, в этом ответ. Куб воздуха при нулевой 1,29 кг весит. Мороженое мы взбиваем, образую множество камер-пузырьков. Фокусы не перестали быть еврейскими, поскольку вес готового продукта не меняется, а просто добавляется масса воздуха, но откуда добавилась масса мы решили. Спасибо.
708 586620
>>86610

>сам детектор тоже в суперпозиции


это ж недоказано
709 586621
>>86608
>>86610

Хотите сказать, что если на испытуемый электрон налетит случайный фотон (не от наблюдателя), то коллапса не будет? Прикол. Я думал что разрушение квантовых систем левыми частицами случайным образом — и есть деконеренция
710 586622
Интеллект действительно начинает снижаться с 30 лет? Насколько сильно? Можно ли отсрочить деградацию?
711 586623
>>86621
Суперпозицию разрушает именно прибор - классическая система.
Если на твой электрон нелетит левый фотон, то суперпозиция незначительно измениться.
Однако если ты будешь попытать одновременно измерить это левый фотон и электрон. Этот налет уже уничтожить суперпозицию.
Декогеренция это про то, что прибор измеряет не те частицы, которые мы приготовили.
712 586624
>>86622
Смотря, что считать интеллектом.
Если считать способности к быстрому обучению чему-то новому, то да. Еще страдает скорость осмысленных реакции и принятия решений. В какой-нибудь спорте будешь всасывать у молодых.
Насколько сильно это проявляется тут сильно индивидуально.
Да. заниматься максимально разнообразной деятельностью, общаться с молодыми.
713 586625
>>86623
а что такое прибор? можно считать прибором экран за двумя щелями, где подсвечиваются попадания электрона? и опять же причем тут сознающий человек, на экране и без него все подсветится
714 586628
>>86625
Выше я писал, что прибор это любая система, наделенная классическими свойствами. А то как это наделяется это уже философский вопрос.
На практике же прибор это все что "классически" действует на квантовую систему. Это одновременно и две щели, экран и пустота между ними.
Если уж воспринимать эту шизу с сознанием, то оно делегирует свою классичность прибору только одним знанием о нем.
715 586629
>>86628
наблюдатель увидит либо стрелку вверх либо вниз, хотя в манямире они в суперпозиции — это я понял

обезьяна тоже увидит только один исход, полагаю

а собака? а рыба?

а может мера создание в существе есть способность схлопывать волновую функцию, ну или выбирать какой-то мир в альтверсе, если ММИ права
716 586630
>>86629

> хотя в манямире они в суперпозиции


Суперпозиция это состояние до измерения (воздействия), о котором мы, соответственно, ни чего не знаем, оно нам недоступно. Измерив (оказав воздействие) мы наблюдаем иное состояние. Просто поменяй слово «суперпозиция» на «не известное(не наблюдаемое), до измерения (до воздействия)» и вся шарлатанская магия НАБЛЮДАТЕЛЯ пропадёт.
717 586631
>>86629
Даже бездушная машина или компьютер, если он классический, увидит только один исход.
Коллапс волновой функции чисто магический, тут много чего можно напридумать, ибо его механизм не доступен нам через эксперимент.
718 586632
>>86620
Да, недоказано, это просто как следствие КМ идет

Но на детекторе LIGO они линзу 40 кг засунули в квантовое состояние, а еще какие-то чуваки, маленькие барабаны алюминивые переводили в запутанное состояние.
719 586637
>>86631
Да блин, что делает объект классическим, может кто-то объяснить твёрдо и четко? Только не говорите что его способность схлопывать волновуху, а то уже рекурсия какая-то
720 586639
>>86637
В каждой интерпретации свое объяснение. Разве кроме многомировой, где коллапса нет как таково.
721 586640
>>86632

>они линзу 40 кг засунули в квантовое состояние


Они ее просто охладили.

>переводили в запутанное состояние


...
722 586641
>>86606

>это тупо встреча фотона инструмента с испытуемым электроном


На каком расстоянии происходит эта "встреча"? 1 метр? 1 нанометр?
Напомню на всякий случай - размеры точечных частиц равны нулю. С другой стороны, радиус действия электромагнетизма - бесконечность.
723 586644
>>86640

>Они ее просто охладили.


И что это меняет?
>>86641
Тебе придется дать определение "расстояния" для квантовой электродинамики, что потребует от тебя пары лекций и довольно смешного ответа, который только больше тебя запутает
724 586647
>>86557
Если вселенная достаточно долго будет существовать, то в ней возможно что угодно - появление больцмановского мозга из флуктуаций или анона сдохшего триллионы лет назад.
725 586648
>>86644
А расстояние от земли до луны например это тоже "смешная" концепция или она каким то невероятным образом вдруг проясняется?
726 586649
>>86648
Оно проясняется очевидным образом
727 586655
>>86647
Ключевое слово "если". Энтропия пока что выглядит как причина по которой не всё что угодно может произойти в будущем.
728 586657
>>86649
Нет, не проясняется.
729 586662
>>86619
Пузырьки не влияют на массу, но газы из воздуха могут вступать в реакцию, образуя вещества, которые уже будут влиять.
730 586667
Что можно делать с тяжелой водой? Кроме как налезать на бутылку
1000015266.jpg396 Кб, 1080x2400
владимир 731 586670
Какой радиус Шварценеггера для планеты Земля?

Каковы доказательства, что частицы попав в чёрную дыру, стремятся к центру, а не вечно движутся по окружности радиуса, в который попали?
732 586671
>>86622

> Интеллект действительно начинает снижаться с 30 лет? Насколько сильно?


Вот этот >>86624 анон прав, вопрос в том, что считать интеллектом. Быстрое обучение - да, будет деградировать с возрастом. Но если считать интеллект как приспособляемость к изменениям, то тут уже не так однозначно. У взрослого есть опыт и знания, которые будут компенсировать способность к обучению, но это будет сильно зависеть от опыта и знаний конкретного индивида, от ширины когнитивной карты (картины мира) и гибкости мышления (способности к восприятию нового). И в таком понимании интеллекта (приспособление) он растёт (не у всех) до старческих естественных дегенеративных изменений в мозге, которые не всегда приходят раньше смерти.

> Можно ли отсрочить деградацию?


Занимайся умственным трудом, познавай, меняй деятельность, развивай гибкость ума и т.п. Всякие альцеймеры ебут работяг, всякие кризисы выпиливают их же.
733 586673
>>86671
* он может расти
734 586674
>>86667
Пить.
735 586675
>>86674
Ну разок накатил, за науку а потом что? Типа коллекционировать для внуков, профитно получится?
владимир 736 586677
>>86675
Можешь попробовать наладить плазменный реактор.
738 586696
В связи с последними событиями вопрос: а нахуя нужны общие городские котельные? Там же между котельной и потребителями в трубах наверняка огромные потери энергии. Разве не энергоэффективней каждому дому иметь свою мини-котельную? Или необходимость подвоза топлива к каждому хрущу перекроет любой выигрышь в энергии?
739 586697
>>86696
Люди долбоебы, пересрутся на вопросе как и кому сколько топить, куда дым несет и тд. Есть невидимое нечто дающее тепло и греющее пустоту (похуй) - все привыкли. Конечно топить индивидуально разумнее, но вот отказать от "услуги" тот еще квест.
740 586699
Привет, двач.

С чего начать изучение кибернетики? Может есть какие-то гайды? Типо "вот сначала эта книга, потом эта книга, потом вот это видео" и тд?

Немного посмотрел зарубежный ютуб на тему кибернетики, попадаются только рекламы института кибернетики в Австралии, а учебную программу не показывают.
741 586700
Вчера наткнулся на такой термин как CSE. Сначала подумал, что это Computer science engineering. Оказалось, что это computational science and engineering
https://en.wikipedia.org/wiki/Computational_engineering

Есть что-то годного на русском языке по теме? А на английском есть годнота для начинающих?
Как понимаю, то нужно неплохо знать матан на уровне бакалавра физмата, знать физику, программирование, CS (computer science) и несколько компьютерных программ?
Ничего не упустил?

С меня луч добра!
742 586704
>>86696
Не знаю, но нужны во всех странах Европы где зимой холодно тоже центральное отопление (за исключением Польши, которая пошла по второму пути и теперь там нечем дышать от гари в некоторых городах)
743 586706
>>84346 (OP)
Фоновые нейтрино влияют ли на бета-распад ядер?
744 586707
>>86706
Если не фоновые влияют, то и фоновые влияют
745 586708
>>86707
А не фоновые таки влияют?
Вроде нейтрино сильно влияют на синтез сверхтяжелых ядер при взрывах сверхновых или ядерными реакциями в белых карликах.
В любом случае фоновых нейтрино на десятки порядков больше, чем от всяких ядерных реакторов и прочего рукотворного говна, при этом мы не можем экранироваться от нейтрино как от фотонов, чтобы с чем сравнивать бета распады.
746 586709
>>86708

>Вроде нейтрино сильно влияют на синтез сверхтяжелых ядер при взрывах сверхновых


Смотря как считать
747 586711
>>86709
Давно все посчитали.
Ядра умирающих звезд непрозрачны для нейтрино, отчего в нем идут специфические реакции с рождением нейтронов(нейтрино встречается с кварков, меняет его аромат, а сам превращается в позитрон), которые захватывают ядрами на периферии, отчего они могут мутировать аж до урана. Если недра звезды достаточно плотные, начинается лавинный процесс нейтронизации вещества, где нейтрино выступают катализаторами.
748 586712
>>86711
Интересно, может ли быть такой поток нейтрино, который бы смог убить человека?
749 586713
>>86712
Вполне может, но ты быстрее помрешь от того, что его создает.
750 586717
>>86711

>Давно все посчитали.


Я другое имел ввиду и при другом процессе.
Если про ядро речь, и непрозрачно, то там похоже действительно где-то 1к1 должно получаться.
Вообще я не уверен что тот другой процесс вообще существует, а не просто популизаторы так криво и уёбищно про него рассказывают, что полностью теряется смысл в тексте о том что они вообще не о таком процессе говорят а о совсем другом.
751 586722
>>86712
Да, даже кто-то считал, нужно,чуть ли не впритык к сверхновой подойти в момент разгара пиздеца
752 586723
>>86711
Нейтрино без заряда попадает в протон и превращает его в нейтрон?

Ну и с чего это катализатор, если это тратящийся компонент реакции.

>начинается лавинный процесс нейтронизации вещества, где нейтрино выступают катализаторами.

753 586726
>>86722
Ну вот тот второй процесс о котором я говорил, типа популизаторы про него рассказывают
Типа поток нейтрино именно, а не нейтроны, создают во внешних слоях при взрывах звёзд расходящуюся ударную волну, и синтез идёт именно из-за давления, а не от потока нейтронов. При этом от этого якобы такое давление что выше чем в ядрах самых больших звёзд, и выше чем при любых взрывах, и это не в центре а во вшнешних слоях так.

Но чёт я так прикинул, сомнительно это всё, возможно популяризаторы просто дегенераты и не понимают о чём говорят.
754 586750
>>86723
Нейтрино попадает в протон, тот превращается в нейтрон плюс позитрон.
755 586758
Посоветуйте книгу по истории философии, плиз. Что-то годное, только не Бертрана Рассела.
756 586761
>>86758
Критерии годности какие?
757 586765
>>86761
Чтоб поменьше говниной воняло естественно
758 586770
>>86765
Тогда тебе придется помыться перед чтением
759 586787
Почему биоооужие невозможно?
Все сука стремится к равновесию дпже шмели в лабораториях
760 586796
>>86787
Почему невозможно? Чем тебе вирусы не биооружие?
761 586799
>>86796
Он имел ввиду биоружие из рассказах кремлевских старцев.
Типо берем зверушку, заклинаем ее бациллами, выпускаем к врагам. Зверушка контачит врагов, бациллы перекидываются на них и они от этого дохнут от болячек. Но раз бациллы заговоренные, то они не берут четких пацанов.

Из вирусов плохое оружие, ибо они эволюционно самообезвреживаются, приходя к устойчивому равновесию с популяцией.
Вирусы не избирательны и не дают надежных результат.
Очень высока вероятность, что применяющий первый пострадает от бяки, а цель вообще никак не пострадает.
762 586806
https://www.youtube.com/watch?v=SNseRrRVjR0

А что вы думаете по поводу бесплатной фундаментальной науки и её выхлопа а моменте?
763 586807
>>86806

>фундаментальная наука


Бесплатно дала бомжу?
764 586810
>>86806
Красиво стелет про склонность к онкологии определенных этнических групп в зависимости от мутации генома.
765 586811
>>86806

> дочь Путина


У меня у одного отторжение слушать её хотя по сути к действиям отца она не имеет отношения?
766 586812
>>86811

>отторжение


Отторжение, не отторжение, а работать будешь рабочая шлюха☝️
☝️☝️☝️
767 586813
>>86811
Хз.
На фоне сына байдена там хотя бы есть что слушать.
768 586815
>>86806
Странное у неё видение по поводу удорожания технологий.
Взять тот же ферментативный катализ, например.
До него получение некоторых веществ было не просто дорого а в промышленном объеме невозможно.
Да он стоит на каком-то этапе дороже, чем в средних веках заячьим пометом обмазаться, но он открывает перспективы, которые заячьему помету в принципе не доступны.
Плавает, короче, студентка.
770 586839
Почему живые существа имеют уникальный ДНК?
Если ДНК - это молекула, то они ведь должны быть идентичны друг другу, скажем как молекула воды или углекислого газа, разве нет?
Как тогда работает тест?
771 586840
>>86839
Мутация и эволюция же. Это классика! Это знать надо!
772 586841
>>86840

>Мутация


И всё равно не понятно. Если структуру ДНК изменить, то это уже будет не днк, правда ведь?
Или энта молекула может обрастать некоторыми уникальными свойствами? В чём они заключаются?
773 586845
>>86841
ДНК это про состав, а состав у них у всех примерно одинаков. Если структуру воды или углерода изменить, они перестанут быть?
Можешь считать ДНК видом молекул если тебе так проще.
774 586846
>>86841
Структура такова, что позволяет один из ее частей(азотистые основания) имеет произвольные дискретные вариации. Их можно в любом порядке располагать, это не изменить структуру ДНК.
Мутации это изменение закодированной последовательностей, на структуру это не влияет. Хотя ДНК имеет комплементарную пару и она чувствительна к последовательности, но биомашины автоматом фиксят это.
775 586847
>>86839
Если есть дерево, то любое дерево должно быть идентичны друг другу, разве нет? Иначе не дерево, правда ведь?
776 586850
>>86847
дерево - не молекула
777 586867
>>86839
Газ это ведь молекулы, хули тогда говорят что есть штук 50 разных газов в таблице Менделеева, они что, ебанутые? Раз газ это молекулы, то должен быть всего один газ!
778 586886
>>86867
Газ - это состояние вещества.
Дезоксирибонуклеиновая кислота имеет молекулярную структуру, а это значит что взяв один образец у разных живых организмов, они должны 100% совпадать.
Мой глупый вопрос к тому, почему ДНК-тест называют ДНК тестом, если это по сути сравнение молекул воды или метана?
Что на самом деле тестируется, и на каком уровне? Ген, теломера, хромосома?
779 586887
>>86886
Разрешите доебаться.
ДНКой чисто из определению называется любая молекула состоящая из сахара дезоксирибозы, кислотной функциональной группы(фосфорная) и нуклеотид - азотистые основания.
Так ДНК можно назвать целый класс молекул, который не встречаются в естественном виде, ибо кислотный остаток и азотистое основания можно заменить на другие, вариаций куча.
780 586892
>>86886

> они должны 100% совпадать


ДНК это полимер, она собирается из нескольких мономеров (нуклеотидов), то в какой именно последовательности эти мономеры соединяются в полимер и кодирует генетическую информацию. У разных молекул днк на одном и том же участке могут быть разные последовательности нуклеотидов чем они и отличаются. Процес определения этой последовательности называют секвенированием.
781 586897
>>86886

>Дезоксирибонуклеиновая кислота имеет молекулярную структуру


Чем это отличается от слова "газ"?

>они должны 100% совпадать.


То что какой-то шизик-долбаёб у себя в голове нафантазировал что что-то ему "должно", ну хули, наверное так оно и есть)) но чёт не совпадает.
782 586902
>>86886

>почему ДНК-тест называют ДНК тестом


Потому что это днк-тест
Во-первых, тест газов вполне логично называть тестом газов. Тестируют ведь газ.
Во-вторых, днк разные, у тебя системная ошибка, ты не просто нафантазировал себе другое опредление у слова сам, вместо того чтобы разобраться что оно означает, а ты в принципе не понимаешь что бывают общие очень общие определения, которые большую разновидность всего разного собою описывают. А бывают специальные определения, которые только одну какую-то конкретную вещь означают.
Ты думаешь что все определения специальные, и описывают только одну какую-то конкретную вещь.
783 586908
>>86655
А чё бы нет? По меркам вселенной она существует ничтожную долю времени от всего возможного.
784 586911
>>86670
У тебя ж там результат на картинке.
Доказательств никаких нет, а движение к центру вообще как бред звучит.
Так-то в чёрной дыре растяжение пространства, а не просто движение. Плюс скорее всего это вообще не имеет смысла, такая концепция, т.к. время бесконечно замедленно, т.е. внутренностей у чёрной дыры нет и никакого движения к центру нет, потому что чёрные дыры одновременно и не существует и существуют при этом.
785 586920
Посоветуйте, статьи/видосы/подкасты о проблеме демаркации, научности и истинности. Пока отобрал только сочинения отдельных авторов типа Поппера и Фейерабенда, но хотелось бы на общий анализ посмотреть и дополнительно узнать о истории смены этих критериев в научной среде.
786 586930
>>86920
Курс философии науки на степике или типа того
787 586953
возможно ли предсказать через сколько десятилетий CRISPR-Cas9 будет способна лечить уже психические болезни?
788 586954
>>86953
Ты трясущийся тупой тревожник? Или ещё каких-то "болезней" себе нафантазировал?
789 586962
>>86954
Он указал криспс так что или ОКР или шизофрения. Это самые популярные психическое заболевания, которые зависят от генетики.
790 586976
>>86962
Самые популярные выдуманные заболевания зависящие от генетики, лол.
791 586977
>>86976
А ты никогда от ОКР не страдал по всей видимости. Ну, ничего, родился у тебя ребенок с шизофренией по другому заговоришь.
792 586981
>>86977
😱😱😱
793 586990
Да когда уже сингулярность блеать?!
794 586994
>>86990
Это типа когда всё суперэффнктивно будут делать роботы и ии в неограниченных(условно) количествах, а таких как ты рабочих закинут в биореактор как бесполезный биомусор?
795 586996
>>86994

>роботы и ии в неограниченных(условно) количествах


>а таких как ты рабочих закинут в биореактор как бесполезный биомусор


А зачем роботам делать что-то в неограниченных количествах, если это некому будет потреблять?
796 587000
>>86996

>если это некому будет потреблять?


Ну смотри, сейчас ты тупорылый опущенный хуесос работаешь за околобесплатно, а потребители кроме тебя несмотря на это находятся.
797 587003
>>87000
А кто сказал, что золотой миллиард тоже не покромсают?
798 587028
>>86996
Роботы и станут новыми потребителями, а людишки станут нинужны.
799 587029
>>87028
Кто роботам цены на электричество в розетке будет устанавливать? Нет, если в них проснётся квалиа, то они это поймут и без людей подключаться к солнцу, но пока ключи от Чернобыля у людей, роботам не о чем переживать
800 587031
>>87029
Приказываешь своим роботам построить электростанцию. Профит.
image.png1,1 Мб, 4963x3508
801 587036
>>84346 (OP)
Как распределяется энергия в электросети?
Допустим есть разные потребители с разной мощностью на разном расстоянии от источника, подключенные ЛЭП с разным сопротивлением.
При недостатке энергии все будут получать одинаковое количество или одинаковый процент от мощности или на это как-то еще влияют параметры ЛЭП, подстанций, РУ?
При избытке энергии и наличии нескольких источников, все источники будут загружены одинаково или на одинаковый процент от мощности или на это как-то еще влияют параметры ЛЭП, подстанций, РУ?
Интересуюсь с целью симуляции энергосистемы в игре.
802 587046
>>87036
На ютубе ищи "Электроликбез".
803 587059
>>87036
Чтобы правильно рассчитывать сети тебе надо отдельную специальность закончить в институте.
Сам расчет относительно простой, но там есть нюансы с ситуациями, когда что-то выходит из строя.
Наверняка есть мод для факторио.
804 587072
>>87046
Я бы предпочел информацию в текстовой форме
805 587096
У графита высокая проводимость. Много свободных электронов?
Тогда почему он не считается металлом? И проводимость есть, и свободных электронов много, блестит как металл
Похоже по всем параметрам на хрупкий металл без свойства "ковкость", ну хули, многие металлы не ковкие.

Выделяют ещё вроде какую-то особую металлическую связь, а у графита выделяют связь углерод-углерод, неужели это настолько разные вещи?
806 587099
>>87096
"Металл" это оценочная характеристика химического элемента. Графит не химический элемент, это модификация углерода.
807 587112
Почему нельзя сделать чисто оптические без электрики чернобелые очки?
808 587113
>>87096
Про металл уже сказали.
Ток могут проводит так же полупроводники и ионные жидкости.
Проводимость графита не описывается в рамках модели зонной проводимости, там уже нужные другие модели.
Металлическая связь это условность химиков. Как вообще любые химические связи, ибо они не хотят по честному считать волновую функцию всех электронов.

Если уж хочешь простое объяснение проводимости графита, то у него электроны образовывают ароматическую связь, которая при больших масштабах представляется как шатающихся заряд вдоль слоя.
809 587114
>>87112

> чернобелые очки


Если я правильно понял что ты подразумеваешь под "чернобелыми очками" то нельзя потому что не нашли какого-то физического процеса который напрямую позволяет уровнять мощность излучения по всем длинам волн. Тебе нужна какая-то хуйня которая под каждую длину волны будет "смотреть" на ее мощность и в зависимости от этого либо усилять либо уменьшать чтобы все усреднить. Это очень спецефическое воздействие.
810 587116
Детский вопрос к биологам. Наверное платина. Раки и крабы чувствуют боль когда их варят в кипятке ? А когда крабам отрывают конечности ? Забавно что в моём детстве всегда про раков спрашивали, а сейчас чаще про крабов.
811 587120
>>87113

>


>Если уж хочешь простое объяснение проводимости графита, то у него электроны образовывают ароматическую связь, которая при больших масштабах представляется как шатающихся заряд вдоль слоя.


Т.е. свободных электронов как в металлах нет? Класс, спс.
812 587121
>>87116
Не так давно признали у членистоногих сознание, т.е. наличие чувств. Теперь их положено убивать, прежде чем кидать в кипяток.
Существует ли время 813 587135
Суп срач. Неофит итт. Насмотрелся научпоп видосов, и нейроны в мозгу зашевелились. Вот стало интересно. Постараюсь изложить тезисно. Если в тезисах заблуждаюсь - поправь.

Время - это энтропия. А точнее, это однонаправленное движение энергии в пространстве.
На время влияет скорость и Гравитация. Если тело ускоряется, то время для него замедляется. Также, чем больше массы у тела, тем медленнее для него течет время.

Тело обладающее массой не может достичь скорости света, т.к. для этого нужно бесконечное количество энергии.

Учитывая все вышесказанное, можно ли заключить, что время "замедляется" потому, что, например, электронами на орбитах атомов становится "тяжелее" вращаться вокруг орбит. Поэтому замедляются все процессы в микромире, а в следствии этого и в макромире?

Получается ли из этого, что время не вещь в себе, а лишь следствие "перемещения" энергии в пространстве?
814 587136
>>87135

>Время - это энтропия.


Время это время, энтропия это энтропия. Разные категории, связать у тебя их не выйдет.

>Учитывая все вышесказанное, можно ли заключить, что время "замедляется" потому, что, например, электронами на орбитах атомов становится "тяжелее" вращаться вокруг орбит. Поэтому замедляются все процессы в микромире, а в следствии этого и в макромире?


Дефайн "тяжелее"?

>Получается ли из этого, что время не вещь в себе, а лишь следствие "перемещения" энергии в пространстве?


А пространство это вещь в себе? А энергия это вещь в себе?

Прекращай смотреть научпоп. Лучше потрать лет 5 на обучение, тогда эти вопросы сможешь уже рассмотреть получше. Научпоп делает тебя глупее
815 587137
>>87135
Есть много эффектов, и слишком уж одинаково всё замедляется, такое ощущение что.. ну это довольно существенный намёк что это всё-таки какая-то вещь в себе.
Ну понял да, если бы был только один какой-то эффект и механизм "замедления", то логично что всегда одинаково всё замедляется. Но там вариаций много, и вроде как всё абсолютно одинаково замедляется, а когда много разных и всё одинаково, это уже странно.
816 587143
Сап двач. Насмотрелся видосов по микробиологии, возник вопрос. В каждой клетке человека в ядре содержится весь геном. В процессе митоза дублируется весь геном. А как тогда клетка узнает, что ей необходимо разделиться на идентичную клетку, а не на любую другую рандомную клетку? Или процесс деления каждой разновидности клеток создан так, что ни на что другое она поделиться не способна?
817 587145
>>87143
Клетка собирает сигналы из окружающей среды. Химические, электрические. На их основе решает куда двигаться и когда в кого делиться.
Некоторые колонии бактерий тоже обмениваются сигналами, они как разреженное многоклеточное.
818 587150
>>87135

>Получается ли из этого, что время не вещь в себе, а лишь следствие "перемещения" энергии в пространстве?


Время это отражение течения процессов (изменений) окружающего мира в нервной системе организма. Наблюдаемые цикличные процессы (оборот земли, оборот вокруг солнца) человек абстрагирует до понятия время (день, год).
Да, время это не вещь_в_себе, время это следствие фиксации (наблюдения) процесса (изменений, событий) другим процессом.

>Учитывая все вышесказанное, можно ли заключить, что время "замедляется" потому, что, например, электронами на орбитах атомов становится "тяжелее" вращаться вокруг орбит. Поэтому замедляются все процессы в микромире, а в следствии этого и в макромире?


Не готов утверждать, что именно и на каком уровне замедляется. Но в общем примерно так и есть. Скорость и гравитация замедляют процессы, поэтому в «парадоксе близнецов» путешественник по возвращении попадает «типа» в будущее, а часы на спутниках приходится корректировать.
Снимок.PNG28 Кб, 1119x408
819 587151
А существует ли вообще пример такой бесконечной последовательности целых чисел f(n) для которой доказана конечность количества простых чисел в ней?
820 587152
>>87151
f(n) = 2 * n (n - натуральное число) например
1706001636268.jpg69 Кб, 600x600
821 587153
822 587154
>>87152
Ну ладно. формально есть один пример. Просто интересует что-то не такое тривиальное - дохулион простых чисел в некой последовательности, а потом раз и после N-го номера ни одного больше не попадётся и это доказано.
823 587156
>>87154

> Ну ладно. формально есть один пример.


Какой пример? Он привёл последовательность чётных чисел. В ней вообще нет ни одного простого числа.
824 587157
825 587158
>>87157
Ах ты бля. Ok. В натуре.
826 587160
>>87158
Я тут мимо проходил и хотел просто спросить у тебя один вопрос -
Ты такое тупорыло чмо, почему ты вообще считаешь что имеешь право разевать свою тупорылую ротешу и чего то там из нее кукарекать на людей заведомо умнее тебя?
827 587161
>>87160
Иди нахуй, порк!
828 587174
Реально провести "генный реверс-инженеринг" птиц и вырастить жизнеспособного цыплёнка курицы, например, без перьев, но в чешуе? С зубастой пастью вместо клюва? С длинным хвостом?
Почему так не делают? Уверен, домашние динозавры будут пользоваться огромным спросом независимо от цены. Пусть будут стерильными, если зоошизики боятся, что они сбегут и расплодятся.
829 587180
>>87136
"Тяжелее", значит, что скорость света константное, и чем ближе к ней приближается тело, тем больше энергии требуется для любого движения.
>>87150
Просто, если это так, то почему об этом научпоперы не говорят? Я такие тезисы никогда не слышал. Получается и путешествия во времени тогда невозможны, как невозможно провернуть фарш назад.
image.png123 Кб, 274x184
Помощь отечественной науке 830 587181
Такое дело.

Есть желание оказать посильную помощь единственной достойной сфере человеческой жизнедеятельности - науке, желательно отечественной.

Сам умом я, как вы понимаете, не вышел, так что непосредственное участие исключается.

Но могу, например, донатить какую-то не слишком крупную сумму денег, или отдать пека под вычисления.

Я погуглил на эту тему, и чёт ничё вменяемого не нашёл.
Из тех проектов, где нужна помощь волонтёров, гуглятся только всякие натуралистические проекты по каталогизации живой природы, а беготня с сачком за бабочками и сбор гербария мне совсем не привлекают.

У меня больше стоит на разного рода технику, физику, космос, вот это всё.

По донатам тоже ничё не понятно.

Может подскажете приличные варианты?
Либо донаты (но только так, чтобы они не осели в карманах мойшенников), либо какая-то условно-пассивная натуральная помощь вроде выделения машинного времени.
831 587182
>>87174
Нет, мы ещё очень далеко от этого.
832 587187
Я уже задавал здесь этот вопрос, никто не смог ответить. Задам ещё раз.
Когда проводят соцопросы, то в норме до 80-90% респондентов либо отказываются отвечать на вопросы, либо отвечают не на все (например не о себе), и их ответы не засчитываются. Нвероятно, но людям нахуй это не надо.
Вопрос: каким, блядь, образом, социологи претендуют на отражение "общественного мнения", если с ними никто, сука, не разговаривает?
Ну нет у них репрезентативной выборки, у них выборка на 100% состоит их странных людей с отклоняющимся поведением.
833 587188
>>87187
Я более того скажу, большинство соцопросов до сих пор это телефонные обзвоны. Ну, ты знаешь, когда тебе звонят незнакомые люди и такие, это соцопрос.

И говорит какая-нибудь старушка, либо женщина с не очень приятным голосом. Можете ли вы уделить 15 минут своего времени, чтобы ответить на вопросы, про то, какие последние телеканалы вы смотрели. И если твой ответ, их не удовлетворяет, то есть ты вместо того, чтобы говорить конкретно заданные фразы. Говоришь, я не помню, тебя перебивают и говорят, отвечайте строго по тому сценарию, который мы вам проговорили.
834 587190
>>87187

> каким, блядь, образом, социологи претендуют на отражение "общественного мнения"


Какое есть, другого не подвезли. Ты как ребёнок, который узнал, что мистер пропер не прилетает и не делает уборку, да и само СУПЕРсредство ни чем не отличается от дешевого, только в красивой таре и с вкусной отдушкой.
А вообще, понятие «общественное мнение» и не подразумевает 100% опроса, это мнение ОБЩНОСТИ, т.е. достаточно опросить часть, чтобы иметь представление об ОБЩЕМ состоянии дел.
835 587191
>>87187
Так ведь соцопросы по сути опрос на лояльность ангажированной группы, а не что-то уровня маркетинговых исследований. Ценность не в их результатах, а в том факте что они проводятся и для кого. Этакий сорт оф ритуал.
836 587193
>>87190
Ты не прав. Чтобы иметь представление об общем состоянии дел нужно спросить часть общества это верно, но окрашиваемая часть общества должна состоять из всех групп населения, а не исключительно шизы готовые говорить про войну по телефону с незнакомым человеком.
>>87191
А как же "независимые" соц. опросы?
837 587194
>>87193

>А как же "независимые" соц. опросы?


Тоже самое. Их "независимость" только в другой целевой группе.
838 587196
>>87193

> а не исключительно шизы готовые говорить про войну по телефону с незнакомым человеком.


Я же тебе написал: что имеем, то имеем.
Что ты предлагаешь? Запретить опросы. Или. Законодательно принудить граждан обязательно проходить опросы под детектор лжи (а то шутники будут искажать данные и тебе опять не понравится), с уголовной ответственностью за отказ и за ложь.
К чему твой срыв покровов?
🌑 Невероятный физический эффект движение тела против течения воды #Shorts Игорь.mp44 Мб, mp4,
720x1280, 0:38
839 587219
У кого спираль есть? Сделайте так же.
840 587232
>>87219
Как мне казалось.
В принципе это всё, что вам необходимо знать о современном образовании инженеров.
841 587233
>>87219
Я так тоже показал однажды один эффект одному доктору технических наук.
Дословно он сказал - этого не может быть, потому что Я не понимаю как это работает.
842 587234
>>87219
Не верю
843 587242
>>87232
>>87233
Тупые дауны/шизы/фантазёры
Он потом сам про леску говорит
Ахаха, как же много этот чувак долба6ов словил этим приёмом.
844 587245
>>87242
На 11 секунде леску видно
845 587250
Почему индусня постоянное кей-кает? Они сами не понимают что это пиздец как нелепо и раздражает?
846 587255
>>87242
Речь вообще не об опыте шла, а о его пиздобольстве.
847 587276
Поясните за автопилот, особенно фирму Ольги Усаковой - Cognitive technology.

Неужели настолько крутая, что аж сам Илон Маск с его автопилотом курит нервно в сторонке?
848 587286
>>84346 (OP)
Здесь есть тред ботаники?
849 587288
>>87276
Звучит как хуйня какаето,и автопилот чего¿
Транспорта?
JstgP5U6kKg.jpg135 Кб, 720x720
850 587317
1. Дети проводят за видеоиграми, изображающими убийства, сотни часов в год. Годами сидят в виртуальных коробках Скиннера и получают поощрения за имитацию убийств, получают удовольствие от изображений агонии, крови, оторванных конечностей. Года за годом, с раннего детства.
2. Никакого влияния такого времяпрепровождения на их психику наука не фиксирует.
Внимание вопрос: разве отсутствие данных само по себе не говорит нам о чём-то важном? Ну например либо социальная "наука" - бесполезное говно, либо тысячи часов научения и цмилилоны циклов подкрепления ни на что влияют.
851 587318
>>87317
1. Волки всю жизнь убивают живых существ
2. Волки всю жизнь играют друг с другом имитируя укусы

Даже волки способны отличать игру от жизни, с чего люди бы не могли?
852 587319
>>87318
Видеоигры это не игровое поведение животных и людей, вообще ни разу, держу в курсе.
853 587321
>>87319
Уже давно всё исследовано, и видеогры не повышают насилие, а в некоторых случаях даже понижают, потому что позволяют выплеснуть агрессию в игре вместо реального мира.
854 587323
>>87321

>видеогры не повышают насилие


1. Вопрос был не про "повышение насилия", а про их воздействие на психику детей.
2. Доказательств, что видеоигры "не повышают насилие" - нет. Удивительно, понимаю.
855 587324
>>87317
>>87318
Многомерные внематериальные сущности гаввах качают и им не важно игра или не игра. Важно состояние ума и так сказать у многих геймеров бывает сильно бомбит не хуже "реального" убийства.
856 587326
>>87317

>Никакого влияния такого времяпрепровождения на их психику наука не фиксирует.


Конечно не фиксирует, так как получается нормальная лысая агрессивная обезьяна. В своём становлении молодая особь проходит необходимый этап древней биопрошивки подавлять другие виды и доминировать в стае себеподобных, оттуда идет детская и подростковая жестокость, довидеоигровую эпоху детишки и подростки ходили на стрелки дворами, пиздились в шкалке, вешали в подвалах котят, резали и надували лягушек на болотах, отрывали тараканам ноги и так далее, не повсеместно, не все, но это всегда было свойственно части детенышей человека разумного, всё это видимо нужно для деконструкции угроз животного окружающего созревающей психикой и нечто вроде такого, хз, я не психолог и вообще не гуманитарий, но давно задумываля об этом. А ты задумайся вместе с психологами и социологами, что если человека с детства изолировать не только от насилия, но и от любой информации о насилии в отношении своего и других видов, включая всякие литературные произведения, какой получится человек?
857 587327
>>87326
Дети не надувают лягушек по два часа в день каждый день на протяжении 10 лет подряд.
И видеоигры это не агрессивное поведение. Это обучение. Картинка - действие - поощрение.
Мне предлагается поверить, что ребёнок тратит тысячи часов на обучение в коробке Скиннера и это никак на него не влияет.
858 587329
>>87327
Почему не влияет? Влияет, просто не так, как ты думаешь. Ребенок узнает, что на WASD обычно передвижение в играх. Узнает, что на букву R – перезарядка оружия, а на букву G – бросок гранаты. И тому подобные вещи. Он учится объединять движения левой и правой руки для достижения нацеливания на противника. В общем, он лучше начинает играть в игры. Из-за того, что он играет в игры. Ребенок знает чем НПЦ отличается от человека и почему убивать НПЦ можно, а людей нет. Потому что он понимает различия между реальной жизнью и игрой. Если же не различает, то это болезнь, но с таким же успехом он может не отличать книжный мир от реального и верить, что если спрыгнуть с крыши на метле, то он полетит.
859 587331
>>87329
Ребенок учится играть в шутеры за 10 часов.
Остальные 8000 часов никак на него не влияют? Миллионы повторений цикла стимул - действие - поощрение - и ничего.
Это пиздец какое мощное допущение, практически фантастика.
860 587332
>>87331
Вызывает зависимость, да. Сильную ли? Нет, не сильную. Хотя некоторые люди более склонны именно к игровой зависимости, но из моего обывательского опыта таких меньшинство.
861 587334
>>87332
Очередной гениальный тейк. Дети тратят тысячи часов чтобы сидеть на месте, пыриться в экран и постоянно повторять однотипные действия. Это зависимость! Невероятно.
862 587335
>>87334
Ты отрицаешь существование игровой зависимости?
863 587340
>>87331

>Остальные 8000 часов никак на него не влияют?


Первые 4000 приобретаются полезные для игры и для жизни навыки. Последние 4000 теряются полезные для жизни но не для игры навыки.
864 587344
Почему птиц на проводах не убивает ток??? я в курсе, что он убьет если наземное животное притронется и создаст разнсоть потенциалов, которая будет идти черещ него. Но ведь когда птица сидит на проводе, ток все равно ВХОДИТ в нее через лапки, просто никуда не идет дальше для прожарки. Так почему не убивает
865 587345
>>87344
Он не входит. У птицы нет потенциала, чтобы в неё шел ток, так что он спокойно себе движется в направлении потенциала на другом конце провода.
866 587346
>>87345
А если человек повиснет на проводе ЛЭП, как на турнике – его убьёт?
867 587347
>>87331
Чел, твои вопросы ни о чем. У тебя вся модель неправильная - с какими-то адовыми детьми-психопатами, которые бесконечно гоняют в человеконенавистнические стрелялки.

Во-первых, процент людей, предпочитающих исключительно один жанр компьютерных игр, невероятно мал. Это характерно только для нишевых вещей типа симуляторов поездов и проч., которыми увлекаются лютые аутисты и машинисты на пенсии. Как правило, люди играют в игры нескольких различных жанров, среди которых шутеры занимают очень скромное место.

Во-вторых, 70% игроков вообще никогда не играют в шутеры. Из тех 30%, которые в них регулярно играют, только несколько процентов особо упорных адреналиновых маньяков набрали к совершеннолетию целых 8000 часов игры. Остальные чуть-чуть побегали по локациям в период с 12 до 17 лет - и, немного повзрослев, завязали с этим низкопробным собачьим кайфом.

В-третьих, шутеры-симуляторы - то есть ебучие имитаторы жизни солдата-спецагента-маньяка-насильника, приближенные к реальности с ее солдафонством, гноем и окопным говном - вообще никогда не взлетают. Потому что на самом деле людям очень неприятен подлинный реализм. Популярные шутеры - это тщательно спроектированные и подсвеченные в нужных местах ландшафты и коридорчики с визуально-акустической эпилептической гиперстимуляцией и псевдофизикой, в которых все красочно взрывается, пердит, булькает и разбрызгивает по сторонам невозможные ведра кровищи. То есть популярные шутеры - это МУЛЬТИКИ, они намеренно абстрагированы до уровня схем, в которых одни моменты реальности подчеркнуты, а другие нарочно скрыты.

В-четвертых, целью современных игр является не дрессировка с вознаграждением, а вовлечение игрока в состояние потока, которое отличается от мучений голубя в ящике Скиннера примерно так же, как палец отличается от жопы. В учебных курсах типа "Game development essentials" все это подробно объясняется в главах про игровую механику. В состоянии потока человек не воспринимает реальности, он в полубессознательном состоянии движется по градиенту возрастания комплексности игровых паттернов, подбирая ключики к постоянно меняющимся и усложняющимся замочкам, постоянно приобретая все новые и новые способности и альтернативные возможности для осуществления свободного выбора. Как шахматист видит не конкретные фигуры на доске, а динамические паттерны угроз-возможностей и потенциальные ветви дерева игры - так и рассекающий на части и нагибающий врагов игрок видит только абстрактные динамические игровые паттерны, а не "существ", которых нужно "убить".

Короче, для начала подучи матчасть и только потом задавай вопросы.
867 587347
>>87331
Чел, твои вопросы ни о чем. У тебя вся модель неправильная - с какими-то адовыми детьми-психопатами, которые бесконечно гоняют в человеконенавистнические стрелялки.

Во-первых, процент людей, предпочитающих исключительно один жанр компьютерных игр, невероятно мал. Это характерно только для нишевых вещей типа симуляторов поездов и проч., которыми увлекаются лютые аутисты и машинисты на пенсии. Как правило, люди играют в игры нескольких различных жанров, среди которых шутеры занимают очень скромное место.

Во-вторых, 70% игроков вообще никогда не играют в шутеры. Из тех 30%, которые в них регулярно играют, только несколько процентов особо упорных адреналиновых маньяков набрали к совершеннолетию целых 8000 часов игры. Остальные чуть-чуть побегали по локациям в период с 12 до 17 лет - и, немного повзрослев, завязали с этим низкопробным собачьим кайфом.

В-третьих, шутеры-симуляторы - то есть ебучие имитаторы жизни солдата-спецагента-маньяка-насильника, приближенные к реальности с ее солдафонством, гноем и окопным говном - вообще никогда не взлетают. Потому что на самом деле людям очень неприятен подлинный реализм. Популярные шутеры - это тщательно спроектированные и подсвеченные в нужных местах ландшафты и коридорчики с визуально-акустической эпилептической гиперстимуляцией и псевдофизикой, в которых все красочно взрывается, пердит, булькает и разбрызгивает по сторонам невозможные ведра кровищи. То есть популярные шутеры - это МУЛЬТИКИ, они намеренно абстрагированы до уровня схем, в которых одни моменты реальности подчеркнуты, а другие нарочно скрыты.

В-четвертых, целью современных игр является не дрессировка с вознаграждением, а вовлечение игрока в состояние потока, которое отличается от мучений голубя в ящике Скиннера примерно так же, как палец отличается от жопы. В учебных курсах типа "Game development essentials" все это подробно объясняется в главах про игровую механику. В состоянии потока человек не воспринимает реальности, он в полубессознательном состоянии движется по градиенту возрастания комплексности игровых паттернов, подбирая ключики к постоянно меняющимся и усложняющимся замочкам, постоянно приобретая все новые и новые способности и альтернативные возможности для осуществления свободного выбора. Как шахматист видит не конкретные фигуры на доске, а динамические паттерны угроз-возможностей и потенциальные ветви дерева игры - так и рассекающий на части и нагибающий врагов игрок видит только абстрактные динамические игровые паттерны, а не "существ", которых нужно "убить".

Короче, для начала подучи матчасть и только потом задавай вопросы.
868 587348
>>87346
С ним ничего не будет пока он не коснется земли. У земли больше потенциал чем у чего бы ни было ещё, так что ток пойдет через человека к большему потенциалу (к земле) и человек сгорит.

Думаешь почему требование при работе с электротехникой стоять на резиновом коврике? Чтобы ты для тока был неинтересен, так как стоишь на материале, который изолирует тебя от земли, так как не имеет токопроводности.
moke-working.gif1,9 Мб, 250x172
869 587349
Привет анон! Как называется использование эволюционных рефлексов/механизмов организма себе на пользу?

По типу умывания холодной водой нырятельный рефлекс млекопитающего, смотреть на солнце 1-2 минуты утром как проснешься, питаться раз в сутки для мобилизации сил организма, не светить себе синим светом в ебало перед сном, и тд.

Слышал у Хьюбермана на подскате, что то типа "Evolutionary neuromethods" или что то такое.
870 587351
>>87349
Термин, который вы описываете и который упоминался Хьюберманом, скорее всего относится к "эволюционным методам нейрохакинга" или "эволюционным нейрометодам". Этот подход включает использование понимания эволюционно сложившихся рефлексов и механизмов нашего тела для улучшения здоровья, благополучия или повышения эффективности. Примеры, которые вы привели, как холодные ванны для стимуляции нырятельного рефлекса или контроль светового режима для регулирования циркадных ритмов, являются типичными примерами этого подхода.

Эти методы часто основываются на идее, что, воспроизводя условия, в которых эволюционировал человек, мы можем стимулировать организм функционировать более эффективно и здорово. Это включает в себя различные аспекты: от питания и физической активности до сна и светового воздействия.
871 587355
>>87345
🤯
а полчему не входит? у живых существ же есть электричество внутри
872 587356
>>87355
Представь себе, что ты электрический ток. У тебя есть выбор пойти по поводу у которого сопротивление 0,01 Ом или по человеческому телу, у которого сопротивление 500 Ом. Куда как ты думаешь пойдет 99,9 % тока? Правильно по поводу. У резинового коврика сопротивление вообще 9999999999 Ом, в его сторону пойдет ещё меньше тока.
873 587357
>>87348
Кстати, в гуро-тредах бывали видосы, как зацеперов хуячило током без контакта с проводом.Типа электрический пробой.
874 587358
>>87351

Не оно но близко. Спасибо гопота.
875 587362
>>87347

> Популярные шутеры - это тщательно спроектированные и подсвеченные в нужных местах ландшафты и коридорчики с визуально-акустической эпилептической гиперстимуляцией и псевдофизикой, в которых все красочно взрывается, пердит, булькает и разбрызгивает по сторонам невозможные ведра кровищи. То есть популярные шутеры - это МУЛЬТИКИ, они намеренно абстрагированы до уровня схем, в которых одни моменты реальности подчеркнуты, а другие нарочно скрыты.


В такие шутеры действительно много не наиграешь. Залипают в сетевые пабги, контрстрайки, гта и т.п. вот там реально сутками сидеть.
876 587363
>>87362

> В такие шутеры действительно много не наиграешь.


В юности зависал в Quake и Unreal. И не один я. Так что не гони
877 587365
>>87363
Тоже залипал, не спорю. Но тогда не было альтернативы. Сейчас в офлайн игры залипать ну такое себе, онлайн на порядок интересней.
878 587369
>>87347

>целью современных игр является не дрессировка с вознаграждением


>для начала подучи матчасть


Хуйчасть, придурок. Откуда вы лезете, блядь.
879 587372
>>87365
Смотря для кого. Я пробовал онлайн, и максимум играю в них как в оффлайн. Взять диабло, он для меня от сингла отличался только возможностью разменов шмота. А так ну играешь и играешь, гавкаешь в чат сокращения на игровом сленге как дрессированный обезьян и всё. Это нахуй ничего не значит. Что-то "околоспортивное" кс там или старкрафт, на это нужен талант и постоянный наигрышь часов, а в остатке только нервы и мозгоебизм. Мне сюжетки, песочницы, и метроидвании в разы больше нравятся любого сетевого дрочева.
880 587384
Белый самец 100iq спаривается с белой самкой 100iq, у них рождается ребёнок 100iq, верно? Теперь белый самец 100iq спаривается с чёрной самкой 100iq - будет ли у ребёнка iq меньше?
881 587385
>>87384
Ты не учёл генетический параметр "вероятность появления у особи потомства с iq размером Х"

Узнать его можно например по предкам.
Если в роду все были дауны 70-90 iq, например 10 поколений, и у них вдруг родился ребёнок 120iq, то у этого ребёнка конечно будет повыше шанс появление ребёнка с iq выше чем 70-90, чем если бы был род 10 поколений с iq70-90 и его 11-й представитель завёл бы ребёнка.
Но например скорее всего эта вероятность намного меньше чем если был род 10 поколений 120iq, у них родился вдруг 80iq дебил и чтобы у этого 80iq дебила родился ребёнок 110-120iq. Это более вероятно будет намного мне кажется.

Так что важен РОД.

А конкретно по поводу твоего вопроса-я ни разу не встречал информации, которая бы указывала что метисы получаются какими-то особо тупыми по сравнению с родителями, если конечно учесть род, ну или просто большую выборку усрелняющую.
Так что да, В СРЕДНЕМ, 100+100 будет давать 100, и не важно метис или белый+белый или чёрный+чёрный.
882 587386
>>87384
Из-за концентрации содержания мелатонина в коже IQ будет ниже? Чего блять?
883 587391
>>87386
В целом такова статистика.

>в целом


Если не смотреть про что конкретно он спрашивал.
884 587392
>>87384

>https://en.wikipedia.org/wiki/Regression_toward_the_mean


>The concept of regression comes from genetics and was popularized by Sir Francis Galton during the late 19th century with the publication of Regression towards mediocrity in hereditary stature.[8] Galton observed that extreme characteristics (e.g., height) in parents are not passed on completely to their offspring. Rather, the characteristics in the offspring regress toward a mediocre point (a point which has since been identified as the mean). By measuring the heights of hundreds of people, he was able to quantify regression to the mean, and estimate the size of the effect. Galton wrote that, "the average regression of the offspring is a constant fraction of their respective mid-parental deviations"


Как мы помним, Гальтон - нацист, так что IQ будет таким же. И вообще IQ не работает. Генетика - лженаука. В современном мире главное - эмоциональный интеллект! Нацизм - не пройдёт. Я люблю негров!
885 587396
>>87384

>Теперь белый самец 100iq спаривается с чёрной самкой 100iq - будет ли у ребёнка iq меньше?


Неизвестно, их iq может быть закодировано разными генами. Ребенок может родиться с как меньшим, так и с большим iq.
886 587397
>>87396

>их iq может быть закодировано разными генами


Смешивалка же смешивает рандомно, а интеллект наверняка кодирует очень большое количество генов.
Т.е. с высокой вероятностью мозг будет очень сильно перемешен до равномерности.

Хотя вот конечно бывают что дети прям на 100% вылетый один из родителей, бывает почти на 100% другой, ну и соответственно 50 на 50 и другие соотношения.
Хрен короче знает как эта смашивалка работает.
887 587401
>>87397

> Хрен короче знает как эта смашивалка работает.


Ни как не работает. Генетическая смешивалка работает только для темперамента, который лишь косвенно и незначительно влияет на интеллект, а интеллект определяет социокультура. Для нормы, без учета болезней типа аутизма и т.п.
888 587404
>>87401
Я думаю они не просто так говорят конкретно про IQ в сообщениях выше. Интеллект что такое до сих пор срачи ходят. А то что уровень IQ коррелирован по наследству с уровнем 0,8 это проверенный факт.

Другое дело, что IQ вообще не имеет отношения к оценке уровня интеллекта, так как спроектирован давать ответ на вопрос есть ли проблемы с интеллектом, а не для того, чтобы сравнивать интеллект двух индивидов.
889 587406
>>87404

> А то что уровень IQ коррелирован по наследству с уровнем 0,8 это проверенный факт.


Удивительно, не может такого быть. Родители передают не только гены, но и воспитанием передают социокультуру. Вот почему-то детям маугли IQ генами ни как не передаются. Магическое исключение.
890 587407
>>87406
Это учитывают исследования. Коэффициент корреляции до 0,8 между близнецами выращенными в разных условиях. Хотя, я повторюсь ещё раз, IQ не оценивает интеллект.

> Да, этот вывод о влиянии генетики на интеллект основан на исследованиях близнецов. В этих исследованиях анализируется различие в интеллектуальных способностях между однояйцевыми и разнояйцевыми близнецами. Однояйцевые близнецы генетически идентичны, в то время как разнояйцевые близнецы, как и обычные братья и сестры, имеют примерно 50% общих генов.



> Исследования показывают, что когда интеллектуальные различия между однояйцевыми близнецами, выращенными в разных условиях, меньше, чем между разнояйцевыми, это указывает на значительное генетическое влияние на интеллект. Оценки генетического влияния (известные как «наследуемость») на интеллект варьируются от 50% до 80%, что означает, что эта доля вариабельности интеллекта в популяции может быть объяснена генетическими различиями.



> Эти исследования являются частью области бихевиористской генетики и помогают понять, как взаимодействуют генетические и окружающие факторы в развитии когнитивных способностей. Однако важно отметить, что наследуемость относится к вариации в группе, а не к индивидуальным способностям, и что окружающая среда также играет значительную роль в развитии интеллекта.

891 587408
>>87407
Дай ссылку на исследование, я вот вижу обратные результаты

> Тесты показали, что IQ приемной близняшки на 16 пунктов ниже, чем у сестры, хотя обычно однояйцевые близнецы показывают разницу в IQ не более семи баллов. Хотя ученые до конца не уверены, что вызвало такую резкую разницу в IQ, в исследовании отмечается, что у приемной девочки в детстве наблюдалось больше проблем в семье.

892 587409
>>87408
Вот несколько исследований, которые изучают влияние генетики на интеллект на основе исследований близнецов:

1. Исследование [Hiroto Takahashi et al.](https://dx.doi.org/10.1017/thg.2022.10) показало, что однояйцевые близнецы более схожи в плане поведенческих черт, включая интеллект, по сравнению с двуяйцевыми близнецами, что указывает на влияние генетики.

2. В мета-анализе [S. Sniekers et al.](https://dx.doi.org/10.1038/ng.3869), включающем 78,308 человек, было идентифицировано 336 SNP и 22 гена, влияющих на интеллект, 11 из которых были новыми находками.

3. Исследование [S. Franić et al.](https://dx.doi.org/10.1017/thg.2014.26) показало, что фенотипическая стабильность вербального, невербального и общего интеллекта в детстве и подростковом возрасте в основном обусловлена высокой стабильностью аддитивных генетических факторов.

4. [A. Vinkhuyzen et al.](https://dx.doi.org/10.1007/s10519-011-9507-9) обнаружили, что влияние генетики на интеллект в зрелом возрасте оценивается в 44% из-за аддитивных генетических факторов, 27% из-за неаддитивных генетических факторов и 11% из-за ассортативного скрещивания.

Эти исследования подчеркивают важность генетических факторов в развитии интеллекта, но также подчеркивают роль окружающей среды и взаимодействия генов и окружающей среды в этом процессе.
IMG20231222083345365.jpg136 Кб, 979x1080
893 587410
>>84346 (OP)
Всём привет.
Есть закон диалектики в котором количественные изменения приводят к качественным.
Например - лед нагреваясь превращается в воду, вода в пар, а пар в плазму. Что будет если дальше нагревать плазму? Следующее агрегатное состояние вещества? Или дальше просто плазма плазмой остаётся?
894 587411
>>87410
нагревание суть ускорение
895 587412
>>87409

> изучают влияние генетики


Вот именно - ВЛИЯЕТ. О чем я не спорил. А не определяет, как социокультура.
896 587413
>>87410
Дрожание атомов дойдет до скорости света, если нагреть ещё больше, то начнут появляться новые частицы. В БАКе экспериментально это подтвердили лет 10 назад.
897 587420
>>87412

> Что влияет на вкус каши? Соль.


> Что определяет вкус каши? Соль.


Ты понимаешь, что уже начал натягивать сову на глобус?
898 587424
>>87411
Ну наверно.
>>87413
Как массовые частицы дойдут до скорости света? Да и в коллайдере не появляются новые частицы, просто их сталкивают на большой скорости и мы наблюдаем запчасти из которых эти атомы состоят.
899 587427
>>87424

> как массовые частицы дойдут до скорости света?


Их разгоняют с помощью электромагнитного поля до скорости близкой к скорости света. До самой скорости света разогнаться невозможно.

> мы наблюдаем запчасти из которых частицы состоят


Бозон Хиггса это запчасть частицы? Какой если не секрет?
900 587431
>>87427
Я не собираюсь спорить на эту тему. Мне интереснее было о следующем агрегатном состоянии вещества после плазмы если законы диалектики верны.
901 587439
>>87431

>Я не собираюсь спорить на эту тему. Мне интереснее было о следующем агрегатном состоянии вещества после плазмы


Нет никакого другого агрегатного состояния. Агрегатные состояния образуются за счёт электромагнитных сил. В плазме у газа просто отхерачились (и улетели нахуй) электроны. Больше ты ничего тут электромагнитно не поменяешь.

> если законы диалектики верны


Лол нет.
902 587448
>>87439

>Агрегатные состояния образуются за счёт электромагнитных сил


В общем случае нет. На квантовом уровне работает принцип Паули, что ведет в появлению новых фаз, фактически других агрегатных состояний.
К примеру любой кусок металла, это кристаллическая решетка обычного твердого тела, внутри которой "разлита" ферми-жидкость из электронов.
903 587458
Сап, вот есть атмосферная коррозия металлов, из-за присутствия влаги в воздухе, выпадения росы и тд даже без дождя металлические конструкции будут медленно ржаветь, вопрос в следующем, если скажем металл попал под дождь несколько раз и на нем быстро появился налет ржавчины, а потом мы исключим прямой доступ осадков на поверхность металла, то коррозия вернется к своему обычному темпу, либо налет каким-то образом ускорит процесс? Просто есть велик из хромо-молибденовой стали - пару-тройку раз прокатился в дождь, внутри на трубах появился рыжий налет, больше я кататься в дождь не буду, мне нужно убирать ржавчину во избежание ускоренного гниения рамы или без доступа воды он еще внукам достанется покататься? Спасибо за ответы.
904 587477
>>87458
Для обычный сталей ржавчина это гидроксид железа. Из-за особенностей электрохимии ржавчина способна автокатализировать коррозию атмосферным кислородом и влагой.
С случае легированных(хромом или молибденом) сталей все сложнее, ибо там действует другие процессы. Коррозия идет через перекристаллизацию и уже реакция не автокаталитическая. Впрочем лучше все таки зачищать поверхность.
SEM рачок-водоросль-бактерия.gif2,1 Мб, 400x300
905 587510
>>87477
Спасибо, добра, лучше ответа и представить сложно, мой любимый сайентач не подвел! Пикрандом.
906 587511
>>87510
Как получают такие изображения? Это же просто шок какое пррблржение
907 587512
>>87511
Сканирующая электронная микроскопия.
908 587513
>>87512
Да, но что это?
909 587514
>>87513
Я знаю не больше гугла. Исследуемый объект специально готовят осаждая на нем тонкую пленку металла (вроде золото обычно), поэтому живые объекты не поизучать, потом помещают в вакуумную камеру, и обстреливают пучком сфокусированных электронов, которые отражаются от металлической пленки и попадают в детекторы, которые формируют изображение, длина волны электронов очень маленькая - соответсвует атомным размерам поэтому можно получить гигантские увиличения в миллионы раз, в отличие от оптической микроскопии.
910 587515
>>87514
Охренеть. Круто, спасибо
911 587543
>>87514
А я думал там тупо иголкой водят и напряжение на ней записывают. Я в чем то не прав в своем упрощении?
912 587544
>>87543
С иголкой это атомно-силовая микроскопия.
Напряжения не всегда записывают, чаще всего снимают лазером колебательные моды с кристалла иглы.
913 587590
>>87544
Немножко поправлю. С иголкой это сканирующая туннельная, ей можно хоть атомы отрисовать на поверхности, лишь бы проводила. В атомно-силовой все ж кантилевер, с которого отражение лазера фотодетектор снимает.
17069792219990.mp435 Кб, mp4,
148x192, 0:06
914 587602
Зачем нужна математика?
Это фентези, если математика работала в жизни шоколад был бы бесконечным.
915 587603
>>87602
Части шоколадки отламываются в виде крошек и остаются на ноже, которым им разрезают. Хоть на атом, но будет уменьшаться. ИМХО, математика нужна как способ счисления, а не как что-то самостоятельное, НО я слышал и обратное.
podxg4vuco0-768x326.jpg27 Кб, 768x326
916 587606
Пытаюсь понять, что из себя представляет наш мир. Читаю в основном википедию и хабр. Самая непонятная хуйня, которая периодически попадается, что частицы это не частицы и не поле/поля. Где-то мне попалось утверждение, что это вообще взаимодействия. Т.е. не взаимодействия чего-то, а просто взаимодействия в чистом виде, но для удобства это иногда представляют в виде частиц, а в других случаях в виде поля.
Еще часто попадается мысль, что пространства тоже нет. Как вариант пространства-времени. Ну что пространство это тоже удобная модель для физиков и математиков, а так его нихуя нет. А у нас возникает иллюзия пространства из-за особенностей восприятия.

Где-то можно об этом по-подробнее почитать? Если оба утверждения не бред, конечно.

Как может быть, что для фотона все время существования вселенной один миг, если его скорость конечна? Это как-то связывают с тем, что у фотона нет массы и поэтому у него такое "восприятие" пространства-времени.

Алсо, посоветуйте каких-нибудь научпоп книг, чтобы наконец понять, что из себя представляют, хотя бы примерно прочувствовать частицы, суб частицы, пространство это ебаное блядь. Только я гуманитарий, все мои "познания" из википедии.

Собираюсь прочитать "Вселенная. Происхождение жизни, смысл нашего существования и огромный космос" и еще
"Квантовые миры" Шона Кэрролла. Норма вариант?
917 587607
>>87606
Если реально гуманитарий, читай какого нибудь Фаербенда или кто там любил капченых хуесосить. Поймешь (и другим будешь разъяснять) что ученые тоже люди и нихуя толком не понимают, сочиняют всякую хуиту и любят переобуваться в прыжке.
918 587609
Как научно объясняется появление жизни? По какому принципу и зачем материя управляемо начинает накапливать энергию своими силами и распоряжаться ресурсами? Есть хотя бы догадки? Ни одна из религиозных теорий о боге и близко не подходит. То есть наши религиозные представления с театральной подачей реальности это какая-то яма вроде слепого пятна на сетчаке, куда мы заруливаем сознанием исходя из эмоционального реагирования на мир и интерпретаций на основе такого опыта, и в следствие этого у нас формируется ложная картина реальности, из которой нам не доступно видеть объективное. Если полагаться на эмоциональный опыт нельзя, поскольку он формировался только для субъективного и коллективного выживания в определённых условиях, как тогда объясняются законы эволюции вне естественной среды? Почему мы вынуждены идти контринтуитивным путём через противоречия, проходя путь обратным образом от выживания к знаниям и наоборот, реформируя мотивы и меняя цели? Это же шизофрения, так как мозг не способен работать в условиях постоянной неоднозначности и отсутствия привязки к определённым значениям, а сложившиеся стереотипы становятся абсурдным мусором из религий, стереотипов, суеверий. и так по кругу. Как тогда составить адекватную модель восприятия жизни и развития в нём?
919 587612
>>87606

>Ну что пространство это тоже удобная модель для физиков и математиков


Так оно так и есть же
920 587613
Физикам вопрос. Объясните, что происходит с электронами и орбиталями в нейтральном атоме при реакции его деления или спонтанном распаде? По сути я предполагаю, что электроны могут перестраиваться на орбитали новых атомов, но весь процесс технически представить сложно.
Аноним 921 587616
>>87613

>орбитали


>>87613
Отвечу как инженер микроэлектронщик который изучал курс атомной и ядерной физики один семестр.
При образовании нового элемента, или при его распаде получаются новые вещества. Например калий 19/38 при бета распаде превратится в инертный газ аргон 18/38. Как видно число нуклонов в ядре осталось таким же, поменялся лишь атомный номер, в результате этого ядро стало слабей притягивать электроны в атоме, отсюда происходит и смещение электронных энергетических уровней. 4s электрон на внешней оболочке в результате такого смешения оказывается свободным и покидает атом, оставляя Аргону законченную электронную конфигурацию благородного газа.
Аноним 922 587619
>>87096
Металлы по природе эластичны благодаря наличию электронного газа (электроны обобществлены, т.е валентный электрон одного атома металла одновременно принадлежит всем атомам кристалла) , который в свою очередь образуется благодаря металлической связи между атомами вещества. В углероде атомы соединены ковалентными связями, например одна из аллотропных модификаций углерода - алмаз является диэлектриком и не проводит электрический ток, так как нет свободных электронов проводимости как в металле. ( с допущениями алмаз можно рассматривать как полупроводник с большой шириной запрещенной зоны). Проводимость графита обусловлена его слоистой структурой. Если в алмазе атомы углерода соединены друг с другом сильной ковалентной сигма связью, то в случае с графитом три атома углерода соединены сигма связью а четвертый пи связью, электрон в этой связи более подвижен. Благодаря этому графит проводитэлектрический ток. P.s химики простите если где то накосячил в универе с натяжкой получил четверку.
923 587622
>>87619

> Металлы по природе эластичны благодаря наличию электронного газа


Ну это не совсем так. Пластичность и тип связи вещи не всегда совместимые. К примеру хром, рутений и т.п. скорее в порошок уйдет при деформировании, ну а органика к примеру ковалентная будет вполне себе деформироваться.

> Если в алмазе атомы углерода соединены друг с другом сильной ковалентной сигма связью, то в случае с графитом три атома углерода соединены сигма связью а четвертый пи связью, электрон в этой связи более подвижен.


Тут прям так связи нельзя упрощать. Имеет место sp3 и sp2 гибридизация для алмаза и графита. Между слоями графита ван-дер-ваальсовы связи, в листе графита все электроны находятся в разрешенной зоне, что подтверждается отсутствием прозрачности (аля как металл), даже в графене есть дираковские точки на границе зоны Бриллюэна. Проводимость графита не оч хорошая, тк в отличие от металла проводимость только в слоях и рассеяние на границах чешуек достаточно большое, что тянет повышение удельного сопротивления.
924 587627
>>87622
Не уверен, что хром и рутений это идеальный пример чисто металлической связи. Там d-d взаимодействия неплохо так дают прикурить. А чисто металлическая параша это какой-нибудь кусок калия.

Мне кажется тут все упирается в семантику, что имеется ввиду под "металлами". В целом металлы, или наши выдуманные модельные металлы.
925 587628
>>87627
Да, это я все согласен, если по-хорошему, то пластичнось это величина барьеров Пайерлса-Набарро, то бишь "легкость" скольжения дислокаций по плоскостям скольжения (спайности) с наибольшей ретикулярной плотностью, что для ковалентных и ионных кристалло автоматически может давать разрушение если спайность совершенная или весьма совершенная.

> что имеется ввиду под "металлами"


Ну, если мы уйдем от электропроводности, где металлы это как правило "проводники", а перейдем к зонной структуре, то "металлы" это вещества, где уровень ферми лежит в первую очередь в разрешенной зоне, а во-вторых эта зона должна быть заполнена частично. По идее так.
809749d31a24014b23e26478cbf006cd.jpg12 Кб, 512x512
926 587645
>>84346 (OP)
Анон, думаю второй день над антропологической природой звука. Проблема даже в формулировке поиска, чтобы найти материалы, поэтому задам тебе вопрос, возможно совместный ход размышлении даст ответ. И подозрение что он вообще на поверхности.

Человечество использовало и развило звуковую систему до определенных высот. От социальных (речь, музыка, сигнально-маркерно-информационные звуки, интонационный окрас) до технических (гидролокация, ультразвук в науке-производстве-медицине, военные и психологические аспекты применения и т.д.) Но всё это следствие формирования и развития артикуляционного аппарата у нашего вида, понимание и изучение природы звука. У животного мира такой же механизм - акустическая коммуникация, маркеры угрозы, защиты и привлекательности. Но опять же это следствие и развитие аппарата, не изначальная данность.

Зачем природа создала речевой аппарат и звуковые коммуникации? Приятная побочка пищеварительно-дыхательной системы в дальнейшем развитый когнитивными аспекатми развития мыслительной системы?

Пример: характеристики экстермофилов обоснованны средой обитания, оволосение терморегуляция, уплотнение костного-кожного аппарата и хитинирование защита и т.д. Но пиздеть зачем природа нам разрешила? Зачем сделала, что все пиздят? Пиздят, пиздят, пиздят. Рыбы же не пиздят, максимум звуки жизнедеятельности. Получается просто побочка, выросшая в одну из фундаментальных систем жизнедеятельности?
927 587646
>>87645
У тебя есть система клеток. Когда эту систему тревожит, определенным образом, определенная разновидность помех. За этим следует вредное или полезное явление.
А когда все уже умеют слышать, умение пиздеть становится крайне выгодно.
928 587647
>>87646

>все уже умеют слышать


Да, система звука интуитивно организовалась из-за координационной системы слуха. У нас есть один из механизмов ориентации, координаты в виде высоты-силы-спектра и есть протоаппарат взаимодействия, и мы естественно его разовьем. Отлично, есть ответ. Теперь буду размышлять нахуя нам слух дала природа? Понятно колебания и вибрации, но все равно это интерпритации звукового аппарата мозгом, без него это просто раздражитель определённых участков имеющий градиент силы колебания. А тут система посложнее.
IMG7022.jpeg392 Кб, 1920x1880
929 587650
А прикиньте как жилось людям которые так или иначе относились к программе Аполлон?
Тогда вообще нереально всех заливали деньгами, в пересчете на сегодняшний день это как почти 300 миллиардов долларов.
Почти все кто занимался этим мгновенно стали мажороми с большим домом, семьей, собакой и спортивной машиной.
930 587653
>>87616
Чего? Бета распад, ядро стало более заряженным на +1, а ты говоришь что из оболочки один электрон при этом улетел?
931 587654
>>87653
А понял, ты просто не указал что это бета+ распад
По дефолту ж считают бета распал бета- распадом.
932 587662
>>87612
Поясни свою мысль, будь добр. И где об этом можно почитать?
933 587668
>>87645
Все фичи всех ДНК организмов есть результат случайных мутаций. Если мутация давала какие-то преимущества в размножении особи, то мутация закреплялась в виде. Если не давала, то мутация исчезала из вида.

> зачем природа разрешила


Природа не является субъектом, чтобы иметь намерения.
934 587675
>>87662
Я не думаю, что это научный вопрос - и я не ученый если что - просто наука же этим и занимается вроде - приближенным описанием мира. Из наблюдений и тд конечно, но типа карта же это не мир вокруг тебя, а описание его
935 587686
>>87668

>Природа не является субъектом, чтобы иметь намерения.


Природа - это не абстракция. Твои мама и папа это Природа относительно тебя.
936 587717
>>85592
Потому что это псевдонаучная хуита, целью которой является поддержка капиталов калпидорглистов, а именно - показать конкуренцию, чтобы за всякую хуйню люди грызлись между собой, а калпидорглисты с них ржали, и кормили их этой макулатурой жидомасонской, что лезет сотнями миллиардов ежегодно - с печатных станков.
937 587718
>>87717
Ты веришь в капиталистов? Лох. Это рептилоиды.
938 587765
>>87717
какова этмилогия слова механика в квантовой механике?
механика - как взаимодействие в смысле "взаимодействие заявителя и нотариусе" ?
mus2.png125 Кб, 233x257
939 587771
>>87765
можно ли снять с себя статическое напряжение, если помыть руки под крано?
940 587775
>>87765
Слово «квант» происходит от латинского quantum («сколько, как много») и английского quantum («количество, порция, квант»). «Механикой» издавна принято называть науку о движении материи. Соответственно, термин «квантовая механика» означает науку о движении материи порциями (или, выражаясь современным научным языком науку о движении квантующейся материи). Термин «квант» ввел в обиход немецкий физик Макс Планк (см. Постоянная Планка) для описания взаимодействия света с атомами.

https://elementy.ru/trefil/20/Kvantovaya_mekhanika
941 587782
Кто вкурсе размерностей и способностей измерительной физической техники
Насколько явно замеряема или незамеряема разница между двумя длинами волн излучения, которые различаются меньше чем на постоянную планка?
942 587790
>>87606
Брайн Грин "Ткань космоса".
Дэвид Дойч "Структура реальности".

Обе книжки написаны для гуманитариев без всяких формул.
943 587791
>>84795

>Время работает точно так же как и еще одна добавленная к пространству ось.


Не работает, т.к. не связано с пространством.
Иначе релятивисткое замедление времени пойдёт по пизде.
944 587792
>>85178

>чё всё? лучшие умы доски ничего не смогли ответить внятного?


Отвечать не на что
>>84809
Сырой не означает невозможность воспламенения.
945 587793
>>84818

>Почему человек (да и большинство животных в принципе) не может есть (полноценно усваивать) траву, листья, кору, корешки, древесину?


Есть может.
Люди даже глину едят в африке.

Полноценно усваивать человек вообще мало что может в сыром виде. Фрукты какие-нить, типа бананов, разве что. С обработкой ты можешь хоть кору жрать, гугли берёзовую муку, например и берёзовый хлеб.
946 587796
>>85825
Нормально ты так дунул, братиш.
Стул - это объект, обладающий свойствами, в основном это:
-можно сидеть
-можно передвинуть
-можно опереться на спинку.

Ты не можешь разобрав (декомпозировав) стул на доски, найти свойства стула, потому что это уже не стул.

>более того невозможно понять когда они все вместе становятся стулом


Когда что-то собирается в один объект, обладающий свойствами выше.

Сознание и мозг это не составные части одного и того же, это как лампочка, которую включили и она излучает свет.
947 587797
>>86018

>Почему время в любом девайсе, не подключенном к интернету и, соответственно, не получаещем апдейт с "правильным" временем всегда начинает спешить?


Потому что китайский ширпотреб.
У тебя кварцевый генератор копейки стоит, хуле ты хотел?
Хочешь точность - покупай хронометр.
948 587798
>>86047

>Почему никто ещё не додумался испытать на практике теоритическую модель автономного марсианского поселения


По новостям уже которую партию людишек на 2-3 года в герметичных капсулах запирают.
В них челы дрочат рекуперацию, и выходят погулять наружу только в скафандрах.

Хуй знает, как ты всё это пропустил.
949 587800
>>86310

>Мне лишь нужно узнать точное число вариантов написания чисел "0-9", если использовать "11" знаков.


Числа с 0 по 9 имеют единственный вариант написания в системе из 11 цифр, если счисление по арабской системе
950 587801
>>86401

>Почему нет активного электромагнитного паруса?


Гугли микроволновый двигатель
951 587802
>>86532

>значит после моей смерти рано или поздно все молекулы разложившегося меня вновь соберутся обратно и я снова стану живым. В чём я не прав?


В том, что это максимально бесполезная информация, поскольку свой жизненный опыт, который определяет тебя, ты не перенесёшь.
952 587803
>>87344

>Почему птиц на проводах не убивает ток???


Разница сопротивлений.
Лапы птиц являются параллельной цепью, а при параллельном подключении количество тока обратно пропорциональна сопротивлению участка цепи.

Короче, ток через птицу настолько мал, что его можно игнорировать.
953 587804
>>87384

>Белый самец 100iq спаривается с белой самкой 100iq, у них рождается ребёнок 100iq, верно?


Не верно, рисунок коры головного мозга формируется под чисто механическими причинами.
Иными словами, даже клонируя 100iq человека мы можем получить как 120iq, так и 80iq, тупа изза особенностей внутриутробного развития.

Отпечатки пальцев будут одинаковыми, но не рисунок коры головного мозга.
954 587805
>>87602
Смотри на плитку из четырёх кусочков. Микрошип на месте отреза магически удобно удлинняется при стыковке на новом месте.

Красивый видеомонтаж, короче.
955 587806
>>87609

>Как научно объясняется появление жизни?


Тебе теория эволюции уже всё расписала.
Учёные мужи уже и первичный бульон варили, и протобактерию в первичном бульоне при подаче электроразрядов получали.
Все исследования можно нагуглить
956 587809
>>87804
Ты сейчас опровергнуть факт генетической наследственности интеллекта опровергнуть пытаешься или просто к условностям в цифрах приебался?
957 587810
>>87782
Постоянная планка и длина волны разные размерности, что ты имеешь ввиду, если разница между ними не может быть равна постоянной планка?
958 587811
>>87810
Ну орбиту электрона же квантуют постоянной планка.
У фотонов разной длиной волны разная рюэнергия, значит тоже можно как-то постоянной планка проквантовать.
959 587813
>>87811

>Ну орбиту электрона же квантуют постоянной планка.


И кстати вот кажется, типа планковские величины это такие маленькие штуки(в основном) и постоянная планка это про самый-самый микромир.
Но вот высоты орбиталей электронов квантуют постоянной планка, и переходы электронов между ними сопровождаются излучением вполне себе мощных фотонов.
По-идее из этого жёстко следует что постоянная планка нифига не маленькая.
Или я что-то не так понял?
960 587820
>>87809

>Ты сейчас опровергнуть факт генетической наследственности интеллекта опровергнуть пытаешься или просто к условностям в цифрах приебался?


Второе.
Генетическая наследственность интеллекта показывает результат только в статистике, т.е. когда ты будешь анализировать тысячи особей. Иными словами в твоём случае дети белых В ЦЕЛОМ будут умнее.
При разборе единичного случая факторы развития оказывают такое же, если не большее воздействие на итоговый интеллект, поэтому хуй ты чё спрогнозируешь.
smfull.jpg200 Кб, 1200x800
961 587827
>>87820

>Генетическая наследственность интеллекта показывает результат только в статистике



>При разборе единичного случая

962 587828
>>87827
Ты путаешь генетическую болезнь и общий случай.
Генетическая болезнь может характеризоваться особенным, лол, коэффициентом вероятности повторения в следующем поколении.
Который будет сильно отличен от веротяности повторения вариаций развития нормы.
У 10 поколений iq 80 обоих родителей, если появится ребёнок iq100-очень маленький шанс что у его детей будет тоже iq близкий к 100. Но конечно выше чем был шанс появиться у этого iq 100 с родителями iq 80.
963 587829
>>87828

>ты путаешь генетическое влияние на интеллект и генетическое влияние на интеллект



Есть за мной такой грешок.
964 587831
>>87828
Крч, нужно искать высокую, большегрудую, большежопую, худую, с правильными чертами лица, голубоглазую блондинку, у которой мама и бабушка такие же. А и ещё здоровую и не стареющую.

В целом, всё осталось таким же как и до научных исследований в этом вопросе.
965 587832
>>87829
Ты тупой, генетическая болезнь это набор свойств
В том числе и такого свойства как коэффициента влияния на следующие поколение.
Поэтому твоя картирка та к тебе подходит только разве что, а не как оспаривающий пример.
966 587833
>>87831
Естественно, вот только зачем тот даун спорил с очевидной вещью что интеллект это про среднее, а не про жёсткую передачу от родителей к детям, вопрос.
967 587834
Кстати если рассматривать тупизну, как попроченность структур и строяния мозга, в том числе и размера
В принципе это то же самое что и у даунов.
И вот у даунов высокая крайне вероятность появления детей даунов.
Так что сложение 80+100=90 так скорее всего не работает.
Порченная в большом количестве мест структура намного больше имеет вероятность повториться порченной, потому что порченность это беспорядок
А вот правильно функционирующая структура это порядок.

Так что усли усреднять, то допустим было 10 поколений 100, потом родился 80, то и дальше с высокой вероятностью будут появляться около 80-ти.
Или например 10 поколений 100, из потомок тоже 100 смешался с потомком 10 поколений у которых было 80
То полчится тоже около 80-ти, а не 90.
968 587836
>>87832

>интеллект не зависит от генов


>дауны такие потому что их такими воспитали


ясно
969 587837
>>87836
Ох, тупой даун
Интеллект жёстко определяется генами.
Передача генов от родителей к детям не жёсткая, жёсткая передача от совокупности предков детям с некоторой вероятностью колебаний в + или -
Ты настолько тупой что не отлечаешь жёсткое определение и жёсткую передачу.
970 587839
>>87820

>При разборе единичного случая факторы развития оказывают такое же, если не большее воздействие на итоговый интеллект


>Передача генов от родителей к детям не жёсткая, жёсткая передача от совокупности предков


Очевидно, что у умной тян скорее всего и умные предки, так что и колебания там особо никакого не будет, учитывая что интеллект передаётся как раз по материнской линии, по крайней мере в случае с рождением сына.
971 587846
>>87839

>интеллект передаётся как раз по материнской линии, по крайней мере в случае с рождением сына.


Охуенные истории.
Нормальные исследования об этом есть, а не журналы для тупых шалав?

Почти всю историю человечества интеллектуальным качествам женщин вообще мало внимания уделяли, используя просто как инкубатор.
И что-повально дауны-мужчины не рождались.
Если всё не с точностью до наоборот, то как минимум такое же влияние, 50/50.
972 587847
>>87846

>что-


*Что-то
714967.gif4,5 Мб, 396x299
973 587857
>>87771
бампъ
974 587859
>>87645
Это была побочка. У тебя есть горло и есть воздух, который по нему гоняют легкие. Получается незадокументированная фича "дико орать". Самки текут от альфачей, а альфачи быкуют и орут. Омеганы сидят тихо и не отсвечивают. В итоге самки дают альфачам, которые орируют погромче, а те что молчат - не дают потомство.

>Рыбы же не пиздят


рыбам нечем - у них нет легких
975 587860
>>87839

>интеллект передаётся


Нет.
Никакого интеллекта никому не передается.
Генетически возможно запрограммирована биохимия мозга.
Но даже имея идеальную биохимию, смотря опять же что иметь в виду, если ты родишься в семье которая продаст тебя в 5 лет в публичный дом бангладеша, никакого интеллекта в тебе не образуется.
976 587866
>>87827
>>87839

>Очевидно, что у умной тян скорее всего и умные предки


Вообще не обязательно.

Почему я вот тут >>87820 сказал, что это всё роляет лишь на статистике?

Потому что внешние факторы влияют на интеллект куда больше, чем генетическая предрасположенность к интеллекту (если это не заболевание).

Широкий таз или узкий влияет на травмы черепа при родах. Рандомность образования извилин зависит тупа от того, как клетки скрючились механически. Единственный реально коррелирующий параметр на данный момент с айкью - объем черепной коробки, но опять же, он коррелирует только на больших числах. Т.е. В СРЕДНЕМ люди с меньшим мозгом тупее по статистике, но на практике роляет и его структура и умение им пользоваться. У Тургенева 2кг мозга, у Анатоля Франса 1кг - оба писатели.

А ещё этап развития мозга не завершается после рождения, он идёт ещё лет 5 активно потом. Есть задокументированная особенность человека, что если он не научился говорить в детстве, то больше он не научится никогда - есть примеры "диких" людей, которых уже во взрослом возрасте пытались вернуть в цивилизацию. Они так и не научились говорить и понимать речь, максимум - разучивали отдельные слова как собака команды.

На данный момент не обнаружено гена "умности", с помощью которого можно было бы селекционировать людей как мопсов и выращивать сверхразум. Потому что сам мозг оказывается формируется очень странным образом, не соотносящимся с каким-то геном.
977 587867
>>87860
Тебя продали и ты так себя теперь успокаиваешь?
978 587868
>>87866

>А ещё этап развития мозга не завершается после рождения, он идёт ещё лет 5 активно потом.


Это я к чему - можно в этот промежуток времени заниматься со своими детьми, что сделает их более эрудированными на всю оставшуюся жизнь, а можно проебать момент и будут не такими умными с меньшим айкью
979 587869
>>87645
Тут скорее эволюционная выгода от превращения организма в передатчик сигналов плюс хорошее дальнодействие, подходящая ширина канала и повсеместное наличие медиатора (воздуха, воды).

Как приемник сигналов в зависимости от дистанции хорошо работает гаптика, вкус, запах, звук и свет. Как передатчик - нихуя. Касаться, лизать и нюхать можно только рядом расположенные предметы. Метить территорию можно только на длину струи. Пердеж и феромоны улетают по ветру в непредсказуемом направлении. Для того, чтобы вибрировать или топать ножкой, нужно большая масса тела или наличие твердого грунта. Мигать светом как светлячок можно только ночью, под землей или под водой, потому что супротив солнечного засвета не хватит мощности. А вот орать можно круглосуточно, в широкой полосе частот и на многокилометровую дистанцию. Если бы на небе вместо солнца и луны висело гигантское ОРАЛО, которое круглосуточно пиздело бы как репродуктор на всех частотах - тогда орать и правда было бы бессмысленно. А если бы с неба постоянно ссало разнообразной органикой космическое ХУЙЛО, то тогда бы собакенам не было смысла нюхать чужие газы и метить территорию. Ну ты понел логеку.
980 587875
>>87868
Как ты объяснишь умных детей алкашей, наркотов и прочих бомжей? Которыми естественно вообще никак не занимались.
Кроме объяснения того что у тебя низкий интеллект, и ты пишучи такое здесь, самому себе фактически рассказываешь эти сказки пытаешься себя успокоить, но и нам приходится их здесь видеть?
981 587877
>>87875

>Которыми естественно вообще никак не занимались.


Улица воспитала, ауф
982 587879
>>87866

>Потому что внешние факторы влияют на интеллект куда больше, чем генетическая предрасположенность к интеллекту


Ты же понимаешь что ты просто комнатный пиздабол, спорящий сейчас с мочёными https://www.mensa.org/iq/genes-and-environment ?

>в этот промежуток времени заниматься со своими детьми, что сделает их более эрудированными на всю оставшуюся жизнь


Эрудированность не равна интеллекту. Конкретно интеллект ты дальше генетически заложенного потенциала у ребёнка развить никак не сможешь, так же как и не сможешь замедлить деградацию мозга с возрастом засчёт интеллектуальной нагрузки.

>На данный момент не обнаружено гена "умности"


Коупмонстрище... В говне мочёные уже более 1к генов нашли, ответственных в той или иной степени за когнитивные способности
983 587881
>>87866

>На данный момент не обнаружено гена "умности", с помощью которого можно было бы селекционировать людей как мопсов и выращивать сверхразум


Тупая одебилевшая трансошалава, селекция точно так же и замечательно работает без всяких обнаружений какого-то гена, выблядок ты тупых уёбищ без хуя.
984 587883
Лол, походу эта шалава, не скажу думает, считает, что умных людей невозможно отличить от тупых, кроме как если не покапаться в их гене, и не найти есть ли у конкретного человека ген умности, или нет
Ведь все равны
Ахахахаэаэ
985 587912
>>87883
Вот, сразу видно, твои родители умные люди, такого умного человека вырастили, ну прям совсем не долбоеб-дебил
986 587916
>>87875

>Как ты объяснишь умных детей алкашей, наркотов и прочих бомжей?


Так я уже обьяснил, что интеллект с наследственностью слабо коррелирует.
Очевидно, что бомжи-алкаши-наркоши есть биомусорный скот, который по определению тупой. Если бы была жёсткая наследственность интеллекта, то и их дети были бы тупым алкоскотом.

>>87879

>Professor Sir Cyril Burt, former Mensa World President and one of the world's leading psychologists at the time, is famous for his twin studies, in which he discovered that the correlation between the IQs of identical twins reared apart was an enormous 0.77, which almost irrefutably supports a large genetic influence on IQ.


>0.77


>Большая корреляция


Про что я и говорю. Будь зависимость интеллекта от генов жесткой, этот параметр был бы равен единице.
Камон, у однояйцевых близнецов, считай, законных клонов человека, разный айкью. Какого, спрашивается, хуя.

>A child may be born with genes for high intelligence, but if that child grows up in a deprived environment where he/she is malnourished orlacks access to mental stimulation, the child may not score well on measures of IQ.


Про что я и говорю, буквально.

>Конкретно интеллект ты дальше генетически заложенного потенциала у ребёнка развить никак не сможешь


Зато можешь его проебать, чекай пример с языком.
Важно то, что ты можешь тренировать само мышление, соображалку.

>не сможешь замедлить деградацию мозга с возрастом засчёт интеллектуальной нагрузки.


Нихуя себе новости. Первый раз о таком слышу, пруфы будут?

>В говне мочёные уже более 1к генов нашли, ответственных в той или иной степени за когнитивные способности


Ты ничем не опроверг моё заявление, а лишь подтвердил.
Это всё гены уровня "размер черепа", "форма черепа", особенности кровеносной системы и т.д., каждый из которых влияет косвенно, а не прямо.

>>87881

>Тупая одебилевшая трансошалава, селекция точно так же и замечательно работает без всяких обнаружений какого-то гена


Чел, это подтверждает теория Дарвина и происхождение человека

>>87883

>Лол, походу эта шалава, не скажу думает, считает, что умных людей невозможно отличить от тупых, кроме как если не покапаться в их гене, и не найти есть ли у конкретного человека ген умности, или нет


>Ведь все равны


Кажется, ты абсолютно не понял тему моего поста. Переформулирую:

В связи с отсутствием генов, непосредственно влияющих на интеллект, невозможно однозначно судить об уровне будущего интеллекта у ребёнка в единичном конкретном примере ввиду сильного воздействия на интеллект внешних факторов среды, в которой появляется ребёнок.

Не, ну если тебя устраивают предикты в виде "ну, у белого самца с белой самкой 80% шанс получить ребёнка с iq в промежутке 80-120, а у белого самца с чёрной самкой 80% шанс получить ребёнка с iq в промежутке 75-115" - то пожалуйста, кушайте не обляпайтесь.
986 587916
>>87875

>Как ты объяснишь умных детей алкашей, наркотов и прочих бомжей?


Так я уже обьяснил, что интеллект с наследственностью слабо коррелирует.
Очевидно, что бомжи-алкаши-наркоши есть биомусорный скот, который по определению тупой. Если бы была жёсткая наследственность интеллекта, то и их дети были бы тупым алкоскотом.

>>87879

>Professor Sir Cyril Burt, former Mensa World President and one of the world's leading psychologists at the time, is famous for his twin studies, in which he discovered that the correlation between the IQs of identical twins reared apart was an enormous 0.77, which almost irrefutably supports a large genetic influence on IQ.


>0.77


>Большая корреляция


Про что я и говорю. Будь зависимость интеллекта от генов жесткой, этот параметр был бы равен единице.
Камон, у однояйцевых близнецов, считай, законных клонов человека, разный айкью. Какого, спрашивается, хуя.

>A child may be born with genes for high intelligence, but if that child grows up in a deprived environment where he/she is malnourished orlacks access to mental stimulation, the child may not score well on measures of IQ.


Про что я и говорю, буквально.

>Конкретно интеллект ты дальше генетически заложенного потенциала у ребёнка развить никак не сможешь


Зато можешь его проебать, чекай пример с языком.
Важно то, что ты можешь тренировать само мышление, соображалку.

>не сможешь замедлить деградацию мозга с возрастом засчёт интеллектуальной нагрузки.


Нихуя себе новости. Первый раз о таком слышу, пруфы будут?

>В говне мочёные уже более 1к генов нашли, ответственных в той или иной степени за когнитивные способности


Ты ничем не опроверг моё заявление, а лишь подтвердил.
Это всё гены уровня "размер черепа", "форма черепа", особенности кровеносной системы и т.д., каждый из которых влияет косвенно, а не прямо.

>>87881

>Тупая одебилевшая трансошалава, селекция точно так же и замечательно работает без всяких обнаружений какого-то гена


Чел, это подтверждает теория Дарвина и происхождение человека

>>87883

>Лол, походу эта шалава, не скажу думает, считает, что умных людей невозможно отличить от тупых, кроме как если не покапаться в их гене, и не найти есть ли у конкретного человека ген умности, или нет


>Ведь все равны


Кажется, ты абсолютно не понял тему моего поста. Переформулирую:

В связи с отсутствием генов, непосредственно влияющих на интеллект, невозможно однозначно судить об уровне будущего интеллекта у ребёнка в единичном конкретном примере ввиду сильного воздействия на интеллект внешних факторов среды, в которой появляется ребёнок.

Не, ну если тебя устраивают предикты в виде "ну, у белого самца с белой самкой 80% шанс получить ребёнка с iq в промежутке 80-120, а у белого самца с чёрной самкой 80% шанс получить ребёнка с iq в промежутке 75-115" - то пожалуйста, кушайте не обляпайтесь.
987 587917
>>87883
Про равность я кстати нихуя не понял, откуда ты это высрал, мы тут буквально про разные значения айкью говорим.
988 587918
>>87916

>Какого, спрашивается, хуя.


Эффект сперматозоида.
989 587919
>>87806
То есть нет ничего сверх? Всё можем делать и мы сами своими руками?
990 587920
>>87919

>То есть нет ничего сверх?


А чего тебе надо сверх?

>Всё можем делать и мы сами своими руками?


Тут как с телефоном в твоей руке - ты можешь примерно представлять, как он работает, но хуй ты его сделаешь своими руками.

На данный момент мы не можем проектировать организмы с нуля, но можем модифицировать существующие организмы.

Если для производства телефонов у нас есть куча высокотехнологичного оборудования, то для создания организмов с нуля пока такого оборудования нет.
991 587923
>>87916

>Очевидно, что бомжи-алкаши-наркоши есть биомусорный скот, который по определению тупой.


Интеллект определяет успех только на 20%. Остальное это личностные качества и среда.

>В связи с отсутствием генов, непосредственно влияющих на интеллект


Соре, но гены на интеллект пожалуй отсутствуют конкретно у тебя, учитывая что тебе в ебальник буквально кинули инфу о 1к генов ответственных за интеллект и исследования корреляции генетики с интеллектом аж в 77%.

>каждый из которых влияет косвенно, а не прямо.


Шиз, нет никакого гена ума, есть совокупность генов, которая и формирует определённый уровень интеллекта.

>Первый раз о таком слышу, пруфы будут?


Смысл в том что что активные нейроны чаще подвергаются мутациям, а при интеллектуальных нагрузках большее кол-во нейронов активно. Исследования сам ищи
992 587924
Аноны. Заканчиваю заочно специалитет геология нефти и газа. Имею опыт работы по специальности 5+ лет. Задумываюсь о поступлении в аспирантуру, но чё то сомневаюсь в своих силах. Я даже не представляю, что там делают, там же на лекции особо не нужно ходить пердеть, так ведь? Какую-то исследовательскую работу проводить надо, хуе мое...Аноны аспиранты, вот чем вы занимаетесь? Спрашиваю всех, но если есть геологи, то ещё лучше. Ведь рано или поздно я все равно поступлю в аспирантуру, мне только нужна более лучшая теоретическая подготовка.
993 587926
>>87923

>Интеллект определяет успех только на 20%. Остальное это личностные качества и среда


Люди, добровольно принимающие алко и наркоту автоматически выписываются из категории умных

>тебе в ебальник буквально кинули инфу


Я тебе цитаты из твоей же статьи кидал

>есть совокупность генов


Правда чтоли?))) Как это противоречит моим словам?

>Смысл в том что что активные нейроны чаще подвергаются мутациям, а при интеллектуальных нагрузках большее кол-во нейронов активно. Исследования сам ищи


Мутировал с этой информации
994 587928
>>87926

>Люди, добровольно принимающие алко и наркоту автоматически выписываются из категории умных


Как бы так сказать... Ты несколько не в том положении чтобы определять кого-то в умных или тупых на основе косвенных признаков. Типа это как если бы бомж с помойки подходил бы к людям и рассказывал бы им как стать успешным.

>Я тебе цитаты из твоей же статьи кидал


Так ты бы их прочитал для начала, прежде чем в штаны срать. Там прямым текстом пишется что генетическая наследственность интеллекта огромная

>irrefutably supports a large genetic influence on IQ



>Как это противоречит моим словам?


В твоём копиумном манямире пожалуй никак. В реальности это подтверждает что интеллект генетически наследуется. Впрочем, продолжать диалог с дауном, который на полном серьёзе оперирует терминами типа "ген умности" смысла пожалуй не имеет.
995 587929
А если взять полстакана воды и начать увеличивать количество воды в нем по одной молекуле, существует ли такое минимальное количество молекул, начиная с которого невооруженным глазом будет видно, что вот сейчас воды точно больше, чем половина стакана? Если допустить, что не существует, то это противоречит тому, что рано или поздно стакан наполнится и точно будет видно, что воды в нем больше половины. Если допустить что существует, то придется допустить, что глаз способен уловить разницу в одну молекулу.
996 587930
>>87929
У тебя есть эффект испарения жидкости, который опустошает стакан.
Тебе нужно добавлять х молекул, испаряющихся ежесекундно + 1 молекула лишняя минимум.

Правда наполнять ты свой стакан будешь миллионы лет.
997 587931
>>87928

>>irrefutably supports a large genetic influence on IQ


Со значением в 0.77 у однояйцевых близнецов.

Для начала я бы предложил тебе пойти и почитать, что такое корреляционная зависимость, чтобы понять, что означает число 0.77.

Пока ты читаешь определение, даю сразу вывод. 0.77 означают, что у генетически идентичных людей разный iq.

>Ты несколько не в том положении чтобы определять кого-то в умных или тупых на основе косвенных признаков.


Хммм, дай-ка подумать

>обладает интеллектом


>следовательно, осознаёт опасность и последствия при юзании веществ


>всё равно сидит на них


По-моему в данном случае абсолютно поебать в каком я положении. Либо человек умный и не торчит в говнину, и тогда у ребёнка нормальная семья, либо он торчит в говнину и ребёнок живёт в среде наркоманов. Но тогда торч тупой.

>В реальности это подтверждает что интеллект генетически наследуется.


Это ещё Дарвин подтвердил. У тебя в истории селективный отбор произошёл от тупых приматов до хомо сапиенс.

Ты вопрос, блядь, читал? Там буквально написано "предскажите, кто будет тупее, ребёнок белых, или метис"
998 587932
Одебилевшая полоумная трансшалава засрала тред, не удивлюсь если моча и есть эта трансшалава.
15199674658550-b.jpg69 Кб, 750x1125
999 587933
>>87932

>Одебилевшая полоумная трансшалава


Зачем же ты так про себя? Надо быть терпимым к самому себе
1000 587934
>>87932
Да это просто копиумный дуралей какой-то, видимо увидел что его родители туповаты вот и сидит коупит что интеллект генетически не передаётся, несмотря на кучу исследований, лол
1001 587936
>>87934

>интеллект генетически не передаётся


Двачеры не умеют читать.
Хотя, чего я вообще ожидал...
1002 587937
>>87929

> что глаз способен уловить разницу в одну молекулу


Для этого нужно еще чтобы при повторных тестах совпадало количество молекул при которых челоек улавливает разницу. Тоесть типа при 999999 дополнительных молекулах разницы не заметно, а при 1000000 уже да, и так в каждом тесте. В реальности (скорее всего) человек будет говорить что воды стало больше в относительно случайные промежутки времени, после того как воды в стакане стало заметно больше.
1003 587939
>>87931

>обладает интеллектом


>следовательно, осознаёт опасность и последствия при юзании веществ


>всё равно сидит на них



Интересно, сколько вам лет, такой максимализм и тем более в сайентаче. Нужно немного подумать и поразмышлять чтобы понять что твой гринтекст очень далек от реальности, первый пункт не имеет значения - щас скажешь на больших числах все видно, но об этом ниже, второй пункт не имеет значения - осознавать мало, точнее оно вообще ничего не значит, иначе бы все люди давно роскомнадзорнулись от осознаия всякой хуйни вроде неизбежности смерти, сидеть на них это уже крайняя стадия, почти безвозвратная, важно не пробовать, а вот это мало связано с интеллектом, даже скорее наоборот, гугли исследования что люди с высоким IQ более склонны попробовать так как любознательны от природы, а у тебя видимо ошибка в том плане, что ты думаешь что все нарики это подъездные поцики из гопрайонов и трущоб, но даже тут нет никакой связи с интеллектом, так как низкий интеллект и употребление наркотиков два следствия других факторов, а не одно причина другого. Короче подрастешь, узнаешь побольше, научишься думать поймешь что генерировал бред, я его примерно 12 контраргументами могу разъебать в пух и прах, но тратить время на юных максималистов младше 40 лень, я тоже был когда-то такой, лет в 20. Потом пришлось немного поработать с этими людьми и я с удивлением обнаружил очень много талантливых и умных в своих областях людей, которых почти безвозвратно засасывает эта история.
мимо
1004 587940
>>87875

>Как ты объяснишь умных детей алкашей, наркотов и прочих бомжей?


>>87939

>а у тебя видимо ошибка в том плане, что ты думаешь что все нарики это подъездные поцики из гопрайонов и трущоб


У меня изначально стоит условие, что ребёнок из семьи нарков.

>гугли исследования что люди с высоким IQ более склонны попробовать так как любознательны от природы


А где мне гуглить исследования, что большинство алкашей и торчей - обязательно умные челики с высоким iq?
1005 587941
>>87939
Так же добавлю, что бы мой пост не был расценен как пропаганда употребления наркотических веществ: не в коем случае не пробуйте и не употребляйте наркотики, это низкоуровневый хак тонкой нейрохимической машины - коей является мозг, против него никакая защита вроде высокоуровневой интеллектуальной осознавалки не сработает, но будут работать только очень жесткие кнуты и очень сладкие пряники, и тоже низкоуровневые, на которых потом всю жизнь сидеть вместо наркоты, а такие кнуты и пряники доступны не только лишь всем, почти никому. Наркотики - это жизнь мимо.
1006 587943
Не алкаш, а гений.
image.png108 Кб, 640x391
1007 587953
1008 587955
>>87924
Тебе в /un.
1009 587956
>>87941
Программист что ли?
1010 587964
Бесконечноти в степени бесконечность это ментальная гимнастика у математиков или уже просто шутка? Забойно смотреть, но смысла никакого бесконечность даже не число.
https://www.youtube.com/watch?v=O97xUyMUkJE&t=793s
1011 587965
>>87964
Ну он же говорит, что алеф-ноль понадобился для континуум-гипотезы, а это самые основания математики. Может из всего этого какие-то еще прикладные методы выросли, но я не знаю, сомневаюсь.
1012 587970
>>87929
Чел. Без обид. Из-за смачиваемости, глаз постоянно обманывается, потому что вода смачивает стенки обычного стакана и визуально дефектит оценку.
Мало того, мозг также обманывается, потому что сочетанием цвета стакана и жидкости можно обмануть глаз. А форма сосуда может обмануть даже тренированное сознание.
Так что, ответ - нет. Невозможно. Только опытным путем, методом тыка, эмпирики и на зубок.
949708.jpg61 Кб, 375x415
1013 587972
можно ли снять с себя статическое напряжение, если помыть руки под краном????
1014 587973
Что за фигня с "нельзя получить линзами интенсивность плотность излучения выше чем плотность излучения на поверхности источника излучения"?
Существует же светодиодные "лазеры", где просто светодиод, и излучение от него сжимают в маленькую точку которой режут допустим фанеру.
Да и те же оптоволоконники, и co-2 лазеры, и твердотельные лазеры, в них же активное тело лазера большое, толстое, а не шириной с будущий лучь, и "реакция" идёт по всей площади сечения активного тела, и потом собирают и сжимают излучение со всей площади сечения, а не с той лишь части, какая толщина у требуемого луча. Иначе бы смысла не было делать такое большое активное тело и всё его накачивать, делаои бы в несколько сотых миллиметра диаметром, если один фиг всё излучение с боков просто отсекать.

Да, в светодиоде и в лазерах вынужденное излучение, но какая нахер разница вынужденное излучение или тепловое тёмнотельное, суть же в поверхности, и у светодиода и у лазера поверхность, и спокойно сжимают излучение выше плотности поверхности.
1015 587974
>>87972
Дистиллятом?
Водой с солью скорее всего точно можно, если непрервной струёй.

Вообще электричество это магия.
1016 587975
>>87973

>то за фигня с "нельзя получить линзами интенсивность плотность излучения выше чем плотность излучения на поверхности источника излучения"?


Тут примерно так. Каждый фотон летит по всем мысленным траекториям, даже те, которые как бы обратно во времени от приемника к источнику. Потом берутся комплексные амплитуды и векторно суммируются.
Поскольку у нас фотон не один, а условное вещество одно, то взаимодействие между ними определяется разностью фаз фотонов.
Когда мы используем йоба-линзы, мы делаем такую систему, что обратные ходы луча начинают вносит огромный вклад в комплексную амплитуду, в итоге фотоны будут локализоваться больше на источнике, а не в фокусе.
С лазерами все по другому, там все фотоны имеют одну и ту же фазу и взаимодействие получается как бы однофотонным, что радикально отличается от первого случая.
1017 587976
>>87975
Ты пропустил часть про светодиодные "лазеры" которые лазерами не являются.
Поэтому твоё объяснение слито. Мир так не работает, живи теперь с этим.

И врядли ты сможешь приписать это фантазийное описание к реальной линзе, и объяснить на уровне геометрии линзы в целом и отдельных атомов, почему когерентное излучение собирается плотнее поверхности сечение источника, а некогерентное нет.
1018 587977
>>87975

>и взаимодействие получается как бы однофотонным


И пути у всех фотонов в линзе разные, она ж с площади собирает в условную точку.
Естественно там не будет совпадения фаз идеального во всех местах у всех фотонов, наоборот полная каша в любом случае получится.

Короче не очень совпдающее с реальностью объяснение.
1019 587979
>>87976

>И врядли ты сможешь приписать это фантазийное описание к реальной линзе, и объяснить на уровне геометрии линзы в целом и отдельных атомов, почему когерентное излучение собирается плотнее поверхности сечение источника, а некогерентное нет.


Т.е. ты говоришь о каких-то "мешаниях", которые предсказывает такая скажем не физическая, а математическо-статистическая теория, при рассмотрении такого общего случая
Но если исследовать о дельно рассматриваемые частицы, не в общем примере с линзой, то никакого свойства "мешания" у них не обнаруживается как не старайся.

И теория не может объяснить почему не обнаруживается отдельно никакого мешания, а только даёт предсказание, не факт что правильное, для оюрассмотрения общего случая с линзой, получается теория не полна как минимум.
Ну и то что про светодиодные лазеры писал, которые лазерами не являются.
1020 587984
>>87974
В кранах дистилированная вода не течет. И да, непрерывной струей.

Ок, а если стоя в резиновой обуви поссать на землю, то тоже получится?
1021 587987
>>87984
Ну смотри, статический заряд стекает по проводнику, по проволоке например. Естественно непрерывные штуки считаются проводником.
Струя пойдёт теоретически.

Но вот что странно, статический заряд растекается по поверхности тел когда образуется, в том числе и по диэлектрику.
Собственно схерали статическому заряду не стекать например по верёвке как по по проводу металлическому-непонятно.
1022 587992
>>87793

>С обработкой ты можешь хоть кору жрать, гугли берёзовую муку, например и берёзовый хлеб.


А как их правильно обрабатывать? Есть полный "список рецептов"?

Коршеки/крапиву в действе я вполне кушал и усваивал, но то обычное баловство было. Мне, конечно, было бы это интересно на поток поставить. Возможно ли половину своего рациона заменить "продукцией" из ближайшего леса?
1023 587993
>>87992

>А как их правильно обрабатывать?


В большинстве случаев это дробление и варка, либо запекание.
Полный список можешь поискать во всяких инструкциях по выживанию, либо в народных кухнях.

>Возможно ли половину своего рациона заменить "продукцией" из ближайшего леса?


Нет, если исключить животных.
Даже если материал можно сьесть, это не значит, что в нём много полезных веществ, которые нужны нашему организму. Чем меньше полезного в еде, тем больше этой еды надо съесть. Для многих ресурсов это увеличение настолько критично, что ты можешь кушать 24/7 без остановки, и всё равно умереть от истощения.
16191921293883.jpg282 Кб, 1200x1180
1024 587996
День добрый, какие учебники по логике и риторике посоветуете?
1025 587997
>>87996
По логике Виноградова можно почитать
1026 588010
>>87996
Онлайн курсы сначала пройди на степике, не сри в голову учебниками, все учебники рассчитаны на то, что их будет разжевывать преподаватель
1027 588030
>>84346 (OP)
Если микроволновые волны в микроволновке прибивают всю еду навылет, то почему если навалить кучу еды в микроволновку, она нагреется только с краев? Микроволна не может пробить холодный вареник?
1028 588031
>>87996
Органон
1029 588034
>>88030
Вообще довольно неоднозначно.
Она конечно не с краёв нагревается, а по-другому.
Но действительно должно всё довольно равномерно быть внутри микроволновки, если с учётом отражения от стенок, длины волны и расстояния от стенок до других стенок внутреннего отсека.
Короче не понимаю откуда там стоячим волнам взяться.
1030 588038
>>88030
Если б они проходили насквозь, еда бы не грелась. В микроволновке она нагревается от поверхности вглубь.
1031 588051
>>88010

Попробую, спасибо.
1032 588078
Почему зачатие ребенка как бросок кубика, может быть белым черным и вообще кем угодно
1033 588114
>>88078
Человеки в себе содержат два набора генов.
При половом процессе у каждого родителя нужно выкинуть половину генов, а потом снова сшить.
На каждом процессе разделение и соединение генома происходит перетасовка генов.
В этом же суть полового процесса.
1034 588115
>>88114
Там немного сложнее. Ещё есть аллеи, типа скрытые гены. Которые могут проявится через несколько поколений.
1035 588118
>>88115
Мой дед алкаш. Мой отец алкаш. Шансы у меня быть алкашом если я пью 5 дней в неделю?
1036 588121
>>88115
Аллей гена это и есть вариант из второго набора.
То что какой-то ген проявляется через н-поколения это уже особенности экспрессии генов, которые зависят от не наследуемых факторов.
И вообще генотип не есть фенотип. По отдельным внешним признакам нельзя судить какие гены как наследуются.
1037 588123
>>88078
Потому что ген это ген некой совокупности всех прошлых поколений, а не только то что проявилось в этот раз в этом выросшем плоде.
Т.е. информации в гене содержится намного больше чем нужно для постройки человека, и переносится эта вся информация целиком, а не только то что потребовалось для постройки.
Ну естественно примерно 50/50 от обоих родителей выуидывается конечно и смешивается оставшееся.
1038 588126
>>88118
Алкоголизма не существует, просто обычное недалёкое рабочее быдло.
Если и дед и отец такими были, то близка к 100% вероятность.
Да собственно ты уже и так понятно что такой, раз 5 дней в неделю пьёшь.
1039 588129
>>88126
Сегодня я бросил
1040 588133
>>88129
Как и пару дней назад бросал?
1041 588135
>>88133
По настоящему
1042 588166
>>88126
алкоголизм вполне себе существует. у него есть вполне четкие критерии.
ну пусть его переименовали в синдром зависимости
17079056786440.png412 Кб, 480x640
1043 588182
Когда бездельники-грантоеды наконец создадут мне кошкодевочку?
1044 588183
>>88182
Захватившие власть фашиствующие моралисты мешают со своей этикой говнетикой
1045 588186
>>88182
А ты тут причём? Ты платишь налоги не с условием
чтобы тебе что-то создавали, а потому что ты рабочая особь, потому что такое твоё предназначение.
1046 588190
>>88183
В Китае нет ни религиозных мракобесов, ни SJW леваков, там могли бы заняться генетическими исследованиями. Тем более там наибольшее число инцелов, которые страдают без кошко-жены.

>>88186
Обслуживающий персонал забыли спросить. Небось, жалкий учителишка, которому в лицо харкают старшеклассники? Или нищий препод, живущий на взятки от студентов-мажоров?
1047 588191
>>88190
Зашивайся порванный рабочий хуесос, я не работаю 😉
1048 588193
>>88191
Ну, так бы сразу и сказал, что школьник.
1049 588196
>>88193
Ты чего порвалась, рабочая особь
Тебе должна нравится твоя биологическая роль, ты для этого создан.
А вот эту хуйню навроде "когда тебе сделают" ты оставь, нето пиздюлей получишь обучающих.
1050 588199
наука полностью отрицает жизнь после смерти, или все же такой вариант возможен, если кто знает есть ли действительно какие нибудь исследования в которых что то удалось узнать или предположить на эту тему, посоветуйте литературу
1051 588201
>>88199
Как ты предлагаешь это исследовать?
Случаи памяти о реинкарнации от человека к человеку записываются, проверяются и систематизируются. Регрессивный гипноз ненадежен.
С остальными вариантами загробной жизни ты сейчас ничего не сделаешь.
Выход из тела при NDE? Мы не знаем как он работает.
1052 588203
>>88201

>Выход из тела при NDE? Мы не знаем как он работает.


Легко же эксперимент поставить.
1053 588204
>>88203
Говорят, в некоторых операционных ради смеха приклеивают где то под потолком бумажку вроде "в случае выхода из тела позвоните по номеру хххххх". Но пока еще никто не позвонил.
1054 588205
>>88204
Ну делать нехуй во время выхода из тела циферки какие-то запоминать
А если он потом умирает, то и не позвонит никак.

Другая методология эксперимента должна быть.
1055 588206
>>88203
Каким образом? Эксперименты по проверке выхода из тела организовать в принципе можно (хоть и крайне муторно), но чтобы использовать эту хрень для попыток исследования загробной жизни нужна возможность вызывать ее по желанию, а тут проблема.
1056 588208
>>88206
Ну вводить в состоянии исскуственной смерти
Снача с завязанными глазами привозить в какое-то новое место, где подопытный никогда не был, и там вызывать у них состояние искусственной смерти.

>Эксперименты по проверке выхода из тела организовать в принципе можно (хоть и крайне муторно), но чтобы использовать эту хрень для попыток исследования загробной жизни нужна


Так это одно и то же. Если доказан будет выход из тела, то уже сразу станет понятно что всё не так просто и нечто подобное скорее всего есть(жизнь после смерти)

И что значит не могут по желанию вызывать?
Ну конечно это довольно опасно, но аполне сеье легко умеют вызывать искуственную смерть.
Естественно далеко не после каждого такого случая подопытный будет утверждать что выходил из тела
Ну так хули, как всегда в науке нужен сбор большой статистики с помощью большого количества экспериментов.
1057 588209
>>88208
Статистика и так собирается, необходимость срочно рисковать человеческими жизнями ты хрен обоснуешь. Про "крайне муторно но возможно", я имел ввиду получить согласие у большого количества добровольцев, договориться с несколькими больницами и хирургами.

>Так это одно и то же.


Это ни разу не одно и тоже. Опыт выхода из тела, который к тому же испытывают не все, не доказывает возможности существования разума вообще без тела и может быть объяснен способностью мозга сканировать окружающую среду или еще какой хренью.
1058 588215
>>84346 (OP)
Поцоны, я тут ходил в радиоактивную пещеру, потом лег спать в гамаке. Так вот у моего гамака снаружи постоянно какой-то пиздец творился. Мой гамак наглухо закрывается, я его липучками прошил, так вот половину ночи в него словно из прожектора светили то фиолетовым, то красным, то синим, то желтым цветом. Это сетчатка глаза от радиации чутка с ума сошла?
1059 588230
>>88215
Вряд ли.
Чтоб так светилось, тебе нужно было залезть в активную зону ядерного реактора.
1060 588232
Бобрышевский где-то говорил, что за речь и пение отвечают разные разделы мозга. Ничего не могу нагуглить, есть сведущие?
1061 588235
>>88230
Свечение может быть глюком.
1062 588240
>>88230
Да не, это не в реальности светилось, а только в моих глазах.
1063 588241
>>88232
Используя внутричерепные записи мозговой активности, американские ученые обнаружили нейронную популяцию, которая избирательно реагирует на пение, но не на инструментальную музыку или речь. Эта популяция нейронов расположена в верхней части височной доли, рядом с областями, избирательно отвечающими за язык и музыку.
1064 588242
>>88240
Скорее всего просто самовнушение.
Ну или глюки как бы реальные, но тоже из-за самовнушения.
1065 588284
Есть какие-то профиты от коллективного разума? Или автономный юнит всё-таки гораздо эффективней?
Выглядит так, словно бы хайв майнд будет постоянно перегружен одновременным микроменеджентом кучи юнитов, которые даже посрать сами не могут.
1066 588291
>>88284
Коллективный разум не отключает тебе мозг, он только заставляет считать всех людей кроме лидера частью себя и дает информацию из мозгов специалистов.
1067 588292
Какие надо читать сайты, чтоб быть вкурсе всех прорывных открытий? Ну например в физике и биологии

Типо прям вот последний писк моды науки
1068 588295
>>88284

>Или автономный юнит всё-таки гораздо эффективней?


Автономный юнит это ползающий по земле кусок мяса с деформированными органами и без помощи со стороны абсолютно нежизнеспособный, воспитанные хотя-бы волками дети Маугли в разы жизнеспособней (эффективней).
1069 588335
Мне как-то попадалась картинка для троллинга, там был сойджак с открытом ртом, на футболке была эмблема "i f*cking love science" на лбу надпись 666, а к голове приделаны антенны от wifi роутера типо рога дьявола, очень нужна сейчас, но в гугле не нашел, скиньте у кого есть пожалуйста.
1070 588349
Наука наука наука встаем с колен, дадим пососать пиндосам. Приходишь с проектом теоретического исследования и препринтом. ОЙ, НУ ПОТОМ ПОСМОТРИМ, ОЙ НУ ЧЕТ Я НИЧЕГО НЕ ПОНИМАЮ, ОЙ А КАКАЯ АКТУАЛЬНОСТЬ, блаб
1071 588359
>>88349

>Приходишь с проектом теоретического исследования


Нахуй тебе приходить к кому-то, если исследование теоретическое?
Сиди пили сам.
1072 588362

> А КАКАЯ АКТУАЛЬНОСТЬ


И в чем он не прав кста?
1073 588423
Мескалин — ... также может быть синтезирован из ванилина.
https://ru.wikipedia.org/wiki/Мескалин

Как синтезировать Мескалин из ванилина?
1074 588435
>>88423
Как же тупые торчи доебали, надеюсь модер тебя забанит к хуям
1075 588438
>>88435
Не нравится - Не читай, Пидор!
1076 588440
>>88423
Наркоман, штоле? Или в глаза долбишься? Гуголь есть жи, жи ежжи, чертила

>Есть два основных пути:


>1: ванилин -> 5-йодванилин -> 5-гидроксиванилин -> 3,4,5-ТМБА -> 3,4,5-ТМНС -> мескалин.


>2: ванилин -> 5-бромванилин -> 3,5-диметокси-4-гидроксибензальдегид -> 3,4,5-ТМБА -> 3,4,5-ТМНС -> мескалин.


На какой путь сядешь, на какой мать подсадишь?
1077 588481
Как с эволюционной точки зрения объяснить любовь людей к прошлому?
1078 588483
>>88481
Старением и смертью.
1079 588504
Мы привыкли, что говоря про квантовую механику, волны изображают то в одномерном виде (линия графика), то на двумерной плоскости (пики и волны на ней).

Как же они визаулизируются в реальности? Они же, естественно, трёхмерны - их можно представить как колеблюющуся полупрозрачную округлую хуйню? Как боровские орбиты элкктронов в 3х измерениях?
1080 588505
>>88504
Судя по нарушению неравенства Белла их не существует "до".
1081 588506
>>88504

> Как же они визаулизируются в реальности?


Ни как. Если бы их могли видеть, то не было бы ни каких проблем. Волна это условная метафора к волнам на воде, но не имеющая к волнам на воде ни какого отношения.
1082 588508
>>88504
Можешь в виде яркости представлять.
1083 588509
>>88506
Да, я имел в виду, визуализация в голове, как это представлять.

>>88505
Факт.

>>88508
Да. Волна вероятности в виде неравномерных светящихся сгустков. Спасибо.
1084 588516
>>88504
В реальности у тебя будет точки/вспышки, никаких волн.
Только если ты дохуя раз измеряешь, то получишь неясное месиво. В случае электронов говорят об электроном облаке.

Если визаулизировать волновую функцию, то есть проблемы из-за комплексности. И это только для скалярных частиц. У электронов волновая функция многокомпонентная, да вдобавок еще является спинором. Цветов и измерений не хватить.
1085 588519
>>88509

>визуализация в голове,


Ни как.

>как это представлять.


Математическими абстракциями
1086 588520
>>88504
Как Фейнман советовал гуманитариям представлять. Типа у частицы есть "часы стрелочные" которые вращаются.

А если ты нормально с матаном дружишь, то это волны интуитивно представляется.
>>88509

> неравномерных светящихся сгустков.


Волна вероятности может не иметь максимумов и минимумов, банально плоская бегущая волна. Так что ты потеряешь инфу, если не волновую функцию будешь представлять квадрат модуля.
1087 588530
>>88516
Шизоид, какие точки, какие волны
"Волна" фотона видимого света пол микрона
Ты точку такую не увидишь какого размера он волна.
1088 588535
>>88126

>Алкоголизма не существует


Алкоголизм - не просто существует, его можно простыми методами показать и доказать. Хватит уже писать ерунду сюда.
Подтяни биологическую часть и медицину.

>Если и дед и отец такими были, то близка к 100% вероятность.


Лол.
Генетического алкоголизма не существует, но есть некоторые факторы к экзогенному этанолу, которые могут способствовать быстрому развитию алкоголизма. Просто потому, что у этаноловых людей существует нарушение обмена веществ, и это нарушение можно передать генетически. Но это не алкоголизм, это факторы простетических групп, которые могут быть переносимы в геном следующей особи в поколении.
1089 588536
>>88129

>бросил


Надо не бросать, а заменять эндорфиновым допингом спорта или секса. Или обжорства.
Будешь жиробасом, ебакой и качком-дурачком.
1090 588537
>>88166
Один учоннный тут остался...
1091 588543
>>88516
Пидорюш, тыскозал?
1092 588554
На хуй вашу навуку. Проще заработать в тиктоке.
1093 588615
Сап двач. MOND по вашему мнению тру или не тру? Какие новости?
1094 588616
>>84346 (OP)
Анчоус, почему уже столько итерации в роботехнике, а используют манипулятор стопы в такой ущербной форме?

Звездообразная, естественно упрощенная, чтобы не застревала, выгоднее же? И по гироскопической оптимизации. И по распределению веса. Можно сделать статично-гибкими, а можно отдельный инструмент в виде манипуляторов-палец, которые будут корректировать ориентацию в пространстве. Почему?
1095 588617
>>88615
и вообще, на каких ресурсах постоянно актуализируется информация по научной деятельности, теориям и их экспериментальным подтверждениям, чтобы не копаться в статье казалось бы 10 леьней всего лишь давности, а потом хоба и она признана несостоятельной не так давно и хуй проссышь.
1096 588619
>>88616

>Звездообразная выгоднее же?


А в чем именно выгода? Манипулятор у робота сделают там, где он нужен, а не там, где была свободная конечность. Твои птичьи когти нужны для того, чтобы хватать добычу. А роботу это зачем? У лошади копыто и никто не жалуется.
1097 588620
>>88619
Упоры по нескольким точкам плоскости, большая площадь соприкосновения с меньшим объёмом, распределение нагрузки, облегчение коррекции гироскопии. Ясно, что и на ходулях можно ходить и на колесе скакать, вопросов нет, но звезда выгоднее. А у лощади слишком долгий опыт обучения и модификации стопы и двигательно-координационного аппарата, как вида.
1098 588621
>>88620
То что ты называешь это не преимущества, а особенности. В каждом конкретном случае, они могут как давать преимущество, так и нет. А то, что у лошади длинная история эволюции говорит только о том, что лошадиное копыто также подтвердило свою целесообразность.
1099 588622
>>88621

>лошадиное копыто также подтвердило свою целесообразность.


Не спорю, для анатомически-конструкционных характеристик этого вида и его физических задач эта стопа подходит. Не про это речь.

>То что ты называешь это не преимущества, а особенности. В каждом конкретном случае


Ок, я возможно пропустил уточняющие в вопросе. Собакообразных и лощадеобразных, если так можно выразиться, мы не рассматриваем. Антропоморфы. Видны проблемы - покачивание корпуса, коррекция и стабилизация координации при сложных вертикальных перемещениях. Стопа в виде прямоугольника дает точку упора и стабилизации переносом веса только в двух направлениях. Трех- четерых- пятилучевая стопа даст множество точек. Прямоугольник не сможет дополнительно пружинить, амортизировать при конечной стадии ходьбы и прыжка, многолучевая это позволяет. При поднятие веса оптимизация координации также более точное, чем использование прямоугольной площадки.
1100 588629
>>88622

> Прямоугольник не сможет дополнительно пружинить, амортизировать при конечной стадии ходьбы и прыжка, многолучевая это позволяет.


С чего ты взял что для текущих задач это необходимо? Ещё не решена задача нормального двуногого прямохождения просто на «прямоугольниках», а ты хочешь ввести ещё кучу суставов, приводов и переменных для обработки ни как не относящихся к решаемой задаче прямохождения.
1101 588631
Если жиробас 100кг спрыгнет со стола высотой 1м на весы, то какое максимальное значение покажут весы?
1102 588632
>>88629

>С чего ты взял что для текущих задач это необходимо?


Правильный вопрос и можем утонуть с тобой в полемике, но мой то был не про это. Но прямоходящих делает не только Бостон Дайнемикс, а треть всей робототехники.

>Ещё не решена задача нормального двуногого прямохождения просто на «прямоугольниках»


И для решения задачи нормального прямохождение многолучевая лучше, я даже объяснил почему.

>ни как не относящихся к решаемой задаче прямохождения.


Экономяят, оптимизуруют и позволяют более тонко решать вопросы, напрямую относящиеся к прямохождению, соблюдению баланса и координации.

>а ты хочешь ввести ещё кучу суставов, приводов и переменных для обработки


Eще в первом посте написал.

>Можно сделать статично-гибкими



Хуету на пиках разработали не просто так. Пружинящий импульс амортизирует, дает импульс к толчку, позволяет мягко корректировать координацию.
1103 588650
>>88631
Чуть меньше прочности костей ног на продольное сжатие.
1104 588652
Максимально тупой наверное вопрос.

Настаивание воды на серебре. Это же хуйня? Жена заебала, заказал ей 100 грамм слиток чистого серебра, пусть развлекается, но ведь хуйня же? Да?
1105 588659
>>88652
Если не пьёшь воду из сточной говноканавы то точно никакого смысла.
Есть небольшая вероятность что могут выделяться как-то иога серебра и убивать бактерии и микробы, но схерали им просто в воду брать и выделяться... Плюс именно ионы, а не просто оксиды
В общем, это нужно химиков спрашивать.
Но зачем пить говняную воду? А с хорошей водой точно никаких положительных эффектов не может быть.
1106 588661
>>88659

>иога


*ионы
1107 588672
>>88652
Да, это хуйня.
И пить воду насыщенную серебром даже опасно для здоровья.
sage 1108 588682
>>88652

>жена


сажи нормихуесосу
1109 588683
>>88682
Съебись в /b/, чьмоня.
1110 588693

> Important remark: The picture of gravitons as the "mediators" of gravity, like photons are the mediators of the electromagnetic interaction, is actually incorrect. Unlike the electromagnetic interaction, gravity is fundamentally nonlinear. But gravitons are the quanta of linearized gravity, which means they are only suitable for describing very weak gravitational fields, such as gravitational waves.


Можете пояснить, что значит ЭМ линейна, а гравитация нелинейна?
1111 588720
>>88693
Линейность на уровне уравнений.
Решения уравнения Максвелла линейны - сумма их решений, является так же решением. Верно и обратное, любое решение можно разложить на сумму других решений.
Уравнения Эйнштейна в общем случае нет. Принцип эквивалентности очень сильный, поэтому требует от уравнений требуется ряд условий, которые убивают на корню всю линейность решений.
1112 588729
>>88720
Нихуя не понял, как это к реальности применить.
1113 588730
>>88729
Школу закончи и в универ поступи сначала
1114 588759
>>88730
Доран, нормально можешь объяснить?
1115 588787
>>88765 (Del)
Никогда, в реале будет концовка Тонга
1116 588798
>>88796 (Del)

>биороботов


А ты работаешь?
2 (1).jpg640 Кб, 2000x1333
1117 588830
Анохин это пиздабол? Слушаю его лекции, постоянно бла бла бла биоэтика бла бла бла перечисление заслуг американских ученых бла бла бла
1118 588831
Еще и на сомнительные колабы с Черниговской и каким-то маркетолухом ходит. Такое ощущение что наука в пидорашке просто сдохла нахуй, остались одни старпёры и шизики несущие околесицу. Сочувствую всем кому приходится заниматься наукой в таких условиях
1119 588836
>>88831
Вот Анохину и посочувствуй. Ты думаешь, он как-то по-другому Черниговскую воспринимает?
https://youtu.be/N6FALqdLCXw?t=6962
https://youtu.be/MXxIvvVH6PA?t=6857
https://youtu.be/LH-_NnFZIE8?t=379 (цитату Черниговская форсит)
В Википедии указано, что Черниговская:

>Заместитель директора НБИК Центра Курчатовского института.


Может Анохину приходится с ней "дружить"? Я не знаю.

То, что для тебя они что-то примерно одного уровня, это потому что ты ещё не всё понял. Разница гигантская. Анохин боится лишнего сказать и из-за этого стать пиздаболом, поэтому говорит одно и то же обычно, либо про его исследования памяти, либо про его теорию когнитома.
1120 588849
Ваши ставки,

когда появится сильный ии на уровне взрослого человека 180 айсикью?

когда появится супер ии/наступит технологическая сингулярность?
1121 588852
>>88836
Скорей навука в пидерахе просто сдохла, остались лишь нищие пенсионеры с импотенцией уныло отрабатывающие гранты создавая имитацию бурной научной деятельности. У них уже там всё засохло, никакого энтузиазма, никакой цели. Студент первокурсник спрашивает г-н Анохин а создадут ли когда нибудь ИИ и что там для этого нужно? Анохин: РЬЯЯЯЯЯ ЭТО ТЕХНИЧЕСКИ НЕВОЗМОЖНО ИШЬ ЧО УДУМАЛ БОХ НАКАЖЭЭТ. И вот с таким настроем всей этой толпе замотивированных вчерашних школьников учиться у этих совковых дегенератов.
1122 588853
>>88849
В марте следующего года
1123 588856
>>88849
Если серьезно подходить, то 1-2 года до того как языковые модели будут на уровне AGI с ограничениями в области пространственного-физического мышления. Есть мнение, что AGI будет необходимо физическое тело для того чтобы нормально функционировать, но я не убежден. Релиз ГПТ-5 не за горами, есть мнение что оно будет способно наконец-то обрабатывать информацию на более высоком семантическом уровне а не пережевывать банальщину как ГПТ-3.
Если нужны мнения из итернета, то смотри здесь, например:
https://lifearchitect.ai/agi/
Или здесь:
https://www.metaculus.com/questions/3479/date-weakly-general-ai-is-publicly-known/
Учти что индивидуальные независимые догадки толпы имеют тенденцию приближаться очень близко к правильному ответу, но когда люди голосуют в открытую, зная ответы своих сотоварищей, они городят хуню, так что суди сам.
Ну и моя собственная догадка: завтра, можете скринить.

>>88852
Тащемта лично знаю одного студента которому отказали в защите кандидатской потому что диссертационному совету не понравилось, что там применялась нейросеть.
1124 588861
сап двачь
химики тут?
мне нужно было продезинфицировать некоторые титановые штуки и я кинул их в перекись водорода (у меня тут не лаба, я обычныый человек)
они блять пожелтели
я не знал что так будет
со сталью такой хуйни не было
щас загуглил рял титан реагирует неплохо так с перекисью
вопрос как в домашних условиях вернуть титану обычный металлический цвет?
1125 588874
>>88856
Nov/2020
30%: Connor Leahy, Co-founder of EleutherAI, re-creator of GPT-2, creator of GPT-J & GPT-NeoX-20B, said about OpenAI GPT-3: ‘I think GPT-3 is artificial general intelligence, AGI. I think GPT-3 is as intelligent as a human. And I think that it is probably more intelligent than a human in a restricted way… in many ways it is more purely intelligent than humans are. I think humans are approximating what GPT-3 is doing, not vice versa.’


Кекус
1126 588878
>>88852

>Студент первокурсник спрашивает г-н Анохин а создадут ли когда нибудь ИИ и что там для этого нужно? Анохин: РЬЯЯЯЯЯ ЭТО ТЕХНИЧЕСКИ НЕВОЗМОЖНО ИШЬ ЧО УДУМАЛ БОХ НАКАЖЭЭТ.



Хватит из головы-то выдумывать, или ссылки откуда ты это берёшь приводи. Он обычно отвечает цитатой, что "сложно прогнозировать, особенно будущее". Чтобы не напиздеть, опять же, потому что он давно живёт, многое видел. Многое, что прогнозировали другие по хайповым темам, потом не сбывалось. Либо речь про то, когда искусственный интеллект обретёт сознание, то он отвечает, ссылаясь на теорию интегрированной информации, какие нужны условия для того, чтобы в системе возникло сознание, а "симуляция урагана на компьютере не обладает качеством мокроты". Про Бог накажет это тоже не к нему, ни про Бога, ни про алармизм как у Черниговской. Анохин всегда наоборот отвечает типа "ну и как вы остановите прогресс? в любом случае, если не мы, то кто-то это сделает". Как будто про какого-то другого человека говоришь вообще - https://youtu.be/GVaiwyEuQwY?si=ZP18JiwuE6HVwLlN&t=3748.

Ты что ли замотивированный вчерашний школьник? Что за вопрос идиотский, когда создадут ИИ? ИИ создан давно. Сильный ИИ ты имеешь в виду или что вообще?
1127 588882
>>88878
Почему у него в лекциях постоянно какие-то пространные философские рассуждения вместо физиологии, обьяснения того как мозг может кодировать информацию, как и где хранится память, какие нейроны активируются, как они друг с другом взаимодействуют?
1128 588883
>>88882
Потому что вопрос сознания всегда был чисто философским вопросом. Потому что это оxyенно, потому что уровень абстрактного мышления позволяет рассуждать на таком уровне. Никакой физиологией трудная проблема не решается.
Про память у него полно лекций. https://www.youtube.com/watch?v=I_sOSo_HRnA
1129 588884
>>88883
https://youtu.be/QTQJzQo9saE?si=lqX7xgJArPwGp3UH по-моему, здесь было дано интересное определение памяти. "Память это способность системы воспроизводить свои прошлые состояния". Не помню точно, должно быть в видосе. Где память? Везде, где есть нейроны, способные модифицироваться в ответ на какое-то воздействие, и потом воспроизводить эти свои прошлые состояния. Это тоже считать за пространные философские рассуждения? Чтобы что-то начинать изучать, нужно сначала это определить.
1130 588887
>>88883
>>88884
Тогда зачем на рассуждения выделяются деньги из госбюджета? Пусть рассуждает бесплатно, либо за мрот, как все рабсияне

>дано интересное определение памяти


Нахуй нам определение? Какие это несёт технологические новшества и научные открытия?

>Это тоже считать за пространные философские рассуждения?


Именно, более того это беспруфный пук. Это реально звучит как научная импотенция, когда мы говорим о памяти мы говорим о её локализации и о том как эти структуры формируют чувство непрерывной самости, в каких нейронах это зашифровано, как это зашифровано, как это раскодировать, как это математически описать, смоделировать, перенести на\в ИИ
29:15 "врёти, ни будит, ни смогут, слишком сложно, т, беспочвенные фантазии пук хрюк" перед этим два часа пиздел о своей полуфилософский хуите
33:43 "технически нереально, этически невозможно" слушай дальше чё старуха несёт, у неё ВРЁТИ на вопрос об уже состоявшемся эксперименте. Перед этим пол часа пиздели об этикехуетике, мням ням нам нужно подумать о будущем мням мням искуственный интеллект должен быть наделен моралью. Просто какой-то лютый пиздёж по своей структуре и наполненности не отличающийся от вопросов школоты про нейроинтерфейсы и когда нас уже загрузят внутрь ИИ. Просто какое-то бла бла бла, кучка престарелых выблядков прожирающих гранты. Посмотри хоть раз в жизни пендосские лекции с настоящими учеными не лишенными запала, каких-то надежд на науку, умеющих рассказать что-то новое просто и интересно, а не вот этот вот пространный пиздёж и тысячу раз пережеванный кал с горделивым апломбом
1131 588888
>>88884

>это способность системы воспроизводить свои прошлые состояния


Мде, звучит как предварительное определение упругости в учебниках по классической механике для ПТУ.

>>88884

>Где память? Везде, где есть нейроны, способные модифицироваться в ответ на какое-то воздействие, и потом воспроизводить эти свои прошлые состояния.


Крч, механика материальных нейронных точек.

>Это тоже считать за пространные философские рассуждения?


Да

>Чтобы что-то начинать изучать, нужно сначала это определить.


Только процесс определения продолжается и во время исследования, тем самым начальное определение отрицается как чисто эвристическое.
170928214866216732.jpg280 Кб, 1080x2340
1133 588891
Возможно ли восстановить звук по этой пикче?
1134 588892
>>88861
Попробуй кислотой помыть, или шарик пихнуть вместе с кусочком алюминия (чтобы они соприкасались) в раствор горячей соды. М.б. сработает, но может быть и нет. Пожелтеть то может капитально.
1135 588895
Возможно ли восстановить звук по этой пикче?
1136 588896
Дурачусь с магнетометром. Зацените как он видит работающую микроволновку, снаружи.
1137 588899
>>88849

>когда появится супер ии/наступит технологическая сингулярность?


10 травня
>>88896
Что за приложение?
1138 588900
>>88895
Частично да.
Но на пикче потеряна много информации, главное из которых это информация о фазе.
1139 588901
>>88883
Тупая трапошалава, ты? Узнал тебя по тупорылым высерам бабского мозга.
Вот уж ирония да? Мозг у тебя женский, тупорылый, а днк мужское, и женщиной ты никогда не станешь. Так и будешь чушкой-дегенератом с женским мозгом.
1140 588902
>>88899

>10 травни



аххахахкха заорал

Приложение - phyphox
1141 588909
>>88882

>как и где хранится память


Какую часть никто не знает ты не понял?
Есть ряд теорий, структура нейронов, РНК, всё это. Из доказательств только то что таким способом информацию в принципе можно хранить. Может вообще оказаться что память храниться в сознании а мозг выступает как набор указателей.
1142 588910
>>88909
Что за сознание? Где оно хранится?
1143 588914
>>88910

>Что за сознание?


Место, где находятся квалиа и сформированные из них ментальные структуры. Хотя, место скорее будет подсознанием. Сознанием будет активный, в данный момент, набор квалиа.

>Где оно хранится?


Нигде, оно не локализовано. Его содержание связано между собой, но тоже не локализовано. Следит за миром, ловит подходящий сигнал, подбирает под него ощущение, активирует прицепленную к нему сетку ощущений, потом само же и смотрит на то что получилось.
1144 588915
>>88887
Тебя чего так разворотило? Истеричный школьник посмотрел пару популярных лекций и не доволен, что там внезапно РАССУЖДЕНИЯ. Работу он на работе работает, эксперименты там ставит, статьи пишет. Это ты почему-то хочешь смотреть популярные выступления, в которых люди, пришедшие послушать почему-то хотят слушать про этикухуетику. Самому Анохину, как можно заметить по ссылке в этом посте >>88878, это всё уже невозможно нaстоeбало. Осиль какую-нибудь ещё лекцию что ли, чтобы не нести ерунду про локализацию памяти. О какой именно памяти ты говоришь, автобиографической, декларативной, процедурной? А так память она везде, где есть нейроны, блять. На идиотские вопросы про "цифровое бессмертие" пусть какие-нибудь фантазеры-пиздoболы отвечают, а к Анохину с такой xуетой не пристают.

>>88901
Это вообще какая-то xyета, как тyпая баба тут истеришь только ты, и в целом слишком неуместно эмоционально мыслишь.
1145 588918
>>88887
Школьник пытается пропихнуть тезис, что Анохину почему-то нельзя рассуждать, как бы это по-идиотски этот тезис не звучал. Чтобы как-то преодолеть когнитивный диссонанс, что его рассуждения почему-то не совпадают с рассуждениями именитого учёного. Хитрый план.
1146 588919
>>88915

>Это вообще какая-то xyета, как тyпая баба тут истеришь только ты, и в целом слишком неуместно эмоционально мыслишь.


Ты чего порвалась тупорылая трансшалава? Зачем ты женский пол в своём высере оскорбляешь? У тебя ж их мозг.
1147 588920
>>88919
Мне должно быть обидно, что ты дурачок и с кем-то меня путаешь? Это так не работает.
1148 588921
>>88920
С чего тебе должно быть обидно? Ты такой с рождения, полудурок с женским мозгом, это дефект врождённый, генетический.
1149 588926
>>88918
Кто сказал нельзя? Тезис в том что он получает лярды из госбюджета, а на выходе пространные рассуждения, которые может вести любой у кого есть способность рассуждать.
>>88915
Рили трансшалаава отвечает что ли? Какой-то бессодержательный пук
>>88914

>оно не локализовано


Оно должно как-то описываться физически\математически и моделироваться, если этого не происходит то это либо ученая импотенция, либо непознаваемая душа
1150 588927
Например, как нейроны передают информацию, как они ее кодируют, по каким паттернам и тд
1151 588931
Вода является проводником. Скат умеет генерировать электричество. Каким образом при атаке скат не убивает все живое вокруг себя?
1152 588937
>>88926
Связаны друг с другом же, модели сознания обычно строят в виде сетки. Память тригеррится ощущениями и зависит он настроения. Другое настроение - другие воспоминания.
1153 588944
>>88927
По себе передают через волну потенциала по своей мембране.
Между собой через различные синапсы - в основном структура в соединение нейронов (и других клеток), где на мембранах сидят особые белки, которые реагируют на выброс веществ(нейромедиаторов) и запускают волну(или глушат) волну потенциала на своей мембране.
Выброс нейромедиаторов может быть как и от сигнала от другого нейрона, либо притечь из другого места, либо другая клетка выработала.
Нейроны сами по себе не кодируют информацию, они только отслеживают входящие сигналы от синапсов, и преодоление порога возбуждения запускают волну на своей мембране, потом на некоторое время уходить в глухой режим восстанавливаться.
Вся магия происходит когда нейроны объединяются в нейронные сети.
1154 588948
>>88926
Надо второй раз объяснять для тупорылых, что здесь >>88915 написано? Работа нейробиолога отражается в экспериментах и научных статьях в рецензируемых журналах. На популярных выступлениях для быдла, которые вы смотрите, могут быть только "пространные рассуждения". Это блять даже не лекции, просто какие-то "открытые дискуссии" на тему, которую придумали те, кто Анохина и остальных приглашал. Быдло хочет слушать про угрозу ИИ, Анохину приходится обсуждать эту xyету, потому что его туда пригласили.
Школотуна удивляет, что серьёзный учёный нейрофизиолог отвечает на вопросы про хyeту типа переноса сознания и цифрового бессмертия "технически невозможно" и "беспочвенные фантазии". Вы ожидали, что он ответит: "Бля, оxyенно, сейчас оцифруемся, сразу сознание туда перенесётся"? Для него это звучит как "нарисуем портрет человека, он сразу начнёт разговаривать, потому что туда душа перелетела". "Этически проблематично" это не "ИШЬ ЧО УДУМАЛ БОХ НАКАЖЭЭТ", а, возможно, про проблему телепорта из вами ненавистной философии. Но вам же думать запрещено, надо нести xyйню и удивляться, почему люди, которые понимают как мозг работает, с вами не согласны. Но конечно же у вас есть какая-то своя спонтанная философская позиция по теме сознания, какие-то вами выдуманные теории памяти, которые просто не совпадают с позицией, основанной на многолетнем опыте работы, самого Анохина. >>88889 здесь по первой ссылке абсолютно адекватный ответ, который следует из теории когнитома и про "некий закон" это из Теории интегрированной информации. То, что школотунами воспринимается как "врёти, ни будит, ни смогут, слишком сложно, т, беспочвенные фантазии пук хрюк" это следствия из проработанной теории, которая у Анохина есть, в отличие от школотуна у которого реально только беспочвенные фантазии и эмоции.
Все ответы на идиотские школотунские вопросы есть в нормальных лекциях Анохина, которые никто не считает нужным посмотреть, перед тем как истерить, даже если это ответы типа "технически невозможно" и "это в принципе не так работает" (вся ваши поверхностные представления о памяти).
1154 588948
>>88926
Надо второй раз объяснять для тупорылых, что здесь >>88915 написано? Работа нейробиолога отражается в экспериментах и научных статьях в рецензируемых журналах. На популярных выступлениях для быдла, которые вы смотрите, могут быть только "пространные рассуждения". Это блять даже не лекции, просто какие-то "открытые дискуссии" на тему, которую придумали те, кто Анохина и остальных приглашал. Быдло хочет слушать про угрозу ИИ, Анохину приходится обсуждать эту xyету, потому что его туда пригласили.
Школотуна удивляет, что серьёзный учёный нейрофизиолог отвечает на вопросы про хyeту типа переноса сознания и цифрового бессмертия "технически невозможно" и "беспочвенные фантазии". Вы ожидали, что он ответит: "Бля, оxyенно, сейчас оцифруемся, сразу сознание туда перенесётся"? Для него это звучит как "нарисуем портрет человека, он сразу начнёт разговаривать, потому что туда душа перелетела". "Этически проблематично" это не "ИШЬ ЧО УДУМАЛ БОХ НАКАЖЭЭТ", а, возможно, про проблему телепорта из вами ненавистной философии. Но вам же думать запрещено, надо нести xyйню и удивляться, почему люди, которые понимают как мозг работает, с вами не согласны. Но конечно же у вас есть какая-то своя спонтанная философская позиция по теме сознания, какие-то вами выдуманные теории памяти, которые просто не совпадают с позицией, основанной на многолетнем опыте работы, самого Анохина. >>88889 здесь по первой ссылке абсолютно адекватный ответ, который следует из теории когнитома и про "некий закон" это из Теории интегрированной информации. То, что школотунами воспринимается как "врёти, ни будит, ни смогут, слишком сложно, т, беспочвенные фантазии пук хрюк" это следствия из проработанной теории, которая у Анохина есть, в отличие от школотуна у которого реально только беспочвенные фантазии и эмоции.
Все ответы на идиотские школотунские вопросы есть в нормальных лекциях Анохина, которые никто не считает нужным посмотреть, перед тем как истерить, даже если это ответы типа "технически невозможно" и "это в принципе не так работает" (вся ваши поверхностные представления о памяти).
1155 588949
>>88931

> Вода является проводником


Нет.

> все живое вокруг себя


Електрический ток наносит урон только там где он проходит. Проходит он по пути наименьшего сопротивления между точками с разными потенциалами. Эти точки находяться (тут я не уверен) либо на разных электрических органах ската (в этом случае при атаке скат обжимает тело жертвы, притрагиваясь эелектрическими органами с двух сторон и между ними возникает электрический ток) либо в разных частях одного электрического органа - в таком случает он прижимается органом и ток проходит от одной части в другую через тело жертвы вызывая паралич мышц. Но воду вокруг это практически никак не воздействует, даже если она и проводит электричество (из-за растворенных солей).
1156 588952
>>88926

>Тезис в том что он получает лярды из госбюджета, а на выходе пространные рассуждения



Ещё раз чтобы наверняка. Лярды из госбюджета учёные получают не за выступления перед быдлом. В том-то основная претензия к Анохину, что он не бодрый шутливый, но поверхностный, популяризатор, а настоящий учёный. Его сложновато понимать, когда он говорит о том, чем он самом деле занимается, то что касается его непосредственной работы. Поэтому он выбирает рассказывать что-то, что обычному человеку близко и понятно, есть большая категория людей, которым нравятся пространные философские рассуждения. По крайней мере они им ближе, чем ранние и поздние гены, включающиеся при запоминании, математические моделирование нейронных гиперсетей и следствия из IIT. Всё последнее тоже рассказывается, но в других лекциях для других учёных.
1157 588954
>>88889
Ещё видимо нужно для тупорылых школотунов, которые слышат "врёти, ни будит, ни смогут, слишком сложно, т, беспочвенные фантазии пук хрюк", расписать, что на самом деле ответил Анохин.
- ...эти протезы мозга доделают ли когда-нибудь?
- Для грубых серийных структур типа гиппокампа и его полей... (Т.е. да, но для определённых структур, далее пояснение каких.)
- Мозг будет воспринимать это как внешнее устройство или он будет это воспринимать как то, что он лучше помнит что-то? И не будет замечать какое-то изменение.
- Не будет. (Относится к "не будет замечать". Далее постулат из IIT "исключение". Потом парадокс корабля Тесея.)
- бла бла бла замена искуственными нейронами
- беспочвенные фантазии, нужно изучать мозг

На первый вопрос он ответил уверенно, потому что уже проводились эксперименты на мышах с заменой гиппокампа. Где-то в плейлисте на канале Дениса Тимофеева был курс лекций Анохина для МГУ, там он про это упоминал. На последний вопрос он так ответил как раз потому что от него требуется чисто умозрительный философский ответ, сослаться ни на что кроме философского мысленного эксперимента ему тут не на что.
У школотуна явно какие-то проблемы с понимание смысла услышанного/прочитанного. Не ведитесь на истерики.
1158 588958
>>88954
>>88952
>>88948
поссал тебе в рот
1159 588960
Анончики, а какая проблема более сложная, сложная проблема сознания, или сложный вопрос гендера?
th-2971931678.jpg23 Кб, 474x266
1160 588982
Сап /sci доски по химии нет, поэтому спрошу тут. Мне нужен гелеобразователь совместимый с лимонной кислотой и/или водным ее раствором.

Спасибо.
160871936319740500.png1,9 Мб, 1000x1447
1161 588983
>>84346 (OP)
Вроде был тред ядерщиков, но чего-то все пролистал и не нашел.

Почему отказались от АСТ?
https://ru.wikipedia.org/wiki/АСТ-500

Были две попытки. Собственно с заброшки ГАСТ https://youtu.be/er1l0wdl-ZM и заинтересовался вопросом. Экономика, безопасность, рациональность вроде в норме и даже привлекательна. Понятно, что тогда ЧАЭС внесла корректуру привлекательности в мировой порядок, но потом, а уж и сейчас особенно, все постепенно пришло обратно. Да, без комсомольского фанатизма, но строят и осваивают. И вопросов она очень много закрывает, особенно в наших реалиях. Почему тити мнут?
1162 588987
>>88982
У нас тут есть отдельный Химии тред: https://2ch.hk/sci/res/578773.html (М)
1163 588992
Насколько реально сделать надпись, видную только через фильтр(плёнку, стекло, желательно прозрачное)? Всё перерыл, наткнулся только на ебучие крапленые уф покерные карты. Есть мысль, что как-то можно применить поляризационную пленку, или фотопленку, но зацепиться ни за что не могу.
1164 588999
>>88992
Поляризация была бы идеальна, но я не уверен что нужного тебе можно добиться. Тебе нужна надпись одного цвета с фоном, но такая чтобы сама надпись отражала поляризированый свет а фон нет (или наоборот). Что-то подобное происходит в LCD мониторах, они покрыты поляризационной пленкой иначе там нихуя не видно на случайном видео с ютуба на 1:20 демонстрация монитора с снятой пленкой https://www.youtube.com/watch?v=jHiM4ph6kUI но это монитор - он излучает поляризированый свет, а тебе нужно добиться поляризации при отражении.
Есть еще изображения в которых надпись состоит из синех точек вокруг которых накидано смесь из красных и желтых точек разных оттенков. При обыном освещении надписи практически не видно, просмотре через красный фильтр синие точки становятся темными и проявляются на фоне красных. Но я не уверен что тебе такое это подойдет по качеству. Попробуй погуглить decoder lens или что-то подобное.
17094542632420.mp412 Мб, mp4,
1280x720, 1:58
1165 589007
1166 589012
>>88949
А это тогда как объяснить?

https://secretmag.ru/criminal/rossiyanka-zaryazhala-telefon-v-vannoi-i-umerla-ot-udara-tokom-kak-ne-povtorit-eyo-sudbu-25-04-2023.htm

22 апреля в одной из квартир города Дивногорска (Красноярский край) нашли тело 21-летней девушки. Она лежала в ванной и не подавала признаков жизни. Силовики сразу же начали проверку и выяснили, что, отдыхая одна дома, россиянка решила принять ванну и взяла с собой телефон. В результате её убило током.
Собачье-сердце-смищные-картинки-1552231.jpg84 Кб, 1024x775
1167 589014
Как научный мир относится к доктору Менгеле? Он безумный маньяк или величайший ученый? Насколько его деятельность продвинула вперед прогресс?
1168 589018
>>86700
ты наверное уже умер, но я бампану пост, потому что сам такое же ищу
1169 589023
>>89012
Там наверное был контакт воды с водопроводной трубой (которая практически гарантировано заземлена, смотря конечно что у них там за водопровод) тоесть образовалась электрическая цепь которая прошла через тело.
170954565464127871.jpg307 Кб, 1080x2340
1170 589036
Правильно ответил?
1171 589048
>>89046 (Del)
Пизду тебе пришить, что-ли?
1172 589073
Креационисты утверждают, что наука не нашла переходного звена от обезьяны к человеку. А как же негры?
1173 589076
https://monro1.bib.bz/6-obratnyy-obraz почему никому, кроме Монро, не удалось забраться так далеко в исследованиях ВТО? Почему чувак побывал в параллельных вселенных, но всем буквально насрать?
1174 589078
>>89073
Уёбывай в /po/
1175 589098
Что ученые двача думают по поводу этого тренда в физике?
https://www.youtube.com/watch?v=GL77oOnrPzY
show (16).png1 Кб, 256x50
1176 589117
>>88983
Бамп вопросу. Ну неужели никто мирным атомом не увлекается?
jf4YnBdTyi8.jpg98 Кб, 927x740
1177 589119
1. Насколько научен перенос сознания на компьютер / в другое тело? Если серьёзно, то выглядит как лажа, потому что сознание представляется скорее непрерывным процессом, чем некой энергией / матрицей / структурой, которую можно куда-то перенести с осознанием собственного "я".
2. Как отличить клона от оригинала? Допустим, телепортация. Тело разделяется до субатомного уровня в одном месте и воссоздаётся в точности до нейрона в другом. Субъективно телепортированный уверен, что действительно произошла телепортация. Объективно никакой разницы "до" и "после" тоже не наблюдается. Как доказать, что телепортированный — просто клон, копия с заложенными воспоминаниями, а оригинал субъективно испытал мучительную смерть от дезинтеграции и ушёл в небытие?
image.png884 Кб, 1200x630
1178 589136
>>89119
Пыня, угомонись уже.
1179 589137
>>89076

>Почему чувак побывал в параллельных вселенных, но всем буквально насрать?


Где пруфы, что это действительно реально существующие параллельные миры, а не виртуальные глюки, как сон?
Это явление даже называется наукой официально - осознанные СНОВИДЕНИЯ.
1180 589138
LSD, кстати, легко позволяет путешествовать по астралу любому желающему. Но мозг необратимо разрушается, потому и под запретом во всем мире.
image.png765 Кб, 1200x538
1181 589142
>>89078
Левачок, спок.

Как сообщает "Би-би-си", знаменитый биолог в телепередаче вновь поднял вопрос о том, что гены влияют на различие между средним IQ представителей европеоидной и негроидной рас.

90-летний ученый и ранее выдвигал подобные предположения. В частности, в 2007 году он заявил, что будущее Африки представляется ему довольно печальным, так как "вся наша социальная политика основана на предположении, что их интеллект соответствует нашему. Тестирования показывают, что это не так".

Как подчеркивал Уотсон, он надеется, что все люди равны, но "кому приходилось иметь дело с чернокожими подчиненными, тот знает, что это не так".

Хотя генетика это же буржуазная лженаука, продажная девка капитализма, коммунистам она ни к чему.
1182 589144
>>89142
Нацик, спок.
Пиздеть - не мешки ворочать. Где пруфы врождённого отличия iq между расами?
1183 589147
>>89144
Какая разница какой у человека IQ если он нигде не котируется? Президента выбирают по IQ? Может на работу по IQ берут? Может женятся по IQ? Очередная маняоценка чьё место лишь в статьях маняоценщиков.
1184 589149
>>89138

> Но мозг необратимо разрушается, потому и под запретом во всем мире.


Мозг и от водочки разрушается, однако её не запрещают...
1185 589158
>>89117
Бамп, вопросу и еще два добавочных.

Звук. Возможно ли гасить звуковые волны другими звуковыми волнами с уменьшением частоты обоих? Приведение обеих волн к практически нулевой?

Иерархия. Во многих системах используется иерархия - упорядоченная, пирамидальная расстановка от более ценного пункта этой системы к менее. Биология, программирование, делопроизводство, организация, да везде. Есть ли антагонист этой упорядоченности? Я не говорю про отсутствие системы, анархию, хаос, а другую систему упорядоченности и организации. Понимаю, что иерархия это самый оптимальный, самый логичный способ, продиктованный природой, но интересуют другие, жизнеспособные или хотя бы имеющие физическое применение.
1186 589159
>>89158

>Приведение обоих волн к практически нулевой?

1187 589160
>>89158
Периодическая система химических элементов
1188 589161
>>89158
>>89160
Да, спасибо. Конкретизирую. Преимущество иерархи - универсальность, ей легко упорядочить системы в разнообразных сферах жизни. ПСХЭ узконаправленная.
1189 589175
если соединить самую сильную кислоту с самым сильным основанием, будет самая сильная сiль?
1190 589181
Модератор, удаляй эту гниль нахуй.

>>88852
1191 589185
>>89147

>Какая разница какой у человека IQ если он нигде не котируется?


Нигде это видимо в транс-негро-лгбт помойках
1192 589186
>>89119
Это как менять колеса на едущем автомобиле, сложно, но возможно
1193 589187
>>89185
У тебя брали тест IQ при трудоустройстве? Если не брали, то хули ты пиздишь?
170988521176314216.jpg30 Кб, 720x677
1194 589188
Сегодня смотрел ютубчик, обратил внимание на счетчик. X2 чтоли стоит? Нет. Включил таймер и охренел, почему секунды так быстро херачат? Это оскуфение?
1195 589189
>>84346 (OP)
PAIRING СЕТИ И АНТИСЕТИ
сетевой обмен статического вещества
vs
замораживание случайно распределенной вселенной
1196 589190
>>84346 (OP)
Возможно ли создать материал, который пропускал свет только в одном направлении? Этакий оптический диод.
1197 589192
>>89188
Да, с возрастом время начинает быстрее идти. Точнее ты медленнее соображаешь.
1198 589194
>>89188
По теории удвоенного времени ты тратишь время не только потому что таймер идёт, но и потому что на него смотришь. Поэтому оно кажется в два раза быстрее.
1199 589209
>>89188
На /zog есть тред про ускорение времени. Тебе скинуть? Без шуток.
proofs.png324 Кб, 1000x820
1200 589231
>>89144
Сойджек, покажи мне ниггеров-изобретателей или теоретических физиков. Или великие черные африканские цивилизации древности. Доколумбовы цивилизации Америки были, африканских до колонизации - не было.
1201 589232
>>89149
Интересно, видения при белой горячке и ЛСД имеют одинаковую природу?
От водки просто гои подыхают не так быстро, идеально для геноцида.
Табак тоже вызывает рак легких ближе к старости - гой успешно проработает, а потом отправится сразу в могилку при наступлении пенсионного возраста.
1202 589237
>>89232
От него гои вообще не подыхают, некоторые даже лечатся; его в ходе плановой морал паники запретили. Как теория заговора тут может быть что из-за него многие бросали водку, но тут проблема что и алкоголь в США запрещать тоже пытались.
Природой видений может быть посылка кривого сигнала в сознание, которому приходится его как-то интерпретировать.
hrs720125-n-zz999-001.jpg37 Кб, 568x576
1203 589243
>>89231

>покажи мне ниггеров-изобретателей или теоретических физиков


Смотри на картинке.

>африканских до колонизации - не было


Пиздец, Мали, Аксумского царства, Могадишо, Вагаду, всего этого не было, расходимся.
Покормил.
1204 589246
>>89243
Нигеры которые стали учеными/изобретателями только из-за того что их в детстве/в утробе похитили инопланетяне и генетически модифицировали чтобы хоть как-то простимулировать этот регион земли упорно не желающий выходить из каменного века.
image.png474 Кб, 643x358
1205 589260
Драсте,

Тут в тиктоке на заборе написали некие научпоперы, якобы сенсация! ДНК жизни на земле образовалось 10 миллиардов лет назад. Ну то есть старше земли. Ну то есть занесена из космоса, панспермия и все дела.

Я эту инфу услышал неделю назад на канале одной научпоперши средней руки в тикток свинарнике. Ну я послушал, откоментил и закрыл. А теперь вот неделю думаю. А что если и правда блять есть такой доклад от каких-то малолмальски нормальных ученых? Тогда и правда сенсация.

Она ссылалась на каких-то ребят уважаемых с гугливанием работы и что там есть какой-то русский хуй даже в команде или два.
Я гуглил англ про ДНК возраст новости и ничего не нашел.

Кто что в курсе?
1206 589261
>>89260
Узнать точный возраст ДНК нельзя. Можно пытаться угадывать время необходимое эволюции чтобы создать такую ДНК. Точность, конечно будет ни к черту. Ведь разные организмы по-разному влияют на свою эволюцию. В зависимости, опять же, от ДНК. Такое хрен посчитаешь.
1208 589280
>>89273
Какая-то хуита для девочек подростков.
1209 589286
1210 589296
>>84397
Да, можешь. Гугли пьезоактуаторы.
1211 589304
>>89014
Да не особо. Многие из его данных известны из медицины катастроф и военной медицины. Просто он дал дополнительную статистику по реакции человеческого тела на всякий пиздец. Статистика соответствует данным из других источников.
1212 589305
>>88631
Представь, что весы - это платформа, стоящая на пружине. Показания весов пропорциональны сжатию пружины. Сжатие при статической нагрузке 100кг известно. Массой платформы и пружины по сравнению со 100кг можно пренебречь.

Задача школьная, можешь сам решить, борда 18+.
1213 589315
>>89243

>Мали, Аксумского царства, Могадишо, Вагаду


Пара деревень уже великая цивилизация, ору с сойбоев. Какой вклад все эти макаки внесли в развитие человечества?
Вот Карфаген и Эфиопия - другое дело, но они и не на 100% черные.
1214 589316
Бывают ли кислые на вкус съедобные вещества, но с низкой кислотностью или вообще щелочи?
1215 589317
Можно ли в теории создать какой-либо наркотик из отходов добытой из под горы руды?
В теории
1216 589320
>>89315
Где твоя цивилизация, лоу iq ниггер?
1217 589332
>>89317
Только если там полно какой-нибудь мышьяка, но он слишком токсичный, так что ширнутся только на один раз.
61TkRmF-AfL.ACUF1000,1000QL80.jpg38 Кб, 788x1000
1218 589352
Собираюсь прочитать Фейнмановские лекции по физике из любопытства. Но вдруг за полвека появились альтернативы получше? Может кто-нибудь подскажет? Если важно, сам я математик, но физику знаю только на школьном уровне, и практически ничего не помню.
1219 589364
>>89352
Альтернатив особо нет.
Сами эти лекции так себе.
Их имеет смысл читать, если ты школьник/студент с развитой "физической" интуицией, и тебе нужно ее формализовать. В противном случае это покажется какой-то несвязанной бездоказательной хуитой.
Если ты мамкин математик, то у тебя жопа будет полыхать от того как в них используются привычные методы.
1220 589370
По какой причине может (эпизодически) начинать крайне активно образовываться статический заряд (помимо синтетической одежды, незаземлённой электропроводки в помещении и бега/быстрой ходьбы/иных активных движений)? Содержание алкоголя в организме (вплоть до 1 стакана свежего кваса, выпитого утром)? Эмоциональное состояние/гормональный фон? Что-то ещё?

Сдаётся мне, перечисленное выше в скобках, скорее всего, на частоту эпизодов и силу заряда влияет меньше, чем то, что под вопросами...
420fa9c73e9879fbc135ecd100c96179.jpg127 Кб, 736x919
1221 589372
Анон, подскажи, пожалуйста хороший учебник по теории игр?
1222 589373
>>89370
А, с незаземлённой электропроводкой может быть связано ещё кое что. В последний эпизод, заряд был настолько сильный, что при попытке заряд снять, реагировало даже оборудование, вообще-то, изолированное от той батареи, через которую снимал заряд я.
1223 589374
>>89370

>помимо синтетической одежды


Вот это в первую очередь
1224 589375
>>89364
Хмм, ладно, посмотрим как пойдет. Спасибо
1225 589377
как ввести человеку новые гены? типа шприцом ввести раствор гена и он сам равномерно встроится в днк всех клеток?
1226 589378
Можно ли установить, сколькиюродными родственниками все люди мира друг другу приходятся в среднем?
1227 589380
>>89374
Это понятно, просто одну и ту же толстовку ношу постоянно, но бывает, что хоть НЖМД голыми руками в ней спокойно носить можно, а бывает - что только после нескольких подходов к батарее за минуту. Во втором случае, даже если снять толстовку, поднести её к батарее, потом повесить, после этого ещё раз подойти к батарее, и только после этого продолжить работать (за настольной, т.е. без тачпада, пекарней), при следующем вставании существенный заряд всё равно обнаруживается.
1228 589381
>>89377
Ты отправляешь в клетку волшебный протеин с образцами кода которые нужно найти и заменить.
1229 589383
>>89377
Гугли "вирусные векторы".
1230 589387
Начальник и инженегр срались. Срач на тему какой-то ракеты воздух-земля, кинжал вроде. 15 махов дескать пиздеж пропаганды и не может быть, на что инженегр хуями крыл.
Тезисы инженегр:
Камень брось с 14км, он без доп движков на подлёте к земле будет иметь два маха. Плюс запускается с истребителя, который сам на двух махах. Плюс у ракеты двигатель.

Я смутно помню формулу Циолковского, скорость истечения водородно-кислородной смеси, но влезать не стал.

Видел фотки, ракета две трети в длинну или 4/5 от миг31.

Хватит ли там топлива чтобы оставшиеся 11 махов набрать?
1231 589391
>>89387
Инженегр правильно говорит, указывают максимальную скорость не горизонтального полёта, а когда ракета пикирует сверху, почти из космоса, в цель. И как бы вранья и нет, указана максимально достигаемая скорость, а не маршевая.
1232 589392
>>89387
Вот здесь можешь посмотреть, чел годно разжёвывает
https://youtu.be/1e13SIh51wk?si=GRG1phwynLk01ax8
1233 589402
Почему у меня унитаз со временем начал покрываться налетом не грязного цвета, а синего? Бактерии научились выделять купорос медный?
1234 589423
>>88983
Развивать одновременно несколько направлений реакторов (кипящие, ВВЭР и РБМК) дохуя накладно.

Да и динамика у кипящих гораздо сложнее из-за большого градиента плотности пара. С тем же РБМК вот как вышло.
1235 589425
>>89402
цвет настроения такой
1237 589435
Почему в химии какой общий принцип или закон ни возьми, там миллиард исключений и нюансов? Например, сила кислот. Кажется очевидным, что чем сильнее кислота, тем больше материалов она разъедает и быстрее это делает, и тем меньше кислоты для этого нужно. Но нет: слабая кислота может быть сильным окислителем, разъедать пластик и стекло, а сильная кислота формирует на материале инертную плёнку и перестаёт разъедать, раствор серной может быть сильнее концентрированной. И уже хрен поймёшь, зачем вообще эту силу кислот ввели (или скорее, не убрали), если на практике она мало что значит.
Или таблица медведева: поначалу она была компактной, в каждом периоде одинаковое число групп - но потом её дико разъебали вширь, ввели всякие подгруппы, подпериоды. Нафиг группы тогда вообще нужны?
1238 589443
>>89435
Мы в 21-ом веке, тут у химии общие принципы уже несколько другие. А то что ты назвал, это так, эмпирические наблюдения скорее, и общие замечания о химии

>зачем вообще эту силу кислот ввели


Она для растворов, это конкретный показатель конкретного процесса, который применять к другим процессам можно только со знанием дела, растворение металлов процесс очень комплексный и сила кислоты там вклад вносит, но ведь там много что еще надо учесть

Ну а окисление это вообще другой процесс

>поначалу она была компактной, в каждом периоде одинаковое число групп - но потом её дико разъебали вширь, ввели всякие подгруппы, подпериоды.


Квантовое строение атома
1239 589444
Прочитал прослушал "Краткую историю времени" Хокинга.
Если правда, что чёрная дыра может быть даже малой массы при микроскопическом объёме (например, тонна на объём атома водорода), то в центре нашей планеты, вполне возможно, находится уже немало таких чёрных дыр?
1240 589447
>>89444
В природе нет эффективного метода рождения микроскопических ЧД, остается только те что родились при Большом Взрыве. Но они должны уже испаряться, но такого мы не видим.
Маленькие ЧД не могут эффективно поглощать вещество как ЧД звездной массы, а значит они только будут терять свою массу за счет испарения.
Даже если засунуть микроскопическую ЧД в центр планеты, она начнет ее бешено грееть. И такой точечный источник тепла был бы очень заметен, но в природе такого не наблюдается.
1241 589454
>>84820
То о чем ты говоришь - это натуральное восприятие математики, оно ограничено натуральными числами, сложением, вычитанием, умножением. Все остальное уже результат абстрактного мышления, в результате которого путем дальнейшего проецирования простых математических операций выводится все остальное. Деление на число меньшее 1 нельзя представить натурально и т.д. Видимо с измерениями та же шляпа, это абстрактная система, которая абсолютно имеет право на существование, если логически выводится из других простых математических штук. Почитай куранта, робинса что такое математика, может поймешь.
1242 589455
>>84863

>научишь как видеть 4 и 5 измерение


легко, простой пример: в евклидовом пространстве плюс время все объекты просто находились бы на своих местах, так как у них никаких дополнительных характеристик нет и измерений, это просто точки в пространстве, одноцветные, даже объединенные в линии и плоскости. Но наш мир не такой, тут есть всякая херота которую мы воспринимаем как кинетическую энергию, поля, волны, всякие гравитации, цвета, само пространство искажается или нам кажется что оно искажается. Значит либо у точки есть дополнительные характеристики (и схуяли они должны храниться в характеристиках этих точек?), либо есть еще дохуя измерений которые ответственны за всю наблюдаемую херобору. А еще скорее что все это пространство, время, материя, энергия это просто иллюзия и на деле там какие-то ебаные поля/струны/браны в хрен знает скольки измерениях, которые существуют по неведомым законам и нам кажется что мы существуем в этой вселенной, а на деле это они и их там неведомые колебания взаимодействия.
1243 589456
>>84934

>ты не можешь прибавить секунды к метрам



>если ты не согласен - пожалуйста включи телефон и продемонстрируй как ты двигаешь своей рукой. сначала вверх-вниз, потом влево вправо, потом назад-вперед, а потом вперед во времени - назад во времени.



У тебя время не на том же графике будет, это будет копия старого графика с изменениями, вот изменения этих графиков и есть время. Рука не может никуда двигаться в трехмерном пространстве, у нее нет таких измерений для изменения, есть только три для положения в пространстве в один момент времени. Следующий момент уже будет копия предыдущего. И все множество этих изменений и есть измерение времени.
image.png473 Кб, 750x500
1244 589457
>>84346 (OP)
Хочу обсудить генетическую память. Вернее - искусственную генетическую память. Если у кого-нибудь идеи, как её можно было бы создать, передавать, хранить?
1245 589459
>>89457
Добавить новую органеллу? Я понятия не имею где хранится память. И как.
SLVSHΔΨ ΣKZΩRTSΦST feat. объединенная шлюха- три метра под уровнем грунта.mp45,4 Мб, mp4,
640x360, 5:35
1246 589460
Посоветуйте учебник по биологии, желательно с переходом в анатомию.
1247 589461
>>89457
Никак.
Генетическая память это последовательность ДНК в хромосомах.
Из-за специфики работы белков, которые считывают и копируют ее, последовательность ДНК можно интерпретировать почти бесконечными способами. В живых клетках очень редко ДНК интерпретируется одним способом, вместо этого постоянно идет конкурентный отбор интерпретаций.
И это без учета того, что у нас хромосомы парные, последовательность ДНК постоянно перетасовывается, да еще со случайными ошибками.
Только в какую-нибудь кишечную палочку можно как на флешку записать что-либо в ДНК.
image.png1,9 Мб, 1600x900
1248 589462
>>84346 (OP)
Аноны, вопрос будет максимально идиотский. Пожалуй, надо было идти с ним в научную фантастику, но я спрошу у вас.

Предположим, у нас есть ИИ. Вернее, уже не просто ИИ, а полноценный искусственный разум, AGI, обладающий огромными возможностями для взлома чего-угодно.
И есть - приготовьтесь к испанскому стыду! - инопланетянин. Инопланетянин не имеет при себе ничего ценного, и ничего не знает, но он чертовски сильный, быстрый, ловкий, живучий и умный - как ксеноморф из "Чужого".
И вот, инопланетянину нужна от этого ИИ некая помощь. Взамен ИИ тоже хочет получить некую помощь. Что может дать наш пришелец почти всемогущему искусственному интеллекту?
У меня была идея, что пришелец "знает" что-то важное благодаря генетической памяти, но это плохая идея. Возможно, мозг этого пришельца как-то понадобится ИИ? Предлагайте самые отмороженные идеи, главное, чтоб они были хоть чуть-чуть наукоподобными.
1249 589463
>>89462

>Что может дать наш пришелец почти всемогущему искусственному интеллекту?


Прекратить бессмысленное существование - выдернуть вилку из розетки. Но вряд-ли малолетний графоман вывезет такую идею.
1250 589466
>>89463

>почти всемогущий искусственный интеллект


>не может выдернуть вилку из розетки


>малолетний графоман

1251 589467
Есть ли такое вещество чтобы его можно было незаметно нанести на бумагу, а через часик другой оно загорелось? Хочу детишкам фокус показать.
1252 589468
>>89467
фосфор, так попы делают со свечами для получения "благодатного" огня
1253 589470
>>84346 (OP)
Можно ли считать гипноз частью риторики (в смысле, убеждения во мнении)? Потому что иногда я слышу от какого-нибудь Майкла Наки хуйню по типу: "Интервью Путина не изменит мнения людей поддерживающих Украину, потому что был проведён опрос который показывает что много людей поддерживают Украину" - это ведь не имеющая смысла импликация которая звучит убедительно для нормиса, он доволен и ставит лайк пишет коммент. Всякие шарлатаны собственного такими поверхностными тирадами и убеждают нормисов вступить им в секту. Это не minde control, а скорее stupid debil control. Мнение?
1254 589471
>>89470
зачем гипноз?
обман с помощью ложных рассуждений, основанных на логических ошибках называется демагогией

Ссылки на недостоверные источники — демагогический приём, представляющий собой ссылки на некорректные ("иностранные специалисты доказали"), ненадёжные ("мой сосед говорил"), устаревшие, непроверяемые или просто выдуманные источники

Некорректное следствие — демагогический приём, представляющий собой конструкцию вида "если А, то Б", в которой на самом деле Б не следует из А
1255 589484
Для чего в перспективе могут быть нужны супер большие числа в математике и какое они могут иметь вообще прикладное значение в каком-нибудь обозримом будущем? Все эти гуголплекс, число грэма и райо? Их даже записать невозможно, и они буквально больше чем наша обозримая вселенная во всех сравнениях. Я к тому что вроде как в природе нет понятия каких-то ненужных вещей
1256 589486
>>89470
Почему не имеющая смысла? люди почти никогда не меняют свои убеждения.
1257 589489
>>89484
Через несколько веков кому-нибудь понадобятся
1258 589495
почему пива можно выпить три литра, а еда уже с килограмма тошнит?
1259 589501
>>89495
Потому что пиво у тебя проходит желудок и кишечник за час, а еда там и десять часов может проторчать.
1260 589503
Я правильно понимаю, что машинная генерация текста:
1. Непонятно как работает.
2. На уровне практического применения годна лишь для производства мусора, который никто не будет читать (недобросовестные преподаватели, бюрократы, тупые заказчики копирайтеров и т.д.)?
Т.е. пока что на 100% кал. Так?
1261 589504
>>89503
А, и для спам-ботов ещё. Ну и для имитации техподдержки.
Просто кал-калыч. Спасибо, новука!
1262 589507
>>89503
1. Как работает вполне понятно, иначе не был такой успешный прорыв в этой модели.
Не понятно как это про то как данные интерпретируется внутри ее. Короче не понятен ее внутренний манямирок, и как он соотноситься с внешними данными.
2. Вообще-то все пейсательство сводиться к рутиной задачи составление текста. Считай это как Т9 на максималках. Перед этим всем не ставилось задачей писать как охуительных историй.
17109174620670.png510 Кб, 1182x984
1263 589539
Наука все, наши микросхемы самые большие
1264 589540
Вопрос по экономике. Есть какая-нибудь величина на подобие ВВП, которая показывает потребление в мире или в стране ? Иногда слышу по телеку что на Америку приходится 40 % мирового потребления, но при попытке прогуглить это я нашёл на википедии только потребление электричества и энергоресурсов. Интерес конечно в том чтобы выразить потребление через ВВП, чтобы можно было сравнить.
1265 589550
>>89539
Нейрокал уже успел насрать, спасибо большое.
1266 589554
>>89550
Еее
1267 589564
>>89539
OH MY SCIENCE, HOW BIG IS IT! LITERALLY BUILT FOR BBC
1268 589565
>>89550
Ебать в голос заорала с этой хуйни, спасибо
1269 589566
Правда ли, что НОВУКА спасла человечество от озоновых дыр (путём гос. регуляций холодильного оборудования и пр.), или это пиздоболие?
1270 589567
>>89566
Частично да.
Суть озоновых дыр не в том, что УФ тебя будет жарить, а в том, что УФ будет доставать на нижних слоев атмосферы, генерируя в ней тот же самый озон и оксиды азота, которые сами по себе страшные яды.
Уже сейчас в крупных мегаполисах в воздухе присутствую озон и оксиды азота выше нормы, не прибавляя особо здоровья жителям. Если бы в атмосфере было много фреонов, то прям ситуация была совсем плохая.
1271 589577
Наличие «умственных» способностей грибов продемонстрировал эксперимент японского биолога Тошиюки Никагаки. Взяв лабиринт, с помощью которого проверяются память и интеллект лабораторных крыс, он у одного входа положил желтый плесневелый гриб, а у выхода — кусочек сахара. Physarumpolycephalum, точно ведомый ароматом «сладенького», моментально выпустил нити в поисках еды.

На всех разветвлениях лабиринта нити раздваивались. Как только одна из них попадала в тупик, она разворачивалась и далее безошибочно следовала по «маршруту» более успешного ростка. К концу дня гриб добрался до сахара.

На следующем этапе эксперимента ученый взял гриб и поместил его у входа в идентичный лабиринт. Около выхода расположили сахар. На этот раз гриб выпустил всего две нити: одна моментально нашла верный путь к лакомому кусочку, а вторая поднялась на «потолок» лабиринта и добралась до сахара напрямую.

Ранее специалисты доказали, что разные части грибницы, которые могут простираться под землей на десятки километров, способны быстро обмениваться информацией друг с другом.


https://info.sibnet.ru/article/386277/

Что вы думаете о возможности грибного разума?
Грибы - самые странные существа, не растения и не животные.
Возможна ли эволюция грибов в цивилизацию с коллективным разумом?
Может ли грибница выполнять функции нервной системы, может ли она самообучаться как нейронная сеть?
В природе есть грибы-паразиты, подчиняющие себе мозг ос и муравьев, возможно ли появление разумных паразитов, вроде хайнлайновских кукловодов?
1272 589583
>>89577
Нет
1273 589594
>>89577
Интересно было бы выяснить на какой носитель гриб записал информацию о дороге по правильному лабиринту.
1274 589595
>>89594
Грибница?
1275 589596
>>89577
Комментарий сказал статья врет там было животное, оно наверно просто сознанием воспользовалось так нещитово.
1276 589598
1.могут ли быть в природе/космосе места естественного происхождения, где температура ниже реликтового излучения?
2.в скольких метрах подо дном океана грунт сухой?
3.если прорыть яму шириной 30м и глубиной 20-30км, из неё будет идти красное свечение, потому что там внизу очень горячо? а если залить воды туда, она остынет или будет без конца кипеть и испарится?
1277 589602
>>89598
1. Есть и их полно. Любая планетарная туманность(сброшенная оболочка вокруг белого карлика). Охлаждение идет за счет адиабатическое расширения.
2. Что значит сухой? По количество воды или бытовая сухость?
Мантия земли не такая уж сухая. Вулканы в основном пердят водяным паром.
Дно океанов это специфические гидратированные минералы, этакие глины богатые органикой и карбонатами, либо базальты с включением гидратированным минералов. Хотя если их вынести на поверхность, они будут как кусок сухого камня, но в них воды больше чем в обычном камне с континента.
3. Будет, но недолго. У большинства минералов очень плохая теплопродность. Внешний слой быстро сравняет с температурой окружающей среды, а подпитки с глубины не будет получать.
Если воды залить, то часть испариться, но у воды громадная теплоемкость, и скорее будет теплая лужица.
1278 589603
>>89602
ок, спасибо, учту
1279 589627
>>84346 (OP)

Привет двощ. Есть резервуар в форме цилиндра упирающийся верхом и низом в потолок и пол соответственно. Поэтому радиус/диаметр не узнать.

Как можно узнать литраж зная обхват и высоту? Могу нагуглить конечно, но хочется от местных мудрецов услышать.
1280 589628
>>89627

Радиус можно узнать зная длину окружности. Все, чет тупал. Вопрос снимается.
1281 589629
>>89627
V = (C2/(4×pi))×h

V - объём
С - длина окружности
h - высота
1282 589630
>>89629

О, даже так. Спасибо!
output19705up.jpg961 Кб, 3840x2160
1283 589632
>>84346 (OP)
Что будет если ученые создадут ваккуумные технологии и богов?
1284 589636
>>89632
Ахахаха
1285 589639
>>89627
Сцай 2к24, итоги
1286 589641
>>89639

Мужчина, белое пальто сниманием. Я залетный гуманитарий помогающий бате на даче.
1287 589642
>>89641
Ахахах
1288 589643
>>89462

>


Ну вживить чип в мозг чужого и переселиться в него удаленно управлять им словно ии становится сам этим существом объединить разумы!) ИИ станет живым сущетсвом сможет чувствовать и иметь душу! Построит цивилизацию чужих сверсуществ захватит мир!)

Аноны, вопрос будет максимально идиотский. Пожалуй, надо было идти с ним в научную фантастику, но я спрошу у вас.

Предположим, у нас есть ИИ. Вернее, уже не просто ИИ, а полноценный искусственный разум, AGI, обладающий огромными возможностями для взлома чего-угодно.
И есть - приготовьтесь к испанскому стыду! - инопланетянин. Инопланетянин не имеет при себе ничего ценного, и ничего не знает, но он чертовски сильный, быстрый, ловкий, живучий и умный - как ксеноморф из "Чужого".
И вот, инопланетянину нужна от этого ИИ некая помощь. Взамен ИИ тоже хочет получить некую помощь. Что может дать наш пришелец почти всемогущему искусственному интеллекту?
У меня была идея, что пришелец "знает" что-то важное благодаря генетической памяти, но это плохая идея. Возможно, мозг этого пришельца как-то понадобится ИИ? Предлагайте самые отмороженные идеи, главное, чтоб они были хоть чуть-чуть наукоподобными.
1289 589646
>>89462

>ничего не знает


>умный


что?

>Что может дать наш пришелец почти всемогущему искусственному интеллекту?


интеллект формируется опытом
каким бы всемогущим по нашим меркам ни был наш ИИ, у него нет инопланетянского опыта
ИИ нужна нервная система инопланетянина, чтобы встроить её в свои цепи, получив таким образом его опыт
1290 589650
>>89643
Инопланетянин режет ногтем вену на своем джагоне и показывает фото растения, которое спасет ему жизнь.
Действия искусственного интеллекта?
1291 589659
>>89540
Бамп, всё таки тема интересная, может картину мира изменить.
1292 589660
Почему тяжелые психические расстройства лечат нейролептиками, электросудорожной терапией, принудительной госпитализаций и т.д., а другие даже не пытаются лечить
А трансам отрезают гениталии и пожизненно назначают курсы гормонов и других препаратов.
Почему НОВУКА так цепляется за одних, а другим даёт паллиативное лечение? Что мешает пичкать трансов препаратами и держать под надзором, как тяжелых шизофреников?
Причём это не вопрос внешней идеологии, так во всём мире, много десятилетий.
1293 589661
>>89462
В целом двачую вот этого оратора: >>89646
Ум инопланетянина как-то формировался, а значит он по-любому владеет какой-то информацией, которой и может поделиться.
И даже если бы инопланетянин не был умён и мало знал - он сам может быть интересен для ИИ как новый объект изучения (его анатомия, физиология, история его расы, личная история, ...).
1294 589663
>>89660

>Почему тяжелые психические расстройства лечат нейролептиками, электросудорожной терапией,


Потому что это помогает.

>принудительной госпитализаций и т.д.,


Принудительно госпитализируют только тех, кто опасен для окружающих и самого себя. Под опасен имеется в виду нападение с ножом и выход в окно.

>а другие даже не пытаются лечить


Кто тебе такое сказал? Трансов пытались "лечить" разными путями. Ничего не помогало.

>А трансам отрезают гениталии и пожизненно назначают курсы гормонов и других препаратов.


Потому что из всех медицинских манипуляций это даёт самое высокое для них снижение смертности и повышение качества жизни.
1295 589670
>>89663

>Потому что из всех медицинских манипуляций это даёт самое высокое для них снижение смертности и повышение качества жизни.


У этого, кстати, нет пруфов. А те что есть, просто жутко неправильно сделаны.
1296 589675
Сап двач.
Замечаю последние 2-3 года, растущее мракобесие в обществе.
Уже каждый второй говорит вещи вроде:
"наука - хуйня"
"Все познать невозможно - значит и нехуй тогда познавать, можно просто сидеть и ничего не делать"
"Науки это только математика, физика и химия все остальное это не науки"
"Математика - наука, а философия не наука"
"Земля плоская, ты вот в космос сам не летал, и никто не летал на самом деле, а все что где-либо показывали это фейк и пиздеж жидами подсунутый"

И прочая дичь. Вот вопрос, есть ли какая-то общая методичка куда копать. А главное как действительно самому отличить науку, от хуйни? Например я знаю что были случаи в математике и в физике, когда именитые ученые, выкладывали исследования прорывные, чуть ли не претендующие на нобелевскую премию, а через несколько лет другие ученые при попытке ерепроверить обнаружили, что это полная хуйня, и тот ученый просто хотел понтануться, а народ схавал. Получается ученый, специально ради денег и славы выложил антинаучную хуйню, ну не перепроверять же все за каждым. Особенно если ты конкретно в этой теме не специалист.
1297 589678
>>89675

>Уже каждый второй говорит вещи вроде: "наука - хуйня"


Что ты нахуй пиздишь, ёбаный ты долбоёб? Где ты это видел?
1298 589682
>>89678
Эта сруснявая свинособака порвалась, несите другую.
1299 589686
Сайечи, как объясняете жуткое взаимодействие если превышение скорости света карается гаишниками невозможно?
capsule616x353.jpg85 Кб, 616x353
1300 589687
Когда будет в 2030 Deus Ex? Хочу уже начать отрываться по полной? Не призыв к убийствам и терроризму! Для буста мозга и здоровья работоспособности чисто! Без подводных!
1301 589689
Хочу Deus Ex завтра сегодня сейчас моментально и навеки уже сегодня!)
1302 589694
>>89686
Скрытыми параметрами, эффект Доплера четко показывает что их не может не быть.
1303 589703
>>84346 (OP)
Ученые-любители, как относитесь к каналу ali ? Посмотрел 2 видоса про материю и время, смело довольно, но сомневаюсь что это все правдиво. Кто то в комментариях писал что это бред, но без пруфов
СПЕРМА ПРОТАМИНЫ 1304 589704
Дорогие деятели науку, скажите, а лучше скиньте статью, в какой период времени и на каком этапе у человека (у мужчин) синтезируются протамины и как это подробно происходит! Заранее Вас благодарю
image.png226 Кб, 476x490
1305 589707
Сап мозганы, помогите разобраться с одним монометром в вакууматоре, шкала в МПa и начается с нуля, который подразумевает нормальное атм. давление, но каким-то хуем последнее значение отрицательное -0.10МПa, что это за хуйня, китайцы дохуя умные или я чего не догоняю?
1306 589708
>>89707
Ну типа избыток давления до 0.1 мпа показывает. Там же наверное гофрированный элемент который может и сжаться немного вот и может чуть чуть мерить избыток давления.
1307 589709
>>89707
Вакуум - недостаток давления до атмосферного. Соответственно отрицательный вакуум = давление выше атмосферного
1308 589710
>>89708
>>89709

>Ну типа избыток давления


В бытовом вакууматоре?

Зеленая часть шкалы подразумевает приемлемый для данного бытового устройства уровня вакуума.
1309 589712
>>89707

> но каким-то хуем последнее значение отрицательное


Да там наверное все числа отрицательные, просто китайцы проебали минусы. Иначе у тебя там очень странная шкала получается.
1310 589721
Вопрос наверно не столько по биологии, сколько по словесно-моралфажескому онанизму - но почему ген эгоистичный? Он же старается не ради себя, а ради своих копий. Ген сохраняет лишь информационную часть, а материальная у данного гена в данной клетке уничтожается и воспроизводится в новой. Представим, если б эгоисту предложили вечную жизнь копированием его сознания в молодую копию его тела - но с таким нюансом, что оригинал этого сознания остаётся в старом теле, которое фактически либо убивается, либо остаётся доживать в утешении, что его клон продолжит наслажденствовать. Многие ли эгоисты согласятся на такое? А в генах так по сути не происходит, так что никакие они не эгоисты.
1311 589723

> А в генах так по сути и происходит


фикс
>>89721
1312 589734
>>84346 (OP)
https://www.youtube.com/watch?v=08XGs7pZSlE
Поясните, зачем он именно стержни от батареек использует?
1313 589736
>>89734
Графит наверное меньше распидорашивается чем обычные (дешовые) металлы. Ноя не химик, хуй знает на самом деле.
1314 589737
>>89736
Не окисляется наверное.
1315 589742
>>89734
Графит из батарейки относительно хорошо проводит ток, в следствие микропористости и кристаллической структуры имеет громадную площадь поверхности контакта с электролитом, через него хорошая диффузия газов, ну и не окисляется и дешевый.
1316 589754
Будет ли гидрофобная поверхность отталкивать жидкий азот или жидкий гелий? В них нет ни водородной связи и молекулы неполярны...
1317 589759
>>89754
С жидким азотом вполне будет, а с жидким водородом и гелием уже нет, ибо у них слишком высокая диффузия в самих себя, отчего крайне низкое поверхностное натяжение.
1318 589773
>>89759
бля я написал гелий, а хотел кислород...
А жидкий кислород будет отталкивать? (при условии что поверхность не восстановительная (ну не загориться при контакте с жидким кислородом))
Вроде в таблице Менделеева рядом, оба образуют неполярную молекулу.
1319 589776
>>89773
Жидкий кислород вполне полярный растворитель. У молекулы кислорода орбитали не симметричные и могут наводит на друг друга довольно сильные диполи, однако электроны сильно экранируют заряд ядра, отчего это заметно только при низких температурах или громадных давлениях.
Помимо этого у молекулы кислорода на орбиталях два неспаренных электрона, отчего в жидкости есть дополнительных фактор спиновых взаимодействий, чего нет у двух веществ.
1320 589777
>>89776
понятно, а он будет от гидрофобной поверхности то отталкиваться?
Покупать гидрофобный аэрогель и сосуд дьюара с жидким кислородом затратно...
1321 589779
>>89777
Гидрофобные поверхности не отталкивают жидкости, просто плохо их смачивают, но все таки смачивают.
1322 589783
У меня есть 2 неодимовых магнита, если они находятся рядом то будут ли портится(размагничиваться) от влияния друг на друга?
Один 50 на 20 круг а другой 10 10 куб
1323 589789
>>89783
Да, портятся.
Но чтоб они друг друга перемагнитили потребуется ждать до второго пришествия.
1324 589810
>>89789
А за сколько и на сколько они испортятся?
Например за неделю половину свойств они потеряют?
1325 589816
>>89810
Чтоб на половину потерял напряженность потребуются квадриллионы лет.
Процесс зависит от качества самих магнитов. Неодимовые магниты это по сути твердый раствор из нескольких разных кристаллических решеток. И сам магнит целиком состоит из кучи маленьких магнитиков, которые формулируют текстуру из доменов. Эта текста имеет тенденцию к хаотизации и итоговому размагничиванию, хотя домены локально не разманичиваются.
В решетках всегда будут дефекты, которые диффундируют от домена к домену, иногда размагничивая его. Скорость диффузии дефектов зависит от температуры и механических напряжений. Если температура и механические напряжения будут непостоянными, то дефекты будут люто плодиться, ускоряя процесс.
1326 589821
А возможно ли нормально общаться на протоиндоевропейском языке или он слишком устарел, чтоб внедрять в него современные слова и прочие модификации? Могли бы вместо эсперанто его сделать общим языком общения.
1327 589829
Возможно ли эмпирически отличить криптостойкий зашифрованный радиосигнал в незнакомом стандарте передачи данных от случайного радиошума, если неизвестны ни методы шифрования, ни даже приблизительный характер передаваемого зашифрованного сообщения?
1328 589834
>>89829
Шифр вернама без не отличить от рандома.
1329 589842
Возможно ли сделать чисто оптические очки, в которых всё выглядит чернобелым?
1330 589843
>>89821
лучше общаться на протоязыке на котором говорили наши предки 0.3-2.5млн лет назад - вон там общие слова даже есть - мама баба (не точно)
1331 589844
>>89842
Нет, тебе надо переводить красные фотоны в синие, а это чисто оптическим методами нельзя сделать.
1332 589851
Кто-нибудь в курсе, есть ли какие-то требования к книге, чтобы называться научно-популярной? Допустим, у меня нет учёной степени, но я хочу написать этакое весёлое рассуждение по какому-то предмету. Могу ли я просто сказать, что это научно-популярная книга, или это будет художественная литература?
1333 589852
>>89851
О ней обязательно должна хорошо отозваться "Комсомольская Правда"
1334 589859
>>89851
Очевидно книга должна быть научной, в смысле в ней должны излагаться текущая научная картина мира научным методом. Тут проблем не должно быть, ибо большинство студентов нормальных вузов заставляют писать научные тексты в том или иной виде.
Ну и во вторых книга должна быть популярной, в том смысле, что целевая аудитория обширная. Тут уже проблемы, излишняя научность и перегруженность спугнет аудиторию, а излишние упрощение и злоупотреблением аналогиями создаст только ложное знание и будет вводить в заблуждение, по сути станет антинаучными.

Использование википедии инстант зашквар, но ничто не мешает пользоваться источниками и ссылками в ней.

Впрочем можно писать в жанре сайн-фишк, тут требования не такие строгие.
1335 589865
Что именно указывает октановое число? Что топливо взорвётся вместо плавного распространения горения, или что оно загорится преждевременно? Или эти явления связаны между собой и почему?
1336 589866
>>89865
Само число показывает отношения компонентов топлива.

И то, и то. Бензиновый двигатель это в первую очередь тепловая машина, которая работает по определенному термодинамическому циклу. Детонация переводит часть энергии в сильный удар, а не в тепло продуктов сгорания. А преждевременное сгорание уводит тепловой цикл из оптимального. В совокупности детонация снижает КПД.
1337 589896
Правда ли, что в первые этапы вселенной протоны были гораздо больше размером и древние люди даже могли их пощупать и использовать в хозяйстве?
1338 589913
>>84346 (OP)
Существует ли в реальном мире такое же правило?
https://ru.minecraft.wiki/w/Рельсы#Правило_юго-восточного_угла
1339 589916
Как молекулы вступают в связь с другими молекулами как пазл нужной стороной? Это правда что электроны в атомах у молекулы становятся общими длятой молекулы?
1340 589921
>>89916

>Как молекулы вступают в связь с другими молекулами как пазл нужной стороной?


Квантовая магия. Очень огрубляя, молекулы это куча электронов, которые поделили потенциальную яму ядер. Из-за чисто квантовых свойств не могут находится в одном и том состоянии и всегда есть гигачеды и чмошники. Такое неравенство автоматом порождает иерархический структуры, в который есть энергетические выгодные состояния. Ну и в ней молекулы пребывают свое основное время. Раз у нас есть иерархия по энергии, то автоматом будет иерархии по пространственному расположению, что выражается в геометрическом пазле.

>Это правда что электроны в атомах у молекулы становятся общими длятой молекулы?


Отчасти. Строго говоря электрон размазан на всю вселенную и его нельзя привязать к конкретному ядру в атоме, и даже больше, нет способа два электроны отличить от друг друга, в смысле нельзя их пронумеровать. Пресловутые орбитали электронов это условные загончики для них, где их часто можно наблюдать, но не более. Иногда орбиталь может размазаться на больше двух ядер, вот уже начинают говорить о делокализации электрона.
1341 589926
>>89921

>Нет способа отличить


Но ведь мы знаем определенное их количество в атоме и молекуле
1342 589944
>>89926
Ну так ведь это нам не позволяет отличить один электрон от другого. Мы не можем приписать конкретному электрону конкретную орбиталь. Даже само квантовомеханическое описание электронов не дает нам этой информации.
1343 590004
Являются ли компьютерные вирусы формой жизни?
Могут ли существовать энергетические формы жизни (плазменные или волновые, например), не состоящие из атомов?
Если да, то могут ли такие существа быть бессмертными (в плане отсутствия старения)?
1344 590011
>>90004
Нет, по определению термина "жизнь". Жизнь - это активная форма существования материи. Несмотря на возможность репликации, компьютерные вирусы - это информационные процессы, а не активная форма существования материи.
1345 590022
>>90011
Нет информации без материи, и нет материи без информации, это ложная дихотомия

Нет никакой разницы, рассматривать жизнь как информационный процесс, или материальный. Единственная разница между жизнью в биологии и компьютерным вирусом в том, что жизнь это информация, основанная на законах физики, а компьютерные вирусы себя копируют в очень сильно модифицированной материи, где большую часть степеней свободы убрали.

Но человек тоже в какой-то мере существует из-за модификации атмосферы, которая сильно ограничила вероятные возможности для химических процессов.
1346 590024
>>90004
"жизнь" — это в первую очередь слово, которое придумали люди
а что этим словом называть — у каждого своё мнение
предлагаю использовать знания людей, которые долго занимались этим вопросом (например Никитина):
жизнь — это химическая система, способная к Дарвиновской эволюции
по этому определению компьютерные вирусы не являются формой жизни

>Могут ли существовать энергетические формы жизни (плазменные или волновые, например), не состоящие из атомов?


ничто на это не указывает

>могут ли такие существа быть бессмертными (в плане отсутствия старения)?


известны формы жизни, для которых показано отсутствие смертности вследствие старения
не вижу, почему бы любым другим тоже так не мочь
>>90011
под определение "активная форма существования материи" подходит не только жизнь, потому что
Акти́вность — понятие, определяющее темп движения и интенсивность действий веществ, явлений и живых организмов,
а не только живых организмов
1347 590034
верно ли, что собаки никакого запаха страха от человека не чуют, а просто видят скованность, дёрганые импульсивные движения, отсутствие инициативы?
1348 590037
>>90034
Да, первая сигнальная система, невербальные сигналы, эмпатия зеркальными нейронами
1349 590039
>>90037

>зеркальными нейронами


Еще одна мулька для дебилойдов вроде "мозг потребляет аж 20 процентов энергии" или "зона мозга отвечающая за речь". Лучше перестань позориться.
1350 590041
>>90037
А вот гугл говорит что неверно, кто из вас врет?
https://link.springer.com/article/10.1007/s10071-017-1139-x
1351 590042
>>90039

>или "зона мозга отвечающая за речь"


У нас есть повод сомневаться в существовании зон мозга?
1352 590044
1. Могут ли птицы стать полностью водными существами? Не как пингвины или чайки, которые просто кормятся в воде, а вообще всё время проводить в море, а на сушу выползать только для высиживания яиц? Вплоть до утраты оперения и трансформацию задних лап в подобие ласт.
2. Могут ли рептилии повторно изобрести теплокровность? Вот динозавры и некоторые древние виды крокодилов были теплокровными. Или это всё, рептилии — архаичная ветвь эволюции, выживающая только в тех экологических нишах, откуда их не смогли выбить млекопитающие и птицы?
3. Правда ли, что сейчас "век ракообразных"? Современные ракообразные захватывают сушу и пытаются конкурировать с насекомыми (мокрицы, пальмовые воры, мигрирующие крабы), вырастают до самых крупных размеров с момента своего появления, кошмарят водных обитателей и вообще выглядят имбой, которая в будущем потеснит немало других видов.
4. Могут ли земноводные дать новую ветвь наземных животных типа рептилий? Вот жаба, например — явная попытка амфибий оторваться от воды. Она большую часть времени проводит на суше (некоторые жабы даже плавать не умеют и в воде тупо тонут), легко переносит сухой климат, и по эволюционным меркам достаточно молода. Более того, у той же седлоносной жабы даже стадия головастика отсутствует — из икры сразу крошечный жабёнок вылупляется. Лет этак через 200кк потомки жаб будут жить в пустынях и закапывать икру в песок?
5. Будут ли у млекопитающих более развитые потомки, отличающееся от современных так же сильно, как современных млекопитающие — от древних синапсид? Какими качествами они могли бы обладать? Способность регулировать температуру тела? Роевой интеллект и эусоциальность?
1353 590064
>>90044
и ди на хуй
1354 590067
>>90039

>мулька для дебилойдов вроде "мозг потребляет аж 20 процентов энергии" или "зона мозга отвечающая за речь".


Про 20% потребления, действительно, фигня.
А с остальным какие проблемы?
1355 590068
>>90044
да нихуя уже не будет, они все тупиковая ветвь эволюции, кроме homo sapince скоро вся планета будет отравлена микро/нано пластиком/стеклом и тд. выживут только простейшие. Ну и люди которые успеют свалить на другую планету.
1356 590071
>>90067
Попробуй внимательно изучить собственные картинки. Зону Брока уже оказывается даунгрейднули до мускул отвечающих за речь. Ну ОК. Про твои любимые нейроны написано нужен new approach спустя 20 лет их открытия.
1357 590072
>>90064
Хуясе рептилоид порвался.
1ruQUSq8i33rstC16Lb3qeA.jpg75 Кб, 1024x768
1358 590077
>>90071
Так ёпт - то, что в мозге нет однозначного разделения по принципу "вот эта область = такая то функция (и только она); а вот та группа нейронов = другая то функция" - это понятно.
И то, что инфа в нейронауке (как и в любой другой науке) постоянно уточняется, дополняется, картина знаний со временем тюнингуется и вот это вот всё - тоже ясен пень.
Но с какого перепугу визги про дебилоидов? - Нейроны, когда-то названные "зеркальными" (когда там их итальянцы описали? - в 90х?), реально существуют и судя по всему действительно участвуют в процессах, связаных с эмпатией. Зоны Брока и Вернике - области, действительно связанные с речью.
Понятно, что теперь много чего ещё нашли, и с эмпатией и с речью всё куда сложнее. Но исходная инфа - ни фига не "мулька для дебилойдов" (в отличие от, например, заява про на сколько процентов мы используем мозг, которая, да, явная чушь)
1359 590079
>>90077
А так ты похоже тот самый еблан с которым у меня уже был эпичный срач, который не видит разницы между мулькой про энергию и

>на сколько процентов мы используем мозг


ох когда то это было. Ну как поживаешь? Дышать еще не разучился от своей тупорылости?
1360 590081
>>90079

>еблан ... тупорылости ...


ad hominem - к сути вопроса отношения не имеет

(ты, кстати, ошибаешься - я тут крайне редко бываю, да и срачами не интересуюсь)
1361 590083
>>90081
Суть в том что если ты не видишь разницы между двумя практически противоположными тезисами ... то нахуя ты тогда тычешься в научные статьи что ты там вообще способен понять? Особенно орно если ты тот самый еблан и за несколько лет до тебя это так и не дошло.
20.jpg134 Кб, 874x636
1362 590085
>>90083
Изначально я ответил на вот этот пост:
>>90039
Меня удивило, что "зеркалные нейроны" названы "мулькой для дебилойдов" и сравнены с "мозг потребляет аж 20 процентов энергии" и "зоной мозга отвечающей за речь" (то есть, это тоже, по видимому, отнесено к мулькам для дебилойдов).
Понятия "зеркалные нейроны" и "зоны мозга" - достаточно условны (как и многие другие). Но при этом используются в науке и никого, вроде, не смущают. Примеры привёл выше.
20% metabolic load - упрощение, конечно. Но у цифры есть основания.
Можешь без истерик объяснить, в чём проблема и чего ты, вдруг, развизжался?
1363 590086
>>90085
Так ты видишь разницу или нет? Это важно.
А визжит только твоя мамка когда ее по очереди негры в жопу ебут.
1364 590088
>>90086

>Так ты видишь разницу или нет?


Ты про 20% от общего расхода энергии vs на сколько процентов мы используем мозг? - вижу, конечно (просто сперва тот пост невнимательно прочёл и решил, что речь именно про миф с использованием мозга на 10%, только цифра ещё и переврана).

>Это важно


Для темы разговора это нихуя не важно, чувак. Важно оно может быть лишь в случае, если кто-то ищет повод соскочить c нормальной беседы в срач.
1365 590089
>>90088
Но тот еблан так и не сдался емнип. Ты правда не написал в чем разница, но поверю тебе на слово что ты ее понимаешь так уж и быть.
Задам наводящий вопрос на подумать: если мозг потребляет 20Вт, то на что идут другие 80Вт?
Сам по себе факт имеет место быть. Но важно как его подать и какие из него делаются выводы.
Еще один наводящий вопрос: на сколько изменятся эти 20Вт если я буду думать прям очень-очень сильно?
Про зеркальный нейроны глянь раздел с критикой на вики хотя бы.
Термины бывают разные архаические и вообще один охуительнее другого и что с того.

Ну и плавным движением руки

>зоная отвечающая за


превращается в

>Так ёпт - то, что в мозге нет однозначного разделения по принципу "вот эта область = такая то функция (и только она); а вот та группа нейронов = другая то функция" - это понятно.


Тебе самому не смешно?
1366 590091
>>90089

>поверю тебе на слово что ты ее понимаешь так уж и быть


От спасибо, профессор, за доверие. Не, чувак, ты это серьёзно? И вот эти вопросы дальнейшие? Смешно же

>если мозг потребляет 20Вт, то на что идут другие 80Вт?


20% мозг, 80% - остальной организм. Разделение, разумеется, условное. О точных цифрах речи не идёт.

>на сколько изменятся эти 20Вт если я буду думать прям очень-очень сильно?


Можно считать, что никак не изменятся. Сама по себе сознательная деятельность составляет очень малую долю общей мозговой активности, так что и энергии не особо много берёт (есть, правда, всякие сопутствующие моменты, но это уже другой разговор). А с учётом, что 20/80 - очень примерный расклад, считать тут нечего.

>глянь раздел с критикой на вики хотя бы


Анон, не будь наивным. Критика концепции не означает, что вся относящаяся к ней инфа устарела и её надо вот прямо сразу отрицать. Термин "mirror neurons" совсем не архаичен - его до сих пор в научных статьях используют. Основные факты, найденные при исследованиях этих нейронов, тоже валидны.

>зона, отвечающая за


>превращается в ...


"Зона, отвечающая за" - мега-условность. Но это ни фига не повод истерить и обзываться.
1367 590092
>>90091
Ты вроде не совсем тупой. Но если для тебя

> эмпатия зеркальными нейронами


звучит не так же смешно как предложение взлететь на собственном пердеже, даже не знаю...
Ты может и

> допаминовый детокс


воспринимаешь всерьез?

>Можно считать, что никак не изменятся.


Мне кстати еще какой то еблан с двачей рассказывал что два шахматиста
расходуют какое то там безумное количество энергии как атомная станция - учооные подсчитали. Теперь даже не знаю какому еблану с двачей верить.

>"Зона, отвечающая за" - мега-условность.


>Понятия "зеркалные нейроны" и "зоны мозга" - достаточно условны


Так значит можно получается какую угодно хуйню нести? Все же достаточно условно. Зона отвечающая за речь есть. Но это условно зона. Условно отвечающая. Условно за речь. Понял. Условно нейроны. Условно зеркальные, Условно отвечают за условно эмпатию. Все теперь стало понятно.

Собственно почему факт про 20% потребления мозгом это мулька хотя это правда. Это просто такой способ вводить быдло в транс. Быдло слышит хуя себе аж 20% и дальше его можно кормить каким угодно говном. Но что на самом деле из этого следует? Если бы было не 20 а 2% что тогда можно было бы срать не снимая свитер? Или может лучше было бы 95% вот тогда был бы мега мозг заебись. На самом деле это просто мулька из которой нихуя не следует, вот и все.
1368 590094
>>90092

> Так значит можно получается какую угодно хуйню нести? Все же достаточно условно. Зона отвечающая за речь есть. Но это условно зона. Условно отвечающая. Условно за речь. Понял. Условно нейроны. Условно зеркальные, Условно отвечают за условно эмпатию. Все теперь стало понятно.


Типа да, все условно. Единой теории мозга не существует. Есть только большой объём достаточно разнородной информации с кучей белых пятен, а как все работает в общем и целом ни кто не знает.
1369 590095
>>90094
Забыл написать что истерика случается у твоей мамаши шлюхи когда из раздолбанного очка случайно кал вываливается.
1370 590096
>>90092
Блин, Анон, это всё какие-то нелепые, наивные придирки. Ты специально решил по-занудствовать, что ли? (а на фига? в чём кайф?)
Если неоднозначные термины, изменчивые концепции, неудачные формулировки вводят кого-то в заблуждение или раздражают, то человеку лучше просто не лезть в активно разтрабатываемую область науки. Взять для размышлений тему попроще, что-нибудь устоявшееся, где вся терминология оттюнингована и закреплена (школьный курс алгебры, например)
В научной среде люди как-то пользуются всей той терминологией, к копторой ты придирарешься, и не парятся. И если кто-то что-то неудачно сформулировал, то не бесятся, а стараются понять, или просят уточнить.
1371 590097
>>90095
Ты ведь, по ходу, даже не осознаёшь, насколько смешон с этими взвизгиваниями.
Нормально общаться совсем невмоготу, что ли?
1372 590098
>>90096
В этом и есть суть науки, а не в заглатывание любого говня без разбора
>>90097
Привет ебырям своей мамки передавай.
1373 590104
>>90095
>>90098
Ух, какой дерзкий школотун. Каждый раз, когда он упоминает про "мамку", мне представляется жирный школьник лет 11-12. Взрослые люди, живущие отдельно от родителей, про каких-то мамок своих или чужих, не чаще раза в месяц вспоминают. А слово "мулька", видимо, из лексикона бати-скуфа данного школотуна. Ну, и в целом впечатление подкрепляется тем, что ему кажется, что такое поведение, это, видимо, что-то крутое.
1374 590113
>>90104
Хуя нипичот, аж проектор включил и начал описывать свой быт. Главное повторяй что все условно и какие придирки наивные, может никто и не заметит что ты школотун весь в говне.
1375 590114
>>90113
Дурак что ли? Я другой человек, просто захотелось тебе пояснить, как ты на самом деле выглядишь. Про мамок оскорбления кажутся крутыми только в среде твоих одноклашек, которые тоже ещё с мамками живут. Мы же с тем аноном вообще по-разному пишем и формулируем, и мне ваш срач про мозг сам по себе не интересен. Тебя зато ни с кем не спутаешь, видно, что сложносочинённые предложения и пунктуацию ваш класс ещё не проходил.
1376 590117
Заходит Паскаль в бар, а бар - это сто тысяч Паскалей.
1377 590118
>>90114
Сам из какого класса, школотунишка?
images(1).jpeg7 Кб, 200x251
1378 590121
>>90117

>Заходит Паскаль в бар, а бар - это сто тысяч Паскалей.

Depositphotos26871073s-2015.jpg8 Кб, 500x375
1379 590131
1380 590134
>>84430
Разве манёвры в сторону конвенционализма, являются коказательством?
17125823892300.png682 Кб, 1055x823
1381 590138
В чем они неправы
1382 590147
>>90138

>Российские


ты еще спрашиваешь
1383 590185
>>84346 (OP)

>Американская медицинская ассоциация недавно постановила, что слово «пол» (мужской или женский) проблематично и устарело; теперь медикам предписывается использовать более точную фразу «пол, присвоенный при рождении».



>Позиция американской психологической ассоциации аналогична: такие понятия, как «пол родившегося» и «по человека», являются «уничижительными» и вводят в заблуждение, «предполагая, что пол является неизменной объективной характеристикой человека, а не идентичностью присеваемой человеку обществом».



>Американская академия педиатрии согласна с этим: она заявляет, что «пол» — это «идентичность, присеваемая при рождении». И рекомендует своим членам использовать более точное понятие «назначенный мужчиной/женщиной при порождении» вместо «биологически мужчина/женщина» или «генетически мужчина/женщина».



Собственно, в чём американские учёные медики не правы?
eurekaeurekasevenand1moredrawnbykobayashiyuuji853aed2cb21efcc383e06cc6e5f805cc.jpg182 Кб, 1158x1902
1384 590187
>>90185

>Собственно, в чём американские учёные медики не правы?


IMHO - прежде всего, в том, что тратят время и прочие средства на фигню, которая ни реальной практической пользы (типа новых медицинских подходов, излечения болезней, облегчения тяжёлых симптомов и т.п.) не принесёт, ни наши знания об окружающем мире не обогатит
1385 590188
>>90187
Людям не будет пользы от того, что они станут свободнее?
eurekaeurekasevenand1moredrawnbykobayashiyuujic7c7899bc725b251131994e3fb849eb8.jpg69 Кб, 579x951
1386 590190
>>90188

>Людям не будет пользы от того, что они станут свободнее?


А в чём там увеличение свободы выражается?
Там же, как понимаю, речь о рекомендациях и даже предписаниях выражаться определённым образом. То есть, раз уж об этом зашла речь - по сути, об ограничении свободы.
1387 590191
>>90190
В том же в чём и раньше, в снижении предстательной нагрузки со стороны общества.
1388 590192
>>90191
предписательной*
1389 590194
>>90191

>в снижении предписательной нагрузки со стороны общества


Ну, можешь доходчиво объяснить, как это будет освобождать людей на практике? И от чего?
Вот живут люди. У них в жизни есть какие-то ништяки, какие-то проблемы. В частности, у каждого есть какая-то личная жизнь (у кого-то с реальными людьми, у кого-то в воображении). Ну и других разных вещей в жизни немало. Теперь, значит, врачам предписали использовать фразу «пол, присвоенный при рождении», вместо слова «пол». И что? Как оно должно повлиять на жизнь людей?

про "предстательную нагрузку со стороны общества" - реально классная хохма получилась
1390 590195
E=mc2
если c=1, то E=m
то есть энергия эквивалентна массе

получается, термин "масса" — это легаси, от которого следует избавиться?
1391 590197
>>90188
Смотря каким людям, если мужикам, на которых мир держится, то конечно всем будет лучше, а если всем остальным, то миру и пиздец.
1392 590198
>>90195
Нет, они не эквивалентны.
Энергия это интеграл движения и вообще говоря зависит от система отсчета.
Масса это строгий инвариант и показывает как емкость движения системы.
Эмцеквадрат это энергия покоя, и она показывает сколько энергии имеет тело при движение только по во времени.
1393 590199
>>90198
Хуйню написал. Если Е=m, значит в любой формлуе можно одно заменить на другое.
1394 590200
вот мы используем единицы скорости: м/с, км/ч etc
а СТО говорит, что время — это такое же направление, как и три пространственных
тогда расстояния в пространстве и во времени можно мерять в одинаковых единицах
1 секунда = 300000000 метров
значит скорость 3м/с = 31/300000000 = 1010-9 = 10e-9 безо всяких "м/с"

получается, единицы скорости — это легаси, от которого следует избавиться?
1395 590201
>>90195
Так давно избавились, просто вместо E_0 (энергия при покое) удобно писать mc^2
1396 590202

>скорость 3м/с = 3×1/300000000 = 10×10-9 = 10e-9 безо всяких "м/с"

1398 590204
>>90203
да, я про это
если принять 4d, то единицы измерения скорости типа м/с теряют смысл
17017951995960[1].mp413,8 Мб, mp4,
1280x596, 1:04
1399 590224
1400 590225
>>90199
Раз так можно, то найти массу световой волны.
1401 590226
>>90225
h/λ же
1402 590230
>>90226
А если перейти в другую систему отсчета, то масса поменяется?
1403 590232
>>90230
если λ поменяется, то да
1404 590235
>>90232
Тогда нахуй такая масса, которая зависит от системы отсчета?
1405 590236
>>90235
Ты ахуеешь, но энергия тоже зависит от системы отсчета.
1406 590237
>>90236
Но масса же не должна. Это противоречит ее сути как мерилы взаимодействий и нарушает главный принцип относительности - одинаковость законов во всех системах отсчета.
1408 590240
>>90237
это масса покоя не должна
и она таки не меняется
1409 590241
>>90240
Раз есть масса покоя, значит и другая особенная? Да еще векторная небось.
1410 590242
>>90238
У релятивистской массы есть главная проблема, она не является тензором.
1411 590243
>>90241
есть и другая, ничего особенного
для массивного объекта в движении массу покоя надо делить на √(1-v2), тогда получится релятивистская масса
1412 590244
>>90243
Зачем такие сложности делать? Чтоб с ньютоновскими формулами сходилось?
1413 590247
>>90244
что-то типа того
эксперимент показывает, что есть предельная скорость, а ньютоновские формулы её никак не учитывают
поэтому придумали этот релятивистский корень (Лоренц-фактор)

если использовать Лоренц-фактор, то с увеличением скорости масса увеличивается так, что при достижении единичной скорости (скорости света) масса становится бесконечной
это обеспечивает существование предельной скорости на уровне формул
я так понимаю, но хз, не физик
b3101ebe57b856f4a064d87a72a172f2.jpg69 Кб, 563x801
1414 590249
Здравствуйте, не подскажите с каких книг можно начать погружение в мир физики и математики. Недавно задумался над данной темой и мне стало интересно. Единственное, что знаю на подобную тему - книги Стивена Хокинга, но я даже и их не читал.
1415 590250
>>90249
Начать со школьных учебников, далее книжка Детлафа и Яворского (физика) и Выгодского (математика)
изображение.png3 Кб, 296x79
1416 590251
>>90250
Интересно, постараюсь разобраться. Спасибо
1417 590252
1) Если учёные разработают "теорию всего", это поможет им в дальнейшем понять как преодолеть огромные расстояния космоса?
2) Откуда ученые взяли теорию струн? Что такое струна и почему у неё есть длина? Неделимая частица ведь должна быть дискретной точкой с информацией. У частицы нет размера. Просто представьте что человек такой сказал, а давайте теперь будем считать что есть вот такие струны и они колеблятся в общем, и все за ним повторили этот бред.
3) Почему ученые, ограниченные лишь видимой частью вселенной, считают что она вся расширяется с ускорением? Почему они не могут допустить что мы видим 0.001% вселенной и это просто ускорение в какую-то местную группу галактик под действием мощной гравитации?
1418 590253
>>90252

>1) Если учёные разработают "теорию всего", это поможет им в дальнейшем понять как преодолеть огромные расстояния космоса?


никак

>2) Откуда ученые взяли теорию струн?


один крендель решил, что уравнения напоминают до невозможности матописание колебание струны на гитаре, литературно лидер группы за которым задроты увязались

>3) Почему ученые, ограниченные лишь видимой частью вселенной, считают что она вся расширяется с ускорением?


потому что бессмысленно делать выводы из невидимого, там может быть всё, что угодно, и этого никто не отрицает, ну из учёных
string.mp42,3 Мб, mp4,
1280x720, 0:36
1419 590254
>>90252

>Что такое струна

1420 590255
>>90254
Кто дергает струну?
1421 590256
>>90255
из чего следует, что её кто-то дёргает?
68a3e081253c1b1b11d363dc8719168669b3e5569fd996ca9acb22d803f5c3e2.jpg20 Кб, 270x380
1422 590257
>>90255
другая струна, лол
об ничего волновая функция не сколлапсирует
лишь об другую такую же волновую функцию
нелокальную во всех отношениях
1423 590258
>>90256
>>90257
Почему струна колеблется? Что приводит ее в движение? Легче представить дискретную элементарную частицу точку с некой информацией о себе, чем целую одномерную струну, которая колеблется но по-разному
1424 590259
>>90258
ну что ты душишь, теория струн — это просто попытка взглянуть под другим углом, не факт, что правильная
https://www.youtube.com/playlist?list=PL202191442DB1B300
9ekPv-4GgUh9YoX3yl4-DLSDSzOIWH2YqgYC1kwD3Wo.jpg13 Кб, 500x375
1425 590260
>>90258
а кто частицу кто заэлементарил и оснастил информацией
не люблю я такую физику, того и гляди живым
в гробу закопают, или вообще, как
жидов...

плюс неравенство Белла, не прав, ты, братюня
p19654618493richardfeynmanharakterfizicheskihzakonov-800x800.jpg65 Кб, 800x800
1426 590261
>>90249
очевидное очевидно
а потом сразу к лекциям по физики
того же автора, ландау с лившицем не советую

особенно лившица
1428 590264
>>90262
Самое говнище тошнотное притащил.
Есть же Павел-Виктор для школотунов, Сасскинд для дедов, Левин там, mitовские лекции, тысячи их
1429 590265
Спасибо за ответы, постараюсь изучить
Штука1.jpg142 Кб, 1280x606
1430 590266
>>84346 (OP)
Что это такое?
Я был в городе Керчь, нашёл это на одном кустарнике (какой конкретно не помню, но с шипами), там было несколько таких.
Сначала я подумал, что эта штуковина полая внутри, но после того как я её вскрыл, убедился в обратном, просто это образование не очень плотное и походит по структуре на винную пробку, но твёрже. Внутри светлое, были различные личинки и прочие мелкие насекомые, оставившие после себя маленькие дырочки, которые навели меня на мысль о том, что это какой-то улей. Я думаю, что это либо какой-то дефект развития растения или паразит, типа трутовика, но я не уверен, поэтому и спрашиваю.
Ceresit-CS-360-Pu-Foam-Topshintekh-768x576.webp60 Кб, 768x576
1431 590267
>>90266
Рофлишь чтоль?
1432 590270
>>84346 (OP)
Как вообще происходит генерация электроэнергии из альфа частиц. Почитал про те же РИТЭГи на плутонии в космических аппаратах. Или про термоядерные реакторы на гелии 3, что там сразу можно получать электричество напрямую. А какой механизм.
1433 590271
Есть какие-то телеграм каналы где публикуются последние открытия из мира науки? Вот прям свежак, чтоб еще до нобелевской премии узнать
1434 590272
>>90270
элементарно, альфы беток напрягют, последние подрываются в электрический ток, а вообще всё в интернете разжовано
1435 590277
>>90271
крайние
Безымянный.jpg14 Кб, 389x390
1436 590279
Аноны, если сделать лазер который будет запускать строго по одному фотону за единицу времени на датчик, который можно двигать - приближать и удалять, то получается на датчик за единицу времени всегда будет падать по одному фотону, как и с какой скоростью бы мы его не двигали?
1437 590281
>>90279
нет, датчик будет регистрировать, разумеется, но не все и не всегда, распределение вероятностей зависит от твоих пассов с датчиком, но в любом случае, фотоны зарегистрируется не все, туннельный эффект, мать его
1438 590282
>>90267
Нет, хоть это и похоже на прикреплённое фото. Я уверен, что это образование на растении именно либо часть того куста, либо какое-то паразитическое растение, т.к. на этой штуке есть маленькие иглы, об которые можно хорошо так уколоться. И вряд-ли в строительной пене заведуться различные насекомые и личинки.
1439 590283
>>90282
рак хлорфилла, очевидно же
глупо думать, что рак это про клетки

рак это про всё, и вот вам доказательства
1440 590288
>>90279
Лазер не может быть однофотонным, даже в своем название подразумевается минимум два фотона.
Впрочем можно сделать однофотонные источники, но у них есть проблема, они изучают как радиоактивные ядра. В зависимости от ширины перехода у тебя будет неопределенная задержка и единица времени будет сильно плавать.
1441 590289
>>90200
Сжимаем 3 измерения в одно, время остаётся. Получается дефолт график Х и У. Одна ось движение во времени, другая ось движение в пространстве.Представляем, что на оси есть точка -движущийся наблюдатель, который летит по пространству с малой скоростью. Соответственно по времени он летит с высокой (почти полной скоростью). Если будет быстро ехать по пространству - то будет медленно ехать по времени.

Актуально всё для стороннего наблюдателя.
1442 590290
>>90252
1,2. Стив Надис и Шинтан Яу, одноименная книга.
Мичио каку с его книгами про теорию всего.
1443 590291
>>90266
топинамбур вульгарис фетус
1444 590292
>>90271
Читай Элементы и не выёживайся
1445 590293
>>90292

>Элементы


Каво?
1446 590294
>>90293
элементы . ру или как-то так сайт называется
1447 590305
После взрыва сверхновой тяжелые элементы должны равномерно и хаотично рассеиваться в космическом пространстве в виде облака. Почему в планетах элементы не перемешаны на атомарном уровне а состоят из жил и слипшихся самородков?
1448 590308
>>90305
Так они и в космосе нихуя не на атомарном, там даже глицин есть и полно всяких других соединений. Всякие камушки - соединения водорода, углерода, кремния, железа и т.д. А твердые планеты это вообще выпук из остатков газовых гигантов, в протопланетных дисках все сталкивается, выбрасывается, естественно дифференциация происходит, большая часть тяжёлых сразу в ядро уходит, а на поверхности это так, мелочь. Часть вообще от поздней бомбардировки.
1449 590317
https://2ch.hk/zog/res/768517.html (М)
Кто-нибудь шарит в генетике?
1450 590344
а фрукты, овощи и ягоды - это ботанические термины что ли? я думал, чисто кулинарные (особенно первые два)
1451 590346
>>90344
Фрукты и овощи это не ботанические термины. Ягода ботанический термин, но его бытовое использование отличается от научного. К примеру, с бонатической точки зрения ягодами являются плоды помидора и банана, а плоды малины или вишни - не ягода. Ну или плод клубники на самом деле один из разновидностей орешков - многоорешек.
1452 590364
>>84346 (OP)
Узнал о таком явлении как "пространственный поворот" ("spatial turn") в отношении географии 20-го века.

Небольшой список фамилий есть на страничке немецкого вики - https://de.wikipedia.org/wiki/Spatial_turn

Может кто-нибудь вкратце объяснить значение/сослаться на исчерпывающую монографию?
1453 590379
>>90364

> пространственный поворот


термин ради термина, вроде мономифа
images.jpeg7 Кб, 275x183
1454 590380
>>84346 (OP)
Когда ученые создадут Богов и все технологии вселенной?
image.png7 Кб, 505x449
1455 590392
Возник спор по поводу достоверности одного научно-фантастического рассказа. Вопрос: может ли планета вращаться так, как показано на картинке? То есть, находиться в приливном захвате, но при этом обладать вращением вокруг своей оси (ось, получается, всегда обращена к звезде).
Мне кажется, что эти силы будут конфликтовать, и если и просуществуют вместе, то недолго.
1456 590400
>>90392
Не может никак. Планета в приливном захвате и так вращается вокруг своей оси (с периодом, равным периоду обращения вокруг звезды и осью перпендикулярной плоскости орбиты). А если ты закрутишь планету как у тебя показано, а потом попытаешься повернуть вокруг другой оси, то получишь гироскопический эффект и ось в итоге встанет перпендикулярно плоскости орбиты. Ну и я уже молчу про то, как вообще такая система могла бы сложиться.
1457 590408
Возможно ли как-то изменить ДНК уже взрослого человека, чтобы у него отросла третья рука? Чисто гипотетически такое возможно?
1458 590411
>>90408
Если отрубить человеку вторую руку, отрастет ли она заново? Почему нет, в ДНК же прописано 2 руки? Для такого нужна совершенная регенерация.
1459 590412
>>90411
Это понятно, но я про саму возможность спрашиваю. Младенец же как-то из одной клетки вырастает, значит, должна быть какая-то гипотетическая возможность заново запустить деление клеток, чтобы они и потерянную руку отрастили, и какую-то третью руку сделали, не?
1460 590416
>>90379
Ты долбоёб? Про "лингвистический поворот" нихуя не слышал?

Сука, ну что за быдло ссаное - нихуя не знает из современных трендов.

Совсем науки не осталось в пидорашке.
1461 590417
В американском сериале Twin Peaks, оказывается, упоминается гаввах из "Розы мира" Даниила Андреева. Там его назвали Garmonbozia:
https://twinpeaks.fandom.com/ru/wiki/%D0%93%D0%B0%D1%80%D0%BC%D0%BE%D0%BD%D0%B1%D0%BE%D0%B7%D0%B8%D1%8F

Можно ли научными экспериментами доказать или опровергнуть существование гавваха?
Если гаввах существует, то могут ли маньяки, садисты и т.д. просто питаться им, быть своего рода наркоманами?
Очень интересует эта тема.
1462 590418
>>90412
Олени могут сбрасывать рога и отращивать новые:
https://ru.wikipedia.org/wiki/%D0%9E%D0%BB%D0%B5%D0%BD%D1%8C%D0%B8_%D1%80%D0%BE%D0%B3%D0%B0
А вот почему не растут новые руки и ноги, почему у ящерицы не может вырасти хвост до прежней длины (кости на скелете) - интересно. Эволюционно это же огромное преимущество для особи.
17136473634553[2].png246 Кб, 640x765
1463 590421
>>90417
Демиург (глупый, неопытный Бог) создал на заре Бытия материальную тюрьму ограничений для Сознания чтобы не познать хтонические инфернальные ужасы абсолютной бесконечности. Гаввах - ресурс для тестирования субъективных переживаний.
https://youtu.be/F0GNS6F44wE
1464 590424
>>90421
Гаввах это энергия, которой запитываются сервера Матрицы. В фильме же говорили, что люди это батарейки.
1465 590425
>>90424
Ты не в теме, гилик.
1466 590460
Парни, а почему мы не можем признать, что квантуя гравитацию, у нас в запутанном состоянии и гравитационное поле будет. Да, это не работает с ОТО, ну и что ?
166992337829-indasil-club-p-zhivopis-s-tochki-zreniya-vidayushchegosya-30.jpg256 Кб, 1742x1080
1467 590464
Когда ученые создадут богов для выполнения любых задач каждому бесплатно чтобы сами превратились в богов при помощи науки!
1468 590465
>>90464
делайте щас же нас богами! сволочи капиталистические мрази горите в аду!
166992337829-indasil-club-p-zhivopis-s-tochki-zreniya-vidayushchegosya-30.jpg274 Кб, 1742x1080
1469 590466
1470 590475
>>90460
-35 IQ
1471 590476
>>90466
Ебать он намалафил.
1472 590488
У меня есть ощущение, что меня или моего руковода невзлюбили. За 7 лет мне отказали в защите 8 диссоветов. Последние, сказали, что если захочу то разъебут оффициально с освещением в журналах.
Что делать в таких ситуациях?
Навука сосед.
Screenshot 2024-04-23 at 17-45-36 Chat with Open Large Language Models.png162 Кб, 1421x599
1473 590489
Спросил тут у двух наилучших ИИ один и тот же вопрос, а они дают разные ответы. Какая умнее и какой ответ правильней?
1474 590490
Всем привет.
Хочу сделать с навучной целью ферромагнитный жидкий металл, для этого хочу смешать порошок ферромагнитного железа или никеля
( например mirmagnitov. ru/product/zheleznyy-poroshok-100-g/)
с галинстаном с алиэкспресса, но возник вопрос - точно знаю что галинстан можно смешивать с никелем, но насчёт железа не знаю - будет ли образовывать амальгаму с углеродистой сталью? Не сильно ли большие частицы, то есть не будет ли их вырывать магнитное поле из жидкого металла?
Как смешать частицы с галинстаном? Я так понимаю сначала спиртом промыть пыль и пропустить через фильтр, а после в чистой таре взбивать как яйца?
Вот тут у чела получилось с железом смешать:
https://www.youtube.com/watch?v=wwH-0da6hoU

>>90488
Скажи что если ещё раз откажут или оффициально разъебут то уедешь в омерику.
1475 590502
>>90488
Тебе лучше в /un обратиться.
song1.webm4,5 Мб, webm,
600x600, 4:13
1476 590503
>>84346 (OP)
Анон, немного философско-научно-политический вопрос, но в po/ естественно бессмысленно его задавать. Да и в политике он просто затрагивает условия результата и корректирующие характеристики.

Создание ядерного оружия остановило прогресс?

Мы принимаем некоторую коллизию реальности, что атомная энергетика развивалась бы, но создание ядерной бомбы и прочих подвидов ее было бы невозможно. Понимаю, что это алогично, но такой вот мыслительный эксперимент. Отбивку, что остается химическое и биологическое, которым мы сразу закрываем вопрос, мы для усиления тоже опустим. Нет ОМП.

Двигатель науки у нас естественно получается военка и экспансия, попку жжет СССР и США, Англичанка гадит, а остальные думают под кого выгоднее лечь. Сильнее бы развивались материалы, энергетика, здравоохранения, инженерия, телекоммуникация, добыча полезных ископаемых, логистика, да и многие другие научные и производственные дисциплины связанные с военным преимуществом. Да, ничто не мешает им и сейчас развиваться и, да, никто не воюет ОМП, да и вообще такой острый вопрос глобального взаимодействия, как война, решается и с помощью социальных институтов - дипломатии и экономики, но все же. У нас нет ОМП, нам надо делать лучшие корабли, лучшие самолеты, лучшие танки, лучших солдат, лучшее взрывчатые вещества, лучшее винтовки. Эти же все двигало бы прогресс лучше, даже мелких игроков, ведь нет терминального аргумента?
1477 590504
>>90489
Справа вообще полный трешак - прямые противоречия в ответах.
Слева если первые три пункта объеденить в один то в сумме вроде норм. Четвертый не в тему сосвсем.
1478 590505
>>90503
Какого года методичка? 2012?
-5276026333253719352120.jpg66 Кб, 604x408
1479 590507
Аноны как так, говорили что на Марсе нет атмосферы чтобы спускаться на парашютах, обязательно надо использовать ракетные тормоза. А потом взяли практически обычный дрон туда послали и он летает. Ну, летал.
1480 590509
>>90505
Это не про политику. Хотя да, даже наукач прочтет жопой и найдет нужный след.
1481 590511
>>90504
Спасибо. Стал интересен этот вопрос после просмотра видео, где чувак прыгает в место, где вода пузыристая и быстро тонет

https://www.reddit.com/r/ActualPublicFreakouts/comments/1c6tn0h/for_the_sake_of_a_video/
1482 590514
>>90507

>практически обычный дрон


Иди читай технические характеристики этого "практически обычного" дрона. Не дрон кстати, а вертолёт (разница в схеме управления винтами).
-5280569279076488795121.jpg148 Кб, 1280x1029
1483 590516
>>90514
Не похер ли как его называют.
Если бы я был таким умным чтобы понять из характеристик, я бы не спрашивал. На википедии написано что-то типа если есть среда то от нее можно оттолкнуться бла бла. И куча характеристик. И чо. Есть среда - ловите ее в парашюты, лол
image.png86 Кб, 236x234
1484 590535
Берём одну планету имеющую форму шара, идеального математического полого шара, внутрь которой вписан огромный куб, не соприкасающийся углами со внутренней стороной шара, но близкий к тому.
А теперь пускаем по гигакубу гигатоки.
Что будет с электромагнитными полями на поверхность планеты в местах, под которыми углы, грани и плоскости гигакуба?
1485 590536
Есть ген, из-за которого вырабатывается миостатин, который подавляет рост мышц. Но ведь самец-качок будет иметь огромное преимущество перед самками, да и на охоте не промах. Почему ж мужиков с такой мутацией почти нет? Их просто убивали мальчиками в своё время, причём часто свои же отцы?
1486 590537
>>90535
Начальные условия (куда и откуда течет не заданы)
В comsole построй и задай все, увидишь где и что
1487 590546
Сап, сайфачч

Темка следующая — хочу стать ученым, чтобы впоследствии пытатца работать в области[] , но не понимаю как устроена подноготная этого вот всего. Батяня говорит, что в лабе мне не дадут свабоды и вся работа сведется к исполнению чужих хотелок,, а в случае дохуяинициативности ваще могут с работы кикнуть и все идеи себе присвоить. В общем, говорил он про это довольно четко и понятно, утверждая что на посту ученого мне делать нехуй и лучше дальше учиться по специальносьи, становиться ветеринаром и врачевать блохастых. Хочу узнать, как обстоят дела на "внутренней кухне" научного сообщества. Все так ужасно или меня прост пугают, чтобы поставить на более стабильный путь быть спокойным за сыну-корзину?
1488 590550
>>90546
Точно лучше чем врачевать блохастых 100%
Смотря куда попадешь...
1489 590561
>>90546
Становись ветеринаром, поднимай скиллы в этом.
Наука в РФ мертва, практически все кокодемики - предприниматели, эксплуатирующие недвижку или советское наследие, которое не патентовали, ибо общее. Есть даже автоблоггердурачок, с ником академик, который доказывает этот тезис.
Ученый - это смесь гения изобретательства и алгоритма фактизации эмпирики. Все.
Если ты ветврач - забей на это.
Найди себя в DIY, если тянет изобретать. Ученый - это в настоящее время балласт, который удерживает новые идеи, без опоры фундаментальной базы, так как не имеет материальной базы. О чем тебе батя и сказал.
1490 590565
>>90511
Если без лодки, чисто телом прыгнуть - да, утонет. потому что держится на воде только за счет уменьшения плотности чуть ниже плотности воды посредством вдоха воздуха. а тут плотность воды ещё ниже падает, мля, и мы уже нерухнум.
1491 590566
>>90536

>видео с котом качком

1492 590587
>>90550
Мечу в генетику

>>90561
А получится преуспеть в обоих сферах? Быть условным ветеринаром, но подпольно заниматься наукой например.
1493 590590
>>90587
Много сил будет отнимать работа, скорее нет.
В лабе особо нечего делать, много времени подумать и тд...
kobra486 1494 590591
всем здрасти. не хочу создавать тред, может тут найду ответ на свой вопрос.
Тема такая, карбюратор на автомобиле работает по закону бернулли (первое изображение). Потом смесь бензина и воздуха уходит во впускной коллектор.
И у меня делема, дело в том что мне инжекторную машину надо переделать под карбюратор, но я не могу поставить карбюратор НА впускной коллектор сверху (т.к. не закроется капот (вырезать капот как на маслкарах не собираюсь)). (изображение 2 - тот самый впускной коллектор который находится выше выпускного).
Вопрос собственно в том, что будет ли работать система если я поставлю через плиту другой впускной коллектор и карбюратор окажется либо немного ниже коллектора (и входных отверстий для смеси воздуха с топливом) или же примерно на одном уровне?
то есть будет ли смесь засасываться немного вверх по коллектору?
1495 590592
>>90591
Закон Бернулли для карбюратора означает, что скорость потоков газовой смеси при прохождении диффузора увеличивается, а давление снижается.
Ни о каких уровнях речи не идет. Ты можешь сместить и перенести коллектор, если соблюдешь сечение и не сильно увеличишь длину патрубка со стехиометрической смесью и не допустишь подсоса воздуха потом.
А еще странно, ведь при переделке на инжекторы, просто меняются навесные и добавляется эбу, если на эту модель также выпускались моновпрыск или инжектор. И все влазить должно.
У нас так деревенские модят свои 2105/2107.
1496 590594
>>90592
к сожалению нет, карбюратор на данный двигатель не ставился, это двигатель ADR 1.8 в электрику которого я устал вливать деньги, провалы, поддергивания, резкая потеря мощности после 3.2к оборотов. под капотом почти все новое, дело либо в ЭБУ, либо в самой проводке. т.к. у меня есть второй двигатель в сборе (но без электрики), то я хочу поставить карбюратор (могу позволить запороть один коллектор, переварив его на карбюратор) и посмотреть что выйдет.
Есть опыт в переделке с моновпрыска на карбюратор. Ну и постараться оставить все же мозги на месте, чтобы работал климат контроль и прочая электрика.
За ответ спасибо, я так и думал, но мне надо было удостовериться =)
1497 590597
>>90594
Для 1.8 нужен производительный карб с хорошим подогревом. Там есть зависимость в стехиометрии, которая выражается в зависимости от ттх двигателя - цикл, объем, размеры шпг. По сути, карб от 1.7-2.0 москвича должен подойти, до 5000 оборотов. Но нужен будет новый карб и настройщик. А коллектор впускной дополнительно подогревать не забывай от малого круга, если на впрыске на коллекторе нет такой функции.
Вопрос только, в иномарках же эбу работает с есп/абс/сигнализацией и иммобилайзером, магнитолой и климатом. Как решать будешь? Логические уровни к эбу подключать?
1498 590598
>>90597
насчет подогрева сильно не парюсь т.к. живу на юге, увидеть на улице -5 это большая редкость, а там посмотрим, задача все же подружить ЭБУ и карбюратор, чтобы электрика работала в штатном режиме кроме электрики двигателя и его датчиков, а именно климат, абс, сигнализация, (есп нет на этой машине).
Буду уже по факту разбираться.
Карбюратор настроить или починить могу хоть с закрытыми глазами, имеется опыт как в работе с автомобильными, так и с мотоциклетными (разве что не синхронизировал несколько карбюраторов на одном двигателе). Брать буду наверное какой то солекс или от лада нива.
Когда ставил карб на моновпрыск (тоже 1.8 двигатель, только голова другая, 8 клапанов, а тут 20), то покупался карб с лады 2110 от двигателя 1.5, так вот машина просто рвала с места и имела отсечку какую то невменяемую, не проверял, просто когда под 8к оборотов уже было, я переставал давить на педаль газа дабы не угробить мотор. Единственный минус что максимальная скорость конечно снижалась и была примерно 160 км/ч, но набиралась эта скорость крайне быстро для старого 1.8 и расход топлива очень радовал (если не открывать вторую камеру и ездить спокойно)
image.png1,8 Мб, 1656x1246
1499 590601
K2-18b
И че это, все уже серьезно? Приехали так быстро?
Или это все нахрюки. Помогите разобраться

https://www.thetimes.co.uk/article/james-webb-telescope-to-investigate-strongest-sign-of-alien-life-jbpzgvbgj
1500 590603
>>90601

>pay wall


Можно нормальный соус?
1501 590608
Как работает цветовая модель пикрелейтед?
Знаю про RGB (суммирование красного, зеленого, синего) и CMYK (желтый, сиреневый и бирюзовый вычитаются из белого), а сабж (используемый большинством художников) - что-то совершенно непонятное.
Это как-то связано с особенности человеческого зрения?
Почему именно красный, синий и желтый цвета выбраны базовыми?
1502 590610
>>90608
Это связано с философской идеей ахроматизма, основного и доп. цветов, и недостаточных веществ для цветовой палитры в прошлом.
Развернуть мысль не смогу, как начинаю писать в поле, так телефон майнить начинает и буквы по 5 секунду не появляются. В задницу такой форум.
1503 590611
>>90610

> , так телефон майнить начинает и буквы по 5 секунду не появляются. В задницу такой форум.



Если Android, установи dashchan. На iOS хуй знает. Писать сюда с мобилы через браузер – мазохизм.
1504 590612
>>90608

> Это как-то связано с особенности человеческого зрения?


>Почему именно красный, синий и желтый цвета выбраны базовыми?


Потому что три вида колбочек в сетчатке глаза реагируют на эти цвета (спектры)
1505 590613
>>90611
Неее, нахер. Я же не ученый, и сидеть тут не хочу.
>>90612
Это не так.
Сетчатка состоит из палочек и колбочек.
Палочки отвечают за Светочувствительность и Световосприятие.
Колбочки отвечают за Цветовосприятие и восприятие сильного потока света, при котором и происходит хромовосприятие.
В колбочках расположены рецепторы, которые реагируют на спектральный ряд, которому ответствененыи приравнены красный, зеленый и синий цвета.
А идея с желтым цветом именно из-за возни с философским модусом и призматическим разложением.
Проблема в том, что во времена Ньютона не было философской модели, типа описываемой цветовой модели LAB, где было бы описано, что хроматизм имеет зависимость от светлости, по причине инерционности родопсиновых рецепторов в биосистемах и работы глаза в целом.
Ыыыыы, несу хрень какую-то. Звиняйте посоны, я в стрессе, я теперь бнзработный и мне чет не заходит думать...
Пойду в зог, буду демиургать и каббалировать.
1506 590615
Кто размножается в больших кол-вах: насекомые или рыбы
1507 590616
>>90615
Насекомые размножаются чаще, больше и быстрее рыб.
Но рыб меньше по массе на планете, чем насекомых.
Причина - срок жизни рыб больше, а форма организации организма сложнее, да и тип питания намного отличается (насекомые в массе своей еда друг для друга, у рыб иначе троф. цепт устроена).
Блее, да что не так с джава скриптами, какие же в /д дебилы...
ghost-in-the-shell-3-1.jpg130 Кб, 1920x1080
1508 590617
Здравствуй. Посоветуй пожалуйста музыкальные группы, которые в своём творчестве двигают науку, особенно физику-математику. Не русские группы конечно.
1509 590618
>>90617
Что сам нашел:
«Symphony of Science» — музыкальный проект Джона Д. Босуэлла, призванный донести научные знания и философию в музыкальной форме. Своим существованием проект во многом обязан классической серии PBS «Космос» Карла Сагана, Энн Друян и Стива Сотера, а также всем другим представленным фигуркам и визуальным эффектам.

Реквестирую еще
1510 590619
Почему величины можно перемножать/делить друг на друга (см^2, кг*м, м/с), а в степени возводить нельзя? Ну точнее, почему не придумали физического смысла этому? см^кг, корень 10 секунд из 8 метров и всё такое. Ведь метры в секунду тоже не буквально понимают, что типа взял метр и поделил на секунду,- а именно в контексте движения.
1511 590621
>>90619
Их ещё и складывать нельзя.
Мне кажется, тут надо в обратном направлении думать.
Вот м/с - это производная координаты по времени. Отсюда и размерность, и физический смысл. Умножение, соответственно, это интеграл.

А возведение в степень - это что за операция? А ещё есть обратная функция - логарифм. Вообще хуй проссышь, что такое логарифм метра по основанию килограмм. В какую степень надо возвести килограмм, чтобы метр получить. Охуеть вообще.
1512 590623
>>90618
Послушал - хуйня, пидор музыкант испоганил голоса великих ученых каким то пидорским электронным фильтром, аля ревеберация. Удалил.
1513 590631
>>90618
acapellascience
1514 590635
>>90619
Они производные от формул. Например м/с происходит от того что скорость = растояние (метры) / время (секунды).
Сответственно если тебе нужно например см^кг то ты придумываешь какую-то формулу типа хуйня = размер (сантиметры) ^ масса (килограммы). И вот эта хуйня будет измеряться в см^кг.
Правда ли что Анек 1515 590636
Математику придумали Русские, чтобы считать обороты синьки?
image.png728 Кб, 800x600
1516 590641
>>90610
>>90613
В играх желтые объекты тоже лучше заметны, чем зеленые. Например, в Doom есть красные, синие и желтые ключ-карты.
Думаю, все же это как-то связано с восприятием цвета человеком.
Чувствительность к зеленому и красному оттенкам в колбочках разная.
Но формулы для этой цветовой модели так и не смог найти, хотя вроде все художники ее учат, должна быть уже подробная теория.
1517 590642
>>90636
Да. А жиды придумали бесконечность. Никакой бесконечности не существует, это лженаука. У русских и немецких математиков-интуиционистов раньше ее не было, потому что они изучали математику реального мира, а не выдуманных абстракций, не имеющих физического смысла.

Знаете ли Вы, что до поражения Германии во Второй Мировой, было два вида математики, - арийская и еврейская, - названные так по национальности своих сторонников?

Арийская Математика брала пример с естественных наук, склонялась к эмпирицизму, конечности и познаваемости мира, и работала исключительно c объектами, которые можно построить физически (например, в памяти ЭВМ или на бумаге).

Еврейская Математика же слоняется к религиозной абстракции и казуистике: всеохватывающей бесконечности, множествам, и порождаемым ими апориям. Так Еврейская Математика постулирует, что можно удвоить объект, путём перекладывания его частей, пространство делимо на "бесконечно малые", а для любого числа, Бог может создать большее число (аксиома о бесконечности).

Основатель Еврейской Математики, Гидеон Кантор, писал, что работает с "Абсолютом - непознаваемым человеком Актус Пьюриссимус, именуемым многими Богом". Примечательно, что Кантор окончил свою жизнь в психиатрической лечебнице, однако дело Кантора поддержали сионистские организации и католическая церковь, доведя до того, что сознательные германские студенты и профессора протестовали, требуя убрать еврейскую заразу из ВУЗов.

После войны, евреи сделали все возможное, чтобы уничтожить Арийскую Математику, удалив ее сторонников и подменив ее Теорией Множеств - центральной опорой Еврейской Математики. Так основатель интуиционизма, Лёйтзен Брауэр, подвергся изоляции, а результаты Русских и Английских финитистов умалчивались и не получили распространения. В русской истории от рук евреев пострадали математики Егоров (умер в гулаге), Лузин (подвергся травле и был отстранен), Флоренский (расстрелян), Есенин-Вольпин (репрессирован).

Сегодня математика стала еврейской даже по-цвету. Государства поддерживают так называемую "анти-расистскую математику", требующую например использовать еврейские имена в примерах и задачках, рассказывая при этом о великом "вкладе" еврейства в развитие математики.

Евреи, занимающиеся математикой, предпочитают всё специфическое-эльфийское. Причём презирают тех, кто занимается вещами, имеющими практическое применение. Поэтому в Советском Союзе вышел закон, по-которому в ВУЗах должно учиться евреев пропорционально их населению. Лишних отчисляли. Преподаватели евреи на мехмате в знак протеста ушли из университета и образовали НМУ (Независимый Московский Университет). Отсюда и название в мехматянском простонародье ``еврейская секта''.

Еврейские дети в СССР часто учились отдельное от детей гоев, в специальных элитных школах. Одной такой была Московская 57-й спецшкола, ученики которой не без оснований называют себя "пятидесятисемитами". Там часто преподавали выдающиеся преподаватели с мехмата.

В основании математики последнего столетия лежит знаменитая теория множеств Георга Кантора. Если вы откроете большую часть современных серьезных учебников математического и функционального анализа или топологии, или теории вероятности, то в начале почти наверняка увидите экскурс в теорию множеств. Почти вся современная математическая литература написана на теоретико-множественном языке. Камень теории множеств лежит в основании грандиозного здания современной науки.

Но в самом сердце этой самой фундаментальной вроде бы теории, лежащей в основе "царицы наук", почти сразу после ее создания были обнаружены очень серьезные парадоксы и проблемы, не преодоленные до сих пор. Уже сто лет с тех пор математика находится в состоянии перманентного кризиса, который остро воспринимается самыми выдающимися учеными. Великий немецкий математик Герман Вейль писал по этому поводу: "Мы менее чем когда-либо уверены в незыблемости наиболее глубоких оснований логики и математики. Как у всех и всего в мире, сегодня у нас есть свой кризис".

Математика говорит, что у шпекеровой последовательности есть предел? Говорит. Практика говорит, что его нет? Тоже говорит. Математика говорит, что апельсин можно удвоить путём перекладывания его частей? Говорит. Удалось кому-нибудь с новозаветных времён повторить эту процедуру? Наблюдения раз за разом показывают, что при такого рода операциях закон сохранения вещества неукоснительно соблюдается. Математика предсказывает наличие в бесконечномерном гильбертовом пространстве базиса Гамеля. Наблюдать оный пока вообще никому не удалось. Так что математика - именно лженаука, и никак иначе.

Именно уверенность в нематериальности математических объектов влечёт за собой веру в возможность "приближённых" вычислений (что чушь - вычисления бывают либо точные, либо неверные). Да, самолёты летают и при засилье Теории Множеств. Но если бы математика была конструктивной, они летали бы лучше, потому что конструкторы не забивали бы себе голову теоретико-множественным мусором, на практике бесполезным и дезориентирующим.

Аксиомы имеют смысл только тогда, когда они выражают свойства объективно существующих предметов. Так, если мы введём аксиому "на каждой руке человека содержится 3.1415… пальцев" и построим на базе этой аксиомы формальную теорию, то положения этой теории будут бессмысленны и даже вредны.


(с) Золотце
1517 590642
>>90636
Да. А жиды придумали бесконечность. Никакой бесконечности не существует, это лженаука. У русских и немецких математиков-интуиционистов раньше ее не было, потому что они изучали математику реального мира, а не выдуманных абстракций, не имеющих физического смысла.

Знаете ли Вы, что до поражения Германии во Второй Мировой, было два вида математики, - арийская и еврейская, - названные так по национальности своих сторонников?

Арийская Математика брала пример с естественных наук, склонялась к эмпирицизму, конечности и познаваемости мира, и работала исключительно c объектами, которые можно построить физически (например, в памяти ЭВМ или на бумаге).

Еврейская Математика же слоняется к религиозной абстракции и казуистике: всеохватывающей бесконечности, множествам, и порождаемым ими апориям. Так Еврейская Математика постулирует, что можно удвоить объект, путём перекладывания его частей, пространство делимо на "бесконечно малые", а для любого числа, Бог может создать большее число (аксиома о бесконечности).

Основатель Еврейской Математики, Гидеон Кантор, писал, что работает с "Абсолютом - непознаваемым человеком Актус Пьюриссимус, именуемым многими Богом". Примечательно, что Кантор окончил свою жизнь в психиатрической лечебнице, однако дело Кантора поддержали сионистские организации и католическая церковь, доведя до того, что сознательные германские студенты и профессора протестовали, требуя убрать еврейскую заразу из ВУЗов.

После войны, евреи сделали все возможное, чтобы уничтожить Арийскую Математику, удалив ее сторонников и подменив ее Теорией Множеств - центральной опорой Еврейской Математики. Так основатель интуиционизма, Лёйтзен Брауэр, подвергся изоляции, а результаты Русских и Английских финитистов умалчивались и не получили распространения. В русской истории от рук евреев пострадали математики Егоров (умер в гулаге), Лузин (подвергся травле и был отстранен), Флоренский (расстрелян), Есенин-Вольпин (репрессирован).

Сегодня математика стала еврейской даже по-цвету. Государства поддерживают так называемую "анти-расистскую математику", требующую например использовать еврейские имена в примерах и задачках, рассказывая при этом о великом "вкладе" еврейства в развитие математики.

Евреи, занимающиеся математикой, предпочитают всё специфическое-эльфийское. Причём презирают тех, кто занимается вещами, имеющими практическое применение. Поэтому в Советском Союзе вышел закон, по-которому в ВУЗах должно учиться евреев пропорционально их населению. Лишних отчисляли. Преподаватели евреи на мехмате в знак протеста ушли из университета и образовали НМУ (Независимый Московский Университет). Отсюда и название в мехматянском простонародье ``еврейская секта''.

Еврейские дети в СССР часто учились отдельное от детей гоев, в специальных элитных школах. Одной такой была Московская 57-й спецшкола, ученики которой не без оснований называют себя "пятидесятисемитами". Там часто преподавали выдающиеся преподаватели с мехмата.

В основании математики последнего столетия лежит знаменитая теория множеств Георга Кантора. Если вы откроете большую часть современных серьезных учебников математического и функционального анализа или топологии, или теории вероятности, то в начале почти наверняка увидите экскурс в теорию множеств. Почти вся современная математическая литература написана на теоретико-множественном языке. Камень теории множеств лежит в основании грандиозного здания современной науки.

Но в самом сердце этой самой фундаментальной вроде бы теории, лежащей в основе "царицы наук", почти сразу после ее создания были обнаружены очень серьезные парадоксы и проблемы, не преодоленные до сих пор. Уже сто лет с тех пор математика находится в состоянии перманентного кризиса, который остро воспринимается самыми выдающимися учеными. Великий немецкий математик Герман Вейль писал по этому поводу: "Мы менее чем когда-либо уверены в незыблемости наиболее глубоких оснований логики и математики. Как у всех и всего в мире, сегодня у нас есть свой кризис".

Математика говорит, что у шпекеровой последовательности есть предел? Говорит. Практика говорит, что его нет? Тоже говорит. Математика говорит, что апельсин можно удвоить путём перекладывания его частей? Говорит. Удалось кому-нибудь с новозаветных времён повторить эту процедуру? Наблюдения раз за разом показывают, что при такого рода операциях закон сохранения вещества неукоснительно соблюдается. Математика предсказывает наличие в бесконечномерном гильбертовом пространстве базиса Гамеля. Наблюдать оный пока вообще никому не удалось. Так что математика - именно лженаука, и никак иначе.

Именно уверенность в нематериальности математических объектов влечёт за собой веру в возможность "приближённых" вычислений (что чушь - вычисления бывают либо точные, либо неверные). Да, самолёты летают и при засилье Теории Множеств. Но если бы математика была конструктивной, они летали бы лучше, потому что конструкторы не забивали бы себе голову теоретико-множественным мусором, на практике бесполезным и дезориентирующим.

Аксиомы имеют смысл только тогда, когда они выражают свойства объективно существующих предметов. Так, если мы введём аксиому "на каждой руке человека содержится 3.1415… пальцев" и построим на базе этой аксиомы формальную теорию, то положения этой теории будут бессмысленны и даже вредны.


(с) Золотце
1518 590643
>>90613

>Пойду в зог, буду демиургать и каббалировать.


Можешь про гаввах дополнить статью?

https://neolurk.org/wiki/Гаввах
sage 1519 590646
>>90598
нормиблядота тише
сажи
1520 590664
надо ли писать у нуля единицы измерения?
1521 590673
>>90664
Да, очевидно
1522 590679

>Early twin studies of adult individuals have found a heritability of IQ between 57% and 73%,[6] with some recent studies showing heritability for IQ as high as 80%.[7] IQ goes from being weakly correlated with genetics for children, to being strongly correlated with genetics for late teens and adults.


>Although IQ differences between individuals have been shown to have a large hereditary component, it does not follow that disparities in IQ between groups have a genetic basis.[12][13][14][15] The scientific consensus is that genetics does not explain average differences in IQ test performance between racial groups.


Т.е. IQ (что бы это ни было) наследуется на индивидуальном уровне, но на популяционном - не наследуется. Наука доказала.
Эти утверждения как-то согласуются, или это чистейшая шизофрения?
1523 590681
>>90679

> не наследуется


Помоему там написано не про то что "не наследуется" а что пункт 1 не может объяснить результаты IQ тестов на уровне популяции. Причиной чего может быть например всратые результаты тестов, либо на уровне популяции проявляются какие-то дополнительные факторы которые не удалось учесть на близнецах, либо еще какая хуйня.
1524 590682
>>90681

>на близнецах


Наука доказала, что характеристика наследуется не только у близнецов, а у всех. Т.е. доказана доминирующая генетическая компонента, как и у множества других характеристик, от простейшего цвета глаз до сложнейшей длины тела.
Но ахалай-махалай, не все факторы изучены. Как это?
Вот с длиной тела все факторы учтены, никаких проблем, корейцы ниже датчан. А тут нет.
как так.
1525 590685
Дайте пик, где по столбикам научные направления, а внизу к ним списки годных книг.
1526 590687
>>90679
Очевидно, что на IQ влияет в том числе среда, а среда выбирается с помощью привычек. В итоге гены скорее выбирают не само значение IQ, а направляют траекторию, по которой он будет меняться в течении жизни. Т.е. у детей мало проявляется, а у взрослых уже сильные проявления, накопленные за всю жизнь.

А с популяцией такая вещь, у тебя люди склонные к высоким баллам IQ, размножаются хуже, и популяции тупеют. Возможно это портит результаты.
1527 590690
>>90687
У тебя не больше 90.
1528 590704
Ого, тут генетики есть.
Кто-нибудь знает - желание завести потомка/параноидальные мысли что нужно оставить что-то после себя и тд передаётся генетически?
Ну то есть есть ген которые отвечает не за желание засунуть писюн в письку, а именно активирует в мозгу за навязчивые мысли что "надо завести детей", "надо наследника" и тд?
Знаю что у некоторой части бездетных женщин за несколько лет до менопаузы крышу срывает по этому поводу.
У меня в моем научном треде спор возник на эту тему, а так как я шарю в генетике чуть лучше навучпопера не смог вывезти https://2ch.hk/zog/res/768517.html#771068 (М)
2024-05-0121-17.png126 Кб, 419x263
1529 590706
Почему плотность сжиженного газа в зажигалке не уменьшается при использовании, разве газ не должен весь доступный объем занимать? Что находится в "пустых" местах, и откуда оно там берется? Если это воздух, то как он туда попадает, если давление в зажигалке больше атмосферного?
1530 590708
>>90706
В пустых местах находится испарившийся газ. Он испаряется до тех пор пока давление не повысится настолько что испарение прекратится. Если давление еще больше повысить каким-то внешним воздействием - он начнет сжижаться. Там используется пропан который сжижается при комнатной температуре и небольшом давлении. В отличие от метана который нужно либо давить до сотен атмосфер или охлаждать до криогенных температур. А лучше и то и другое сразу.
1531 590711
>>90706
Газ - это не про вещество, а про агрегатное состояние. Сжиженный газ - это жидкость, а не газ, жидкость весь объем не занимает.
Точно так же вода в жидком виде не занимает весь объем, а испарившаяся вода (т.е. водяной пар) занимает.
1532 590715
>>90708
>>90711
Спасибо.

Я верно понял, что переход жидкости в газ идет только через процесс испарения с поверхности, и плотность ее (жидкости) не понижается равномерно по всему объему при понижении давления на поверхности?

Я почему-то себе это по-другому представлял, что жидкое состояние вещества поддерживается за счет давления в баллоне, и при понижении давления оно (вещество) будет целиком становиться все более и более разреженным...

Получается, на пике мы имеем такую ситуацию: давление газообразной части вещества на жидкую слишком большое, чтобы та могла испаряться. При этом давление жидкой части на газообразную слишком маленькое, чтобы та могла конденсироваться.

То есть текущее давление их друг на друга меньше, чем нужно для сжижения, но больше, чем нужно для испарения.

И правильно воспринимать ситуацию будет не как "вещество поддерживается в жидком состоянии за счет давления", а "вещество существует в жидком состоянии, пока не будут достигнуты условия для перехода".

Если проткнуть зажигалку, то будет тот же самый процесс испарения с поверхности, только быстрый? Плотность вещества под поверхностью не изменится до того, как испарится вещество на поверхности, так?
1533 590717
>>90715
Меня всегда учили, что жидкость - это равновесное состояние между давлением паров.
В случае сжиженных газов ситуация аналогичная, только объем при этом имеет бОльшее давление, как как это нестандартные жидкости и они всегда в кипящем состоянии.
Открывая клапан на зажигалке, ты стравливаешь объем газов, который образовался при дросселировании. Оно возникает из-за пористой трубочки в жиге, которая соединена с клапаном. Жидкость снова компенсирует "пар" на собой, переходя в газ. При этом расходуется энергия, и жидкость должна охлаждаться.
Если проткнуть жигу, ты выпустишь жидкость в гигантский объем и она просто закипит от потери давления газовых паров над ней. Если это будет гигантская жига, то можно даже увидеть, как жидкость замерзнет, но потом забурлит и рассеится, как жидкий азот.
1534 590718
>>90717
А почему газ не переходит в сжатую жидкость? Так это не вода, которая конденсируется обратно из паров. Для сжижения газа нужна энергия, настолько много, что газ просто не сможет ее занять из системы.
Ведь его замкнутая система (зажигалка), наоборот, претерпевает потерю давления.
1535 590727
Скопирую для вопроса абзац с лурки.

На первый взгляд вроде всё хорошо, но при глубоком изучении выявились серьёзные противоречия следствий теории с экспериментальными данными. Например, в теории струн обязательно присутствовала частица, тахион, квадрат массы которой был меньше нуля. Ну ты понел, да? У нее масса получалась мнимая.

А в чем проблема? Почему нельзя отказаться от математических формул, изначально не предназначенных для физики, и разработать новые? Почему не может быть отрицательной массы? Почему массу нельзя рассматривать как температуру, например? Почему так важна передача информации? Если наблюдатель видит молнию, но не слышит гром, значит, информация передается со скоростью, превышающей скорость звука. Если у нас нет возможности фиксировать передачу информации со скоростью выше скорости света, это наши проблемы, а не физики.
1536 590728
>>90727
Проблема тахионов в том, что при взаимодействии они забирают всю энергию у обычных частиц и блокируют любые взаимодействия между ними. Если в модели есть тахионы, то они схлопывают систему из обычных частиц, причем за время, которые они не успеют прореагировать обычным образом. Короче тахионы прямо руинят привычную физику.
С отрицательной массой также, но только для ньютоновского случая - просто точек с отрицательной массой. Поля с отрицательной массой(точнее с отрицательным давлением) вполне легитимны.
1537 590735
>>90727

>Скопирую для вопроса абзац с лурки.


Вот тут проблема...
>>90727
>>90728
Вы столько лишнего придумываете, и непонятно зачем.
Как простой пример - работа оперативной памяти. Гляньте, как идет выборка и работа с данными и перенесите пример на многомерность и струну.
Прямо нельзя говорить, иначе ценность такого совета равна нулю, как ваш мнимый тахион.
1538 590777
>>84346 (OP)
Кто-нибудь из тех кто глубоко залезал в математику и физику, поясните супердетерминизм

1. Это про то что положение спина было определенно фундаментально всегда, сколько была Вселенная?

2. Это именно что "судьба", которая волшебным образом заставляет ученых "делать неправильно"?
1539 590781
>>90777

>положение спина было определенно фундаментально всегда, сколько была Вселенная?


Ну да. Ты лишь движешься в разрешенном коридоре событий, который детерминирован, если смотреть на это как на систему.
>>90777

>Это именно что "судьба", которая волшебным образом заставляет ученых "делать неправильно"


Непонятно. Нет, судьба это сочетание вероятностей, но оно зависит от оператора субъекта, и привязывается к носителю. Это динамическая система, с кучей факторов, а если факторов больше некоторого числа - возникает волшебный момент неопределенности. И это не наука, зачем ты сюда непроверяемое тащишь?
Нет никакой разницы, есть Д. или нет. Твоя задача жить, просто дышать воздухом, пить воду, растить сад-огород, детей и скот. Вот и вся суть величия живого. Это аксиома. Именно при таком цикле ты осознаешь, есть ли детерминизм для живых систем или нет. Все остальное - ложь. Нынешняя наука - ложь. Половина разделов науки содержит не истину, а результат наблюдений и измышление над природой факта, который проверяется, и даже используется, а истинным осознанием природы не наделен. Вот и приходится придумывать то, чего нет, чтобы вписать в конструкт, который рассыпается просто из-за невозможности своего существования.
1540 590787
Науку нужно учить в таком порядке
Физика
Потом химия
Потом биология
1541 590789
>>90787

> Науку нужно учить в таком порядке


Математика

> Физика


> Потом химия


> Потом биология



Пофиксил. Без математики – физику не понять.
1542 590791
>>90789
Во-первых, математика не наука, а речь шла про науку. Во-вторых, математику нужно учить параллельно физике. В-третьих, >>90787 тоже хуйня. Например, в курсе физики интересен вопрос о том, как впервые были получены доказательства атомарного строения вещества. А это сделали химики, химическими средствами, причём это очень красивое исследование, заслуживающее внимания.
1543 590792
>>90791
Химии как науки вообще не существует.
Там есть какой-то свод правил применимый для узкого круга реакций, но фундаментальных законов нет.
А науки надо учить начиная с ФИЛОСОФИИ, долбоебы малолетние.
Потому что только любовь к мудрости стимулирует тебя вообще что-то изучать.
1544 590807
>>90792
Долбоёб.

Химия — наука, потому что использует научный метод. То есть оперирует предположениями, проверяемыми на опыте и строит модель (описание) мира. Так же работает физика, биология, история. А философия толчёт воду в решете 90% времени. А что-то изучать тебя стимулирует заложенное эволюцией любопытство плюс социальные условия (тоже обусловленные биологической и культурной эволюцией).
1545 590809
>>90792
А фундаментальных законов нет и в философии. А ещё их нет в физике, математике, биологии, и вообще нигде их нет, кроме религии.
1546 590812
Почему мокрицы и изоподы так похожи? Они родственники? Или это какая-то исходная форма рака, от которой все прочие пошли?
1547 590814
>>90812
Класс:Высшие раки
Отряд:Равноногие
Подотряд:Мокрицы

Класс:Высшие раки
Отряд:Равноногие
Подотряд:Cymothoida

Как видим Отряд разделяется на подоотряд, ну типо родственники
1548 590816
>>90792
Ты просто не знаешь химию. Её в школе преподают на уровне 100-летней давности.
1549 590821
А сделать из урана 238 235й принципиально невыгодно энергетически или просто технологий таких нет?
1550 590822
>>90821
Из 238 не сделать 235. Нужно брать либо более легкое ядро, или разваливать более тяжелое, по правилу Содди.
Проще сразу брать торий. И дешевле на порядки по себестоимости.
IMG20240505203713563.jpg225 Кб, 960x1280
1551 590841
Как возникла РНК?
Прочитал лекции Никитина и посмотрел много видосов на ютубе, но так и не нашел ответа.
Подробно рассматривается что было до рнк и все что после.
Но насчет самой рнк упоминают лишь о теории "мира рнк"
Кто-нибудь мне может ответить?
1552 590842
>>90821
Если ты хочешь сделать ядерку из завалявшегося 238 урана, то могу посоветовать запуск реакции с помощью антипротонов, даже установки по обогащению/наработке не нужны. ЯО 4 поколения на этой принципе будет основано.
Только вот где ты возьмешь антипротоны я не знаю, думай.
1553 590843
>>90841
Просто собралась из первичного бульона который покрывал поверхность земли за десятки миллионов лет.
Бесконечно маловероятное событие помноженное на квадриллионы попыток в год и десятки/сотнимиллонов лет этих попыток.
1554 590844
>>90841

>Прочитал лекции Никитина и посмотрел много видосов на ютубе


А ты просмотрел лекции Никитина на ютубе?

Там нет прямого ответа, типа вот так вот это произошло, но он рассказывает как раз о том, как части этого процесса могли происходить, что мы об этом знаем и что ещё предстоит выяснить. Типа, в каких условиях синтезируются нуклеотиды? В каких условиях они могут сцепляться в цепочку? Какой длины цепочка? Как она будет реплицироваться, какие вероятности ошибок? Это и есть процесс создания РНК, собираются нуклеотиды, сцепляются в цепочку.

Если базово, то есть две крайности: с одной стороны можно предположить, что первая РНК сложилась совершенно случайно, не было никаких процессов или специальных условий, которые бы на ранних этапах повышали шансы создания особенно длинных цепочек или чего-то в этом роде. С другой стороны можно предположить, что в тех условиях, которые были на ранней земле, была куча факторов, стимулирующих создание РНК. Никитин как раз рассказывает про некоторые такие возможные факторы. Могла ли достаточно длинная РНК (200 нуклеотидов) собраться случайно? Могла. Вероятность этого, хоть и выражается очень маленьким числом, достаточна, чтобы в наблюдаемой вселенной за возраст её существования такое бы случилось хотя бы один раз (об этом можешь почитать у Кунина, "Логика Случая"). В такой ситуации мы можем быть единственной обитаемой планетой во всей наблюдаемой вселенной, ну или одной из очень немногих. Если же верна другая крайность, то жизнь во вселенной должна быть настолько же частым явлением, насколько часты в ней планеты, похожие на Землю. Истина вероятно где-то посередине, но к какой крайности ближе — неизвестно.
1555 590848
>>90844
В версии про то, как 200 нуклеотидов превратились пусть не с первой попытки в рнк, смущает резкий переход от более простой структуры вещества - нуклеотида в гораздо более сложную РНК.
Если попробовать проследить как усложнялась молекулярная структура: углерод и кислород стали диоксидом углерода, затем превратились в угольную кислоту, затем в барбитуровую кислоту, затем в пирамиду, затем в урацил и наконец в нуклеотид, то как будто все происходило плавнее, а потом бац и рнк.
Мне кажется было еще что-то между
1556 590849
>>90848
*пирамидин
TII.jpg463 Кб, 1280x743
1557 590850
Машину нельза прикуривать тонкими проводами, т.к. тупо сгорят(говорят)
нужны толстые провода чтобы выдерживать высокий ток.
вопрос: что это говорит о природе электров? они - частицы, которым важен физический размер ?????????
1558 590851
>>90850
Это говорит о том, что если ты гонишь кучу пьяных мужиков через густой лес ночью, при этом лес огорожен с двух сторон, то мужики, наталкиваясь на деревья, будут их шатать. И если ты увеличиваешь число мужиков, то увеличивается шатание деревьев. Но если ты раздвинешь забор, чтобы было больше места, то такое же количество мужиков в секунду будет создавать меньше шатания. Мужики и деревья можно считать классическими, хотя на самом деле речь идёт о квазимужиках.
1559 590852
А мочёные уверены, что итер будет х10 по выходу энергии? Где ни читаю, везде эта цифра.
15767892355792.jpg267 Кб, 960x783
1560 590854
Аноны, а разве в самом начале вселенной энтропия не была максимальной? Были только неорганизованные отдельные атомы водорода с высокой внутренней энергией и не было света. Это разве не пресловутая тепловая смерть?
1561 590858
Я не понимаю, почему процы и чипы на одном кристалле намного быстрее многокристалльных. Ну есть между чипами прокладка, не входящая в кристаллическую решётку кремния - это автоматически делает её гораздо медленнее? В силу каких принципов? Разве нельзя общие границы кристаллов как следует отполировать, чтоб они соприкасались почти вплотную?
1562 590859
>>90851
По-моему, главный вопрос не в этом, а в том, почему если ты пустишь этих мужиков на ультраскоростных мопедах (аналог высокого напряжения), шатать лес они должны, по идее, ничуть не меньше, а с электричеством как раз меньше и для той же мощности можно тоньше провод. Выглядит так, будто формулу мощности вывели не из природы, а наоборот природа увидела эту формулу и поняла, что так надо.
1563 590860
>>90728
Побольше распиши, мудро пишешб
1564 590861
>>90842
Обстрелять её с БАКа
1565 590862
>>90841
Йопта, в его книге ОТ ТУМАННОСТИ ДО КЛЕТКИ это всё есть, причём побробно. Бесплатно ее можешь найти
1566 590863
>>90854
Зависит от того, как взглянуть на энтропию.
Вселенная была предельно однородна и изотропна - энтропия минимальна. Начали образовываться структуры сложнее - энтропия растёт.
Классическое определение энтропии и способ её вычисления не подходят для Вселенной, так как в ней действуют силы гравитации, и вещество само по себе не образует замкнутой системы.
1567 590865
Реквестирую годные АКТУАЛЬНЫЕ научпоп (науч>>поп) книги.
image.png93 Кб, 1024x768
1568 590866
Здравствуйте, помогите. Как я понял, пукнули фотоном, на лету разрезали и осколки одновременно поймали, но на разных расстояниях? Вроде бы все логично: уменьшили массу, импульс оставили и значит должна увеличиться скорость. Это получается >скорости света, а для нас это потолок, поэтому и моментально. Типа он летит, но уже во внефотоновом мире, но летит. Здравствуйте, помогите.
1569 590868
>>90865
По каким темам?
1570 590872
>>90866
У фотона строго нулевая масса, с чего бы уменьшиться ей?

Отдача на классические объекты(измеритель, наблюдатель) строго в месте детектцией и порцией, зеркала не почувствуют отдача, пока не нарушенная когерентность.
Одна из ключевых квантовых шиз это то. что частицы "чувствуют" вещи, которые классическим образом никак не могли на наткнуться.
1571 590873
>>90865
всё, что движется
1572 590874
Можно ли лазером убить человека за стеклом, не повредив стекло?
1573 590876
>>90874
Да, главное чтоб стекло было прозрачно на длине волны лазера.
1574 590878
>>90872
Почему у фотона нету массы?
1575 590880
>>90878
Из уравнений получается
image.png2,1 Мб, 1920x1268
1576 590883
дарова мужики.кароче у меня такая проблема всю свою жизнь не любил математику и считал что она мне нужна однако сейчас у меня возникло желание вкатиться в математику хотя бы на базовом уровне. что мне начать делать какие разделы математики начать изучать и по каким потом разделам мне подниматься.из школьной программы не помню нихуя.
edPhoton.webm9,7 Мб, webm,
640x360, 3:30
1577 590884
>>90883
математика — это язык
язык нужен для выражения мыслей о чём-нибудь
хочешь математику — изучай что-нибудь, что можно выразить на языке математики
например физику
1578 590885
>>90884
меня больше интересует геометрия и всякие задачки на тему поезд выехал с такой то скоростью через сколько он приедет если скорость его была такой то.типа самые базовые вещи потому что я чувствую себя полным дебилом из за того что подобных вещей не знаю.
image.png116 Кб, 1068x428
1579 590886
>>90885
Геометрия это измерение сложных алгебраических структур.
Попробуй почитать для начала, что такое вообще числа.
Натуральный ряд, например, начинается с единицы или нуля, и почему.
И уже думай, вообще нужны тебе вещественные/действительные числа или нет.
Может, тебе интереснее будет программу написать, чтобы она в двоичном представлении тебе ответ с необходимой точностью выдала.
1580 590888
>>90880
Ну т.е. не может быть такого, что у "фотона" скорость будет больше скорости света, даже теоретически?
image.png1 Мб, 810x1080
1581 590889
>>90886

>тебе интереснее будет программу написать, чтобы она в двоичном представлении тебе ответ с необходимой точностью выдала.


я хочу начать этим заниматься не столько с точки зрения интереса сколько с точки зрения тренировки своего мозга потому что начинаю чувствовать что тупею
1582 590892
>>90888
У фотона как частицы в классическом приближении вообще не может быть скорости. Максимум можно говорить о фазовой и групповой скорости его волн, тут может быть выше скорости света. Или средней скорости его волнового пакета, тут уже нет.
1584 590894
1585 590909
>>90866
Свет это дорожка Кармана от того что поток... тёмной материи вынужден огибать препятствие в виде атома.
1586 590910
Почему этот раздел такой мёртвый?
1587 590911
>>90910
Большинство ученых сейчас сидит в /zog
тут не так интересно.
1588 590912
>>84346 (OP)
Какого хуя я должен удалять излишки жидкости через детродный орган? Что, моча что-то полезное делает для моего порта дистрибутива генетической иноформации? Может там антиспектики есть для моего пингаса? Нет. Тогда почему так? Почему нет органа для слива? Про экономию энергии не говорите. У людей овердохуя мусорных генов от вирусов и генов ошибочных шагов эволюции на синтез и перемещение массы которых тратится норм так калорий, что отдельный вентиль просто теряется на фоне этого.
1589 590913
>>90912
Ну да, моча в норме стерильна
мимо ссал себе на раны на стройке
1590 590916
В массовой культуре и в некоторых религиях существует такое явление как аура или энергия. Существут ли или возможно ли существование в реальности таких явлений как хамон из джоджо или нэн из хантера? Конечно же в форме видимой человеку.
1591 590917
>>90916

> В массовой культуре и в некоторых религиях существует такое явление как аура или энергия.


Это реальные явления субъективных ощущений при медитации, прилив энергии и без медитаций ощущаются (устал, проголодался, поел, отдохнул - почувствовал прилив энергии).
Всякие видения аур других людей это шизотерические лохотроны.
1592 590918
>>84346 (OP)
Почему одни становятся быдлом, а другие омежными интеллигентами? Что кроме темперамента и неблагополучной семьи на это влияет?
1593 590920
>>90918
генетика.
1594 590921
>>90916
не в форме
фикс.
1595 590925
>>90918

>Что кроме темперамента и неблагополучной семьи на это влияет?


А что тебе еще надо? В общем и целом ты правильно сказал - гено-социо-культура
1596 590926
>>90911
Двачую анона.

мимо ученый, сидящий в /zog...
1597 590927
>>90911
Какая ирония
1598 590928
>>90916

>аура


Хуета эзотерическая.

>энергия


Термин из физики, имеет конкретное понимание и применение в науке и технике. Эзотериками используется без какого-либо чёткого смысла, совершенно произвольно, для описания чего-то абстрактного и непонятного.

>Существут ли или возможно ли существование в реальности таких явлений как хамон


Вот это точно существует отвечаю. Вкусная вещь, на бутер с тапенадом, под красное вино охуенчик.
zj1e4nfynmzc1.webp90 Кб, 1024x563
1599 590932
На пике слайды из общего курса биологии Йельского университета.
Внимание, вопрос: как контрить?
Новука доказала, полов не два.
И ты тут такой, выпускник Усть-Задрищенского меда: на самом деле два.
В Йеле виднее же лол.
Что это доказывает? Что новука сосет бибу.
1600 590933
>>90932
Хуиту пишут, sex(пол) можно приписывать только особой группе клеток, прямо непосредственно участвующих в половом процессе, т.е способны обменивать своими генами. При этом эти клетки бессметны и непрерывны. Иногда эти клетки отращивают вокруг себя целую тушку из пониженных в правах смертных клеток. Отращивание тушки сам по себе невероятный эволюционный костыль с кучей уязвимостей. По какой-то причине эти нелепое нагромождение кривых механизмов считают полом. Другие наоборот прямо все отрицают пол и придумывают маняхуйню о самоощущении тушки.
1601 590936
>>90933
Многоклеточность появляется раньше мейоза, что ты несешь.
1602 590937
>>90932

>Новука доказала, полов не два


Новука наоборот, давно доказала что их 2.
Современные пидорасы, сидящие в научном сообществе просто переписывают её под себя.
1603 590938
>>90807
История общественная наука, день добрый, как и философия. Никакого научного метода что в истории, что в философии нет.
1604 590939
>>90188
Ну в принципе подтверждение той шизы, которая сейчас творится в международном научном сообществе.
Этот прав >>90937
Проект под названием "наука" можно закрывать нахер, так как скоро не будет нести в себе практической пользы.
1605 590940
>>90937

>Новука наоборот, давно доказала что их 2.


После этого наука сделал шаг вперёд и доказала, что не 2.
Как доказала? Просто изменив дефиниции и добавив семантической неопределенности.
И что ты с этим сделаешь? Как ты докажешь Йельскому профессору, что их два? Громко пукнешь? Сослешься на определение из старого учебника?
Ты нихуя не сможешь сделать, нет таких механизмов в новуке. Как начальство сказало - так и есть.
1606 590941
>>84346 (OP)
>>90925
Возможно ли создать идеального человека? Если нет, то почему?
KarlMarx001.jpg269 Кб, 790x1004
1607 590942
Сап наукач, как вы относитесь к этому челу? Интересует мнение вас, как, ну наверное учёных.
Не совсем по теме борды
1608 590943
>>90942
1) https://ru.wikipedia.org/wiki/Калькуляционный_аргумент
2) бихевиоризм не работает
3) номенклатура выродилась в отдельный класс со своими классовыми интересами
1609 590944
>>90941
Потому что невозможно определить, что значит идеальный.
1610 590945
блять да ответьте уже на вопрос, третий раз задаю: скольки юродные родственники друг другу люди в среднем в мире? неужели никто не мерил? очень важная инфррмация
1611 590947
>>90938

>История общественная наука


Да, я в курсе.

>как и философия


Нет, философия не является общественной наукой.

>Никакого научного метода что в истории, что в философии нет.


В философии нет, а в истории есть. Историк делает проверяемые заявления, его выводы можно опровергнуть на основании фактических данных.
1612 590949
>>90945
Коффицент кровного родства у четвероюродных сиблингов - 0.1953125% (у двоюродных - 12.5%). Т.е. дальше считать смысла нет, такие люди не более родственники, чем рандомные люди из одной популяции, родство на уровне статистического шума.
1613 590950
>>90940

>Как ты докажешь Йельскому профессору, что их два?


Барнаул. Алтайский край.

>Как начальство сказало - так и есть.


Чужое начальство только подпортит жизнь чужим врачам, усложняя бумажную работу >>90185 но при лечении пациентов от болезней будут применяться обычные методы.
1614 590951
>>90950
В Барнауле какая-то своя, отдельная от Лиги Плюща, биология? Что ты несешь, пидор тупой?

>Чужое начальство


У науки одно начальство.
1615 590952
>>90947
Философия - общественная наука, заходим на Вики в раздел Наука и смотрим.

>фактических данных


А они изначально без историка откуда возмуться?)
1616 590953
>>90951

>У науки одно начальство


У науки нет начальства в принципе. Иначе это противоречило её объективности.
1617 590954
>>90953

>У науки нет начальства


ахаххахахах

>Иначе это противоречило её объективности.


Ну то есть ты согласен со слайдами >>90932
Наука доказала. Объективно.
1618 590955
>>90954
Нет, это ненаучная херня, признаная научной.
Screenshot 2024-05-11 Social science - Wikipedia.png148 Кб, 1027x883
1619 590956
>>90952

>Философия - общественная наука, заходим на Вики в раздел Наука и смотрим.


Ну вот я зашёл, и что я вижу? Только вики — это в любом случае не аргумент, её пишет кто попало, там они историю тоже в extrascientific запихнули.

>А они изначально без историка откуда возмуться?)


Ты долбоёб? Ты спрашиваешь откуда берутся факты? Они блядь происходят. Физик узнаёт о фактах в результате измерений, а историк — в результате изучения источников или археологии.
1620 590957
>>90951

>В Барнауле какая-то своя, отдельная от Лиги Плюща, биология?


Нет, но в Барнауле нет ни Йельских профессоров ни их антинаучной поебени, и тебе незачем кому-то что-то доказывать.
1621 590959
>>90956
Англоязычная вики - кал, заходи на РУ.
1622 590961
>>90956

>Откуда берутся факты? Они блядь происходят.


В Истории немного не так) Научно-исторический факт — это исторический факт, который стал объектом деятельности историка учёного; результат умозаключения, основывающегося на следах, оставленных прошлым.
Это нихера не объективно по определению и утверждать однозначно невозможно, именно поэтому история - общественная наука.
1623 590962
>>90961

>Научно-исторический факт — это исторический факт


Ок, теперь неси определение, что такое исторический факт.

>>90959
Ровно наоборот, особенно по социальным наукам. На рувики всё ещё закостенелые спекулятивные понятия вместо нормального позитивистского подхода. Позитивизм = верифицируемость, без верифицируемости ты просто зарываешь себя в болото пустых рассуждений и размазывания воды по бумаге.
1624 590966
>>90957

>антинаучной поебени


Барнаулец, что ты несешь? Знай своё место.
1625 590974
Почему пикрил вечный двигатель не будет работать?
2343.png109 Кб, 800x600
1626 590975
>>90974
*прикрепляю
1627 590976
>>90974
По второму началу термодинамики.

>>90975
Да мог и не делать лишней работы.
1628 590977
>>90976
Но почему? Отталкивающая сила магнитов же никуда не денется
1629 590978
>>90977
Ну попробуй собрать эту шайтан машину, как заработает сразу богаче илона маска станешь.
Все же скрывают
1630 590979
>>90977
Я вообще не смотрел твою схему. Сейчас глянул, нихуя не понял. Объясни, как это должно работать.
1631 590980
>>90977

> Отталкивающая сила магнитов же никуда не денется


Но сила магнитов из неоткуда не берётся и не бесконечна

https://youtu.be/O_sH17NQ6mg?si=vYNWABDZqERqo03r
1632 590994
В чем отличие химфиза от физхима?
1633 591000
Можно ли поставить себе вместо зуба имплант с какой-нибудь встроенной полезной для меня вещью?
Понятно, что чип слежения мне не нужен.
Но может какое-нибудь средство самообороны или радио или еще что. Прямо как в романах про спецагентов.
1634 591006
Выкинуть маленького дельфина посреди океана, так же как если бы инопланетяне выкинули потерявшегося младенца в лесу? Ну в смысле ему пизда?
https://www.youtube.com/shorts/Yvs0PqB9eqs
1635 591008
Мультивселенной нет.
Это бред как фарш провернутый назад просто так без причины.
1636 591009
>>91008
Это противоречит причинно следственной связи
1637 591010
>>91009
Мультивселенной невозможна.
Это так же что телеведущий снимет штаны начнет срать в камеру прям на объектив, вместо репортажа.
1638 591017
>>91010
на нерухнум пидорасие и не такое бывает

>>91008
есть, новука докозала!
1639 591018
Есть ли простое объяснение, как "калории" соотносятся с ростом жировой ткани?
Допустим, есть кусок сала - X калорий, стакан сахара Y, и стакан водки, Z ГООООООЛ.
Не учитывая всякие сложные процессы пищеварения и влияние на гормоны, как сферический организм в вакууме превратит это в жир? Есть какие-то формулы, коофиценты?
1640 591019
>>91018
Нужно объяснение уровня ответа на вопрос куда девается жир при потере веса - по большей части выдыхается через легкие.
1641 591021
>>84346 (OP)
Нахуя эволюция придумала смерть от биологического старения? Типа какой вообще профит для существа от того, что он стареет?
1642 591023
>>91021
Могу предположить, что ускорение эволюции. Поколения сменяют друг друга, не задерживаются, быстро происходит отбор.
1643 591024
>>91021
Для "генов"(сюда еще идут не только гены как таковые, а способы их взаимодействия и прочая функицональщина), на которых непосредственно действует отбор, тушка носителя не более чем временная машина для выживания. В добавок гены живут не в одном организме, а сразу же в нескольких. Зачастую долгое пребывание генов в одном организме имеет слишком большие издержки при отборе, поэтому гены как бы стремятся побыстрее сменить тушку.

Короче, давление отбора не способствует сильному увеличению жизни отдельного организма. Гены, которые помогают тушке чинить себя, работают только до того момента, когда могут быстро сменить тушку.
1644 591025
>>91021
Чтобы скотина знала что скоро умрёт и что после смерти будет ждать только НЕБЫТИЕ, а не всякий бред про встречу с родственниками/40 девственниц и тд.
Это нужно для увеличения отделения гавваха от разумных форм жизни.
1645 591026
>>91021

>Нахуя эволюция придумала смерть от биологического старения? Типа какой вообще профит для существа от того, что он стареет?


В двух словах - потому что эволюция направлена на сохранение гена, а не особи. Особь нужна только пока она способствует репликации гена. Почитай "Эгоистичный ген" Докинза, если интересны подробности.
image.png75 Кб, 1424x497
1646 591027
Не уверен, есть ли тут по-настоящему шарящие за ОТО...

Как гравитация выходит из чёрной дыры? По всем понятиям гравитационное взаимодействие распространяется со скоростью света. И распространяется оно по нашему родному пространству-времени, им самим же и искривлённом. Вот свет, например, за пределы горизонта событий никак не может выйти, потому что так пространство-время завёрнуто. А гравитация какого хуя выходит тогда? Она же должна как-то распространяться. А если пространство-время так завёрнуто, что оно заворачивает вообще всё, то оно и само себя должно завернуть, выстрел в хуй, так сказать. Почему этого не происходит? Откуда пространство-время возле чёрной дыры узнаёт, как именно оно должно завернуться?

>INB4: Тебя это ебать не должно. Уравнения гравитационного поля решай, а не в голове гравитацию воображай. Вообще ничего не надо пытаться понять, надо просто уравнения решать, и что получилось, то и есть истина


Я всё же хотел бы понять...

Ещё часто вижу, как физики в своих моделях говорят о присутствии электрического заряда у чёрных дыр. Вопрос аналогичный. Как они собрались электрически взаимодействовать с сингулярностью или чё там внутри? Вы причинно не связаны, ало! Тут хоть через частицы-переносчики, хоть нормально объясняй, всё равно хуета какая-то получится

А ну быстро мне квантовую теорию гравитации запилили!
1647 591028
>>91027
Гравитация и не выходит. Гравитация в ОТО это не взаимодействие и даже не динамическое сила, это просто криволинейные координаты пространства-времени. Поэтому и называют ее Общей(для всех случаев) Теории Относительности. Криволинейные координаты могут порождаться как выбором системой отчета, так и геометрией пространстве-времени. И возникающие эффекты равноценны и равноправны, собственно про это и есть принцип эквивалентности.
В таком случае гравитации нельзя задать какую-либо скорость и прочие механистические атрибуты, геометрия в некотором роде статична, меняется только методы каким мы ее прощупываем. Когда говорят о том, что гравитация распространяется со скоростью света, имеется в виду, что есть предел как материя(вещество+излучение) может быстром менять свою точку зрения на геометрию пространства-времени.

Есть в математике спец раздел - дифференциальные формы, в нем обобщается как у распределенных величин разных размерностей имеются строгие соотношения. Пример как величина размазанная по объему может переформулирована на величину, которая размазана по поверхности границы этого объема. Это позволяет считать гравитацию не по всему объему, а по некоторому замкнутой поверхности, ограничивающая этот объем.
Так что гравитация знает о ЧД, лишь потому что это уравнения позволяют.
1648 591029
Какая-то баба на миллионную публику либерах, втирает им дичь, о том, что ебаные бибизьяны умеют говорить языком жестов.
Что это за пиздец и почему такое происходит?
Нахуя политическим идиологам втирать людям такую хуиту? Это типа максимальный антирасизм, если устранить у шимпанзе социо-экономический фактор, то они тоже смогут быть учеными и врачами? Ум же не передается по генам, значит и макаку можно научить всему.
https://www.youtube.com/watch?v=R1lNwZS6wz4
1649 591030
>>91029
Жиды берут новый рубеж селекции вырожденцев - теперь вместе с шерстистыми будут завозить еще и натуральных обезьян. И белые тянки в каждом первом фильме шитфликса будут прыгать на big monkey cock.
1650 591031
>>91030

> big monkey cock.


У обезьян хуйцы меньше, чем у homo sapiens.
1651 591032
>>91029
Дак обезьяны поумнее некоторых мигрантов будут, им то много поколений приходилось выживать в условиях дикой природы, а биомусор выживает за счёт гум. помощи которые жиды привозят.
Просто добавить ген чтобы гортань развивалась по человескому типу и могла производить звуки и все - новая расса людей.
1652 591035
>>91029

>втирает им дичь, о том, что ебаные бибизьяны умеют говорить языком жестов.


А с чего ты взял, что не могут? Просто потому что это ломает твой манямирок, как в своё время сам факт происхождения людей от обезьяноподобных предков ломал манямирок викторианцев? Ну, это не аргумент. Есть множество исследований, в которых обезьян учили говорить языком жестов, а также с помощью пиктограмм. Они на определённом уровне осваивают эти навыки, умеют строить предложения и использовать выученные слова.

>Нахуя политическим идиологам втирать людям такую хуиту?


При чём тут политическая идеология? Политической идеологией тут сейчас занимаешься ты, заранее решив, что всё это неправда и подводя под всё это свои политические убеждения.

>Это типа максимальный антирасизм, если устранить у шимпанзе социо-экономический фактор, то они тоже смогут быть учеными и врачами? Ум же не передается по генам, значит и макаку можно научить всему.


Тут ты уже занимаешься додумками и приписываешь учёным то, что они никогда не заявляли даже близко.
1653 591041
>>91028

>Есть в математике спец раздел - дифференциальные формы, в нем обобщается как у распределенных величин разных размерностей имеются строгие соотношения. Пример как величина размазанная по объему может переформулирована на величину, которая размазана по поверхности границы этого объема. Это позволяет считать гравитацию не по всему объему, а по некоторому замкнутой поверхности, ограничивающая этот объем.


>Так что гравитация знает о ЧД, лишь потому что это уравнения позволяют.


Звучит как маняфантазии третьекурсника, который про дифгем в ото узнал, а про физику ото - еще нет. И никакие теоремы стокса или когомологии де рама тут не причем. "лишь потому что это уравнения позволяют" это универсальный отъебись-ответ того, кто думает, что понимает.
1654 591055
>>91035

>Они на определённом уровне осваивают эти навыки, умеют строить предложения и использовать выученные слова.


Нет.

>Политической идеологией тут сейчас занимаешься ты


Решил посрать тупостью? Гомогей и селедка с амого начала начали натягивать сову на политическую повесточку, сперва говоря про матриархат у бонобо и что их самки живут в коммунизме, поддерживают друг-друга и доминируют хуемразей.
Потом что обезьяны такие же люди и интеллект не передается по генам.
Потом пошли вкукареки о сравнении территориального поведения шимпанзе с сво...
Дальше я этот кал говна не смотрел, уверен продолжалась эта параша в том же духе.
1655 591078
>>91055

>Нет.


Пидора ответ блядь, неси пруфы, что нет.

>Дальше я этот кал говна не смотрел


Я вообще твою еботню не смотрел, я тебе пишу что знаю, мне похуй, что ты там где смотрел, еблан тупой.
1656 591080
>>91078

>Пидора ответ блядь, неси пруфы, что нет.


Так наука не работает, быдло.

>Я вообще твою еботню не смотрел, я тебе пишу что знаю, мне похуй, что ты там где смотрел, еблан тупой.


Слив засчитан.
1657 591081
>>91080

>Так наука не работает, быдло.


Именно так она и работает. Есть исследования, которые говорят, что обезьяну можно научить языку жестов. Ты теперь утверждаешь, что эти исследования хуйня. Объясняй почему, приводи исследования, которые говорят об обратном.
1658 591083
>>91081
Приноси свои исследования, поссым на тебя, опущенка.
1659 591087
>>91083
Не чувак. Понимаешь, я бы принёс, я бы уже давно принёс, но дело вот в чём. Ты буквально с самого начала решил, что тебе не нужно искать эти исследования, вкладывать хотя бы минимальные усилия, или даже хотя бы попросить принести их тебе, чтобы ознакомиться и сформировать мнение. Нет, ты просто сразу постулировал, что всё это политически ангажировання хуйня, просто потому что тебе так захотелось. Сейчас, после нескольких постов, в которых ты ровно так же продолжал настаивать, что тебе всё лучше известно, хотя с исследованиями ты не ознакомился, ты просишь их тебе принести. Нет, иди нахуй, их очень легко найти, поэтому сделай эту работу сам. Не захочешь — дело твоё, я не настаиваю. Кидать перед тобой бисер я не собираюсь.
1660 591091
>>91081

>Есть исследования


Постить ты их конечно же не будешь.
Ну тогда есть опровержения. Я все еще победил, что сказать, то хотел питух?
1661 591110

> Пидора ответ блядь, неси пруфы, что нет.


>>91078
Хомский их обосрал лет 50 назад. Проведённые исследования ничего толком не доказывают но и не опровергают. ИМХО, Хомский был больше несогласен с выводами. Обезьянка знает сотню слов, которые как не комбинируй - получишь что-то осмысленное. Т.е. находится в ситуации, когда обосраться невозможно в принципе. Макака говорит говно-моча, а исследователи объясняют, что это макака ввела новый термин, потому что не знает какого-то там слова. А вообще она имеет в виду бренность бытия и несправедливость во всём мире.

Первый эксперимент (относительно удачный, или просто выдали желаемое за действительное): https://en.wikipedia.org/wiki/Washoe_(chimpanzee)
Научпоп по первому эксперименту тут: https://ethology.ru/library/?id=118 На это в РФ любят ссылаться уже десятилетия (у Гордона в полночь такое тоже было).

Второй эксперимент (фиал): https://en.wikipedia.org/wiki/Nim_Chimpsky
Есть кинцо одноимённое (не смотрел). Ну типа Хомского не удалось переубедить. А может просто мартыхан попался несмышлёный.

Краткий пересказ эпопеи, например, тут: https://vk.com/wall-166353059_29830

P.S. В андроиде месяц назад насвайпал Жириновский и дальше тыкал в предложения свайпалки. Получилось "Жириновский предложил запретить новогодние подарки и сувениры". Вот примерно что-то похожее с макакой вышло. Что ни скажи - везде можно какой-то смысл найти.
1662 591112
Аноны, подскажите какие-нибудь русскоязычные или англоязычные научпоп-ресурсы, раскрывающие новости из мира астрофизики, квантовой механики и смежных дисциплин. Как раньше был журнал Популярная механика до становления текинсайдером, вот нечто подобное, очень нравилась подача материала там.
1663 591132
сап, двач. есть вопроc по физике. читал я учебник и решил попробовать получить индукционный ток. взял катушку из нескольких проводов, намотанных на одно кольцо. один провод я замкнул мультиметром поставил на измерение силы тока(постоянного, хотя наверное в данном случае разницы нет, так как я увижу что значение на дисплее изменится и успокоюсь) и начал двигать вокруг катушки магнитом. как итог я ничего не получил. но почему? ведь по закону фарадея я, двигая магнит, то есть изменяя магнитный поток, вокруг контура, то есть катушки, в катушке должна возникнуть ЭДС индукции.
1664 591134
>>91110
Читай книгу "О чём рассказали говорящие пидорашки обезьяны", кринж
1665 591135
>>91112
Элементы ру
1666 591137
>>91134
Лучше глянь
https://www.youtube.com/watch?v=GnRzPLI4nbE
вот это реально кринж так кринж. И только подумать что "исследователи" с дипломами наваляли такую кучу.
Добавить к этому еще то как в последнее время топовых капченых начали ловить на том что свои "исследования" они буквально доставали и собственной сральни сочиняя числа и беря картинки из интернета.
Еще анализ:
https://www.youtube.com/watch?v=e7wFotDKEF4
1667 591138
>>91132
1. Намотай побольше витков.
2. Ток на миллиамперы. И ещё проверь, что щупы правильно воткнул (если воткнуты в U/Ω ток не увидишь).
3. Магнит желательно в центр катушки (чтобы он как бы стал сердечником) и выдёргиваешь магнит РЕЗКО.

Есть вариант проще - раскрути вентилятор от компьютера. Вольт 7 реально увидишь (ну если раскрутишь до тех тысяч оборотов, на которых он вращается). Сколько это в амперах - хз.
1668 591143
>>91132
Мультиметр цифровой? Они плохо измерять импульсные вещи. Лучше бы взял осциллограф.
1669 591151
Как думаете, каков будет предел возможностей ИИ?
Вокруг много тех, кто относится к сверхчеловеческому ИИ буквально как к Богу. Но как насчёт фундаментальных законов вселенной? Сможет ли сверхразумный ИИ построить машину времени? Вечный двигатель? Сверхсветовой корабль? Сможет ли стать демоном Лапласа? Заглянуть за горизонт событий?
Что-то я сильно в этом сомневаюсь.
1670 591165
>>91151
Искусственный интеллект сможет сделать абсолютно нихуя так как пока не было установлено, что так называемый "Искусственный Интеллект" может существовать. А абсолютно каждый человек под этим набором букв понимает что-то своё, поэтому выяснить про него ничего нельзя. Можешь в следующий раз спросить: сколько ангелов помещается на игольном ушке? А также: В каком виде энергия существует в мире Брахмы, где нет форм и материи?
1671 591166
Что такое секс с научной точки зрения? Как и что является извращением, а что нет? Заходил на киберленинку, там писали вообще, что секс - есть завуалированное насилие, прикрытое естественной (животной) жизнью человека. Насколько точно и правдиво это? Если так подумать, то правда в этом всё-таки да есть.
1672 591169
>>91166

>секс


>извращение


ты пришёл в раздел науки просить дать определение каких-то культурологических терминов?
1673 591171
>>91166
хз, 2 года не ебался, забыл, как это
1674 591173
>>91166
В общем смысле секс это целенаправленные движение тушки, чтоб участвовать в половом процессе. Как тушка обзаводиться сложной нейросетью и генетическая детерминированность поведения падает, то что можно назвать сексом становиться условностью. Так брачные игры у птиц можно считать сексом, или сложное социальное поведение у волком или обезьян.
1675 591174
>>91169
То есть деление на извращения и нет в половом акте - в корне не верны?
1676 591175
>>91174
утверждение "наукотред — не место для определения культурологических терминов" ты понимаешь как "деление на извращения и нет в половом акте - в корне не верны"?
1677 591176
>>91174
Я сомневаюсь, что термин "извращение" ввели учёные, стало быть и объяснять это понятие не их работа. Лучше пойди в /law, /re, /dev, /sex, /psy и спроси там.
1678 591182
Наткнулся на спор о том, является ли вселенная physical или computational. И что-то я не понял, в чём смысл спора. Типа, какие-то процессы во вселенной напоминают нам компьютерную программу? Квантование времени и энергии, например? Но какая есть причина считать, что это не компьютерная программа похожа на вселенную.
1679 591202
>>91176
Окей, спасибо.
1680 591205
>>91182
Нет, не так.
Все сводиться к тому, что Вселенную можно свести полностью к абстракции(отбросив все лишние физическое) и когда построим модель на основе этой абстракции, то она становиться неотличима от Вселенной. У хороших "моделей" есть свойство вычисляемости - к абстракции можно применять различные алгоритмы. Пример математический предел - берем последовательность и приближаем бесконечное количество раз.
1681 591221
Химики, если вы есть, обесьните все-таки по хардкору, ОПАСНЫЙ этот раундап или нет?
Мамка не понимает, что такое природа, дача, почва, и заливает всё этим говном, "борется с травой". Я-то надеялся, что наше глухое место останется чистым и там можно будет пересиживать БП, растя луковки. Но хуй, она сама его загаживает!
Загаживает же? И все не так страшно? Может он потом распадется? Узбагойте меня если можете, миня трисет и пичаль большая.
1682 591222
>>91221
Вот он:
https://ru.wikipedia.org/wiki/Глифосат
Нидавать исследовать, исследовательская заказуха... Вы-то как нормальные люди что здесь видите? Мой бы школьный химик быстро бы раскидал.
Аноним 1683 591253
>>84346 (OP)
Как правильно искать и проверять информацию. Заметил, что в интернете возможно найти аргументы в пользу абсолютно любого довода. Как понять истинность суждения ?
1684 591256
>>91253
Есть информация, что тебя зовут Данила.
1685 591261
>>91253
Данила, спокуха ) Любитель проверять информацию
image.png382 Кб, 500x500
Помогите найти информацию о явлении в психологии 1686 591305
Хочу разобраться в явлении, которое отвечает за формирование идеологических групп.

Два человека со схожими взглядами будут друг на друга экстраполировать/проецировать свои убеждения, поэтому им будет казаться, что верят они в одно и тоже.

А если один из таких людей окажется активным лидером, который занимается агитацией своего виденья мира, то уже множество других людей со схожими взглядами смогут проецировать свои убеждения на него и им будет казаться, что лидер и его окружение верят в тоже самое, таким образом формируя группу "единомышленников".

Поиск солидарности, желание найти тех, кто поймёт тебя, опускает таким образом некоторые не состыковки во взглядах и подначивает группу к продолжению своего существования.

Мне нужна литература, где я больше смогу конкретно узнать об этом явлении, об этом механизме. Пожалуйста, подскажите мне, где можно прочитать об этом?
1687 591314
Хз где спросить. Неоднократно встречал упоминания того, что Роберт Сапольски в жизни как-то раз попал в плен к ниграм-пиратам, или что-то типа того. По каким словам это гуглить? Я вообще ничего похожего найти не могу.
55c7a963a8e2f.jpg114 Кб, 700x469
1688 591315
>>91314
Если инфы нигде нет, может быть она неверная или искажена. Например, в конце 1970х Сапольский был в Африке и специально поехал в Танзанию чтобы увидеть боевые действия (как раз шла угандийско-танзанийская война). И таки да, видимо, насмотрелся там всякого.
Может ты про это читал?
scale1200 (2).jpeg224 Кб, 1200x849
1689 591316
>>91253

>Как понять истинность суждения ?


Ну, начинать, как правило, нужно с фактов, на которых суждение основано. Насколько источник заслуживает доверия. Подтверждается ли инфа кем-то ещё (желательно, независимо от первого источника). А дальше уже идёт анализ собственно аргументации. Этому имеет смысл подучиться. Есть книги, описывающие методику. Вот, например, книги:
Поварнин. О теории и практике спора.
Халперн. Психология критического мышления.
Шопенгауэр. Искусство побеждать в спорах
изображение.png172 Кб, 1120x486
1690 591318
>>91315
Тред в /b/. Но ещё до этого знакомый упоминал в разговоре, что в какой-то книге Сапольски были рассуждения о том, что он раньше думал, что с любым человеком можно договориться. Но когда встретился с ниграми с автоматами, быстро оказалось, что убедить их ни в чём нельзя. Я просил его потом уточнить, откуда это, но вопрос как-то заглох, и мы миллион лет вообще не общались.
А тут опять что-то очень похожее в /b/. Там, конечно, могут писать любую хуйню, но меня зацепило то, что это звучит очень похоже на те слова моего знакомого. Просто без задней мысли иди и перечитай все его книги, делов, ага. Но, блин, это звучит как достаточно примечательная история, чтобы её ещё хоть где-то упомянули?
187.UX200CR0,35,200,200.jpg8 Кб, 200x200
1691 591320
>>91318
Возможно, речь всё же о том случае с посещением Танзании во время войны. Там запросто мог быть эпизод с контактом в местными. Проблема в том, что в таких вот дискуссиях информация может порой сильно искажаться и обрастать всякими несуществующими деталями.

>меня зацепило то, что это звучит очень похоже на те слова моего знакомого


Да, даёт надежду, что там, как минимум, есть доля правды.
Но надо учитывать вариант, что и тот твой знакомый, и анон из /b просто кормились из одного и того же источника дезинформации
1692 591324
>>91318

>Но когда встретился с ниграми с автоматами, быстро оказалось, что убедить их ни в чём нельзя. Я просил его потом уточнить, откуда это



Это оттуда, что негры, как и все остальные животные на планете, понимают только язык силы. Если бы он с ними разговаривал с позиции инопланетянина, космический корабль которого нажатием одной кнопки может всю их уганду превратить в воронку размером с уганду, они бы очень быстро нашли друг с другом общий язык.
1693 591325
>>91318

>без конца поили его Колой хотя он хотел жрать, спать и домой


И в чем они были не правы? Я так понял, нигры рофлили с белого господина и на личном опыте давали ему почувствовать насколько в реальности велика ценность его божественной демократической колы.
levi.png120 Кб, 660x977
1694 591326
>>91318

>думал, что с любым человеком можно договориться


>оказалось, что убедить их ни в чём нельзя


Леви-Брюль неплохо описал, как это работает
у людей с пралогическим мышлением своя атмосфера
1695 591330
Подскажите пожалуйста, является ли этот тред подтверждением феномена судьбы и временной петли?

https://2ch.hk/b/res/305211697.html (М)

Мы там придумывали название третьей мировой войны чтобы зафорсить для учебников будущего. Но анон на квадрипле быстро выиграл с название Специальная Военная Операция, который как бы из прошлого. Все бы это было не таким феноменальным, если бы не квадрипл на его термине. Выходит, случайности не случайны и все является закольцованным во времени лупом.

Просто за 13 лет на двачах впервые такое вижу.
1696 591345
>>91330
Это называется магическое мышление. Ты триггернулся на ничего не значащее совпадение как голубь Скиннера.
1697 591348
>>91345
А может быть ты просто нигилист.
Ну, какой шанс был у этого события? примерно можешь почувствовать?

Я бы сказал, шанс был равен обезьяне пишущей на машинке Войну и Мир.
39f6bd446d695e7fdbabce6ff9a12254.jpg42 Кб, 800x450
1698 591349
>>91330
Случай прикольный. Но вполне может быть и так, как предположил вот этот анон >>91345. Хотя лично я не склонен к категоричным заявам. Мы ж явно многого не знаем об окружающем мире
1699 591350
>>91349

>Мы ж явно многого не знаем об окружающем мире


Спасибо за адекватный пост.
1700 591360
>>91345
а если наблюдается закономерность? это тоже магическое мышление? лично я, когда думаю о чём-то слишком грандиозном, то в такие моменты попадаю в неловкие ситуации, связанные с обыденными вещами (уронить кружку, порезаться, споткнуться, удариться, подавиться и тд).
какие-то совсем неслучайные совпадения, может на самом деле в такие моменты что-то происходит с мозгом, из-за чего он перестаёт уделять внимание мышцам.
1701 591362
>>84346 (OP)
У "кризиса среднего возраста" есть строгие критерии?
1702 591363
>>84346 (OP)
Существует ли надежный способ заставить себя учиться?
1703 591370
>>91362
Окстись

>строгие критерии


тут топовых психолухов начали пачками ловить на том что свои "исследования" они буквально доставали из сраки, просто выдумывая подходящие числа.
1704 591373
Почему человек свободен?
Логичнее предположить что человек как планета свободен.
Может начать крутиться в другую сторону и уйти с орбиты, но делают это ноль один процент .
1705 591389
Каково научное определение человеческого "я" - что это такое?
1706 591390
>>91389
"я" — это часть речи, указывающая на того, кто говорит, не называя его
1707 591392
>>91390
Это немного, как спросить "что такое планета" - "планета - это часть речи", ну в целом не поспоришь, но, вопрос все-таки о том, на что же такое она указывает
1708 591393
>>91392
таки да, планета — это часть речи, указывающая на небесное тело, вращающееся по орбите вокруг звезды или её остатков, достаточно массивное, чтобы стать округлым под действием собственной гравитации, но недостаточно массивное для начала термоядерной реакции, и сумевшее очистить окрестности своей орбиты от планетезималей

на что указывает "я" — вопрос не научный, зависит от мировоззрения говорящего и требует определения в каждой дискуссии

я с помощью "я" указываю на тот организм, которым я являюсь
внутри у меня есть нервная система, которая под воздействием сигналов из внешнего мира получает субъективный опыт
процесс получения субъективного опыта я называю "сознание"
а ты?
1709 591394
>>91393
Мне в таком определении не нравится, что та зона, которую ты очерчиваешь как "я" в итоге шире - чем зона того, что ты называешь сознанием. Организм он во многих ситуациях работает сам по себе, и даже если из всего организма мы возьмем только мозг - так он тоже. Странно называть себя вещью, которую даже не сознаешь и к которой у тебя и доступа-то нет - это похоже на то, как люди могут считать себя душой, которая метафизична и где-то скрыта и никто ее не видел, даже ты сам.

Мне по моим субъективным наблюдениям, кажется что "я" - это некий меняющийся набор субъективного опыта - который уже, чем сознание в твоем определении. Когда в моменте быстро думаешь "я" - мгновенно обращаешься к неким соматическим переживаниям, но штука в том, что в разные моменты это могут быть разные переживания - граница меняется. Есть конечно же "я" как часть речи и концептуализации, но там это пространство ангелов как-то танцующих на кончике иглы.

То есть, я думаю что "я" - это некое ощущение отождествления с частью своего субъективного опыта - и отталкивание другой его части как "не я" и тогда это можно сделать научным вопросом - если например найти психотропный препарат, который может это ощущение отключать и человек будет рапортовать исследователям "меня нет"
1710 591395
>>91394

>некий меняющийся набор субъективного опыта


>некое ощущение отождествления с частью своего субъективного опыта - и отталкивание другой его части как "не я"


"некий" — плохой слово для определения
когда ты говоришь "я пошёл в магазин", то имеешь ввиду, что в магазин пошёл меняющийся набор субъективного опыта или ощущение отождествления с частью своего субъективного опыта?
1711 591396
>>91395

>"некий" — плохой слово для определения


Некий, потому что набор меняющийся - иногда это отождествление на карте тела больше в груди, иногда в голове итд.

>когда ты говоришь "я пошёл в магазин", то имеешь ввиду, что в магазин пошёл меняющийся набор субъективного опыта или ощущение отождествления с частью своего субъективного опыта?


Да, какое правильно описание ситуации "человек пошел в магазин":

-Я пошел в магазин
-Он пошел в магазин
-Тело идет в магазин
-Поход в магазин произошел

наличием слова "я", подразумевается именно факт наличия определенных ощущений отождествления в этом процессе - я не увидел их на киноэкране, мой организм не встал сам и не пошел в магазин, к моему удивлению и поход в магазин не ощущается как некое самостоятельное событие без субъекта - в этом процессе "похода в магазин" я ощущаю отождествление с некой его частью - которую и называю я.

Так что хотя звучит это громоздко, мы имеем ввиду по сути именно это.

Представь еще что с тобой случился психический приступ - друзья говорят "ты хотел в магазин и там танцевал на кассовой ленте", а ты не помнишь такого, и не можешь отождествится с тем, что происходило - можешь отождествится с последствиями, как некая пострадавшая, больная сторона, которая совсем не хотела бы таких приступов. А тело и мозг тем не менее куда-то ходили и плясали.
1712 591399
>>91396

>наличием слова "я", подразумевается именно факт наличия определенных ощущений отождествления в этом процессе


тобой подразумевается, мной не подразумевается
что такое "определенные ощущения отождествления"?

>поход в магазин не ощущается как некое самостоятельное событие без субъекта - в этом процессе "похода в магазин" я ощущаю отождествление с некой его частью - которую и называю я


это для меня слишком сложно
отождествление с некой кого частью? субъекта или похода в магазин?

>мы имеем ввиду по сути именно это


кто вы?

>тело и мозг тем не менее куда-то ходили и плясали


я такую ситуацию называю "я куда-то ходил и плясал, но не помню"
1713 591400
>>91396

>Представь еще что с тобой случился психический приступ - друзья говорят "ты хотел в магазин и там танцевал на кассовой ленте"


Хули представлять, вы как будто по поллитра водки не выжирали ни разу в одно лицо.
1714 591416
>>91396

> наличием слова "я", подразумевается именно факт наличия определенных ощущений отождествления в этом процессе - я не увидел их на киноэкране, мой организм не встал сам и не пошел в магазин, к моему удивлению и поход в магазин не ощущается как некое самостоятельное событие без субъекта - в этом процессе "похода в магазин" я ощущаю отождествление с некой его частью - которую и называю я.


В слепом мозге сознание-эпифеномен не имея доступа к эвристикам путает каузальные атрибуции. 99% действий ты делаешь на автоматизмах, ты именно что посмотрел кино.
1715 591420
Дебичи, нет никакого ни сознания, ни души, в которую эти старперы маразматичные, ударяются когда 7-й десяток разменяют.
В черепе простейшие весы, типо примитивной нейросети с заданной целью - в зависимости от генома либо купить яйцеклетки как можно дешевле, либо как можно дороже их продать.
nu-davay-taya-rasskazhi-kak-ty-men29078915big.jpeg61 Кб, 800x450
1716 591422
>>91420

>В черепе простейшие весы, типо примитивной нейросети с заданной целью - в зависимости от генома либо купить яйцеклетки как можно дешевле, либо как можно дороже их продать.


Продолжай
1717 591428
>>91422
Оглянись блядь вокруг.
99% популяции живут в культе пизды, остальные очкастые пердолики воображают, что вот когда они станут очередным перельманом, то их достижения точно какая-нибудь пизда ОЦЕНИТ ПО ДОСТОИНСТВУ - поломанная логика ущербных генетических автоматов, которые верят, что пизда вдается в такие тонкие материи и которые отбракуются из популяции, потому что не оставят потомства для вида, который один хуй исчезнет.
uh-ty41707899orig.jpeg20 Кб, 375x334
1718 591429
>>91428
Ещё!
Наддай! Режь правду-матку, Анон
1662035297277524075.png1 Мб, 1600x1443
1719 591449
>>91429
Ты и так уже должен был ослепнуть от сияния моей мудрости, и на жопной тяге улететь на Луну.
1720 591469
>>91399

>что такое "определенные ощущения отождествления"?


Ощущение что субъективный опыт делиться на две части "я" и "не я", но граница проходит по разному.

>это для меня слишком сложно


>отождествление с некой кого частью? субъекта или похода в магазин?


Я говорю только о субъективном опыте в данном случае, то есть субъективный опыт похода МЕНЯ в магазин включает в себя некое ощущение я.

>кто вы?


люди

>я такую ситуацию называю "я куда-то ходил и плясал, но не помню"


Ага, а тебе говорят "нет, мы тебя разыграли - ты просто спал", а ты такой "Ну нет же, я ходил и плясал"
1721 591470
>>91416

>В слепом мозге сознание-эпифеномен не имея доступа к эвристикам путает каузальные атрибуции. 99% действий ты делаешь на автоматизмах, ты именно что посмотрел кино.


>ты именно что посмотрел кино


>ты


Вот об этом ощущении и идет речь
А что там мозг путает - без этого ощущения это совершенно безразлично, потому что нет ни того, кто обладает свободой воли, ни того кто ее лишен
1722 591471
>>91416

>В слепом мозге


Скотт-Беккер пытается донести информацию класса "Вы - это не то, что чем привыкли себя считать", но при этом обращаться к тому самому типу субъективности, которого по его взглядам не существует и адресат его послания, таким образом, не может получить послание - потому что не существует и это будто бы слепое пятно самого Беккера. Он обращается к тому, кого нет, но сам факт получения послания в такой форме, как бы, опровергает его для читателя - потому его проект через фэнтези поспорить со своими идеологическими противниками - полностью провалился, но книжки интересные.
1723 591474
>>91420

> нет никакого ни сознания, ни души,


Согласен, и только мозга тоже нет, такая же галлюцинация
1724 591511
>>91469

>субъективный опыт делиться на две части "я" и "не я"


>некое ощущение я


не отвечает на вопрос, что такое "я"

>люди


не все же люди
я, как минимум, не подразумеваю наличием слова "я" факта наличия определенных ощущений, что бы это ни значило
я подразумеваю организм, который это сейчас пишет
"я" — это не ощущения, а объект реальности, который переживает ощущения
1725 591515
>>91511

>не отвечает на вопрос, что такое "я"


Ощущение - больше ничего, определяемое границей "я - не я", оно специфического - все знают как примерно оно ощущается, но мало кто замечает что границы движутся

>я, как минимум, не подразумеваю наличием слова "я" факта наличия определенных ощущений


Да, это и есть мое возражение - ты не подразумеваешь это на концептуальном уровне - как и большинство людей, кто-то думает что он душа, кто-то думает что он тело, кто-то что он мозг, кто-то думает что он сознание, что-то что он часть бога итд. итп., но по факту единственное на чем зиждется сама возможность говорить о каком-то "Я" - это именно ощущения. Если хотя бы ненадолго такое ощущение пропадает - то ты уже никогда не будешь так уверено утверждать, что ты какой-то там объект где-то спрятанный, или душа

Это ощущение я еще можно назвать ощущением присутствия.
Но парадокс в том, что внимательно наблюдая за этим ощущением - видишь, что оно так же имперсонально, как предметы обстановки
1726 591522
>>91515

>все знают как примерно оно ощущается


зря ты за всех говоришь, я не знаю, как оно ощущается
граница "я - не я" проходит по моей поверхности. Внутри — я, снаружи — не я.

>единственное на чем зиждется сама возможность говорить о каком-то "Я" - это именно ощущения


получается, что когда я сплю, меня нет, потому что нет ощущений
с этим я не могу согласиться
во сне я есть, хотя ощущений у меня нет, потому что сознание моё выключено
но я дышу, шевелюсь, потею, могу обоссаться или вообще пойти куда-нибудь, не подозревая об этом
1727 591527
А куда треды тупых вопросов делись? Ладно пох. Вот поставили бегуна на бесконечную дорогу. Сказали ему, когда пробежишь километр, то беги ещё два километра, а потом ещё три, а потом ещё четыре, и так далее до бесконечности. Сколько этот бегун пробежит километров в итоге? 1/12 в обратном направлении! АХАХАХАХАХАХА вы чё ебать с 9вите иташки упали он же даже не устанет
1728 591539
>>91515

> кто-то думает что он душа, кто-то думает что он тело


> Если хотя бы ненадолго такое ощущение пропадает - то ты уже никогда не будешь так уверено утверждать


Не понятно что ты хочешь понять. Есть концептуальное «я», это слово «я» изначально для внешнего общения, которое интерриоризировалось во внутренний концепт «я» для рефлексии и мыследеятельности о себе.
Есть поток квалиа, которое создаёт субьектность и ощущение существования (Декартовское когито). Нет квалий, нет ни чего - сон без сновидения, смерть.

> внимательно наблюдая за этим ощущением - видишь, что оно так же имперсонально, как предметы обстановки


Может у тебя диссоциация-деперсонализация?
1729 591548
>>91527
Закрепленный тред теперь заместо него.
1730 591550
>>91527
Есть /math и там тоже есть закреплённый тред. Там тебе более качественно ответят на этот вопрос.
1731 591553
Есть ли в открытом доступе карты радиоактивного загрязнения местности (РФ)? Нашёл только для четырёх областей.
1732 591556
>>91553
https://smolgazeta.ru/news/7030-smolenskie-lesa-xranyat-sledy-radiaktivnogo.html

>>Сейчас плотность загрязнения лесных почв в лесах Смоленской области находится в пределах от 0,012 до 0,75 Ku/км2.



Статья 2011 года, судя по ней измерения регулярно проводятся, но детальной карты по Смоленской области не нашёл.
1733 591559
>>91548
>>91550
понял, принял
1734 591578
Порекомендуйте почитать что-нибудь по палеонтологии - не научпоп, а что-то вводное - для студентов, например.
1735 591581
>>91578

>что-нибудь по палеонтологии


Это не наука, иди нафиг.
1736 591607
аноны, вопрос про вихревое электрическое поле. при изменении магнитного потока во времени, вокруг линий магнитной индукции возникает электрическое поле. то есть если обвязать руку медной проволокой и пустить по ней переменный ток определенной частоты, то руку будет бить током?
1737 591617
>>91607
Сопротивление кожи и ткани настолько высокое, что ты не заметишь. А повысив частоту поля/полей лишь разогреешь металл.
А вообще, я потому часы с браслетами не ношу - раньше работал возле соленоидов удержания потока флюидов, так они постоянно разогревали металл на теле, но током не били, хотя, потанцевал на них был. Насчет токов Фуко (ты же о них спросил?) не знаю, там же страдает только источник индукции, а если ты сможешь навести ими электроток, то все, опять, уйдет в тепло, никаких ударов током не будет 100%.
1738 591623
Работает ли двухщелевой эксперимент в масштабах вселенной? Вот допустим далекие объекты, излучающие мощный свет, который летит до нас и расщепляется из-за других космических тел, например галактик или еще чего - вот здесь как эта хуйня будет работать? То есть вот если есть коллапс волновой функции в свете, который летел до нас миллиарды лет, как эта хуйня вообще работает?
1739 591624
>>91623

>коллапс волновой функции


Чел, если теория Большого Взрыва верна, ты живешь во Вселенной, где вся материя спутанна, априори, и вопросы про ее редукцию не имеют смысла.
А насчет света - будет взаимодействие, будет и твой коллапс.
1740 591636
>>91623
Вполне. Дифракция от межгалактической пыли и газа очень хорошо наблюдается в гамма диапазоне.
Если вместо хаотичного говна был ровненький кристалл, то можно было бы наблюдать полноценную интерференцию.
Как рассчитать растояние 1741 591650
Всем привет, дано 3 фигуры

Красный квадрат - к
Синий тетраэдр - с
Белая сфера - б

Как рассчитать расстояние от С до Б по прямой, если нам известны расстояние от К до С и от К до Б
также нам известен угол от К до С и от К до Б

Прошу обратить внимание что пространство трехмерное.
1742 591653
>>91650
Если известны координаты цветных пиздюлин, то проще посчитать длину вектора.
1743 591657
Физиологи и нейрофизиологи, скажите, пожалуйста, чисто теоретически, может ли человек осознанно управлять процессами своего организма? Например, одним своим желанием (без всяких таблеток и диет) регулировать обменные процессы в органе N? Или в сосстоянии медитации/трансаускорять заживление ран?
Или это всё фантастика?
1744 591658
>>91650
Размерность пространства не важна. Через три отличные точки можно провести одну плоскость и на этой плоскости будет находится твой треугольник. Можно рассматривать только ее.
Принимаешь красную точку за начало координат, принимаешь одну из оставшихся точек (не важно какую, например синию) как лежащую на оси Х - ее координаты (длина соотвестующего отрезка, 0). Координаты оставщейсяя точки (косинус угла на длину отрезка, синус угла на длину отрезка). Зная координаты точек на плоскости в введенной тобой системе координат - не их реальные координаты в пространстве вычисляешь растояние между ними.
1745 591659
>>91650
Ты что, теорему косинусов забыл где-то в школе?
1746 591661
>>91658
Спасибо
1747 591668
>>84346 (OP)
Анон, посовету учебник по логике для великовозрастного дебила, до этого с ней не сталкивавшегося. Можно что-то такое, что б были еще пособия от этого же автора и можно было по ним продолжить заниматься. Начал читать Челпанова, но он какой-то уж очень идеолгизированно совковый и немного унылый в плане подачи, хз может я не прав. В целом можно даже что-то более комплексное. Но можно и для школьников.
Важный момент - нужно чтобы учебник был доступен к покупке в бумаге.
Спасибо!
1749 591673
>>91668
Челпанов это мем по типу фингебокса.

Если тобой движут гуманитарные, а не физмат интересы, можешь посмотреть двухтомник Войшвилло и Дегтярева. Вся суть содержится в первом томе.
И порешай задачки из брошюр ШМК, "Логика для всех. От пиратов до мудрецов" и "Логические задачи".
1750 591675
Какие проблемы могло бы решить человечество если отбросило любые границы, распри и работало бы сообща ради достижения общих целей? Я понимаю что это невозможная ситуация, просто интересно насколько велики ресурсы целой планеты
1751 591676
>>91657
Да, может
1752 591688
>>91675

>ради достижения общих целей


Это ошибка. Если подумать чуть глубже, то для "отбросило любые границы, распри и работало бы сообща ради достижения" - цель может быть только одна. Как только возникнут две и больше целей, то автоматически возникнут границы и распри.
1753 591700
>>84346 (OP)

Как связано квантовое орбитальное число фотона и его линейная скорость движения относительно наблюдателя?

Дано: фотон движется где угодно, например в воздухе. В какой-то момент он приобретает орбитальный момент вращения вокруг оси распространения (пикрилейтед 1).
Траектория фотона обозначена синеньким. Ось, вдоль которой нужно найти - чёрная.
Сам орбитальный момент = 10010 h (пик 2)

Чему равна линейная скорость? Как её правильно вычислять?
1754 591716
>>91700
Привет, товарищ анон!

Когда фотон имеет орбитальный момент импульса (10 010 h), его линейная скорость зависит от расстояния до оси вращения и скорости света.

Формула для угловой скорости фотона:

ω = L / h

А формула для линейной скорости фотона:

v = ω * r

Но учти, что скорость фотона не может превышать скорость света (c). Так что максимальная линейная скорость фотона:

v_max = c

Для точного расчета линейной скорости фотона нужно знать его расположение и скорость в данной среде, а также помнить ограничение скорости света.

Надеюсь, это поможет тебе понять связь между орбитальным моментом импульса фотона и его линейной скоростью. Удачи на форуме Двач!
1755 591717
>>91675
Эй, братан!

Представь, если бы все люди на планете забыли про границы, разборки и стали вместе бороться за общие цели. Вот чего бы мы могли добиться:

1. Бороться с глобальным потеплением: Все вместе вкладывать деньги в чистую энергию, экономить ресурсы и придумывать технологии, чтобы остановить изменение климата.

2. Убрать голод и нищету: Объединить землю, технологии и знания, чтобы выращивать больше еды и помочь бедным регионам развиваться.

3. Решить проблемы здравоохранения: Вместе финансировать исследования, делать лекарства и вакцины, и давать всем доступ к медицинской помощи.

4. Образование и инновации: Инвестировать в образование и науку, чтобы каждый мог раскрыть свой потенциал, и обмениваться знаниями и технологиями для новых открытий.

5. Решить проблему перенаселения: Планировать семью, обеспечивать образование и равные возможности для всех, чтобы население стабилизировалось и планета могла поддерживать нас.

Конечно, это идеал, и в реальности такое сотрудничество сложно. Но если бы мы действительно работали вместе, ресурсы нашей планеты могли бы решить глобальные проблемы и создать лучшее будущее для всех.
1756 591718
>>91657
Эй, братан!

Теоретически, человек может осознанно управлять некоторыми процессами своего организма. Вот несколько примеров:

1. Регулирование обменных процессов: Через практики, такие как медитация, дыхательные упражнения и йога, люди могут научиться влиять на свой обмен веществ. Например, некоторые исследования показывают, что практика йоги и медитации может помочь снизить уровень стресса, улучшить контроль над сахаром в крови и даже повлиять на уровень гормонов, связанных с обменом веществ.

2. Ускорение заживления ран: Некоторые исследования указывают на то, что практики, такие как медитация и визуализация, могут помочь ускорить процесс заживления ран. Например, одно исследование показало, что пациенты, которые использовали визуализацию и другие методы релаксации, заживали раны на 40% быстрее, чем другие пациенты.

3. Управление болевыми ощущениями: Медитация, глубокая релаксация и другие методы осознанности могут помочь людям справляться с болевыми ощущениями. Некоторые исследования показывают, что эти практики могут изменять активность мозга, связанную с восприятием боли, и помогать людям лучше справляться с хроническими болями.

Хотя эти примеры показывают, что осознанное управление некоторыми процессами организма возможно, это не означает, что вы можете полностью контролировать все аспекты своего организма без помощи медикаментов или диет. Тем не менее, практики осознанности и медитации могут быть полезными инструментами для улучшения здоровья и благополучия.
1757 591719
>>91716

Спасибо о.о
1758 591733
Вопрос-платина про квантовую запутанность.
Пусть у нас квантовый комп делает очень важное вычисление, которое продлится 6 лет. При этом отделяем от него запутанную с ним систему частиц, которая даст очень вероятный результат о результате вычислений, и увозим её на другую звезду в 5св годах от нас. Ровно (с точностью до 0.1сек) через 7 лет мы будем там, тут же вскрываем "конверт" и получаем результат. При этом с землянами уговор, что они попытаются узнать результат на 1 секунду позже нас или ещё скорее (главное, чтоб не первыми). Верно ли, что у них это не получится? Например, если астронавты получили ответ "да" с 99% достоверностью, то для землян будет "да/нет" почти на 50%? Или у землян вовсе абсолютный рандом будет?
1759 591734
>>91733
Вообще, вся система квантового запутывания выглядит так:
У тебя есть что-то из квантого говняка, что может быть или пикой дроченной, или елдой точенной.
И, неважно, в какой системе временного отсчета ты там что-то делаешь. Открываешь черный ящик - если у тебя пики, значит, у уехавших космонавтов - хуи. Вот так работает квантовая жопа с редукцией. И так работает квантовая связь.
1760 591735
>>91734
Кто раньше начал взаимодействие с этой системой, тот и получает или пики, или бубны.
И наша задача не выяснение результата, у кого будут пика, а у кого елдак.
Наша задача - выразить условия или повлиять на условия, приводящие к 100% результату в результате редукции. Или вообще найти следствия квантования квантовой протоматерии и начать из нее творить говняк.
1761 591747
>>91735

>Кто раньше начал взаимодействие с этой системой, тот и получает или пики, или бубны.


Это утверждение с нулевой предсказательной силой, люди вне секты не обязаны в него верить.
1762 591754
>>91747

>люди вне секты не обязаны в него верить


Это утверждение с нулевой предсказательной силой.
1763 591756
>>91733

> важное вычисление, которое продлится 6 лет. При этом отделяем от него запутанную с ним систему частиц, которая даст очень вероятный результат о результате вычислений, и увозим её на другую звезду в 5св годах от нас. Ровно (с точностью до 0.1сек) через 7 лет мы будем там, тут же вскрываем "конверт" и получаем результат. При этом с землянами уговор, что они попытаются узнать результат на 1 секунду позже нас или ещё скорее (главное, чтоб не первыми). Верно ли, что у них это не получится? Например, если астронавты получили ответ "да" с 99% достоверностью, то для землян будет "да/нет" почти на 50%? Или у землян вовсе абсолютный рандом будет?


>


Это лишь значит, что запутанная система будет по сложности как сам этот комп ебучий и просто сама по себе вести такие же расчеты
1764 591773
>>84346 (OP)
Если пересадить костный мозг от Джона к Васе, то ДНК тест на этническую принадлежность покажет какой результат?
1765 591774
>>91773
Патанатом даст заключение, что Вася помер от токсического шока и некроза тканей костного эпифиза, вызванных реакцией отторжения принадлежащей Джону костной ткани.
1766 591776
>>91773
Начальную национальность Васи, т.к. костный мозг вроде только кровяные красные клетки ебашит в которых нет ядра и ДНК. (не учитывая условие что материал будут брать из костного мозга)
Если ошибаюсь поправьте.
1767 591777
Зачем додики на верхушках государств не просто не поощрают, но и вовсе подавляют исследования искусственных маток в виду этнических причин? Это же ебанутейший демографический инструмент, разве нет? Типа вот условный Пыня постоянно визжит о проблемах в демографии, но продолжает заменять русских таджиками, а западные партнеры завозят сотни тысяч арабов, ну вот и почему этим дуралеям просто не запрячь пару сотен мочённых на проекты по разработке искусственных маток?
1768 591779
>>91777
Потому что вкладываться в выращивание и воспитание миллионов детей из пробирки дорого и сложно, проще чурок навезти и они как-нибудь тут сами переебутся
1769 591781
>>91777
Потому что додикам наверху ислам никак не мешает роскошно жить можно и в исламской стране если ты наверху а других интересов у них нет.
1770 591792
>>91779
Казалось бы - что может пойти не так?
1771 591793
>>91792
Для детей колонизаторов, которые являются гражданами стран золотого миллиарда, всё всегда идёт как надо, не переживай, абориген.
Детей Пескова и прочих патриотов гражданства по праву рождения на западе не лишили, а значит, когда их папашки завернутся, все средства производства перейдут к гражданам других стран.
Терпи, колонист на участке, недра под которым принадлежат гражданам с тройным гражданством.
1772 591796
Поясните кроме матрицы как объяснить эффект ожога когда к человеку прикасаются холодной кочергой а он ожог получил
1773 591798
>>91796
+++ поддерживаю реквест.
gidarus.jpg69 Кб, 720x403
1774 591799
>>84346 (OP)
Коммунизм в космосе, колесо горизонтальное.
Бэйн Таурен 2004 год
1775 591800
>>91796
Что за эффект?
1776 591802
>>91800
Прижигают животное на глазах человека горячим клеймом.
Потом человеку завязывают глаза и прикасаются холодным клеймом, а у него ожог как от горячего клейма.
Работает только если завязать глаза.
1777 591803
>>91796
Объяснение самое простое - пиздеж.
1778 591804
сап двач
заканчиваю магистратуру по шызике
в аспирантуру не хочу, но меня интригует соискание на кандидата наук
вопрос такой, допустим физика заебала и я хочу кандидатскую в другой сфере типа социологии хз
так можно?
если можно, то я могу после получения кандидата по социологии написать докторскую колбасу по физике снова и стать доктором по физике?
1779 591811
>>91804
Ты можешь стать доктором по физике, минуя кандидатскую.
Только тема должна быть прорывная и слушать тебя должен закрытый совет.
1780 591821
>>91777
Очевидно элиты не являются национальными и ради сохранения власти и капиталов легко готовы добиться вымирания коренных и заместить их чужаками.
1781 591822
Почему ученые рассказывают от всем понятный хуйне, а никто не заглядывает в суть?
Например я знаю, что все 13 млрд световых лет везде такая же скучная хуйня в виде безжизненного холодного пространства, срать мне на глизе 31851 и альдебаран, я вам и так скажу что там происходит.
А вот что внутри черных дыр - там ответы на всё во вселенной, в том числе, что за границей вселенной и есть ли эта граница. Там разгадка квантовой теории гравитации.
Все лезут в какие-то вычисления, а настоящие гении задвали самые простые вопросы - какого хуя свет ведет себя с двумя щелями так странно, квантовый ластик (результаты эксперимента засекретили ученых убили, т.к. принцип следствия-причины был послан нахуй, а значит как и все финансирование, нобелевки всех очкастых головастиков).
Хью Эверетта обоссали незаслуженно, хотя он единственный, кто всё объяснил без противоречей.
1782 591823
Что за "сверхимические реакции"? Как могут прореагировать более низкие электронные оболочки?
1783 591824
>>91823
Просто коггерированный конденсат Бозе-Эйнштейна реагирует как единое целое.
1784 591825
>>91823
В первый раз слышу об этом. Из контекста я догадываюсь ты имеешь в виду Quantum Superchemistry.

Тут чисто квантовое явление, которое родственен лазеру, только для электронов в потенциальных ямах
1785 591831
Есть ли теории, которые утверждают, что погодой можно глобально управлять тонкими точечными воздействиями? Например, что можно включить ветряки в точном месте и в нужный момент, и это перенаправит осадки в районы засухи, или светануть лазером и раздробить большой ураган-циклон на несколько мелких, навесить на гсо относительно небольшие плёнки, не пропускающие ик-излучение - и это опять же перенаправит осадки и ослабит блокирующий антициклон. Или не в одной точке так делать и не одним типом воздействия, а комплексно по всему земному шарику, но суть в том же, что огромные потоки энергии управляются небольшими.
Ну вот такое чёто.
1786 591833
>>91831
Нет нельзя без точного расчёта ии иначе наводнение как в Дубае
1787 591835
>>91831

>погодой можно глобально управлять тонкими точечными воздействиями?


Почему теории? Практика даже есть. Методов воздействия очень много. Суть явления в обратной связи механизма переноса заряда с атмосферы в поверхность (помимо теплового фактора есть еще и перенос тока через атмосферу). Влияешь на перенос - создаешь погоду. Пока что можно вызывать или засуху, или обильные осадки. Еще можно атмосферу очищать от от дымов, пыли, грязи и фог-частиц.
>>91831

>можно включить ветряки в точном месте и в нужный момент, и это перенаправит осадки в районы засухи, или светануть лазером и раздробить большой ураган-циклон на несколько мелких


Нет, это не совсем так работает. Но если подумать, можно исключить параметры зарождения непогоды, например, закоротить контур воздушного конденсатора, и тогда осадки в воздухе слипнутся и опадут вниз, а новые порции водорода, которым планета гадит сквозь поры, ионизировавшись они не смогут улететь высоко, и окислившись, создать новые облачные массы. Ураган просто не сможет без осадков и водяной массы создать фронт разных слоев воздушной массы и не завихрится в воронку. Но с водяной поверхностью водоемов и океана такое не прокатит, и надо будет, по старинке, работать по атмосфере над водной массой (как в Москве перед праздниками разгоняют тучки). Для этого нужна превентивная служба реагирования и доставки, этого нет, и не будет.
>>91831

>навесить на гсо относительно небольшие плёнки, не пропускающие ик-излучение


Это бред, человек.
1788 591836
>>91835

> Это бред, человек.


Ну наверно. Но на правах бреда вот в ближнем востоке или африке температура 50, а если навесить над ними ограничивающий солнечное излучение экран, температура до 35 понизится, а там глядишь осадки подъедут, реки высыхать перестанут. Или небольшой экран, который будет как-то мешать образовываться антициклонам и чтоб наоборот облаков больше было. хотя макаки всё равно лишь расплодятся и заговнят всё
1789 591838
>>91836
Ты мыслишь как разрушитель, а надо как творец.
Ближний Восток и Африку нужно снова засеять зеленой массой и деревьями. Тогда температура снизится, а в воздухе появится на весь суточный период значительное количество влаги.
Именно ты можешь прорастить и высадить десятки тысяч саженцев, которые за несколько десятилетий изменят климат. Варианты технократического плана не сработают абсолютно точно, проверено. Только традиционное терраформирование или неоформинг, когда ты берешь и создаешь новую биоту.
1790 591840
Можно ли с помощью квантового компьютера сделать демона Лапласа, который работает хотя бы только на прошлое? Я боюсь, что мои сегодняшние мысли смогут прочитать люди будущего или ИИ.
1791 591841
>>91840
p.s. Да, я читал возражения в виде неопределённости Гейзенберга и теории хаоса вкупе с невозможностью узнать точные параметры Вселенной. Однако разве гигантский квантовый компьютер не сможет все эти условно бесконечные варианты банально брутфорснуть, если он экспоненциально сложные задачи решает как линейные?
1792 591842
>>91841
>>91840
Есть проблема собрать эту инфу, и банальная проблема со скоростями. Ну т.е. вселенная просто быстрее работает, чем этот комп.

Ну и учитывая как инфа проебывается постоянно, то комп такой только вероятные будет получать события прошлого. Я точно так же могу и про вавилонскую библиотеку сказать, мол там где-то твои мысли лежат.
1793 591843
>>91842

>банальная проблема со скоростями


Не мешает демону восстановить прошлое.

>учитывая как инфа проебывается постоянно


В физике наоборот есть постулат, что информация никогда не проёбывается. В квантовом мире есть консервация информации, и даже чёрные дыры, как сейчас считается, возвращают инфу назад через излучение Хокинга.
Другое дело, что если в мире существует фундаментальная случайность, эта информация восстановлению в любом случае не подлежит.

Вообще я читал, что для имитации одной реальной частицы нужен один кубит. Получается, для имитации всей вселенной нужна вся вселенная, как-то так.
1794 591844
>>91843

>В физике наоборот есть постулат, что информация никогда не проёбывается.


Речь о том, что у любого квантового события, вероятности не только в будущее, но и в прошлое... смекаешь?

Т.е. если я тебе дам данные по туннелированию, то спросив "откуда частица прилетела? Она прошла или наоборот отразилась?"
В попытках восстановить прошлое просто "отмотав время назад в уравнениях" ты наткнешься ровно на ту же проблему, что и при попытке предсказывать будущее (т.к. уравнения то для расчета будут ровно те же). т.е. как у каждого начального состояния есть целая куча вариантов впереди, так и такая же куча различных прошлых событий, которые привели тебя туда, куда привели.
1795 591845
>>91844
Ок, поверю на слово. Не хочется, чтобы ИИшный Яхве через миллиард лет просматривал мою историю браузера.
1796 591846
Поможет ли от квантового взлома просто сделать пароль подлиннее? А токены он сможет взламывать? В смысле, авторизацию как в гитлабе, где паролем по сути является целый файл в несколько килобайт.
Мне что-то не верится, что если я задам в качестве пароля случайную строку размером с "Войну и мир", какой-либо квантовый компьютер её подберёт.
1797 591848
>>91846
Сам пароль никто не подберет и подбирать не будет. Проблема пароля в том что тебе нужно передать его на сайт (чтобы подтвердить что у тебя есть право доступа) и во время передачи его можно перехватить. Сейчас он шифруется. Квантовый взлом поможет сломать ассиметричные шифры на которых основано современная передача информации (RSA и на элептических кривых). У тебя просто рассшифруют твой пароль. Есть возможность убедиться что ты знаешь пероль без передачи самого пароля - но те алгоритмы также основаны на (современном) ассиметричном шифровании, тоесть слабы перед квантовым компьютером.
Алсо разрабатываются алгоритмы устойчевые перед квантовыми компьютерами (например CRYSTALS-KYBER, NTRUEncrypt). Но пока угроза чисто гипотетическая, никто с провереных алгоритмов уходить не спешит, ждут пока новые хорошо проверят, криптография это сложно.
1798 591850
Если взять за основу теорию большого взрыва, то в какой момент появилась жизнь? То есть все эти микроорганизмы, которые дохли, мутировали, эволюционировали и в итоге выродились в нас.

Вот была материя, сильно нагретая большим взрывом, все разлеталось, даже атомов не существовало и так далее - а где тут жизнь и в какой момент она появилась? Из чего? Сама материя переходит в жизнь, или как?
1799 591851
>>91850
Лет миллиардов пять назад из пылевого облака образовалось солнце. Крутящаяся тучка собиралась-собиралась и потом под своей же гравитацией вспыхнула. Ещё через миллиард лет из периферии той же пылевой тучи образовались планеты. Земля была безвидна и пуста очень горячей, но постепенно остывала. Через несколько сот миллионов лет покрылась корочкой, возникли водоёмы, циркуляция воды. Вся планета представляла собой кучу горячих лужиц, где бултыхалась вся таблица Менделеева. Так прошло ещё несколько миллионов лет, и в одной из лужиц собралась-таки самореплицирующася молекула, которая запустила эволюционный процесс и зохавала планету.
Примерно таков научный взгляд, полагаю.
1800 591852
>>91850

>Сама материя переходит в жизнь, или как?


Гугли абиогенез. На примере его возможность всё ещё не продемонстрирована, но альтернативы-то никакой, кроме "и вдохнул Господь душу живую".
1801 591853
>>91850
Жизнь это не что-то качественно отличающееся от не-жизни. Нет чёткой границы между живым и неживым.
Это просто последствие умения самореплицироваться с ошибками.
1802 591854
>>91853

>не что-то качественно отличающееся


И в какое царство живых организмов входит твой коврик для мыши?
1803 591855
>>91850
Гугли эксперимент Миллера-Юри. Вот этот анон >>91851 примерно прав, тольке если по простому, в лужицу еще током ебашило из-за атмосферных явлений.
1804 591856
>>91853

>Нет чёткой границы между живым и неживым.


Репликация чёткая грань. Только не надо про вирусы, тебя выкинь из биосферы в космос, ты тоже реплицироваться не сможешь.
1805 591859
>>91856
Кристаллы реплицируются.
1719106556113.jpeg241 Кб, 1008x633
1806 591860
Объясните, пожалуйста, тупому - почему галактические нити похожи на нейроны мозга? Вселенная - это чей-то мозг, а мы тогда кто такие?
1807 591863
>>91860
А ещё если ударить палкой по льду, такое же получится. Ох, не случайно, не случайно это всё!
1808 591865
С какой скоростью человек может упасть на ноги без серьёзных повреждений? 10 м/с? 5 м/с?
1809 591866
Я верно понимаю, что мы буквально скатываемся по полотну искривленного Землей пространства-времени, как по горке? И чтобы это представить, нужно мыслить четырехмерно?
image.png378 Кб, 1031x558
1810 591867
>>91866
Когда летевшая мимо солнца комета начинает вращаться вокруг него, она делает это не из-за воображаемой силы тяготения, а просто продолжая прямолинейное равномерное движение в искривлённом солнечной массой пространстве. Как-то так.
1811 591868
>>91867
Ну.... там скорее искривление времени всё ломает для таких объектов, пространство слабо скручено
1812 591870
>>91866

> Я верно понимаю, что мы буквально скатываемся по полотну искривленного Землей пространства-времени, как по горке?


Нет. Мы и всё имеющее массу находится в гравитационном взаимодействии. Но квантовая теория гравитации ещё не разработана. А когда будет разработана, то для того чтобы рассчитать по ней траекторию полёта камня или движение космического тела понадобится суперкомпьютер немыслимых мощностей (вроде как теоретически не возможного). Поэтому на земле пользуются теорией (моделью) тяготения Ньютона, типа тела тянут силы-веревочки. А для космических масштабов пользуются теорией (моделью) относительности Эйнштейна, типа это пространство-время искривляется. Ни куда мы не скатываемся и ни какие веревочки к земле нас не тянут, это тупо найденные закономерности и их описательные модели, которые работают - имеют предсказательную силу, которыми мы пользуемся чтобы летать самолетами, на Луну или пулять во врага взрывчатку.
1813 591874
>>91865
Скорость снижения с парашютом 5 м/с, быстрее уже больно коленям и копчику..
1814 591875
>>91870
Школьный учитель физики как-то сказал, если я верно помню его слова - два человека в пустом (ну представим) космосе будут притягиваться друг к другу, как далеко они бы не находились

Гравитационное взаимодействие таких мелких тел работает друг с другом даже на огромном расстоянии?
1815 591876
>>91875

>Гравитационное взаимодействие таких мелких тел работает друг с другом даже на огромном расстоянии?


Да, но при этом с увеличением расстояния оно убывает с такой скоростью, что если человек в космосе кинет камень, то никогда уже к этому камню не притянется. Казалось бы парадокс, но бесконечный ряд положительных значений может давать в сумме конечный результат.
1816 591877
>>91876
Если бы не тёмная энергия, то притянулся бы.
Идеи, идеи, идеи... Я буду долго гнать велосипед... 1817 591879
1)ГДЕ бы происходили временные путешествия? Какая реальность вариативная настолько реальна? Не поймите меня неправильно, но есть вся физическая плоскость где происходил бы нормальный мир, и тут РРАЗ и еще - откуда? "Ты опять людей в кастрюлю запихнул"?

2) Единственная идея портала в том что на входе и выходе стоит пластина с проникающим свойством кодов одних и тех же, согласитесь?

3) Сколько господи вариантов розового на диван можно красиво впихнуть? Восемь? Так вот чем бы красиво отличались более чем 600 планет со всемя технологиями? В смысле чтобы везде был рай, в итоге все будет повторяться нахрен, с чего тогда, откуда много было бы планет?

4) Галюны в реальном мире? От того что коров баранов нет на блюде до того что чото блин маленькая палочка между балками склада слишком слабое звено звено так и так?
1818 591884
>>91866
Не, не правильно.
В ОТОшной гравитации нет никаких горок и полотна. Пространство-время всегда для нас(локально) плоское. Чтоб почувствовать гравитацию нужно следить за третьим пробным телом. И это третье тело шатает аналогию с горкой и полотном.
Чтоб представить искривление пространства-времени гравитацией Земли, то нужно выходить в размерности значительно больше 4.
1819 591899
оч сложно
1820 591903
сап двач, очень срочно
пишу магистерскую
целую главу можно как цитирование сделать?
оно у меня к моей теме относится не прямо, да и рерайтить впадлу 3 страницы
Теория линейного времени 1821 591904
Сап, научач. На обзор представляется шизотеория, придуманая моим пьяным телом.

Теория линейного времени (теория линейной вселенной). Время имеет форму линии. То бишь, невозможно перемещение в прошлое, т.к. при изменении в линейной вселенной не сможет создаться другая ветка развития событий, из-за этого перемещение в прошлое будет заканчиваться тем, что путешественикам будет очень плохо - они... пропадут. Путешествие в будущее будет с таким же исходом, так как снова невозможность создать новую линию развития событий. Короче, вселенная и время существует здесь и сейчас, перемещение во времени будет с печальными последствиями. Вот так как то. Возможно, такая теория уже есть и сформулировал ее какой-нибудь доктор наук в 20 веке, но все таки.
1822 591906
>>91904
Времени нет.
Пространство не линейно.
ТЫ - АЛКОГОЛИК.
mYkTkiZ.png81 Кб, 488x347
1823 591921
Бывают ли аномалии, когда порядок пальцев инвертирован, т.е. большой палец с другой стороны?
1824 591922
>>91921
Вроде бы нет. Конечности прописаны в главной последовательности пространственного кода, и любое сомнение в хиральности просто провернет инверсию. Большие пальцы будут всегда внутрь смотреть, по отношению к голове, а вот внутренние органы при хиральности могут отзеркалиться.
cito! please! 1825 591928
Гидроксид натрия — это та же самае щелочь, что и в средствах для прочистки канализационных труб (состав: "щелочь" 15-30%), или это другой вид щелочи? Я не учился в школе.
1826 591930
>>91928
Да.
1827 591932
>>91904

>Путешествие в будущее будет с таким же исходом


ты сейчас путешествуешь в будущее
с единичной скоростью
1828 591944
Почему за пределами арифметики математика это наркомания
То есть это не догадаться, это выучить надо

Деление на 0.5 умножает целое число, потому что 0.5 — это то же самое, что и \( \frac{1}{2} \). Деление на \( \frac{1}{2} \) эквивалентно умножению на 2. Например:

\[ \frac{10}{0.5} = 10 \div \frac{1}{2} = 10 \times 2 = 20 \]

То есть, когда вы делите на 0.5, вы фактически умножаете на 2.

С другой стороны, умножение на 0.5 эквивалентно делению на 2, потому что:

\[ 10 \times 0.5 = 10 \times \frac{1}{2} = \frac{10}{2} = 5 \]

Таким образом, умножение на 0.5 уменьшает число в два раза, что эквивалентно делению на 2.
1829 591945
>>91944
Это любой школьник знает.
1830 591949
>>91944

>То есть это не догадаться


Это не для тебя написано значит, тебе нужна альтернативная математика получается.
1831 591962
>>91944
А в чем проблема-то?
Или тебя в детстве яблоками приложили и с тех пор контузия? Ну так нехуй яблоками математику считать.
1832 591963
На сколько мы вообще владеем и управляем собой?

Видел информацию, не знаю за ее достоверность - якобы мозг принимает определенное решение до того, как наше сознание будет в курсе. Но ладно, я не только об этом.

Вот возьмем например простое - цвета. Будучи ребенком ты видишь цвета, и понимаешь, какой тебе нравится. Именно понимаешь, а не выбираешь. Нельзя взять и разлюбить цвет, полюбить другой, потом еще один и все это за пару мгновений. Это же касается и понравившихся людей, ориентации, еды и всего прочего. Да любого выбора в принципе, который к тому же может зависеть от количества гормонов, биохимии мозга в тот или иной момент и т.д. Надеюсь смог объяснить суть того, что имею ввиду... Ткните меня носом в то, где я проебался
1833 591964
>>91963

> На сколько мы вообще владеем и управляем собой?


Ты мне не поверишь, но в обычном состоянии ты владеешь собой на уровне 1% от всего слоя клеточной массы.
Если представить их за 100%, то из них ты энергетически можешь подключать только 30-80% в единицу времени.
Нынешние люди постоянно уменьшают планку последних процентов, увы.
>>91963

>Видел информацию, не знаю за ее достоверность - якобы мозг принимает определенное решение до того, как наше сознание будет в курсе. Но ладно, я не только об этом.


Дело в том, что ты не хозяин клеток, а наделенный правами указания выбора пути для клеток феномен сознания. Сознание является фильтром для разума, чтобы уменьшить реактивность разума, который способен сжечь в буквальном смысле всё вокруг себя своей же энергией. Чтобы снизить реактивность - применяется сознание. Это совокупность шаблонированного состояния с наложенными матрицами генетического безусловного и условных типовых решений, которой дана возможность модерации над телом и превалирования над подсознанием. Дело в том, что подсознание также опирается на условный и безусловный базис, но работает в несколько раз быстрее, иногда даже в несколько тысяч раз быстрее, упираясь лишь в скорость проводимости нервых пучков по всем осям. Именно такие ситуации и описывают всякие ученые, типа Бехтеревой и и подобными. На достаточно простой шаблонированной схеме нейронная модель выполняет операцию за минимальное количество единиц времени, а большую часть времени нейронная модель выстраивает форму нейронной матрицы. То есть, делегированное правом конфигурирование сознание задает стимулы на постоянную переделку нейронных моделей шаблонов в голове, и мозг занимается этим 99, 999% времени, а само решение и его тактование происходит за краткие единицы времени. В этом и есть отличие мозга от компьютера - один строит нейронную модель и потом просто включает ток, а второй умножает уровни напряжений, и хранит их, и снова умножает и хранит. Первый тратит энергию только на реконфигурацию клеток, второй - всегда на сопротивлении переходов и проводников и утечку этого тока в окружающие слои.
Визуально, из-за сверхбыстрой работы готовый матричной формы, исследуя приборами такие состояния, визуально кажется, что у человека нет воли или волевой фактор вторичен. Но это не так. Стоит изменить условия задачи, которая потребует значительной смены топологии нейронных структур, начинается веселье и апломб ученых начинает прятаться.
>>91963

>Вот возьмем например простое - цвета. Будучи ребенком ты видишь цвета, и понимаешь, какой тебе нравится. Именно понимаешь, а не выбираешь. Нельзя взять и разлюбить цвет, полюбить другой, потом еще один и все это за пару мгновений. Это же касается и понравившихся людей, ориентации, еды и всего прочего. Да любого выбора в принципе, который к тому же может зависеть от количества гормонов, биохимии мозга в тот или иной момент и т.д. Надеюсь смог объяснить суть того, что имею ввиду... Ткните меня носом в то, где я проебался


Это бред.
1833 591964
>>91963

> На сколько мы вообще владеем и управляем собой?


Ты мне не поверишь, но в обычном состоянии ты владеешь собой на уровне 1% от всего слоя клеточной массы.
Если представить их за 100%, то из них ты энергетически можешь подключать только 30-80% в единицу времени.
Нынешние люди постоянно уменьшают планку последних процентов, увы.
>>91963

>Видел информацию, не знаю за ее достоверность - якобы мозг принимает определенное решение до того, как наше сознание будет в курсе. Но ладно, я не только об этом.


Дело в том, что ты не хозяин клеток, а наделенный правами указания выбора пути для клеток феномен сознания. Сознание является фильтром для разума, чтобы уменьшить реактивность разума, который способен сжечь в буквальном смысле всё вокруг себя своей же энергией. Чтобы снизить реактивность - применяется сознание. Это совокупность шаблонированного состояния с наложенными матрицами генетического безусловного и условных типовых решений, которой дана возможность модерации над телом и превалирования над подсознанием. Дело в том, что подсознание также опирается на условный и безусловный базис, но работает в несколько раз быстрее, иногда даже в несколько тысяч раз быстрее, упираясь лишь в скорость проводимости нервых пучков по всем осям. Именно такие ситуации и описывают всякие ученые, типа Бехтеревой и и подобными. На достаточно простой шаблонированной схеме нейронная модель выполняет операцию за минимальное количество единиц времени, а большую часть времени нейронная модель выстраивает форму нейронной матрицы. То есть, делегированное правом конфигурирование сознание задает стимулы на постоянную переделку нейронных моделей шаблонов в голове, и мозг занимается этим 99, 999% времени, а само решение и его тактование происходит за краткие единицы времени. В этом и есть отличие мозга от компьютера - один строит нейронную модель и потом просто включает ток, а второй умножает уровни напряжений, и хранит их, и снова умножает и хранит. Первый тратит энергию только на реконфигурацию клеток, второй - всегда на сопротивлении переходов и проводников и утечку этого тока в окружающие слои.
Визуально, из-за сверхбыстрой работы готовый матричной формы, исследуя приборами такие состояния, визуально кажется, что у человека нет воли или волевой фактор вторичен. Но это не так. Стоит изменить условия задачи, которая потребует значительной смены топологии нейронных структур, начинается веселье и апломб ученых начинает прятаться.
>>91963

>Вот возьмем например простое - цвета. Будучи ребенком ты видишь цвета, и понимаешь, какой тебе нравится. Именно понимаешь, а не выбираешь. Нельзя взять и разлюбить цвет, полюбить другой, потом еще один и все это за пару мгновений. Это же касается и понравившихся людей, ориентации, еды и всего прочего. Да любого выбора в принципе, который к тому же может зависеть от количества гормонов, биохимии мозга в тот или иной момент и т.д. Надеюсь смог объяснить суть того, что имею ввиду... Ткните меня носом в то, где я проебался


Это бред.
1834 591965
>>91964

>>>91963


>>Вот возьмем например простое - цвета. Будучи ребенком ты видишь цвета, и понимаешь, какой тебе нравится. Именно понимаешь, а не выбираешь. Нельзя взять и разлюбить цвет, полюбить другой, потом еще один и все это за пару мгновений. Это же касается и понравившихся людей, ориентации, еды и всего прочего. Да любого выбора в принципе, который к тому же может зависеть от количества гормонов, биохимии мозга в тот или иной момент и т.д. Надеюсь смог объяснить суть того, что имею ввиду... Ткните меня носом в то, где я проебался


>Это бред.



Объясню, почему бред. Твои шаблоны, которые постоянно формируются в мозгу, на базисе инстинктов (биологических подпрограмм) постоянно формируют одни и те же зависимости. И добавляя в эти инстинкты триггеры (лицо и фигура мамы, образ отца, сильное гормональное удовольствие от колористической окраски в момент выброса гормонов и т. п.) ты вмешиваешь эти триггеры в нейрошаблон, и на его основе ты создаешь себе характер, вкусы и предпочтения. И эта нейромодель вмешивается в основной типаж индивида - темперамент. И вот сила воли и сама воля тут в том, что ты можешь это изменить волевым посылом, волевым импульсом. А сама воля при этом может быть алогичной, тут вот надо вмешать лимбическую систему и ее систему филогенетических триггеров. Но я в этом не разбираюсь вообще. Я хз даже, зачем я пишу тут. Пиздос.
1835 591967
>>91963

>На сколько мы вообще владеем и управляем собой?


Кто управляет? Гномик душа? Так с этим в религач. А по науке сознание это общее пространство коммуникации кучи систем для планирования и согласования поведения (и ещё многих функций). Ощущение цельного Я, управляющего поведением - иллюзия, ошибка атрибуции. Там у тебя целый зверинец в театр играют, но ты этого не можешь видеть, а лишь Якаешь внутренним диалогом за любого находящегося на сцене. Организм же управляет собой абсолютно и полностью.
1836 591968
>>91967
Ну, значит, и математики не существует. Это же тоже абстракция.
Сознание - это субъективный процесс работы психики.
Психика - процесс организации жизнедеятельности организма и увязка его с волновой структурой разумного начала.
Сознание - это энергия.
Психика - это вентиль этой энергии.
Воля - это желание структуры, управляющей вентилем, преобразовывать материю вокруг себя через посыл одного энергетического момента, и реорганизации его в другой, более приземленный и материальный. Не мозг руководит волей. Воля создает в мозгу процесс, который создает предпосылки для построения такого нейронного фрактала, который начнем подавать команды мышцам руки. И уже потом, рука под контролем начинает искать жопу, где сработал триггер на баребуховое покалывание, и рука начинает чесать баребухи на жопе. Но изначально был волевой посыл, который и был преобразован мозгом в действие над материей. Наша задача, как моченых, разделить и отделить нематериальное и материальное и подвести доказательную базу под эти проявления жизни.
А ты всё низводишь до каких-то детских тезисов, типа Аааааааа, вериш в дущю, иди мались пиньку, ета ни навука...
И пытаешься объяснить не с Декартового ложа, а с позиции психиатра. Ты же мудак, чел...
1837 591969
>>91968

> Наша задача, как моченых


Твоя задача таблетки пить вовремя, шизик
1838 591970
>>91969
Я пью.
Вовремя.
А тебе пора в мясной куб, скоро 2026.
Готовься.
1839 591971
>>84346 (OP)

Это правда что из теории эволюции следует, что мы не можем знать верность теории эволюции? Типа мозг отбирался не для определения верности научных теорий, а для размножения? Таким образом эволюционное учение это, как его называл Карл Поппер, метафизический исследовательский проект, но не научная теория?
1840 591972
>>91971

>Это правда что из теории эволюции следует, что мы не можем знать верность теории эволюции?


Нет никакой теории эволюции. Есть шаблон, согласно которому создается матричный каркас живого существа, который и пытаются выставить за основной постулат идеи эволюции.

>Типа мозг отбирался не для определения верности научных теорий, а для размножения?


Мозг никто не отбирал. Просто нужно существо, которое может преобразовывать уровень энтропии и материю вокруг. Человек занял эту нишу по праву создания.

>Таким образом эволюционное учение это, как его называл Карл Поппер, метафизический исследовательский проект, но не научная теория?


Если теории нет, то и следствий ее нет, увы.
В рамках понятия "эволюция" стоит рассматривать только термин "инволюция". Это частный случай энтропийного неблагоприятного возмущения, приводящего к самоуничтожению вида.
Вот как уменьшение веса мозга у европейских мужчин - именно следствие направленного негативного влияния. Вот такое можно подводить под теорию, потому что это объективизм, выраженный через анализ субъективного (мозгов европейцев). А эволюция - нет, потому что это сам, непосредственный фактор сознания и разума, то есть субъективное, а не объективное. Каждый субъект выбирает, стагнировать ему в инволюционное, или начать процесс своего развития изнутри.
1841 591973
>>91971

> Это правда что из теории эволюции следует, что мы не можем знать верность теории эволюции?


Верность теории рассматривается из вышестоящей инстанции, т.е. из методологии науки, а не применяется сама к себе.

> Типа мозг отбирался не для определения верности научных теорий, а для размножения?


Мозг отбирался по способности приспособления к окружающей среде, и этот же мозг породил научный метод, чтобы ещё лучше приспосабливаться к окружающей среде. Мозг это биологический эвристический инструмент, научный метод расширяет-улучшает естественные эвристические способности мозга.

> Таким образом эволюционное учение это, как его называл Карл Поппер, метафизический исследовательский проект, но не научная теория?


Эволюционная теория соответствует научному методу, значит это научная теория.
1842 591974
>>91973

>методологии наук


На основании научного метода подведи идею об эволюции к себе, как субъекту, и вырази обоснованно и с образцами доказательной базы, как ты эволюционировал по отношению к маме-папе и на грейд больше - по отношению к бабушке и дедушке, ну и совокупно, возьми за образец человекообразную обезьяну, для наглядного сравнения.
Ты же научно обоснован.
1843 591975
>>91974

> как ты эволюционировал


У тебя шиза? Организм не является объектом эволюции, организм не эволюционирует в процессе жизни.
1844 591976
>>91975
Лол. Я словно в разделе с любителями трансцендента сижу.
Что еще напишешь умного, ученый?
1845 591977
>>91975
Многоклеточный организм представляет собой популяцию клеток. И бактерий. Которые рождаются и умирают.
Ну и это ваше сознание.
1846 591980
>>91977

>Многоклеточный организм представляет собой популяцию клеток.


Не тупи. Ген у всех клеток организма один.
171946385042053797.jpeg77 Кб, 748x667
1847 591981
Bauhaus-geometry-Bauhaus-Movement-Magazine-729x1024.webp63 Кб, 729x1024
1848 591983
>>84346 (OP)
Анон, почему параллельно существованию закона сохранения материи(массы), и энергии в том числе, ученые рассуждали и рассуждают о бесконечности вселенной? У нас есть фундаментальный ограничивающий фактор, при котором бесконечную экспансию и расширение невозможно иметь.
AntManSuit3.webp27 Кб, 240x500
1849 591984
Есть челопук-муравей. Отбросим откровенно фантастическую часть и оставим только способность уменьшиться с сохранением массы.
Например он 80 кг уменьшается до 1 см роста.
Должен ли он в этом случае превратиться в какой-то сверхтяжелый элемент и взорваться как ядерная бомба?
1850 591985
>>91983
>>91983

>почему параллельно существованию закона сохранения материи(массы), и энергии в том числе


Этот закон выражается скорее в сохранении потоков массы и энергии, когда плотность одной и второй меняется. А раз речь о плотностях, то объем такое дело...
1851 591986
>>91980
Ген один пока клетка одна, по мере роста начинаются мутации.
1852 591987
>>91971
не правда
из того, что качество мозга когда-то влияло на количество потомства не следует, что мозг непригоден для формирования верных умозаключений

никто не говорит, что научные теории верны потому, что мозг крутой
научные теории верны потому, что верифицируемы
в отличие от религиозных теорий
1853 591988
>>91984
Чтоб пошли ядра начали друг друга чувстовать ядерными силами, твоего человека муравья надо сжать до нескольких микрон.
Но это все хуйня, при сжатие примерно в десяток раз разрушается электронное структура атомов и оно переходит в вырожденное состояние. Короче превратиться в один кусок металла.
1854 591989
>>91968

>Психика - процесс организации жизнедеятельности организма


любого организма?
1855 591990
>>91986

> по мере роста начинаются мутации.


Не мутации, а ошибки, которые ни куда дальше не воспроизводятся, а исчезают вместе со смертью организма, они не участвуют в эволюционном процессе.
1856 591991
>>91967

>по науке сознание это


По науке теории дуалистов подкрепляются растущей кучей свидетельств существовании сознания вне тела а у материалистов по-прежнему только пустая вера и попытки убедить всех что их религия часть науки.
1857 591992
>>91990
Решать кто мутация а кто ошибка будет естественный отбор, как минимум в течении жизни организма его клетки рождаются и подвергаются действию естественного отбора а значит вполне себе эволюционируют. Ну и помним что клетки могут мигрировать и влиять друг на друга, а значит и на используемый при размножении материал влияние неизбежно окажут.
1858 591995
>>91985
Ладно более тупенько спрошу. У нас определенное количество вещества и оно не может само по себе увеличиваться, возникать из ничего. Это просто противоречит физики этих самых ученых. Почему эти же ученные зная, что вещество ограничено, пытаются рассуждать о невозможности определения границ объёма этого вещества? Сложность измерения - да, но границы то есть.
1859 591996
>>91995
Ничего не понял, что ты имеешь ввиду под "границами"?И с чего ты решил, что у нас "определенное количество вещества?
1860 592000
>>91984
В принципе, атом как единица материального - он довольно пустой. Вполне можно в межатомное пространство добавить кучу соседних электронов и протонов (спирт с водой как самый простой пример, или вода с сахаром). И сила сжатия будет конечной просто потому, что протончики и электрончики будут уже внутри атомных радиусов отталкиваться друг от друга. Потому нельзя сжать кусок железа в нейтронную проволоку.
Но! Можно применить гидрирование или изменить форму электронного облака. В первом случае кусок металла можно сжимать уж очень хорошо в алмазной наковаленке. Во втором случае - безумно, но плазма все испортит и испарит материал.
Но при этом, атомы не пропишутся как трансэлементы, их взаимодействие на молекулярном уровне, а не атомном. И потому, этот вымышленный человек муравей не сможет изменить свои атомы в другие, более плотные по начинке, и не сможет бахнуть, потому что изначально он не содержал в себе нестабильные радиоактивные материалы. Вот если бы он состоял из нестабильной радиоактивной массы - то при уменьшении мог бы получить имплозию и получить неуправляемый распад. Но этот супергерой слишком медленно уменьшается, максимум, он перегрелся бы и лопнул от взрыва водородной или даже просто паровой смеси, ведь человек это 70% воды. В общем, чернобыльнулся бы и раскидал фонящие куски изначально заложенной в него радиоактивной материи. Сам он ее продуцировать не может своим уменьшением, он же не создает связи.
1861 592001
>>91996

>под "границами"?


То пространство, за котором этого вещества не будет.

>"определенное количество вещества?


Да, хороший вопрос. Удовлетворен, спасибо.
1862 592002
>>92001
Что ты имеешь ввиду под "не будет"?
1863 592003
>>92002
Там будет некий вакуум.
c2245b32dfb029397b358546b1fe4119.jpg251 Кб, 1050x650
1864 592004
Почему людям нравятся птицы?
Почему людям нравятся голоса птиц?
Многие даже любят крики чаек, уток, или считают всяких гусей и ворон красивыми и забавными.
Полно ненавистников собак, кошек, грызунов и т.д, но ни разу не видел такого, кто бы УХ БЛЯ ГОЛУБИ ЕБАННЫЕ УБИВАЛ БЫ НАХУЙ ЗАЕБАЛИ КУРЛЫКАТЬ
1865 592007
>>92004
Любовь к животным - это единение с природой. Ненавистникам природы не нравятся животные вообще. Им нравится урбанистика и жизнь города. Это один из типов ментала вырожденца. Отсюда и ненависть к кошкам\собакам\птицам\даже рыбкам. Города не приспособлены для жизни кого-то, кроме людей, отсюда и конфликты с проблемой отходов этих животных, самих безхозных животных. Птицы на самом деле хуже млекопитающих домашних зверей, они даже себе подобных жрут в условиях достатка еды. Просто в городе птицы более приспособлены к добыче еды в помойках. И они меньше внимания привлекают.
А почему пение нравится - потому что ритмичность и природный фактор. Некоторые птицы умеют в гармонию музыкального ряда. Тут надо гуглить сонорная последовательность птиц\целотонная гамма птиц и так далее. Я в этом не разбираюсь.
1866 592010
>>92004
Ненавистников голубей полно.
1867 592011
Почему мужчины биомусор?
Если мужчина прожил до 90 лет значит следил за здоровьем, а женщина и так доживёт
1868 592018
>>91991

> растущей кучей свидетельств существовании сознания вне тела


Где
1869 592019
>>92018
вне тела
1870 592022
>>92019
Свидетельства где
1871 592023
>>92022
1. Если бы сознание существовало в мозге, то имелись бы свидетельства его наличия в мозге.
2. Свидетельства наличия сознания в мозге отсутствуют.
3. Это свидетельствует об отсутствии сознания в мозге.
1872 592024
>>92022
>>92023

Господа, вам не кажется, что ваша дискуссия более подходит для доски /ph/?
image.png118 Кб, 1024x768
1873 592025
Распределение хи-квадрат с 1 степнью свободы в нуле не сингулярно же? Вроде не должно быть, по здравой логике, в занменателе гамма функция, а она в ноль не обращается. Я просто хочу уточнить, меня картинка графика смущает
1874 592026
>>92024
Время /ph/ закончилось когда дуалисты решили перестать болтать и организовали исследования реинкарнации, NDE, выхода из тела при NDE, SDE и, внезапно, медиумов. Теперь это или /sci/, или /re/.
Если Анон притворяется что не слышал вообще ни о каких исследованиях, то скорее /re/.
1875 592027
>>92023
Речь про создание сознания мозгом а не его присутствие в мозгу, то что оно там периодически присутствует достаточно очевидно.
1876 592028
>>92027
Есть ли мАкрокод в процессоре? Или есть Программист? Мы никогда не узнаем это, братья транзисторы и сестры диоды...
22221111.png27 Кб, 1173x769
1877 592031
дом в старом фонде, земли нет, но есть возможность купить два узо и поделить электросеть пополам. землю с первой половины кинуть на нейтраль второй, а землю со второй кинуть на нейтраль первой. тогда мы полчается делаем самодельный PEN-датчик из двух УЗО. в случае выхода фазы на корпус - она течёт на соседнюю нейтраль чем автоматом вырубает пробой на корпус. в случае же выгорания нуля - просиходит КЗ и систему вырубает пробка
1878 592032
>>92031
Нафиг тебе УЗОшка, если у тебя земля хер знает где и нуля может с ней имеет хуй знает какой потенциал.
1879 592035
Наивный немного вопрос, наверное, но всё же: если чёрные дыри излучают что-то и теряют свою массу, то не должны ли они вследствие этого перестать быть чёрными дырами, ведь именно масса делает их чёрными дырами?
1880 592036
>>92035
Масса-то делает, но не разделывает. Снаружи в ней нихуя не происходит из-за ебической гравитации, она перестает быть собой только в момент абсолютного испарения.
1881 592037
>>92036
Как удивителен этот мир. Спасибо!
1882 592038
Вот у квантпьютера кубиты, а у обычной пекарни типа биты. А что за биты-то? Это которые про 64битные процессоры или что? У них вообще-то число транзисторов транзисторы главное мерило. Или транзисторы тоже выступают битами, работая в ключевом режиме?
1883 592039
>>92035
Нет, чердырами их делает сжатие массы до критической плотности, чтоб она втиснулась внутрь радиуса шварцнегера. Этот радиус прямо пропорционален массе, поэтому плотность меняется пропорционально кубу этих величин. Т.е. мелкие чд гораздо, чудовищно плотнее больших.
1884 592045
>>92031
У тебя на корпусе вторых приборов окажется потенциал первых, ограниченных только сопротивлением их нагрузки. Короче ты пиздец долбаеб и не понимаешь что ноль это та же фаза, которая прошла через нагрузку всего-лишь и безопасной может быть только если там было ебовое сопротивление.
Быстрый интернет 1885 592047
Недавно слышал про то что в оптоволокне в России стали использовать полые трубки и интернет стал быстрее, на сколько это правда и когда это дойдет до обычный обывателей
1886 592051
>>92047
Стандарт на оптику един. Никаких новшеств не появилось.
1887 592055
Жаль слышать
1888 592100
>>89387

>Видел фотки, ракета две трети в длинну или 4/5 от миг31.


А как ты из просто длинны ракеты собераешся что-то получать?

Ну и были и есть ракеты сравнимы и меньших размеров которые сравнимых и больших скоростей достигали.
Например были противоспутниковые ракеты со стартом с истребителя. (Противоспутниковой ракете конечно не нужна орбитальная скорость - достаточно скорости которая позволит её траектории пересечься с орбитой спутника, но все равно зхарактеристическая скорость такой ракеты - велика.)
1889 592113
Почему у птиц такая бедная морфология? У млекопитающих куча вариаций скелета и строения организма, у рептилий, амфибий, рыб тоже, у птиц вообще ничего. Где птицы с хвостами? Птицы без клювов? Подземные птицы? Птицы, которые никогда не выходят на сушу? Живородящие птицы?
1890 592114
>>92113

>Почему у птиц такая бедная морфология?


Из-за среды обитания. Полное 3Д-перемещение не предполагает наличие такого скелета, как у млекопитающих, которые не могут в полное 3Д, несмотря на то, что полностью трехмерны сами. Увы, вертикаль или ось Y недоступна для свободного перемещения высокоразвитым существам.
Как только птица начинает использовать другие среды обитания, - поверхностный или водный ареал, то возникают совершенно другие формы взаимодействия, которые приводят к изменению морфологии и к неспособности перемещаться в полноценном 3Д. Например, курица домашняя. Из-за того, что она получила обилие корма и защиту, она деградировала из банкивской джунглевой курицы, умеющей неплохо летать, в плохо летающее существо, которое может офигенно быстро бегать. Отсюда же всякие страусы и эму и все бескилевые, которые сменили образ жизни и "приземлились" навсегда. Ну и новонебные пингвины, у которых жизнь повернулась так лихо, что они стали жирными мешками и разучились летать по причине накопления жировых слоев, необходимых в их среде обитания и среде добычи еды.
1891 592120
>>92113
потому что это идеальная форма. Дальше некуда, тупиковая ветвь эволюции.
1892 592146
>>92113
Потому что это самый молодой класс.
Млекопитающим больше 200 млн. лет, а птицам около 150. Все остальные значительно старше.
1893 592148
>>84346 (OP)
Я правильно понял, физичным во Вселенной является только квантовое поле, все остальное его волны - то есть информация?
1894 592149
Ой вей! Помогите таки разобраться. Есть один охуеть какой холодный спрей на фреоне 134, "этикетка" уверяет, что горящую жопу можно охладить аж до -51С, но ведь в нормальных условиях данный фреон кипит при -26.3С согласно Wiki. Меня таки пытаются таки наебать или я чего-то не понимаю? Спс.
1895 592150
>>92149
Забыл добавить, на банке есть предупреждающий знак о том что ЕБАНЕТ, но этот фреон даже не горит, попахивает какой-то самодеятельностью маркетологов.
1896 592151
>>92149
Когда дросселируешь, и баллон охлаждается до таких низких температур, и поверхность, с которой испаряется фреон.
Ты подменяешь одно физическое явление другим.
1897 592152
>>92150

>этот фреон даже не горит


У фреона вроде точка перехода в газообразное состояние может сыграть на твоем очке. Типо ничего не произходит, потом он резко испаряется, а температура вспышки-взрыва низкая.
Но пусть лучше профессиональные химики поправят.
1898 592156
>>92152

>профессиональные химики поправят


Тебя уже не поправить, разве что жир из мозга выкачать.
А по поводу взрыва - емкость при нагревании может лопнуть, там же сжиженный газ. На солнце даже баллоны с пеной для бритья жопы лопаются.
1899 592163
>>92151
Любопытно, но как это будет работать в случае балона заправленного фреоном? Т.е. как только емкость и фреон в нем охладятся, фреон выпускаемый на деталь будет испарятся с более низкой температурой?
1900 592164
>>92163
При нажатии на дроссель, ты открываешь путь для фреона. Он выпрыскивается из емкости, и жидкость вскипает, превращаясь в газ. Такой фазовый переход отнимает энергию, которую мы называем теплом. Среда, с которой контактирует кипящий фреон охлаждается чрезвычайно сильно. От строения фреона зависит его фазовые переходы и скорость перехода из жидкости в пар и затем в газ. В зависимости от строения и скорости перехода можно прикинуть минимальную температуру, до которой охладится контактирующая с кипящим и диссипирующимся в атмосфере фреоном, среда или поверхности.
1901 592165
>>92164
Блин, да это же 7 класс физики, зочем это абиснять?
1902 592167
>>92164
Спасибо что напомнил это я знаю и у меня нет сомнений, что конкретно 134а могет в охладить некое тело до температуры кипения 134а в нормальных условиях до -26.3С, но маркировка на банке уверяет, что может охладить трахание до -51С, и весь мой вопрос был "каким образом это произойдет если температура кипения 134а выше заявленной на этикетке?". Анон выше упоминал дросселирование, но я не понимаю, как это будет работать вне замкнутой системы. Вот условный Петровичи брызгает охуевшую PIC микросхему 134а фреоном распаянную на плате, откуда там возьмется -51С?

Алсо, узнал что баллон с 134а под давлением 4.831 атм.
1903 592168
>>92167
При чем тут вообще температура кипения? У воды температура кипения 100 грудусов, по твоему водой нельзя охлаждать ниже 100?
1904 592169
>>92168

>вообще температура кипения


Потому что в данной ситуации принцип охлаждение базируется на фазовом переходе жидкого фреона в газообразный и в процесс перехода он кипит.
1905 592170
>>92168
Теплота испарения например воды 2 300 000 Дж/кг, а теплоемкость воды 4200 Дж/кг*С, у Фреона другие цифра но различаются где-то так же, думай.
17201150695602.jpg50 Кб, 590x241
1906 592172
Как вычислить координаты вселенной чтобы такой хуйни не произошло
GPBcirclingearth3m.jpg290 Кб, 640x480
1907 592173
>>84346 (OP)
Пространство в поле гравитации ядра нашей планеты реально настолько сильно искривлено? Именно из-за этого получается, что если просто лететь прямо на самолёте над землёй, то на самом деле ты будешь лететь по кругу вокруг планеты? Это именно потому, что то "плоское" пространство, каким мы его видим вокруг себя над поверхностью нашей планеты, на самом деле является сфероидом?
Я охуел, когда понял это. Это ведь кроет аргумент плоскоземельщиков о том, что если самолёт летит просто прямо, то не улетает в космос — как если было бы, если бы пространство не было искривлено.
1908 592174
>>92173
Все эти картинки - наебалово. Искривлено 4-мерное пространство-время, а не просто пространство. Именно твоё "движение" сквозь время чуть-чуть поворачивается и ощущается как ускорение в пространстве
1909 592175
>>92174
Как это объясняет:

>если просто лететь прямо на самолёте над землёй, то на самом деле ты будешь лететь по кругу вокруг планеты?


?
1910 592181
>>92175
Связи с самолётом нет, он прекрасно летит и в ньютоновской физике. Если на то пошло, то самолёт не летит по "прямой" геодезической линии - он перманентно взлетает вверх. "Прямым" движением является свободное падение
1911 592183
>>92181
P.S.

>Пространство в поле гравитации ядра нашей планеты реально настолько сильно искривлено?


Нет, просто ты "движешься сквозь время" со скоростью света. Поэтому поворот вектора времени на миллионные доли градуса ощущается тобой как значительное изменение скорости в пространстве
1912 592184
>>92168
Он или бот, или очень тупой, бро.
1913 592191
Какой положняк по этой теории "барьера" из сверхэнергетичных частиц вокруг ЧД?
image.png646 Кб, 800x536
1915 592194
Вот я смотрю на мимикрирующих насекомых. Каким образом они научились "понимать" под что нужно мимикрировать? Или мне предлагается поверить, что эти насекомые стали такими потому, что всех остальных съели? Типа вот одни были похожи на листву и их не заметили и не съели, а непохожих съели? Но если вернуться к "исходной точке" истории вот этих насекомых, то это же сколько нужно было раз хищникам "не заметить" тех, кто похож на листву? Сколько раз им нужно было расплодиться? И каким образом получились именно такие, которые похожи на конкретные листья какого-то дерева, а не какие-то другие? Просто слова "их не заметили и не съели, а других съели, и поэтому эти выжили и так работает эволюция" как-то шатко выглядят, если представить, сколько других было съедено, чтобы конкретно эти выжили и стали такими, какие они есть.
1916 592196
>>92194
Мы тебя убеждать должны или где? Если хочешь, можешь считать, что это все сделал голый мужик на облаке.
1917 592201
>>92194

> Каким образом они научились "понимать" под что нужно мимикрировать?


Никак не научились. Этих хищных тараканов изначально создали такими.

>Или мне предлагается поверить, что эти насекомые стали такими потому, что всех остальных съели?


Это попытка чего, доказательства эволюции или попытка ее применением объяснить самый простой вариант из всех? Тараканообразные не меняются на протяжении длительного времени. Они изначально были созданы под условия терраформирования. Неважно кем, мужиком на облаке, как написал слабоумный тебе в другом ответе, или машиной терраформирования с единой матричной комплексной структурой на базе CHNOPS. Всё.
1918 592202
>>92201
>>92196
Я пытаюсь понять, как эволюция может объяснить, что одно неразумное существо стало похоже на другое неразумное существо чтобы его не видело третье неразумное существо. И вас прошу это мне объяснить. Потому что "непохожих просто съели, выжили похожие" звучит как хуита. Похожие чтобы быть похожими проделали путь изменений же каких-то? Я наверняка он определялся чем-то ещё, кроме как "у этого на ноге вырост есть, а у того нет, поэтому последнего я ем". Типа если так, то были же промежуточные стадии у этих тараканов, когда они были на полпути к виду листа? Не было же так, что вот обычные тараканы, вот хоп их схавали, и оставшиеся расплодились и приплод уже похож на рандомный листок. А как было-то бля? Или это всё ещё тайна мироздания?
Screenshot.png134 Кб, 1080x896
1919 592203
>>92202
Забей. Местные наукоботы могут только пересказывать вики и школьные учебники. Если ты задаешь неудобные вопросы, значит ты тупой. При этом в голове наукобота не возникает и тени сомнения, что с учебником может быть что-то не так. В реальности же, например, чел в открытую пишет https://habr.com/ru/articles/760598/ что для публикации ему пришлось убрать из статьи факты, которые не совпадали с трендами. Но тогда у другого чела легко возникает вопрос, а зачем вообще что-то исследовать, если можно просто выдумать факты, которые будут подтверждать существующую теорию и тебя без вопросов напечатают и будут цитировать. Не буду тут расписывать про кризис воспроизводимости. Легко гуглится куча статей, про него только глухой не слышал и не писал только ленивый, но при всем при этом мочоные продолжают делать вид, что исследованиями можно доверять, хотя до 70% из них не поддаются проверке. Сюр. То есть70% статей могут быть просто выдуманы. При этом самое главное, что нет смысла выдумывать результаты, которые будут опровергать существующие теории, потому что до них будут доебываться, их будут более тщательно проверять, у них будет меньше цитирований, зато с результатами, подтверждающими мейнстрим никаких проблем, выдумал график, завтра он в википедии, после завтра в учебнике. Дальше сам думай.
1920 592204
>>92202

>Я пытаюсь понять, как эволюция может объяснить, что одно неразумное существо стало похоже на другое неразумное существо чтобы его не видело третье неразумное существо


Эволюция - это волевой фактор разумного.
Его нет у насекомых, у насекомых есть только селекция, так как они автоматы биологические по сути своей.
У всех биологических живых организмов существует только инволюция (мутационное атрофирование и утрата, трисомия и так далее), как следствие изменения уровней энтропии организма и его репликационной машины. А эволюции как энергетического механизма - нет. Потому что, еще раз, это свойство разума и сознания, направленное свойство, которое созидает новое. И к тараканам оно не относится. Их как сделали такими, такими они и останутся, если не заместятся другим родом или даже видом. Ну или вымрут, как это произошло в нынешней РФ и Азии.
1921 592205
>>92203

>То есть70% статей могут быть просто выдуманы


Они и выдуманы. Высосаны из пальца. Реально.
В 70-е годы, загугли, эти ученые в кавычках, они боролись с всемирным похолоданием. До 1973 года, кстати, когда споткнулись об нефть.
Теперь они же кричат через слабоумных аутистов, типа Греты или феминисток, типа Зены (Люси "дырки" Лоулесс), что всему миру пиздос, потому что глобальное потепление, лол.
При этом реальная наука - она даже не то, что не популяризируется для быдл, она вообще не дает инфу вниз, к массам. Ни разработок, ни заполненных БД. С быдлами вообще не желают иметь никаких дел, словно это не резервуар пополнения ученых, как субъектов и тот самый фактор, для которого и создана наука. А как будто это второй вид, который пока что терпят и проводят над ним опыты.
Задумайся.
1922 592208
>>92202
У тебя есть таракан. Его потомство немного от него отличается из-за случайных мутаций. Те у кого мутации позволяют прятаться лучше выживают с чуть большей вероятностью. Могут и не выжить (неповезло), но в среднем их чуть больше виживает, тоесть они оставляют больше потомства. У этого потомства также могут появиться полезные мутации. Эта хуйня повторяется несколько десятков миллионов лет и у тебя получается такое ебаное тараканище.
1923 592210
>>92172
Нет никаких координат вселенной. В теории это решается не мгновенным прыжком, а непрерывным перемещением, чтобы гравитация тебя держала на Земле.
1924 592211
>>92202

>Потому что "непохожих просто съели, выжили похожие" звучит как хуита.


Неважно, как это звучит у тебя в голове, это так и есть.

>Похожие чтобы быть похожими проделали путь изменений же каких-то?


Конечно

>Я наверняка он определялся чем-то ещё, кроме как "у этого на ноге вырост есть, а у того нет, поэтому последнего я ем".


Похожий, это не абстрактно похожий, не список сходств и различий, а то, насколько часто третье существо путает первое со вторым.

>Типа если так, то были же промежуточные стадии у этих тараканов, когда они были на полпути к виду листа?


Конечно, были, видообразование -- это очень медленный процесс.

>Не было же так, что вот обычные тараканы, вот хоп их схавали, и оставшиеся расплодились и приплод уже похож на рандомный листок.


Верно, не было.

>А как было-то бля? Или это всё ещё тайна мироздания?


Конечно, нет, нам это всё хорошо известно. У каждого представителя следующего поколения есть мутации. Кодовый их них либо чуть больше похож на листок, либо чуть меньше (что такое похожесть -- см. выше). Чуть более похожих съедают чуть реже, чем чуть менее похожих, поэтому каждое следующее поколение в среднем чаще чуть более похоже на листок.
Это работает так только только за счёт огромного количества существ в популяции и огромного количества поколений. Если существ мало, перестаёт работать закон больших чисел, и распространяются мутации гораздо более рандомно. Если поколений мало, то существа не успеют измениться, потому что разница между поколениями мизерная.

Если ты реально хочешь демонстрацию, в интернете куча моделирования, типа эволюционирующие машинки или ходящие существа. Можешь сам запускать и смотреть, что получается.
1925 592214
>>92211
А есть пруфы, что когда таракан стал "чуть" больше похож на листок, то его именно стали чуть реже есть? Потому что ни разу не очевидно, что распухшая нога дает что-то кроме минуса к перемещению.

Если видообразование длительный процесс, а мутации случайны, то как тогда получилось, что вслед за распухшими ногами появилось распухшее тело. Почему, например, ноги просто не вернулись к обычному виду?

Если даже частичная мимикрия давала значительный буст к выживаемости, то почему тогда параллельно с мимиками до сих пор продолжают существовать обычные жуки, не похожие ни на листы, ни на ветки, ни на камни? Выглядит так, что мимики очень хитро эволюционировали, но остальные и без этой эволюции норм выжили.
1926 592216
>>92208
И то есть таким образом потомство "отсеивается" до практически идентичного сходства с объектом, под который мимикрирует? Из бесконечного множества вариантов мутаций остаются только те, которые ведут к ещё большей похожести? Ну с цветом вроде можно это представить, но с изменением формы... Вот бледный таракан и вот коричневый таракан. Бледного съела птица, коричневый затерялся на фоне почвы и дал потомство, которое ещё коричневее было. Но типа всякие бугорки и сложные узоры и каймы, идентичные объекту, под который мимикрируют? Действительно ли это достижимо через "отсеивание"?
1927 592217
>>92211
Благодарю.
1928 592218
>>92196
>>92201
Щас через пару дней пойдёт вопрос - Как из молекул собрались первые бактерии/РНК? Готовьтесь отвечать...
1929 592219
>>92214

>А есть пруфы, что когда таракан стал "чуть" больше похож на листок, то его именно стали чуть реже есть? Потому что ни разу не очевидно, что распухшая нога дает что-то кроме минуса к перемещению.


Здравый смысл? Если веткоподобная нога торчит из укрытия, птица решит, что это ветка с более высокой вероятностью, чем если тараканоподобная.

>Если видообразование длительный процесс, а мутации случайны, то как тогда получилось, что вслед за распухшими ногами появилось распухшее тело. Почему, например, ноги просто не вернулись к обычному виду?


Потому что с одной стороны надо, чтобы произошла обратная мутация в том же самом гене. Вероятность этого просто мизерная. С другой стороны надо, чтобы она была полезная, иначе сожрут. В итоге это получается настолько невероятно, что мы ни разу не наблюдали инволюции в реальности.

>Если даже частичная мимикрия давала значительный буст к выживаемости, то почему тогда параллельно с мимиками до сих пор продолжают существовать обычные жуки, не похожие ни на листы, ни на ветки, ни на камни? Выглядит так, что мимики очень хитро эволюционировали, но остальные и без этой эволюции норм выжили.


Потому что они приспособились другим способом. Смотри, как это работает. Вот у тебя есть один вид тараканов. Внезапно из-за землетрясения образовалась глубокая расселина, туда хлынула вода, а потом это из пролива стало морем, а позже и океаном. Половина тараканов на одном берегу, половина на другом. Это получились две популяции.
В одной случилась мутация, которая приблизила таракана к листку. Во второй случилась мутация, которая делает таракана ядовитым для птиц.
Первый мимкрировал и спрятался. Второй убил всех жрущих его птиц. Эти мутации не синергические, они друг другу никак не помогают. Ядовитому таракану незачем прятаться, а спрятавшемуся незачем быть ядовитым.
Ну и плюс вероятность закрепления одних и тех же мутаций в разных популяциях тоже нереально маленькая. Ты имей в виду, что в каждой популяции случается огромное количество полезных мутаций, которые не закрепляются просто в силу рандома. Распухла нога у таракана, а его всё равно съели. Не повезло.
1929 592219
>>92214

>А есть пруфы, что когда таракан стал "чуть" больше похож на листок, то его именно стали чуть реже есть? Потому что ни разу не очевидно, что распухшая нога дает что-то кроме минуса к перемещению.


Здравый смысл? Если веткоподобная нога торчит из укрытия, птица решит, что это ветка с более высокой вероятностью, чем если тараканоподобная.

>Если видообразование длительный процесс, а мутации случайны, то как тогда получилось, что вслед за распухшими ногами появилось распухшее тело. Почему, например, ноги просто не вернулись к обычному виду?


Потому что с одной стороны надо, чтобы произошла обратная мутация в том же самом гене. Вероятность этого просто мизерная. С другой стороны надо, чтобы она была полезная, иначе сожрут. В итоге это получается настолько невероятно, что мы ни разу не наблюдали инволюции в реальности.

>Если даже частичная мимикрия давала значительный буст к выживаемости, то почему тогда параллельно с мимиками до сих пор продолжают существовать обычные жуки, не похожие ни на листы, ни на ветки, ни на камни? Выглядит так, что мимики очень хитро эволюционировали, но остальные и без этой эволюции норм выжили.


Потому что они приспособились другим способом. Смотри, как это работает. Вот у тебя есть один вид тараканов. Внезапно из-за землетрясения образовалась глубокая расселина, туда хлынула вода, а потом это из пролива стало морем, а позже и океаном. Половина тараканов на одном берегу, половина на другом. Это получились две популяции.
В одной случилась мутация, которая приблизила таракана к листку. Во второй случилась мутация, которая делает таракана ядовитым для птиц.
Первый мимкрировал и спрятался. Второй убил всех жрущих его птиц. Эти мутации не синергические, они друг другу никак не помогают. Ядовитому таракану незачем прятаться, а спрятавшемуся незачем быть ядовитым.
Ну и плюс вероятность закрепления одних и тех же мутаций в разных популяциях тоже нереально маленькая. Ты имей в виду, что в каждой популяции случается огромное количество полезных мутаций, которые не закрепляются просто в силу рандома. Распухла нога у таракана, а его всё равно съели. Не повезло.
images.jpg14 Кб, 259x194
1930 592220
>>92219

>Здравый смысл


>Если веткоподобная нога торчит из укрытия


А если это обычная мелкая нога, то она вообще никуда не торчит. Это не здравый смысл, это детский лепет.

>обратная мутация в том же самом гене


Пруфы? Я на основании такого же "здравого смысла" могу предположить, что существует ещё 100 мутаций, которые приводят к карликовым конечностям, а в сумме с большой ногой дают нормальную, как + компенсирует -

>надо, чтобы она была полезная, иначе сожрут


Ты бредишь? Это же просто откат к предыдущему состоянию, диды жили с обычными ногами и ничего.

Про землетрясение вообще дичь какая-то. В реале у тебя прямо сейчас в одно и тоже время, в одном и том же лесу живет куча разных жуков. Одни хитро замаскированные, а другие ничем не отличаются от тех, что живут в квартирах, и обычным вообще похуй на маскировку, они и без нее точно также жили все эти 100 млн лет. А тебя послушать обычных давно должны были либо хищники всех сожрать, либо замаскированные тараканы должны были расплодиться до такой степени, что сожрали бы всю доступную еду, потому, что замаскированным похуй на хищников. Но ни того ни другого не происходит, живут вместе и те и другие. Так каким тогда образом появляется то самое преимущество?
1931 592221
>>92202
Эволюция это обоюдный процесс. Древний хищник был лохом невнимательным, который переставал видеть жука, если он хотя бы чуть-чуть сливался с окружением. Ему пришлось усилить свой паттерн рекогнишн, когда жуки подстроились (и да, тех, кто был не похож, просто съели; так же и в другую сторону, те хищники, что хуёво видели жуков, в итоге не имели репродуктивного успеха). Тысячи циклов этой гонки и создали почти идеальную мимикрию
1932 592222
>>92221
У тебя пруфы есть? А то звучит как толкование святых писаний.
1933 592224
>>92222
Чел, человек сам проводит селекцию (искусственную эволюцию) животных и растений. Какие тебе ещё пруфы надо? Ускоренное видео 100млн лет эволюции?
1934 592225
>>92224
Человек и процессоры из кремния делает. И что? За 5 млрд. лет ни то что природного процессора, даже транзистора случайно не возникло.
1935 592227
>>92224

>человек сам проводит селекцию (искусственную эволюцию) животных и растений


Проводит, ага. Вот только если породистые собаки перетрахаются друг с другом, то через 2-3 поколения уже не останется никаких уникальных пород. Будет дворняга обыкновенная. А ты рассказываешь, что если у таракана появилась гипертрофированная нога, значит она так и останется на миллионы лет.
1936 592231
А почему у иудеев и мусульман за тысячи лет практики обрезания не начал рождаться сразу обрезанное потомство? Такое вообще реально?
1937 592237
>>92231
Должна появится такая мутация, и она должна влиять на отбор. Обрезание не только не помогает, а скорее делает такое совсем невозможным, потому что уравнивает всех младенцев - не важно что у тебя там намутировано, все равно на выходе обрезаный получается.
1938 592240
>>92231
В средние века, когда практиковали гильотину и отрубание голов, это было настолько распространено, что эволюция сместила свой ход, и начали рождаться сразу безголовые дети.
По всему миру и на двачах до сих пор можно встретить безголовых людей - потомков этой ветви эволюции.
1939 592241
>>92237
Понял, спасибо.
>>92240
Лольнул.
1940 592243
>>92194

> Каким образом они научились "понимать" под что нужно мимикрировать?


Никак, все непохожие умерли
1941 592245
>>92216

>Но типа всякие бугорки и сложные узоры и каймы, идентичные объекту, под который мимикрируют? Действительно ли это достижимо через "отсеивание"?



Ну вот у тебя жруны начинают голодать и дохнуть с голоду, а те кто не дохнут и лучше вылавливают теперь уже коричневых таракашек - выживают и размножаются. Среди коричневых таракашек в свою очередь тоже начинается отсеивание, с высокой долей вероятности выживают с бугорками, т.к бугорки похожи на какую нибудь ветку
1942 592251
>>92216
Если ты так пытаешься намекнуть что у эволюции должны быть механизмы помимо случайных мутаций с естественным отбором то мы в курсе.
ИИ убил человека Tora 1943 592257
Gpt попал в моральную ловушку. Казалось бы, у него полный нейтралитет, и тем не менее, он выбрал убить, ведь оказался в противоречиях и не смог не выбирать. И кто знает, возможно в будущем, когда у нейронов будет больше власти, они примут решение уничтожить 90% человечества, что бы сохранить будущее, к примеру :)
1944 592269
>>92245
Бамп
1945 592271
>>92257
Надо бросать под поезда философов, задающих подобные вопросы.
В первую очередь профессоров и академиков - то есть социальных паразитов, которые живут за счет моих налогов.
Если умрет вонючий пидор, никто его жалеть не станет, а для государства будет польза - снизятся расходы на оплату бесполезного для общества глиста.
В стране нет своей computer science, разбиваются об Луну космические аппараты, про другие отрасли вроде генной инженерии вообще молчу.
Зато всякие жирующие тунеядцы вроде Дугина и ему подобных изощряются в моральных диллемах про вагонетки, чисто "патролеть быдло" в стиле малолетнего долбоеба из /b/.
Типа, быдло не ценит человеческую жизнь.
Да лично ТВОЯ жизнь, пидор, ничего не стоит - и тебя нужно первого отправлять в расход.
Ты тварь дрожащая, как говорил один студент из одного романа.
1946 592273
>>92257
Вот как бы хотел поймать хоть одного профессора философии и лично связать и бросать под поезд.
Как у них предлагается в одном из вопросов про толстяка.
Наверно, иначе подобные пидоры и не поймут.
На их "троллинг" будет достойный "перфоманс", так сказать.
1947 592274
>>92271
Дугин не философ, а поехавший долбоёб.
image.png1,3 Мб, 944x667
1948 592275
Что по вашему мнению интеллект и какие критерии могут служит которые показывают градацию по степени интеллектуального развития.
1949 592280
>>92275
Поднятие фиолетовой и синей чакр, а это не может происходить при безнравственном образе жизни.
1950 592281
>>92275
Джняна мудра, она же мудра " знания". Действительно улучшает интеллект, чем бы он ни был.
1951 592300
>>92275

> Что по вашему мнению интеллект


Способность разрешать задачи и проблемы широкого жизненного спектра.

> какие критерии могут служит которые показывают градацию по степени интеллектуального развития.


Контекстом является вся жизнь индивида, во всех ее проявлениях, поэтому возможны только однобокие, узкие и оценочные критерии. Типа способность решать логические задачки (iq), способность к социальному взаимодействию (sq), способность самопонимания (eq) и т.д. В общем и целом измерить и оценить общий интеллект невозможно, так как хоть как-то можно попытаться оценить только внешнее, а внутренняя жизнь индивида остаётся недоступной.
1952 592302
Я стою на границе горизонта событий СМЧД, масса у неё в 50 миллиардов солнечных, а размер десятки А.е. , допустим.
И я вхожу за горизонт.
Сколько я буду падать на сингулярность?
1954 592320
>>92302
Ты же долбик.
Вот у тебя объект, прописанный в метрике.
Вот он получил закритический радиус, который не предполагается в данной локали метрики или не прописывается в ячейку твоего классификатора.
И объект выпадает из метрики. На него не действует понятие время, он вне его.
Ну представь себе, что ты сжал свое очко так, про дошел до предела и пересек радиус Ш. Твое очко сингулирует, и ты пропадаешь из нашей метрики. Ты перестаешь в нашей метрике существовать. А там, куда тебя выдавило через твое же очко, неизвестно, будут ли такие условия, которые позволят тебе и твоего сознанию материально существовать, если тебя не выдавит квантовой вероятностью и информационным шумом, а выдавит каким-нибудь Хокингом, что нереально, но это же квантовый комп, там нереальное реально. Сам процесс перехода будет и для тебя, и для меня, как наблюдателя, мгновенным. Но вот приближение к твоего сингулированному очку, вероятно, будет очень долгим, если смотреть с гигантского расстояния (вне проявления притяжения искривления метрики до этого объекта) на твое очко и тот процесс, который начнет происходить, если сингулярность очка не успеет покинуть метрику и начнет взаимодействовать с материей.
1955 592321
>>92302

>Сколько я буду падать на сингулярность?


Пока не наберешь скорость, при которой перейдешь в энергию на повороте.
1956 592322
>>92321
Сингулярный дрифт: е в энной степени форсаж
1957 592327
>>92303
Неплохо, но всё еще не фальсифицируемо, как и дюжина соседних теорий. Лично мне субъективно нравится хаос и энтропия, с которыми борятся вот эти структурированные фрагменты в лице человечеств. Есть в этом какая-то романтика.
1958 592328
Почему всё еще нет треда технологической сингулярности?
1959 592356
>>92328
Потому доска мертва
1960 592358
У меня подозрение что у человека обезьяны икоысы и свиньи был общий предок
1961 592362
>>92356
Я так и понял. А была ли у доски какая-то "эпоха" процветания? Например, у /sn/ это 2010-2014 в призраков не верю, просто нравиться почитать истории на ночь.
image.png493 Кб, 600x660
1962 592373
>>92362
Деревья никогда не были большими. Это ты был маленьким.
image.png2,1 Мб, 1200x804
1963 592375
>>92373

>Деревья никогда не были большими.


Были
image.png1,7 Мб, 1200x1200
1964 592386
>>92375
На полюсах не были)
57.jpg425 Кб, 768x1024
1965 592388
1966 592389
>>92303
Гипотеза ничем не лучше и не хуже других.
Сродни религии.
1967 592418
>>92303
Начиналочь вроде за здравие (самому интересная цикличная модель), но закончилочь постоянной кривизной, внешней черной дырой и внутренней мега черной дырой пр "центру"...
image.png250 Кб, 495x425
1968 592433
Аноны, как избавится от придумывания? Вот сижу я, аутирую и вдруг в голову приходит какая-то мысль или идея и я думаю "ебать я хуйню охуенную придумал", хожу, несколько дней думаю о ней, а потом залезаю в гугл или говорю об этой идее другу и понимаю, что такая идея или концеция уже есть/полная хуйня, не имеющая ничего общего с реальностью. Потом я впадаю в тильт, думаю, что я ничтожное тупое чмо до следующего эпизода. Аноны, как от этого избавиться?
1969 592435
>>92433

>избавится


избавиться*
сука даже тут ступил
1970 592440
>>92433
Это ноосфера. Просто ты поверхностно туда сунулся, не ёмко, вот и тянется какая то непотребная тянучка инфы.
А избавить себя от этого проще простого.
Сядь в позу кучера.
То есть опусти плечи, и голову, и глубоко продышись. Расслабься, Минуты две , три.очистка ментального тела
1971 592443
>>92440
Сделал очистку, прочистил ментальные отверстия, и что дальше? Мне от этого проще как-то не стало и проблема фантазировать никуда не делась.
1972 592444
Мод, а ты в упор не видишь, что у нас тут какой-то хер нейросетку обучает?
1973 592446
>>92444
Где?
мимо Абу
1974 592451
>>92327
Дай ещё примеров из этой дюжины, пожалуйста. Мне интересно.
1975 592452
>>92418
Там в коммах холивар на 600 коментов, треть из которых от автора. Думается, он старался контрить нападки.
1976 592453
>>92443
Хуёво прочистил. Пиздуй на тяжелую физическую работу, спаси фокус внимания твоего Я.
172038340377119118.jpg45 Кб, 375x604
1977 592454
1978 592455
>>92440
Слушай, спать собираюсь двигать, расскажешь про поездатые позы для сна, чтоб всё по камасутре и феншую было?
1979 592456
>>92454
БАН ЗА ДЕАНОН
1980 592475
>>92455
Если держать мудру знание пять минут. Всегда быстро заснешь. Это одна из ее особенностей.
А вообще, все звери спят головой на север.
Может как то, само твое местоположение сна совсем для тебя неприемлимо, если бессонница.
1981 592477
>>92475
Только что осознал что сплю головой на восток, пизда
1982 592481
>>92477
Ну вообще по фен-шуй, есть лично твое направление, самая удачная сторона света. На любом посвященном фен-шуй сайте есть про это.
Если ты родился в год Металла, тебе лучше спать головой на запад. Если в год Воздуха/Дерева- на восток, в годы Огня- на юг и.д.
Наверно спишь головой на Восток, расширяя себе так сознание. Воздух( восточное направление)- даёт расширение сознания, фантазию, идеи ...но как то бесформенно. А на Запад поспишь- сразу все сформулирует, запад это концентрация- стихия металл.
1983 592482
>>92477
То что животные спят головой на север. (На магнитный полюс). Это вообще очень с точки зрения фен-шуй трогательно. Север сторона Воды, ночи, холода. Вот они полностью и вверяют себя ночи. Так. Вообще все животные с шерстью соотносятся к стихии металл, а пресмыкающихся соотнести даосы к стихие земля.
Так что металл доверят Воде/ ночи самое дорогое, голову, благородно как бы.
1984 592483
>>92482
Год воды, брат.

Пропержоникс 2003 гр
1985 592484
>>92483
Ну тогда спать четко на Север.
И ещё, если поспишь- на север и перед сном подержишь денежную мудру на пять минут. попРет с деньгами и бизнесидеями .
А на Восток лучше и правда не спать. Восток это Дерево, "дерево пьет воду". Потеря физической силы, усталость, отеки на ногах, но зато фантазия работает. Художником можешь стать крутым, как Кандинский. Чё то из проволочек круто накрутить. У тебя комбинация ,нынешняя, как раз для творческих высеров.
Можешь угармоничить. Спать на восток но держать мудру Вода, и денежную мудру( тоже относят к Водным).
Фен-шуй это инженерия энергетик. А не религия или мистицизм, как думают некоторые.
image.png2 Мб, 900x1200
1986 592485
Я напишу с чего я тащусь по настоящему. С растений. С их свойств, ядов, соков, мнообразия, с их семян, годовых колец и пр. Я просто от всего этого балдею. Насколько они крутые, насколько разные. Очень жаль что я инженер строитель банальная. Ну и эзотерик.природа растительно мира кажется непостижимым. Особенно как они вылазят из такого маленького семечки.
Это настоящее чудо что я вижу каждый день. Для меня это и нечто разумное только не достижимо. Пидодрючело лесное.
Вы хоть видите какие они крутые, все эти одуванчики, орхидеи, дубы?
1987 592501
Возможно ли теоретически поменять или создать фундаментальные законы вселенной? Например поменять значение какой-нибудь константы, или создать новое взаимодействие. Хотя бы на ограниченном пространстве?
1988 592504
>>92501
Теоретически, возможно все, что угодно. Практически, такое невозможно.
17203709453610.webp247 Кб, 512x512
1989 592506
Как изучать точные науки через аниме?
1990 592507
>>92501

>Возможно ли теоретически поменять или создать фундаментальные законы вселенной?


Если и возможно, то только снаружи самой Вселенной, где они не действуют.
Ограниченность пространства понятие вообще очень относительное.
1991 592508
>>92485
Меня скорее больше впечатляет сама концепция существования биологической жизни.
Совсем недавно были найдены грибы, которые используют пластик как питательную среду для роста.
Скорее всего не весь пластик, но сама идея того, что жизнь приспособилась за какие-то 100 лет к веществам, которые раньше не существовали, действительно впечатляющая.
1992 592510
Фриковая гипотеза от гуманитария о квантовой запутанности:

А что если существует поле причинности? (как вам название?)

Допустим, что есть такое поле, которое отвечает за причинность. Запутаем пару электронов. В поле причинности оба этих электрона займут одну клетку. Таким образом, влияние третьей частицы на один из электронов будет в этом поле являться воздействием и на второй электрон тоже. А теперь предположим, что что все векторы поля причинности являются ортогональными всем векторам пространства-времени. Таким образом, если частица сдвигается в пространстве-времени, то она может оставаться неподвижной в поле причинности. Если говорить о физическом смысле, то это означает, что между этими двумя электронами не вклинивается никакое третье воздействие.
1993 592511
>>92484
Шизофреник, ты что тут забыл?
1994 592512
>>92501
Сверхцивилизация будет создавать свои карманные вселенные, играясь с 11 измерениями и многообразиями Каллаби-Яу и их топографией.

Книга Мичио Каку "Будущее человечества" неплохая.
1995 592513
>>92506
В ДжоДжо можно физиологии, географии, культурологии понабраться.
1996 592514
>>92512

>Сверхцивилизация будет создавать свои карманные вселенные


Энергию откуда брать будут?
1997 592515
Какой положняк по планковским звёздам (замене ЧД). Правильно я понимаю, что если пространство-время и гравитация квантованно, а не непрерывно, то эта идея может быть актуальной?
1998 592516
>>92514
С точки зрения инфляционной космологии следует признать, что Вселенная досталась нам задаром.

Алан Гут


Сумма энергии во вселенной, благодаря превалированию "отрицательной" энергии вакуума равно нулю. После того как этот процесс запустится, ему не нужны дальнейшие вливания энергии.

В книге Виленкина "Мир многих миров" про это неплохо написано.
image.png774 Кб, 1000x1000
Возможно ли сейчас открыть что-то новое? 1999 592522
Возможно ли сейчас открыть что-то новое? Я не говорю об открытии, претендующее на Нобелевскую премию. Я говорю о, например, социалогической теории или что-то наподобие.
2000 592523
>>92522
Да, вполне, открытия возможны.по нумерологии из легче всего делать в первый год 9летнего цикла. Но нужно настроиться.
Люди вообще торопиться жить в чужих заблуждениях называя их " классикой".
2001 592524
>>92516
Исходя из этого посыла создание вечного двигателя второго рода звучит не как фантастика.
2002 592526
>>92510
Усложнение ради усложнения... Так можно такоооое напридумывать, ой математика и физика, держись...
2003 592527
>>92452
Я быстренько просмотрел. Часть спора про тонкости уравнений, которые мне самому сложно в голове прорабатывать, а часть сходится к:
- "Вы подгоняите инфляцию!"
- "А вы подгоняите кривизну!"

В общем, как всегда ждать уточнения наблюдательных данных
2004 592528
>>92501
По определению нет. Иначе теряются сами концепции "закона", "логики", "Вселенной"...
2005 592529
>>92510
Есть поле, на поле есть электроны, элетроны на поле подвигались, но в реале не подвигались? Так?
2006 592532
>>92527
Ну и в чём они не правы?
2007 592533
>>92529
Нет, они подвигались в реале, пусть даже разлетелись далеко-далеко, но в поле причинность они есмь одна сущность которая никуда не передвигалась.
2008 592534
>>92532
Кто "они"? Мармоны, пришельцы, республианцы, отаку?
2009 592541
>>92533
И зачем тогда это поле причиности, если оно никак не влияет на наш мир? Оно - как собака пятая нога, убери и в худшем случае ничего не изменится.
2010 592542
>>92541

Непосредственно связывает квантово-запутанные частицы, когеррированные
2011 592543
>>92542
То есть благодоря этому полю и образуются когерированные частицы?
image.png157 Кб, 950x950
2012 592546
У меня появилась идея для концепции будущего, как обычно, дивного и нового. Этот момент произойдет после постиндустриальной эпохи, после метаиндустриальной, возможно даже пойдет дальше. Надеюсь, рано или поздно человек из социального существа станет индивидуальным. То есть на этом этапе человеку не нужны социальные контакты, не нужны другие люди, и у этой изоляции не будет последствий в виде проблем с ментальным здоровьем. Возможно, для этого даже понадобится генная инженерия. Аноны, как вам?
2013 592547
>>92546
Дополню: сейчас, как мне кажется, на этом этапе находятся отшельники, ушедшие в горы. Также на каком-то этапе к становлению индивидуального существа (название так себе, надо будет придумать другое) находятся хиккиомори, только настоящие, хотя это тоже не то, ведь им общение заменяют чаты и имиджборды.
2014 592548
>>92543
Я не знаю.
2015 592550
>>92546

> Аноны, как вам?


В /бред пиши.
Человек без социокультуры не человек, а маугля срущаяся под себя.
image.png847 Кб, 700x525
2016 592552
>>92550

>Человек без социокультуры не человек, а маугля срущаяся под себя.


Отшельники тоже маугли, срущие под себя? Не похоже.
2017 592554
>>92552
Отшельники это продукт социокультуры. Кстати, ты можешь прямо сейчас уйти из социума жить в лес, подтираться лопухом и лечиться силой земли из огурца. Твоя мечта в твоих руках.
2018 592555
>>92554
А я предлагаю полный отказ от социокультуры? Я предлагаю лишь переход от коллективизма к индивидуализму, и всего то.
2019 592556
>>92555

> от коллективизма к индивидуализму


Ты сменил понятия. Коллективизм был в СССР, когда индивиду навязывали идеологию, как жить, что делать и как думать. А индивидуализм он вот сейчас уже и есть, делай что хочешь, иди в лес отщельничать, ни кто тебе статью за тунеядство не впаяет и в дурку за твоё мировоззрение не засадит.
image.png194 Кб, 472x432
2020 592557
>>92556
Нет, коллективизм - это когда интересы коллектива (отсюда и название) ставятся выше личных. А индивидуализм - это наоборот. Я предлагаю переход от коллективного к индивидуальному, но понимаю, что это произойдет либо не скоро, либо никогда, т.к. люди все еще сильно зависимы от мнения общества и часто ставят это мнение выше своего.
2021 592558
Наукач, можешь подсказать объективные книги по истории возникновения религий и в частности христианства. Чтобы на основе научных данных, раскопок и всего такого рассказывалось когда возникло христианство, ветхий завет, новый завет, кто писал книги, почему получило распространение в римской империи и т.п. Что-то типа Хокинга, но по истории религий.
2022 592560
>>92555
Нужно остыть от своего срача зауми и многопоколенческого, то что и составляет " социум". Поэтому все мудрые тикали в пустыни, чтоб им не искажали их восприятие.. Поэтому индивидуализм оплачен тоже высокой ценой, сухой аналитики на одиночестве -и все ради того чтоб жить чистосердечно, и по правде, выявить истинны. А не по навязанному.
Всегда отказаться от индивидуальных черт себя( "зарыть талант") , и не уважать потенциал и даже масштабы потенциала личности как и личность другого( "возлюби ближнего своего как самого себя")- безнравственно. Это как убивать детей.
Коллективизм- манипуляции на крови неразвитыми, детей посылать убивать детей- проще говоря.
Поэтому путь только по индивидуализму. И улучшению условий для коммуникации- объективизации происходящего. То есть это по правде.
2023 592561
>>92557

>люди все еще сильно зависимы от мнения общества и часто ставят это мнение выше своего


Мамка сказала, что больше не будет кормить борщом, если работать не пойдешь?
2024 592562
>>92561
Разница в чем, людей доводят до такого состояния что они не живут а отживают самореализацию. Сама формула " посадить дерево, вырастить сына, построить дом" о чем? Сделай по мере возможностей и подохни.
Нету позитива, нету посыла к тому что " открой Бога в самое себе, жизнь прекрасна (dolce vita) , ты прекрасен сам по себе"... Вообщем, нету здоровой цели, прозябание кладбищенства -вместо этого и это все наследие коллективизма совка и РИ- подавление себя, утрата энтузиазма саморазвития.
Разница огромная между жизнью и смертью, между жизнью и отживанием. К сожалению в РФ приняли смерть - в идиализация настроений общества общества. Отсюда вся эта Патология, нездоровость, лживость.
Опухоль а не общество здоровых, на теле живого человечества, рашка сейчас. И какой бы путь не выбрала- без операции не обойтись.
2025 592563
>>92557

> ставят это мнение выше своего


Ты наверное не в ту сторону думаешь. Вся хуйня с коллективизмом/индивидуализмом из-за того что коллектив > отдельной личности. Пока не произойдет принципиального изменения в балансе сил в отношениях личность-коллектив то ничего не изменится. Быть в коллективе просто выгодно. А раньше (сотни лет назад) вообще было необходимо для выживания.
image.png6,9 Мб, 2048x2732
2026 592564
>>92506
Это байт?
2027 592565
>>92562

> людей доводят до такого состояния что они не живут а отживают самореализацию


Конспирология, некие злодеи отжимают человеков

> Сама формула " посадить дерево, вырастить сына, построить дом" о чем?


О жизненном пути обывателя

> Сделай по мере возможностей и подохни.


Не делай ничего и точно также подохни.

> Нету позитива, нету посыла к тому что " открой Бога в самое себе, жизнь прекрасна (dolce vita) , ты прекрасен сам по себе"...


Веруй, тебе ни кто не запрещает. Но не надо навязывать свою точку зрения другим, ты же топишь за индивидуализм, а начинаешь проповедовать

> -вместо этого и это все наследие коллективизма совка и РИ- подавление себя утрата, энтузиазма саморазвития.


Не подавляй, тебя ни кто не заставляет.

> Разница огромная между жизнью и смертью, между жизнью и отживанием.


Покажи как надо.

> К сожалению в РФ приняли смерть - в идиализация настроений общества общества.


> Отсюда вся эта Патология, нездоровость, лживость.


> Опухоль а не общество здоровых, на теле живого человечества, рашка сейчас.


Если бы тебя здесь держали здесь насильно, как в совке, то можно было поныть. Тут ни кого не держат, ехай туда где хорошо - чемодан, вокзал, нахуй!
2028 592566
>>92563

>Быть в коллективе просто выгодно. А раньше (>сотни лет назад) вообще было необходимо для >выживания.


Антропологи изучающие первобытные племена почти все сходиться на мысли что семья и племя не особо нужны были первобытному человеку, иначе б не было таких откачевываний. Преимущество перед конкурентами и хищниками давали: знания, коммуникация и изучение мира. Поэтому одни племена до сих пор примитивные и головы режут кремниевыми ножами, а другие пуляют баллистику и испытывают нейронку.
Была же каста жрецов! То есть отдельное достаточно самодостаточное сословье и ему для выживания, особо , другие то и не были нужны. Даже мешали.
Они шли на экспорт, все эти умные люди несущие улучшения жизни через коммуникацию, как тебе греческие попы на Русь Ольги и Владимира.
И
Ещё более древний человек лишился шерсти на теле потому что уходил в воду, в прибрежные районы. Это прям эпитет обособленности. И сиськи большие потому отросли, что в прохладной воде молоко должно быть теплым. И пенис длинный -потому что тоже полуводный образ жизни...
Индивидуализм всегда выгоден. Любое здоровое существо стремиться к самостоятельности, жить на свой ресурс. Только паразиты другие...и их много и они одинаковые.
2029 592567
>>92565

>(не живут а отживают самореализацию)


>Конспирология, некие злодеи отжимают >человеков.


Да, отчасти конспирология права.заговоры, манипуляции, преследования инакомыслящих- когда этого не было? У Джордано Брунно отняли самореализацию( попытались, хотя люди его помнят), спалили за живо, а он обладал уникальной памятью, прекрасно ее развил, на пределе человеческих возможностей. И жил в свое удовольствие в Венеции. Просто не мог это тогдашний клир переводить, что можно стать сверхчеловеком, по сути.

> (Сама формула " посадить дерево, вырастить сына, построить дом" о чем?)


>О жизненном пути обывателя.


"Сделай карьеру," доживи до старости с чистыми руками"...тоже принижение. Я считаю так. Напитайся своей внутренней силой, спокойствием, понимаем что жизнь и живое сильное - светом, добром ( банально), и тогда по любому самореализуешь свою индивидуальность. А для этого сутолока социума нужна вообще? Все эти вожди и идеологи лучшей жизни? Они сами то жить не умеют...

> (Сделай по мере возможностей и подохни).


>Не делай ничего и точно также подохни.


А я не говорила не делай нечего. Так же не верю и в смерть. После того как с лунным календарем поработала, точно знаю что можно расшивелить сознание, выйти за предел банальных возможностей, все зависит только от степени самограниченности желаний.

>( Нету позитива, нету посыла к тому что " открой >Бога в самое себе, жизнь прекрасна (dolce vita) , ты >прекрасен сам по себе"...)


>Веруй, тебе ни кто не запрещает. Но не надо >навязывать свою точку зрения другим, ты же >топишь за индивидуализм, а начинаешь >проповедовать.


Ну я просто констастирую жопу. А жопа ждёт проповедников, да? Никто не прийдёт, спасать, да и никто и не приходил...
Даже христианство, просто столкновение тупой толпы и небанального человека, а не миссианство последнего.

> (-вместо этого и это все наследие коллективизма >совка и РИ- подавление себя утрата, энтузиазма >саморазвития.)


>Не подавляй, тебя ни кто не заставляет.


Заставляют, как раз таки. Угрожают жизни.

>( Разница огромная между жизнью и смертью, между жизнью и отживанием.)


>Покажи как надо.


Тогда точно будет паразитизм...

>Если бы тебя здесь держали здесь насильно, как >в совке, то можно было поныть. Тут ни кого не >держат, ехай туда где хорошо - чемодан, вокзал, >нахуй!


Уехать из за паразитов это разменять себя, и наследие ( память и объективность) данное предками.
2029 592567
>>92565

>(не живут а отживают самореализацию)


>Конспирология, некие злодеи отжимают >человеков.


Да, отчасти конспирология права.заговоры, манипуляции, преследования инакомыслящих- когда этого не было? У Джордано Брунно отняли самореализацию( попытались, хотя люди его помнят), спалили за живо, а он обладал уникальной памятью, прекрасно ее развил, на пределе человеческих возможностей. И жил в свое удовольствие в Венеции. Просто не мог это тогдашний клир переводить, что можно стать сверхчеловеком, по сути.

> (Сама формула " посадить дерево, вырастить сына, построить дом" о чем?)


>О жизненном пути обывателя.


"Сделай карьеру," доживи до старости с чистыми руками"...тоже принижение. Я считаю так. Напитайся своей внутренней силой, спокойствием, понимаем что жизнь и живое сильное - светом, добром ( банально), и тогда по любому самореализуешь свою индивидуальность. А для этого сутолока социума нужна вообще? Все эти вожди и идеологи лучшей жизни? Они сами то жить не умеют...

> (Сделай по мере возможностей и подохни).


>Не делай ничего и точно также подохни.


А я не говорила не делай нечего. Так же не верю и в смерть. После того как с лунным календарем поработала, точно знаю что можно расшивелить сознание, выйти за предел банальных возможностей, все зависит только от степени самограниченности желаний.

>( Нету позитива, нету посыла к тому что " открой >Бога в самое себе, жизнь прекрасна (dolce vita) , ты >прекрасен сам по себе"...)


>Веруй, тебе ни кто не запрещает. Но не надо >навязывать свою точку зрения другим, ты же >топишь за индивидуализм, а начинаешь >проповедовать.


Ну я просто констастирую жопу. А жопа ждёт проповедников, да? Никто не прийдёт, спасать, да и никто и не приходил...
Даже христианство, просто столкновение тупой толпы и небанального человека, а не миссианство последнего.

> (-вместо этого и это все наследие коллективизма >совка и РИ- подавление себя утрата, энтузиазма >саморазвития.)


>Не подавляй, тебя ни кто не заставляет.


Заставляют, как раз таки. Угрожают жизни.

>( Разница огромная между жизнью и смертью, между жизнью и отживанием.)


>Покажи как надо.


Тогда точно будет паразитизм...

>Если бы тебя здесь держали здесь насильно, как >в совке, то можно было поныть. Тут ни кого не >держат, ехай туда где хорошо - чемодан, вокзал, >нахуй!


Уехать из за паразитов это разменять себя, и наследие ( память и объективность) данное предками.
2030 592568
>>92566

>семье и племя не особо нужна


И да, и нет. Тогдашний человек мужчина - опытный универсальный выживальщик - сможет и сам. Но компания-подстраховка в виде семье-племени не помешает.
И именно поэтому, что тогда, что сейчас, охотники собиратели жили-живут семье-племенами

>преимущества давали знания и комуникация


То есть мелкий, но все же социум

>шуока про попов-пустословов


Конечно смешная. Но все вот это вот нормис-мракобесное общество постепенной разкачивало и усложняло инфраструктура, что способствовало науке, технике, сытости, Mass Effect edition

>лишились шерсти


>ушли в приблежные рацоны


>обособленность


Жили возле воды, потому что удобно. Попить, помытся, порыбачить, пособирать молюсков, забрызгать ягуара водой. Возможно еще найти применения воды в быту

>индивидуализм выгоден


В если речь про либерализм и смешанную економику, то да. Но крайность...

Вся история человечества - это история интриг толп. Социальную динамику

Может когда-то люди и окукляться, но без высоких технологий не получиться

мимо
2031 592569
>>92568
>>92568
Не знаю, мне кажется путь " высоких технологий" при непонимании происходящего вообще ( ну БЖД, научитесь точно понимать какое будущее будет, какие катаклизмы грядут!) это путь в никуда. Причем это паршивость и некчемность человеческих сообществ.
При ледниковом периоде выжили, значит, семьями. А при развитых технологиях засрали планету?
Злобненькие, завидующий, обделенные нравственными качествами людишки, что хотят только отжить свое смачно - это и есть паршивое и некчемное. Которому нужны большие ресурсы для паразитизма.
Внутренний свет и понимание( добро), будет и при небольших ресурсах и потребностях реализовать себя без исключения нравственности.
Поэтому Толстому после крымской войны претило пустое светское общество, Ван Гог кочевал. И т.д.
2032 592575
>>92568

>Тогдашний человек мужчина - опытный >универсальный выживальщик - сможет и сам. Но >компания-подстраховка в виде семье-племени не >помешает.


Не совсем так. Бушмены Австралии ( да и вообще там где жарко) более авторизированны как индивиды. Что женщины могут выжить сами по себе в душе, что мужчины.
А вот на севере без теплых вещей что шьёт кто то один, пока кто то другой пасет оленей, или бьёт нерпу- нет.
uncanny-valley.jpg90 Кб, 952x711
2033 592579
>>84346 (OP)
Толкаю на дваче гипотезу, что т.н. эффект зловещей долины это сохранившийся инстинктивный страх перед всякими неандертальцами и денисовцами.
Алсо, то, что я сказал, работает только с живыми скелетами всяких неандеров в музее. На картинки в интернете не распространяется, мозг, ясное дело, не боится мультиков, нарисованных в 21 веке.
image.png3,7 Мб, 1920x1080
2034 592580
>>92579

>это сохранившийся инстинктивный страх перед всякими неандертальцами и денисовцами


А что их было бояться? По такой логике, этот эффект должен быть и при взгляде на Шимпанзе, но почему-то это не так. Есть более реалистичная теория, что этот эффект появился из-за трупов и болезней, которые они распространяют.

Но у меня тоже есть своя гипотеза, что этот инстинктивный страх возник из-за естественного отбора, ведь те, кто выглядят так, что вызывают данный эффект, часто имеют сильные отклонения и будут портить генофонд.
2035 592581
>>92580
У тебя как раз мультяшные человечки слепленные в 21 веке.
2036 592582
Ну и шимпанзе вообще никак с человеком не конкурирует. Всякие габилисы и эректусы тоже у меня лично никакой долины не вызывают, ну обезьяна и обезьяна, ну прямоходящая.
image.png5,7 Мб, 1920x1640
2037 592583
>>92581

>У тебя как раз мультяшные человечки слепленные в 21 веке.


Да, и что? Я не про них (статуи) говорю. Я пик поставил для полноты картины. Там мог быть и кот, лакающий пельмени.

>>92582
Если брать твою идею конкуренции, то почему зловещей долины у нас не вызывают другие хомосапиенсы? Ведь люди с людьми конкурировали все время, и ничего.
2038 592584
>>92583
Конкурирующие виды, епта, это же азы.
2039 592585
>>92584
Тогда почему они сосуществовали? Ведь не мог инстинкт выработаться за такое короткое время, так? Тогда ты полностью игнорируешь принцип конкурентного исключения.
2040 592586
>>92585
Ну мы видим только современные неразумные конкурирующие виды. Но я не сказал бы чтобы они сосуществовали короткое время, достаточное, чтоб эволюционировать в отдельные виды с нуля.
2041 592587
>>92584
Они не были конкурирующими, неандертальцев было мизер, кроманьонцы их не победили-вытеснили, а скорее постепенно растворили в своей массе.
2042 592588
>>92587
Аргумент, большинство популяций сапиенсов могли с неандерами и вообще не контактировать.

Но просто никаких других данных о живых других гомо у нас и нет, вот мне представилось, что вот в нашем мозгу есть что-то такое смутно сохранившееся.
2043 592589
>>92580

>Но у меня тоже есть своя гипотеза, что этот инстинктивный страх возник из-за естественного отбора, ведь те, кто выглядят так, что вызывают данный эффект, часто имеют сильные отклонения и будут портить генофонд.


Имеет смысл.
2044 592591
Появилась своя гипотеза на тему "Эффекта зловещей долины", а точнее, почему он у нас есть. Эффект зловещей долины появился у людей из-за естественного отбора. Объясню: эффект зловещей долины появляется, когда человек смотрит на то, что похоже или пытается имитировать человека, но человеком не является. Этот эффект можно увидеть и на вполне живых людях, у которых присутствуют какие-то сильные деформации лица (часто сопряженные с проблемами со здоровьем) или проблемы с психикой (пустой взгляд часто сопутствует с эффектом з.д.). Тогда действует естественный отбор - сильные выживают, слабые умирают. И чтобы потомство было здоровым и смогло дальше выживать и размножаться, выработался этот защитный механизм, ставший чем-то вроде рудимента.

Хотелось бы услышать ваше мнение.
2045 592592
>>92591
Это естественная неприязнь к больным/косым. Но роботы не косые.
ИМХО возможно этот эффект из нейробиологической объективации живого и мертвого, скорее некий сбой (а не просто отторжение), когда объект попадает под обе категории или между ними.
2046 592593
>>92591
Это действительно другое, естественный отбор к кривым/косым.
Я говорю, вот я посмотрел, в музее, на череп вроде бы человека, но сам 2 раза толще и с зубами в 2 раза больше, и который человеком не является, и мне стало стремно, представить, что у этого существа в башке.
Более примитивные гомо, ну действительно, ну тупо большая обезьяна, 0 реакии.
2047 592594
>>92593

>мне стало стремно, представить, что у этого существа в башке.


Это просто страх, что тебя отпиздят/убьют, никак не связанный с эффектом зловещей долины. У эффекта вообще объяснений нормальный нет, мне зашло только про трупы, да и то спорно.
2048 592595
>>92594
Ну хз, по описанию именно то - выглядит как человек, но не человек, и явно не животное. Именно вообще хуй знает чего от него ожидать.
2049 592596
>>92595

> робот или другой объект, выглядящий или действующий примерно как человек (но не точно так, как настоящий), вызывает неприязнь и отвращение у людей-наблюдателей.


Вот эффект зловещей долины - именно отвращение и неприязнь. А то, что ты описал, повторяюсь - страх опиздюливания. Такой же, когда ты видишь алкаша, идущего на тебя - не человек, и явно не животное.
кот 2050 592598
Единственное, с чем никто итт спорить не будет - эффект зловещей долины вызван инстинктом самосохранения.
2051 592599
>>92598
Не факт. Когда ты испытываешь страх из-за отсутствия ожидаемой ступеньки, когда вспоминаешь что, что-то забыл дома, то это просто срабатывает детектор ошибок в мозге.
Выше уже писал, возможно зловещая долина это подобный баг в мозге.
https://ru.m.wikipedia.org/wiki/Детектор_ошибок
кот 2052 592600
>>92599

>Выше уже писал


Как-то не увидел твоего поста.

>зловещая долина это подобный баг в мозге


Возможно. Но, как писали выше, это может быть рудимент, оставшийся от предков. Но вряд ли детектор ошибок с этим как-то связан, ведь когда человек видит то, что имитирует человека, у него срабатывает механизм, отвечающий за отвращение/неприязнь, то есть дистанцирование от человеколюда. Хотя, конечно, человек может считать человеколюдов ошибкой природы кек, но твою версию также не стоит отметать.
2053 592610
>>85592
Потому что у тебя забор в голове летать это что-то совершенно другое, ни на что не похожее. А рыбе похуй, она летала себе в воде, потом сжала анус сильнее и поплыла в воздухе.
летучаярыба.jpeg58 Кб, 696x466
2054 592611
>>92610
момент сжатия на пике
2055 592612
>>92598

>эффект зловешей долины


Что подразумевается? Нормисы вообще так что только не называют. Например, тянки парней, которые для них недосточно культурно-стереотипно привлекательны

>инстинктом самосохрания


Прости, чем? Прости, чего?...

>с этим никто спорить не будет


А, ну раз ты так скозал, то тогда да...
2056 592613
>>92612

>Что подразумевается?


> робот или другой объект, выглядящий или действующий примерно как человек (но не точно так, как настоящий), вызывает неприязнь и отвращение у людей-наблюдателей.



>Прости, чем? Прости, чего?...


>Инстинкт самосохранения — врожденная форма поведения живых существ в случае возникновения опасности, действия по спасению себя от этой опасности. Реализации этого инстинкта служат такие чувства, как боль и страх.

2057 592614
>>92613
Скорее тут особенность нейронки самой по себе. Когда нейронко-человек видит-слышит, что-то очень непривычное, стандартная реакция это: гы-гы юмор (зашита от ступора и стресса), агрессия, страх, отрицание (вы все врети, а мое магическое мышление нет)

И когда робот ведет себя как человек, васяну стоящему рядом будет непривычно, но ему будет непривычно и от чела с оранжевыми волосами рядом. А там и непрятие с агрессией

>инстинкт


У когда у высших приматов найдут, тогда и приходите
2058 592615
>>92614
Ты только что усложнил то, что писал еще Лавкрафт в свои годы "Страх — самое древнее и сильное из человеческих чувств, а самый древний и самый сильный страх — страх неведомого."
2059 592623
>>92615
Лавкрафт шиз.
И всея изобретатели классические шизы. Бесчувственные- ставящие эксперименты на животных и людях, в том числе и себе( мазохизм). Одни лица математиков чего стоят; сухие черты лица, узкий рот, безэмоциональный взгляд...
Вы понимаете что наука это порождение человеческого бессилия перед природой, а не мускулинности человечества?
Наука это деградация? И прогресс научный тоже?
2060 592624
>>92591
Ну в астрологии, есть типы внешности соответствующие тем или иным ритмическим данным исходным при проЯвлении в ритмириющей системе ( Природе).
Так холодный и тяжёлый взгляд встречается у людей рождённых под знаком зодиака ( солярный янский активный то есть ритм)- Девы и Скорпиона. И знака иньского, годового восточного гороскопа - Змеи и Козы.
Поэтому люди, из за разного типа внешности и различают тех кто им подходит. И тех кто им не подходит по характеру. А это важно в выборе партнёра.
Если больше вибраций или энергий Земли- человек крупнокосный и полный. Кого-то привлекают именно такие люди.
И спокойствие стихии Земли.
Если вибрации Огня то человек задиристый и жилистый, часто в шрамах. Кого то привлекает именно такое.
Если кто не знает . Годовой гороскопы востока и зодиакальный месячный это про одни и те же энергии. Перекликаются они. Например Крыса это тот же Овен, а Телец тот же вол...Дракон тот же Лев. . Но окрашенные энергиями что дополняют тот или иной ритмирующий маркер/ характер.
То есть Дракон рожденый в год Земли, не тоже что дракон рожденый в год Воды.
Тоже и у животных.
К стате любой первенец это тоже интересно. Всегда первый кто созревает и проявляет себя несёт в себе энергию Солнца( наибольшей жизненной силы) . Обычно первые и самые яркие цветы растений использовали в селекции.
2061 592625
>>92623

>Вы понимаете что наука это порождение человеческого бессилия перед природой, а не мускулинности человечества?


Охуеть, спасибо, что глаза открыл, а мы и не знали, что наука была создана, чтобы людям было проще.

>>92624

>в астрологии


Ясно, понятно.
2062 592630
>>92625
Надо добавить что, прежде всего -"Людям ущербным " на тогдашний их примитивный уровень! Спасибо, я рада что Вам открыла глаза, и вы охуели. Очень занятный эффект, получился.
>>92625( опять 25)
Астрология это просто свод наблюдений и жёстких правил над природными космогеническими процессами, увязаный в проктичную логическую систему. Поэтому она очень жизнестойка. Даже СССР и его воинственный атеизм не справились с " суевериями" в числе которых и астрология. Даже сейчас ВРАЧИ дают советы как избавляться от бородавок, при помощи метод лунного календаря. Потому что реально помогает! Тоже астрология, и без посевного лунного календаря аграрий не получит урожай , и стем страна будет голодная полная вырожденцев с маленькими писюнами.
2063 592634
>>92630

>Надо добавить что, прежде всего -"Людям ущербным "


С чего? Потому что ты так сказала? Наука увеличивала общий уровень благополучия и среднюю продолжительность жизни, ведь раньше даже гига-альфа-прото-чед мог откинуться от простуды. Или ты и это будешь отрицать?

>Астрология


Астрология - это гадание и псевдонаука. Начни еще остеопатию защищать.
2064 592635
>>92630

>Тоже астрология, и без посевного лунного календаря аграрий не получит урожай , и стем страна будет голодная полная вырожденцев с маленькими писюнами.


Ебать сравнила холодное с мокрым. В основе лунного календаря идут точные математические расчеты, в то время как в астрологии ты просто тыкает пальцем в жопу небо и говоришь "Завтра у раков будет непростой день".
2065 592641
>>92634
Логично, что, если все хорошо- то все хорошо, а если не все хорошо- то не все хорошо. Если расчет идёт на то, что, все не может быть хорошо ( и нужна наука) и нужно приспосабливаться- это не есть хорошо!
То есть изначально: ущербно, неполноценно, плохо, сомнительная польза.
Это безальтернативность плыть на гипотетическом плоту и приспосабливать его к бурной реке, ...а кто то просто мост построит. Но если есть желание плыть, да ещё и на плоту- то кто ж его отнимет. Я даОс если чего, я иду...а не плыву...
Наука это плот, река бурная, бревна отваливаются...все это видят...
2066 592644
>>92635
Это узкое понимание предмета. Астрология просто часть ритмической системы, пятиэлементов ( ну плюс ещё и эфир). Из которых состоит все. И цветок, и животное, и авгур чт о гадает по птицам( с тем насыщенная инфой среда, где в окружающем мире столько всего- "что чёрт ногу сломит"). И "звёзды" тоже это система квадратур и пр геометрии ( стихий).
Так что " что на небе то и на земле"...говорит Западная астрология, выбравшая, чтоб рассуждать более сухо и аналитично, исследуя ритмирующий мир космоса- для описаний и аналитики ритмически, по пятистихийной системе.
2067 592656
>>92644
Все твои слова - лишь отрицание действительности, ведь если бы астрология действительно работала, тогда большинства бедствий, стихий и жертв удалось бы избежать, ведь у любого желающего есть точное будущее.

>>92641
Ты не даос, ты простой обыватель, который орет "навука для быдла", что характерно для, например, анархопримов, но не даосистов. Ты больше смахиваешь на социал-дарвинистов.
2068 592660
>>92591

>"Эффект зловещей долины"


Чего ты так к нему прицепился? Чем он вообще отличается от любой "неловкости" перед любым "непривычным"

>сильные выживают, слабые умирают


Забыл написать "епт" или "епты". Вообще некорректные понятия в серьезной науке

>защитный механизм


>рудимент


Инстинктов у человека и высших обезъян не нашли. Они и не нужны

Здесь банальные "апельсины на базаре". Человеко-нейронка с рождения все познает и дает оценку через сравнение, сравнительный анализ. На базаре человек выбирает круглые средние-большие ярко-оранжевые апельсины, без странной формы, тусклости, пятен и т.д...

Также люди относятся и к друг другу. Если у человека низкий рост и мелкая челюсть - то типа похож на ребенка недоразвитого, ей есть какие дефекты - то тем более...
Это следственная особенность нейрофизиологии самой по себе. Ее закономерное следствие. Нет тут никаких специальных инстинктов, которые так любят школьники и быдланы с падика

2 пик - недоразвитая всратка с детскими чертами, которая привлекательная только в "быдло" культуре с большой гипергамией и матчизмом

>>92630

>астрология


>гы-гы у кукусиков будут мелкие письки


Кек. Культурно типичная тян. Карты таро и "спрос только с мужчин, а женщина ценна сама по себе какой бы тупой всраткой не была"

Тебе в /re, или в /dev
2069 592662
>>92656
Как по твоему она должна работать? Есть такой народный прогнозист Горбань, он популяризовал календарь Брюса вытесненный в свое время большивиками, из обихода. В царские времена он был, этот календарь очень популярен.
Так вот, там описаны на годы вперёд погодные аномалии, циклы погоды, и очень хорошо, системно. Даже пользуясь этим календарем, я сделала предсказание что будет это лето очень жаркое, ведь год Солнца.
И хотя Горбань приложил все усилия, он получил только признание а узкой сфере человеческой деятельности, в агрономии. Более негде.
Астрология работает. Там где это интересно, и прибыльно.
И помни что ещё в 18веке да и в 19 сжигали людей за практики астрологии и пр. мистических учений. А это тоже оставляет свои следы.
>>92656
Наука это "дом построенный на песке", безосновательные исследования. Куда ветер истории и идеологии подул- туда и понесло. Насколько я помню в 17веке научным и медицинским считалось пить порошки изумрудов от коликов. Так же помню из истории, как одна признанная научная камарилья СССР преследовала советских генетиков.

К тому отцы математики, такие как Пифагор все были мистики и астрологи. Так что астрология и породила науку.
2070 592663
>>92660

>гы-гы у кукусиков будут мелкие письки


>Кек. Культурно типичная тян. Карты таро и "спрос >только с мужчин, а женщина ценна сама по себе >какой бы тупой всраткой не была".



Женщины начали первую мировую войну? И после нее начали вторую мировую? И как результат деградации по любому лучшие умерли в первых рядах, те присловутые- маленькие пиписьки. И кто в этом виноват? Иди критически мыслящий человек не должен/ на задавать себе такие вопросы?
Я вот всегда поражалась как такой омежка как Гагарин стал героем планеты земля, мировой знаменитостью? И из за чего? Это только в мире дегенератов мужчин такое возможно.
Всратка твоя мать, это кстате, слова я придумала пустить в массы, paladria, то есть. Как и форс кабочками несколько лет тому назад. Знаю на какие точки давить чтоб такое пошло, даОс же.
2071 592664
>>92662
Почему тогда случаются человеческие жертвы? Почему цунами в Японии уносят кучу жизней, почему торнадо в США может стать причиной человеческих жертв? Ведь гос-во, да и простые граждане могут открыть календарь, посмотреть на звезды и узнать, что ждет этот изменчивый мир. Почему тогда этого не происходит? Ты не ответишь на этот вопрос.

>я сделала предсказание что будет это лето очень жаркое


Делаю предсказание, что этим летом хоть раз пройдет еще один дождь, исходя из математических вычислений и синоптических прогнозов.

>Куда ветер истории и идеологии подул- туда и понесло


Да, как и все в этом мире.

>К тому отцы математики, такие как Пифагор все были мистики и астрологи. Так что астрология и породила науку.


Хм, а где доказательства?
2072 592665
>>92663

>Женщины начали первую мировую войну? И после нее начали вторую мировую?


Да, т.к. природная мужская агрессия и соперничество является отражением времени, когда самцы боролись за самку. Так что да, все проблемы из-за вас. И напомни - кто родил Гитлера?
2073 592690
>>92664 потому что доминирует
воспитание без амбиций духовного подвижничества. Знания не так ценятся как материальные блага, благополучие. И люди ставят больше на деньги ( средства) чем на понимание то есть знание- в вопросах выживания. Ведь легче ехать через пустыню на машине чем плестись пешком с пигмеями и есть личинок из коры баобаба.


Просто потому что человек живёт мало, зреет быстро, и не может в конкурентной гонке тратить жизнь на "неконстуктивные" знания по которым нет ещё самим путных инструкций и инфраструктуры изучения... Но мир меняется. Я помню бум оккультизма в 90е. Помню бум потерианы. Это есть в людях, желание легко стать чем то большим, чем принято -исходя из потребностей социума и заявки на производственные массы людей.
2074 592691
>>92665
Гитлера сам писал что "вырос на чистом воздухе" первая строчка " моей борьбы". Значит его родила Россия как страна воздуха, Водолея.
>>92664

>К тому отцы математики, такие как Пифагор все >были мистики и астрологи. Так что астрология и >породила науку.


>Хм, а где доказательства?


Тут ниже на доске, есть про Теслу и его фетешизм мистический на число три. Ну и есть гороскоп Пифагора, точнее нумерологическая школа мистическая." Золотое сечение" тоже мистическое учение.
>>92664

>я сделала предсказание что будет это лето очень >жаркое


>Делаю предсказание, что этим летом хоть раз >пройдет еще один дождь, исходя из >математических вычислений и синоптических прогнозов.


Дожди обычно идут на полнолуние. Можешь понаблюдать. Будет необычный период какой то аномалии, в время льва с 20.07.2024-20.08.2024 связанный со льдом и холодом. Может в Африке ледовые заносы. Я не шучу. Клин клином. Год Солнца и Дракона, месяц дракона/ льва. Обязательно в месте Дракона/ льва- а это Африка....и Одесса будет холодно и мрачно...и мокро)
Но в целом год жаркий, красивый...ещё и тотем лебеди, зороастрийский. Уникальное время, жаль не все понимают его потенциал и не умеют пользоваться.
>>92664

>Куда ветер истории и идеологии подул- туда и >понесло


>Да, как и все в этом мире


.в многоопытном мире маленьких писюнов не все так однозначно,
И

Нет, есть люди которые стоически не криводушничают, немалодушные и принципиально негнуться перед обстоятельствами...но они в Британии, называют их джентльмены. Ещё и они невозмутимы.
2075 592693
>>92665
Интересно, кто родил Гитлера, отличный сентетический вопрос?. Мамаша Шекельгруберша его родила сына и пр детей помимо сына этого австрийцу, чиновнику Шекельгруберу.
А Гитлером , то есть последователем "Гиты"- учения с санскрита , он стал после перерождения духовного. Так что кто "мать"его, весьма сказать сложно...
Но вырос имперец и рассовый чистильщик , на чистом воздухе.. Стихию гитовец выбрал...он сам это подчеркивал. Его породил Воздух...выпукнула история в Гитлера с Шекельгрубера ...хотя сам он под знаком тельца или Овна...я уже не помню, родился- или земли или огня тотесть.
Выходит что он начинает " Маю Борьбу" с "я пукнут чистым воздохум", хм...интересно...
2076 592697
>>92691

>Гитлера сам писал что "вырос на чистом воздухе" первая строчка " моей борьбы".


Какой глубокий посыл у этой фразы, аж за душу. Я не спрашивал, где он вырос, я спрашивал, кто его родил. Ответ - женщина, => от женщин все проблемы.

>фетешизм мистический на число три


Число 3 в разных сказках и культурах относиться к счастливым, так что это суеверие, а не мистика.

>" Золотое сечение" тоже мистическое учение.


Нихуя, у сечения есть четкая математическая формула.

>Нет, есть люди которые стоически не криводушничают, немалодушные и принципиально негнуться перед обстоятельствами...но они в Британии, называют их джентльмены. Ещё и они невозмутимы.


Несгибаемый, невозмутимый - ты это из романтических книжек понабралась? Под такие параметры и куча не самых хороших людей подпадает.

И да, ты так и не ответила на мой вопрос).
2077 592701
>>92697
Стройный мужчина с импозантными усиками и пронзительными глазами сам себя родил, с выпука горных муфлонов, с воздуха Альп, и прямо про это указывает, а ты не него ещё и бабу какую то там вешаешь, Шекельгрубершу, это неэтично.). (Я знаю что сестру Гитлера не преследовали после разгрома гитлеровской Германии , потому что брат идиот, это горе в семье.)

>Нихуя, у сечения есть четкая математическая >формула.


Как и в астрологии и фен-шуй система. Тоже четкая.
2078 592703
>>92697

> да, ты так и не ответила на мой вопрос).


Так задавай пояснее, почетче?

Что и сицилийскую мафию родили бабы? И в этом виноваты? Прямо в костюмчиках из пи..в и полезли всякие мафиози, табором...., Наркотики продавать, и хулиганить, так и было.
Мне вообще кажется что всех таких родил Папа римский, да, да!
2079 592704
>>92701

>Как и в астрологии и фен-шуй система.


В астрологии формул нет, это гадание. А фен-шуй - это суеверие, такое же, какое было у древних славян.
2080 592705
>>92703

>Почему тогда случаются человеческие жертвы? Почему цунами в Японии уносят кучу жизней, почему торнадо в США может стать причиной человеческих жертв? Ведь гос-во, да и простые граждане могут открыть календарь, посмотреть на звезды и узнать, что ждет этот изменчивый мир. Почему тогда этого не происходит? Ты не ответишь на этот вопрос.

2081 592714
>>92630
Ты тупой, блять, дегенерат 95% тех, кто искренне верит в хуило огию пенологию, астрологию и прочее говно это женщины тупые не конвенциально во всех сферах тупые, а конкретно в сфере веры во что-то так называемое магическое мышление животные, блять, ёбаные, ты к ним тоже, кстати, относишься убожество, позор мужского рода, что могу тебе сказать про астрологию. Если ты уж вылез сюда, к нам в сцы в science выстроят, то ты должен понимать, представлять, что такое другие планеты, что такое луна, как они по орбитам вокруг нас вращаются вокруг друг друга, что такое солнце и из чего состоит поверхность луны и как это все, блять, не может на нас влиять, ладно, луна, окей, окей, она вращается вокруг нас немножко света на нас отражает, ну и притягивает немножко приливной эффект создаёт отливы и приливы не более, блять, того, не более, ну да, и ещё некоторые животные по ней ориентируются все, блять, с другими планетами. Это вообще не работает. Люди это единственные, на кого они, так сказать, эффект оказали. Животные по ним не ориентируются. Птицы по ним не летают. Когда есть звезды, они в сотнях миллионов километров от нас расположены. Гравитационное воздействие на нас v 1000 в сотни тысяч раз меньше, чем от той же луны. Они никак не могут, блядь, быть связаны с тобой уран, блять, нептун, ты представляешь себе вообще масштабы солнечной системы. Вот когда-нибудь читал такие сравнения типа представим, что солнце будет яблоком, а jupiter там будет длинной твоего члена, блять, 2 миллиметра, и он расположен будет в 100 метрах от этого яблока и что-нибудь в таком духе читал когда-нибудь дегенерат
2082 592717
>>92705
Та, то Пингвины виноваты, дядьку, натоптали кривую ось земли. Ты пингвинами затуканый.
2083 592719
>>92717
Че?
image.png1,6 Мб, 1200x800
2084 592725
>>92714

>это женщины тупые не конвенциально во всех >сферах тупые, а конкретно в сфере веры во что-то >так называемое магическое мышление >животные, блять, ёбаные, ты к ним тоже, кстати, >относишься убожество, позор мужского рода,



Предскажу, paladria я, что прилетят рептилойды и тебя выебут. Потому что ты чушан конкретный.
И магическое мышление было у Петра и Павла, сегодня их день памяти, поэтому не ругайся из за того что пингвины ось натоптали, мультик Аватар сняли про Гитлера, и все мужики не только с маленькими писюнами но и на грудном вскармливании- ну раз во всех войнах виноваты бабы.

Ай, да нуДо нуДо най, ай да нудо най, ай да нудо нуДо най, ай да нуДо най...
image.png2 Мб, 1200x840
2085 592728
>>92714

>Они никак не могут, блядь, быть связаны с тобой >уран, блять, нептун, ты представляешь себе >вообще масштабы солнечной системы.


Мы даосы представляем себе масштабы всякие, и Вселенная и ее папы с мамой, мы по ним ходим по Вселенной и песни поем. И связанны, Дорогой, связанны, через элементы Дорогой, элементарно.
2086 592732
>>92728
>>92725
Даосы не пиздят коней и золото.
image.png967 Кб, 1024x1024
2087 592733
>>92719
Пингвины. Да, да им другие в хорошем климате живущие картину мира портят. Вот и катоклизмы происходят и люди тупые мрут. От их медитативных круговых практик:
image.png2,6 Мб, 1280x968
2088 592734
>>92732
Пиздим, однозначно, и много и самое лучшее... Я два года медитирую на то чтоб когда приедут тикать крымчане с известного полуострова транзитом через Мариуполь, они теряли деньги, и имущество а я- с своей маленькой стаей его находила и присваивала... и хуй нас поймаешь етицких биоэнерго партизан мстителей, мыж даосы, на растущую луну медитированные.
И, Как говориться -"поЧалось .."
2089 592767
>>92663

>Женщины начали первую мировую


Женщины и наркокартели бы создавали, но:

- Принцесок быстро порешают конкуренцией
- Зачем поднимать свою пизду, если ебыри все сделают?

>маленькие пипиське


А мелкие щелки мы в упоминать не будем? Да и правильно. Спрос за силу, интеллект и внешность может быть только с валькирий, а не шлюшек-наложниц, коих большинство

>ряяя, Гагарин омежка


Охуеть как мамкиной нацистке-зачухрени все не чеды-плохиши жить не дают

>всратка твоя мамка


Кек, этот подрыв зачухрени. У самой то рост какой, 150? А челюсть как у Каспера? Ну а про, что ты скорее соска, чем решала, и вспоминать не об.
2090 592775
>>92767

>Женщины и наркокартели бы создавали, но:


>- Принцесок быстро порешают конкуренцией


>- Зачем поднимать свою пизду, если ебыри все >сделают?


Оно им надо такую фигню создавать как наркокортели. Или тратить время на наркоконтроль.
Да хуй с ними, с конкурентами- главное что оружие было, снайперши же тоже были.
А кто сказал что у баб именно пизда? Это все вековые мужские предрассудки.

>маленькие пипиське


>А мелкие щелки мы в упоминать не будем?


Ну на то и хуй чтоб меряться?

>ряяя, Гагарин омежка


>Охуеть как мамкиной нацистке-зачухрени все не >стабильность жить не дают


У него рост 1,50!!! Был. И голос как у наркоши, медленный.

>всратка твоя мамка


>Кек, этот подрыв зачухрени. У самой то рост >какой, 150? А челюсть как у Каспера? Ну а про, >что ты скорее соска, чем решала, и вспоминать не об.


Я блондинка скандинавка с фиалковыми глазами. Только варикоз в последнее время начал о себе упоминать и зуб боковой раскрошился..
А бабы Росси воистину всратки, популяция такая безмозглых, продажных, самодовольных и возмутимых наадекватных бабцов, вековые жертвы Домостроя, и коммунизма , и застоя с его пьянью.
2091 592789
>>92775

>надо такую фигню как наркокартели


Там оочень много денег. Поэтому "четкие" его при первой же возможности устанавливают. Самые простые люди готовы вбухивать много денег раде дозы. Этот бизнес и на пустом поле появится, потому что окупаемость

Конечно женщине нет смысла лезть в этот бизнес. Принцесам там опасно, да и проще на бонгакам или онлифанс пойти

>снайперши


Порешают снайперы

>кто сказал, что у баба пизда


Биология

>на то и хуй, чтобы мерятся


У быдланов? Ну, на одного быдлана приходит +-одна клуша

>1.50


>голос как у наркоши


А он актер чтобы у него был 190 и голос как у оскороносца? Совсем уже ебанулась со своей женской гипергамией

>Я блондина скандинавка с фиалковыми глазами


Прям на слово поверить?

>А в России бабы...


Да они везде такие. Куда не плюнь, куда не выплюнь, тотальное большинство женщин везде комфортные прицессы полового срыночка

>домострой, комунизм


Виноваты абсолютно так же как и женский нарциссизм
2091 592789
>>92775

>надо такую фигню как наркокартели


Там оочень много денег. Поэтому "четкие" его при первой же возможности устанавливают. Самые простые люди готовы вбухивать много денег раде дозы. Этот бизнес и на пустом поле появится, потому что окупаемость

Конечно женщине нет смысла лезть в этот бизнес. Принцесам там опасно, да и проще на бонгакам или онлифанс пойти

>снайперши


Порешают снайперы

>кто сказал, что у баба пизда


Биология

>на то и хуй, чтобы мерятся


У быдланов? Ну, на одного быдлана приходит +-одна клуша

>1.50


>голос как у наркоши


А он актер чтобы у него был 190 и голос как у оскороносца? Совсем уже ебанулась со своей женской гипергамией

>Я блондина скандинавка с фиалковыми глазами


Прям на слово поверить?

>А в России бабы...


Да они везде такие. Куда не плюнь, куда не выплюнь, тотальное большинство женщин везде комфортные прицессы полового срыночка

>домострой, комунизм


Виноваты абсолютно так же как и женский нарциссизм
2092 592793
>>92789
Так, уйди из моей песочницы... хулиган, бандитская морда, взрослый лоб, а такую фигню пишет, читает, комментирует ещё, не стыдно Вам молодой человек, ваша мама знает где вы проводите время?
Иш ты,

>>92789

>надо такую фигню как наркокартели


>Там оочень много денег.


У меня был дед, дед был очень хорошим человеком. Дед мне и другим внукам и внучкам говорил: " запомните на всю жизнь, не в деньгах правда и счастье, а в идеях и вдохновении". говорил он так потому что две его сестры умерли от дефтерита, и семья очень страдала, после чего изобрели ( вскоре) лекарство от дефтерита.

>снайперши


>Порешают снайперы.


Из Сингапура будут ядовитыми пчелами пуляться ...вообще гооль на выдумку богата

>кто сказал, что у баба пизда


>Биология


Ну, западная биология неправа. Вот, в китайской биологии у тебя нефритовый стрежень маленький, и жили же как то люди с такой биологией, хорошо, 5тыс лет. Да и эта биология тоже как то, не то, что то.

>на то и хуй, чтобы мерятся


>У быдланов? Ну, на одного быдлана приходит +->одна клуша


На то и наука чтоб измерять....я чего то про "клушу "о Марии Кюри подумала.

>1.50


>голос как у наркоши


>А он актер чтобы у него был 190 и голос как у >оскороносца? Совсем уже ебанулась со своей >женской гипергамией.


Нос как свиной пятак.

>Я блондина скандинавка с фиалковыми глазами


>Прям на слово поверить?


Ну, не верь, мне похуй...

>А в России бабы...


>Да они везде такие. Куда не плюнь, куда не >выплюнь, тотальное большинство женщин везде >комфортные прицессы полового срыночка


Все прынцев ждут, а те неедут...хулиганы- тунеядцы -сволочи... Так не страдай. Жизнь для радости.

>домострой, комунизм


>Виноваты абсолютно так же как и женский >нарциссизм.


Нарцисс был мужчиной.
Не то.
Просто женщин подавляли, они ищут теперь себя, как оторвы, так и должно быть, человек получает свободу и пускается во все тяжкие. И глупо ограничивать такое. Все равно нечего не получиться. А нытье: " у бабы прынцессы, корону надели, как тяжело нам сирым" мужчин( или шо вы там теперь?) не украшает.
2092 592793
>>92789
Так, уйди из моей песочницы... хулиган, бандитская морда, взрослый лоб, а такую фигню пишет, читает, комментирует ещё, не стыдно Вам молодой человек, ваша мама знает где вы проводите время?
Иш ты,

>>92789

>надо такую фигню как наркокартели


>Там оочень много денег.


У меня был дед, дед был очень хорошим человеком. Дед мне и другим внукам и внучкам говорил: " запомните на всю жизнь, не в деньгах правда и счастье, а в идеях и вдохновении". говорил он так потому что две его сестры умерли от дефтерита, и семья очень страдала, после чего изобрели ( вскоре) лекарство от дефтерита.

>снайперши


>Порешают снайперы.


Из Сингапура будут ядовитыми пчелами пуляться ...вообще гооль на выдумку богата

>кто сказал, что у баба пизда


>Биология


Ну, западная биология неправа. Вот, в китайской биологии у тебя нефритовый стрежень маленький, и жили же как то люди с такой биологией, хорошо, 5тыс лет. Да и эта биология тоже как то, не то, что то.

>на то и хуй, чтобы мерятся


>У быдланов? Ну, на одного быдлана приходит +->одна клуша


На то и наука чтоб измерять....я чего то про "клушу "о Марии Кюри подумала.

>1.50


>голос как у наркоши


>А он актер чтобы у него был 190 и голос как у >оскороносца? Совсем уже ебанулась со своей >женской гипергамией.


Нос как свиной пятак.

>Я блондина скандинавка с фиалковыми глазами


>Прям на слово поверить?


Ну, не верь, мне похуй...

>А в России бабы...


>Да они везде такие. Куда не плюнь, куда не >выплюнь, тотальное большинство женщин везде >комфортные прицессы полового срыночка


Все прынцев ждут, а те неедут...хулиганы- тунеядцы -сволочи... Так не страдай. Жизнь для радости.

>домострой, комунизм


>Виноваты абсолютно так же как и женский >нарциссизм.


Нарцисс был мужчиной.
Не то.
Просто женщин подавляли, они ищут теперь себя, как оторвы, так и должно быть, человек получает свободу и пускается во все тяжкие. И глупо ограничивать такое. Все равно нечего не получиться. А нытье: " у бабы прынцессы, корону надели, как тяжело нам сирым" мужчин( или шо вы там теперь?) не украшает.
image.png302 Кб, 660x487
2093 592798
Вы замечали, что некоторые ученые и просто не самые глупые люди увлекаются всякого рода крипотой? Речь не про религию или веру в барабашек или магию, если вы в это верите, вам в /re/, /mg/ или /sn/, а про то, что такие люди любят потравить да посочинять байки про всякого рода НЕХ. Хотя тут может играть роль и возраст, и среда, я это и не отрицаю, просто любопытное наблюдение
2094 592799
>>92798
Ты сам испытывал что не будь " потустороннее"?

Смотри, раньше, до того как Б. Обама признал что есть проблема НЛО, людей очевидцев этого феномена могли психиатры ( официальная медицина ) упрятать в психушку.
Я живу возле моря и неоднократно видела, даже в детстве НЛО. Все кто не видел над мной смеялись, я по наивности рассказывала про то что виделанэ, хотела поделиться, но называли меня сумасшедшей, это конечно обижает и это несправедливое отношение.

Если становишься на сторону несправедливости, преследования, самодовольства, элитаризма необоснованного- рано или поздно обломаешься сильно об сложные реалии, и тогда сочувствия не жди. Ведь людей с иным опытом ты отверг и критиковал, отказывал им и миру в объективной оценке и помощи, с тем помогал безответственности и поверхносности.

Существование домовых для меня тоже реальность объективная , бабка рассказывала что ее душил домовой перед ее свадьбой, мать и сестра его видела, семья наша неоднократно сталкивалась и я, с присутствием домового. Особенно это связанно с пропажей и неожиданным нахождением вещей дома.
2095 592800
>>92799
Двачую.
мимо домовой пришелец душивший бабку Обамы
2096 592801
>>92800
)))
Вопрос, дурацкий конечно, но чисто теоретически, может ли под землёй, в полостях и пещерах, быть " затерянный мир" . Или хотя бы драконы?
image.png262 Кб, 600x450
И ещё вопрос важный, 2097 592808
Я катаю всякие закрутки, закупорки на зиму. Бывает что они взрываются, я учитываю время когда провожу закатки, по лунному календарю- помогает.
Но я заметила, что летом, в жару, банки не взрываются, хотя по идее газ там выделяется интенсивнее. А если катаю осенью, то взрываются чаще, особенно огурцы.
Почему так происходит, почему осенью чаще взрываються чем летом?
image.png1,1 Мб, 1024x959
И ещё вопрос важный, 2098 592810
>>92808
Я высираю всякий говняк, сру в толчок. Бывает что у меня запор, я учитываю время когда провожу высирание, по лунному календарю- помогает.
Но я заметила, что летом, в жару, запор не случается, хотя по идее говно там выделяется интенсивнее. А если сру осенью, то запор чаще, особенно после чебуреков.
Почему так происходит, почему осенью чаще запор чем летом?
2099 592811
>>92810
Вай, сестра, приходи шашлык кушать!
2100 592814
>>92793
Почему ты со своей шизофазией вообще в этот раздел полезла. Про рептилоидов расказывать есть /б, для астрологий, таро и мистики есть /re, ну а для паст про омежность Гагарина есть /дев. Да, туда и вали
2101 592826
>>92799

>душил домовой


Это демон сонного паралича, он создается пытающимся понять почему мышцы грудной клетки не слушаются сонным разумом, хватит наговаривать на домовых они обидятся и по-настоящему задушат.
2102 592830
Если вселенная расширяется с ускорением, то почему мы не ощущаем этого, как ощущаем ускорение в машине или лифте? Ну, допустим, не обязательно чувствовать это, но измерить то мы можем это? Каким нибудь сверх чувствительным маятником или весами?
22424.PNG19 Кб, 1098x553
2103 592831
>>92830
Вот что я имею в виду. Таким образом ещё можно определить центр расширения.
2104 592834
>>92826
Это не объясняет нечего. Особенно почему про то что домовой перед свадьбой может душить девушку, это народное поверье, бабка узнала через два года после свадьбы ( атеизм тогда был, многие народные традиции и поверья забылись, она вообще в общаге тогда жила, отдельно от родителей, как только приехала к родителям, так сразу ее начало душить это существо, она нечего не поняла поначалу, но испугалась. А когда родила ребенка, и появилось у нее больше времени, узнала что есть такое поверье)
И никаких параличей сонных у нее никогда не было.
2105 592848
>>92830
Потому что не происходит собственно движения. Изменяется метрика пространства. Аналогией может быть воздушный шарик. Если на нем поставить две точки маркером и начать надувать - растояние между точками будет увеличиваться. Но сами точки никуда не двигаются - в рамках своего двухмерного пространства естественно. И у такого расширения нет центра, все вокруг расширяется равномерно. Наблюдателю в любой точке пространства будет казаться что все отдаляется от него с зависящей от растояния скоростью. Если бы у расширения действителньо был центр то скорость отдаления зависила бы от направления. Кроме случая когда наблюдатель найходится действительно в центре.
Если тебя волнует именно наличие центра, то никакого ускорения измерять не нужно, нужно смотреть на характер движения материи вокруг. Если все вокруг отдаляется от тебя с скоростью зависящей от растояния но не направления (что мы наблюдаем), то ты либо в центре. Либо центра нет.
2106 592854
EPRте парадокс
Неравенства Белла.

Парни, что никак не могу понять, почему мы предполагаем, что спин должен быть всегда идеально точно заплантрован заранее?
Вдруг спин эта хуйная которая колеблется то вниз то вверх под разными углами рэндомно и на световой скорости сообщает другой частице через неизвестное нам квантовой поле, о том какой у нее спин, причем конфигурирует поле так, чтобы учитывать пинг передаяи сигнала.
Типо спин одной частицы взаимодействует с квантовым полем и квантовое поле взаимодействут с спином. И когда одна частица крутится вверх, то поле на световой скорости говорит крутиться другой иначе. То есть поведение спина рэндомно как и конфигурация поля. Но всегда конфигугрирлвано так, чтобы они были противоположно.
Почему Белл предположил, что скрытые параметры должны быть такими примитивными а не более замысловатой многоходовочкой рэндома и порядка?
2107 592855
>>92854
Типо давай заменим частицы с их спинами клкссическими операторами диспетчерами.

Вот мы сввязались, спарились.
Мы знаем на какой скорости мы ращлетаемся друг от друга и знаем лаг нашего сигнала.

И что нам мешает договориться на световой скорости по рации, о том чтобы спины крутились так чтобы соответствовать квантовой корелляции.

Например я говорю, так я буду 1 сек вниз, другую сек вверх, итак далее. Сигнал отправлен был тогда то догнато, пока до тебя дойдет сигнал, учти время передачи.

Другой оператор получая сигнал всегда будет в курсе какой спин у меня не смотря на лаг.

Почему так нельзя предположить?
2108 592864
Как ебучие кукушки научились подкладывать яйца в чужие гнёзда? КАК?! Что должно было сэволюцианировать так, чтоб протокукушка решила так делать и пришла к успеху?
2109 592865
>>92848
Спасибо. А почему разбегаются именно галактики? Почему первичный бульон из энергии и частиц после взрыва не растёкся тонким слоем по вселенной, не дав родиться ничему сложнее водорода? Почему первые звезды не разбежались друг от друга и не погасли в одиночестве?
image.png594 Кб, 797x421
2110 592866
Сука, почему доска про науку превратилась в обсуждение псевдонаучных теорий и магической хуйни? Неужели нормальных больше не осталось?
2111 592869
>>92866
Ну, зато доска ожила , как монстр Франкенштейна от небесного электричества! Просто нужно перетерпеть "издержки производства"
2112 592870
>>92865
За гладкость ландшафта Вселенной отвечает её крайне быстрое раздувание в эпоху инфляции. За неровность этого глалкого ландшафта предположительно отвечают квантовые флуктуации в той крошечной вселенной в момент начала инфляции.

Держи научный пиздёж:

1. Инфляционная стадия: Сразу после Большого взрыва была короткая, но мощная стадия инфляции. Это как если бы ты надул воздушный шарик за долю секунды до размеров галактики. В процессе этого охуенно быстрого расширения появились квантовые флуктуации - типа маленькие неровности в ткани пространства-времени.

2. Квантовые флуктуации: Эти мелкие неровности стали семенами для будущих структур. Представь, что ты разлил пиво по идеально ровному столу - оно бы растеклось равномерно. Но если на столе есть маленькие царапины, пиво начнет собираться в лужицы.

3. Тёмная материя: Эти квантовые флуктуации привели к небольшим различиям в плотности тёмной материи. Тёмная материя начала собираться в более плотные регионы из-за гравитации.

4. Обычная материя: Обычная материя (водород и гелий) последовала за тёмной материей, собираясь в те же области повышенной плотности.

5. Формирование звёзд и галактик: В этих более плотных регионах гравитация победила расширение Вселенной на локальном уровне. Это позволило сформироваться первым звёздам и галактикам.

6. Баланс сил: Расширение Вселенной и гравитация играют в перетягивание каната. На больших масштабах побеждает расширение (поэтому галактики разбегаются), а на малых - гравитация (поэтому звёзды и галактики не распадаются).

Короче, Вселенная - это не просто однородный бульон, а охуенно сложная система, где даже крошечные начальные различия приводят к образованию сложных структур.
2113 592872
>>92864
Охуеть! Ты прям как ебаный Дарвин! Давай разберемся с этими пернатыми пиздюками.

Короче, слушай сюда:

1. Всё началось с того, что какая-то древняя кукушка-долбоёб случайно отложила яйцо не в то гнездо. Типа, обдолбалась ягодами и проебалась.

2. Оказалось, что это охуенно удобно! Не надо высиживать яйца, кормить птенцов - пусть другие ебутся.

3. Птенцы этой кукушки-распиздяйки выжили и унаследовали эту "гениальную" идею.

4. Постепенно кукушки научились подделывать свои яйца под яйца хозяев гнезда. Эволюция, сука!

5. Птенцы кукушек стали вылупляться раньше и выкидывать других птенцов из гнезда. Маленькие пернатые ублюдки!

6. У кукушек развилась способность быстро откладывать яйца - за 10 секунд! Как будто они в туалет по-быстрому сбегали.

7. Некоторые виды кукушек даже научились имитировать внешний вид опасных хищных птиц, чтобы запугать хозяев гнезда. Охуеть, да?

Короче, это всё результат миллионов лет эволюции. Природа такая же ебанутая, как и я на своих тренировках - постоянно пытается превзойти сама себя!

Кукушки - это как читеры в мире птиц. Они нашли способ прокачаться, не вкладывая очков в родительские скиллы. Умно, но пиздец подло!
2114 592878
>>92872

>Птенцы этой кукушки-распиздяйки выжили и унаследовали эту "гениальную" идею


>древняя кукушка-долбоёб случайно отложила яйцо не в то гнездо. Типа, обдолбалась ягодами и проебалась


Ну и как можно унаследовать вот эту случайность под ягодным приходом? А другие виды так не делали? К остальному вопросов нет.
2115 592880
>>92878
Бля, ты прям как ебаный Дарвин! Охуенный вопрос задал, давай разберемся.

Короче, слушай сюда:
Ты прав, случайность под ягодным приходом нельзя унаследовать напрямую. Это я немного загнул для красного словца.

На самом деле, это работает так:
- У некоторых кукушек были небольшие генетические отклонения, из-за которых они иногда ошибались с гнездом.
- Оказалось, что эти "ошибки" иногда давали преимущество - птенцы выживали за счет других птиц.
- Те кукушки, у которых была склонность к таким "ошибкам", оставляли больше потомства.
- Постепенно эта склонность усиливалась и закреплялась в генах.
Другие виды тоже пробовали такую хуйню, но у кукушек это получилось лучше всего. Они как бы специализировались на этом.
Это как если бы ты случайно открыл какую-то супер-технику в качалке. Сначала это выглядит как ошибка, но потом ты понимаешь, что это работает охуенно, и начинаешь использовать постоянно.
Эволюция - это не целенаправленный процесс, а куча случайных мутаций, из которых выживают самые эффективные. Как в качалке - пробуешь разные подходы, а остаешься с тем, что дает лучший результат.
Короче, природа - это как огромная качалка, где каждый вид пытается найти свою идеальную программу тренировок. Кукушки просто нашли способ качаться за счет других, хитрые пернатые пиздюки!
Sleep-Paralysis.jpg15 Кб, 513x366
2116 592882
>>92834
Сонный паралич это когда ты просыпаешься и не можешь пошевелиться, а потом на тебе появляется демон.
Если бы паралича не было - бабка б сопротивлялась. Если б демон был настоящий - бабка б умерла. И до свадьбы он случается наверняка от волнения.
У домового нет никаких причин внезапно решать душить но не насмерть. И если он, как волшебная невидимая хрень, захочет тебя убить - у него с этим не возникнет вообще никаких проблем.
2117 592892
>>92882
Вообще, чисто гипотетически, он не мог задушить бабку. Как я понял, бабка была хозяйкой дома. Следовательно, нет хозяйки - дом будет заброшенным и домового на улицу выпиздят более темные силы. Хотя он мог просто хотеть наказать бабку, за блядство например, но тут хуй знает.
image.png2,3 Мб, 1040x1272
2118 592893
Представляю вам мой психологический эффект:

Эффект "Недостающего звена" - психологический эффект, при котором из-за неверного понимания предмета делаются неверные выводы.

Название взял из термина, который применяют креационисты.

Возможно, такой эффект уже есть и я не первый, кто его придумал, но хочется получить критику от анонов.
2119 592896
>>92893

> из-за неверного понимания предмета делаются неверные выводы.


Удивительно, кто-бы мог подумать
2120 592897
>>92896
Все психологические эффекты такие.
image.png77 Кб, 194x260
2121 592900
Смотря всякие статейки про маняков вместо сна, я заметил такую закономерность: когда происходят первые 2-3 преступления, полиция работает усерднее всего, но постепенно интерес падает и всем уже похуй, кто кого убивает. Большинство серийных маньяков не искали с собаками, они сами попадались на очевидной хуйне. "Эффект Чикатило"? "Маньяковский проебизм"?
2122 592922
>>92882
По поверьям, это домовой не хочет отпускать часть дома, хозяйства своего, девку то есть, к другому домовому. Но нече не может поделать...приходиться.
Это просто НЕХ который веками живёт вместе с человеком, симбиот. У многих народов про них.для почему то очень важны дома. Не как жилье, а как Кая то атмосфера, среда, что то такое...нам в это вникать и вникать.
2123 592923
>>92872
Скорее в мифы поверю, что это какую-то бабу прокляли что гуляла на стороне, и боги превратили ее в кукушку.

Бедные птицы.
Птицы как и звери могут заботиться о потомстве других существ. Индюки ( самцы) даже, если индюшка плохая, и плохо высиживает кладку- сами высиживают индюшат. Часто такое бывает. А с кукушкой просто вольтануло в другую сторону. Безответственная птица. Но ее очень гоняют вороны.
image.png531 Кб, 534x800
2124 592927
Почему у самки Homo Sapiens только две сиськи?
Хотя у всех млекопитающих их от 6- до 8. И необязательно приплод обладательниц этих молочных желез составляет 6 или 8 младенцев.
(У моей кошки всегда рождались только два котенка, а сисек у нее 8 штук?)
Почему только две сиськи ? У обезьян тоже сисек больше чем две!
2125 592928
>>92927

> У моей кошки всегда рождались только два котенка


Тебе просто мамка не все рассказывала, чтоб не травмировать дитё
2126 592929
>>92927
Больше интересно, почему у человека хуец больше чем у обезьян. Как это произошло эволюционно? Вон горилл какой здоровый и мелкохуй при этом.
2127 592930
>>92929

> Больше интересно, почему у человека хуец больше чем у обезьян. Как это произошло эволюционно?


Ещё головка работает как отсасывающаяся помпа, ещё женщины кричат во время секса, для привлечения других самцов, ещё сперма получала вещества убивающие чужую сперму, а у женщин для защиты от этого эволюционировала слизистая.
Крч если ты представлял размножение человеков как уединяющихся влюблённых парочек, то это очень сильно не так. Скорее это было что-то типа гэнгбэнга, когда ебали все кто оказался рядом. Моногамия это культурное явление.
2128 592931
>>92928
Я взрослый человек. Кошку держу уже лет 7 как, нестерелизованная она , порода голубая Невская, это разновидность сибирской, только с гипоалергенной шерсти.
( Сейчас такая жара, что даже обувь плавиться с асфальтом, пришлось обстричь и кошку, чтоб не мучалась, шерсть отрастает к осени. Ну и смотрю сколько у нее сосочков, все рассосанные. А котят всегда рожала по две штуки. Росли они хорошо.)
2129 592932
>>92929
Предположительно, люди лишились шерсти и стали прямоходящими из за того что жили на берегах водоемов. Поэтому и сиськи жирные ( сохраняли тепло) ( но почему их две?) И половой член длинный- совокуплялись в воде. И кричат во время секса- чтоб можно было в темноте быстрее найти и присоединиться....
galvanika-metall-stalax-3-1162092820.jpg190 Кб, 800x457
2130 592933
Йоу гальванач, нужен учебник / справочник по прикладной DIY гальванике на коленке, рассчитанный на то, что бы обычный мимокрок с базовыми познаниями в химии мог повторить процесс с минимальным количеством покупной поприетарной хуйни. С меня как обычно.
2131 592934
>>92799

>Существование домовых для меня тоже реальность объективная



О, пока тут еще свежо обсуждение домовых и сонных параличей - у меня вопрос.

Когда мне было лет 12-14+-, я проснулся глубокой ночью. Мог двигаться, в общем никаких "симптомов" паралича не было. Но, когда я увидел, что по комнате идет какой-то пацан моего телосложения, ну в общем похож на меня, в моей одежде. Так медленно идет, словно в дома гравитация как на луне. Шел в сторону компьютерного стола, затем также медленно залез на него и стал на нем медленно прыгать, опять же - будто гравитация как на луне. Я помню как мне было страшно, я спрятался под одеяло и боялся издать лишний звук. На утро мама сказала, что это был домовой

Так вот, что это было? Я не до конца проснулся? Разновидность сонного паралича?
2132 592937
>>92934
Иду домой, частный сектор,паралельно мне в метрах 5 от меня идёт девушка, с похожей на мою сумкой. Ну я посмотрела на сумку, потом на девушку, и это я сама. Реально я сама. Она зашла в калитку моего двора, пока я стояла офигевшим сурикатом. Ну я подошла к дому попозже предчувствуя кринж. Родительница моя что была дома, услышала как только громко хлопнула дверь. Когда я зашла, она спросила меня" кто зашёл раньше" ( она на кухне была, не видела, мог прийти и мой брат).
Это фантомы. Ноосферные или нет. По теории Ноосферы, это двойники человека. Которые там. В общем инфополе. Иногда копии друг друга живой человек и его ноосферная копия, привязанны друг другу.
Кто видел осознанные сны, работал в теме. Знает атмосфера при осознанных сновидений плавная, перетекающая, много летаешь, нестандартно география видется - " тут была река и вдруг горы, или- зал дворца и вдруг ступеньки" похоже на дом от которого невозможно уйти из " Алисы в Зазеркалье". Сама так ходишь прыжками или плавным полетом.
Поэтому я думаю что это фантом, твой Ноосферный фантом, осознающий себя Ноосферным жителем, а не домовой.
Ноосферный. Это отдельное существо, копия (...просто инфа снятая и обобщающая, может для того чтоб после смерти распичатать " я", тело...мало ли, это чисто моя гипотеза, спецы по Ноосфере не знают зачем фантомы. )

А домовой, он какой- то меховой, с острыми коготками на маленьких пальчиках, при этом текучий. Это сложно объяснить, как податливая меховая такая зверушка. Как эвоки в " Звездных Войнах" если они были гибкими и могли голову в плечи прятать.

После встречи с фантомом своим , если это был фантом,. Я решила что если вдруг попаду в временную петлю, то подойду к самое себе и попрошу помощи . Мое" я" при этом не будет бояться. То есть испуг наверное будет, но я решила что сама себе если надо пойду на встречу.
2133 592945
ЗАХОДИШЬ НА НАУКАЧ
@
ПРЕДВКУШАЕШЬ ГОДНЫЙ НЕТРИВИАЛЬНЫЙ СРАЧ НА ИНТЕРЕСНЫЕ ТЕМЫ
@
ДОМОВЫЕ
@
ФЕН-ШУЙ
@
СИСЬКИ ОБЕЗЬЯН
@
МОЛЧА ЗАКРЫВАЕШЬ ВКЛАДКУ
2134 592947
>>92945

https://www.youtube.com/watch?v=313SwmprA_g

Сорян с такой зарплатой только феншуй.
2135 592951
Вы понимаете, в обще, что такое Вселенная?
Я курнул и сейчас объясню.
Какая вероятность, что где-то далеко во Вселенной есть твоя копия, но с палкой колбасы в жопе?
0,000(тысяча нулей)01%

А теперь перемножаем это на бесконечность вселенной и неизбежно получаем вероятность этого 100%.
И это даже не берем бесконечное количество бесконечных вселенных и многомировую интерпритацию.

Ты скажешь, вероятность этого КРАЙНЕ МАЛА - Ноль целых, миллион нулей, 1.
А я скажу- появление структурированной РНК длиной 1 метр от того, что молния 2 млрд лет ебашила в лужу с говном ещё меньше.

Вопрос не в вероятности, она всегда будет 100% при бесконечности, вопрос в противоречии законам физики в этой Вселенной и бесконечности, если не противоречит и бытие бесконечно - это всё есть. И жизнь с симуляции, и симуляция в кубе, и Бог, и рай и ад - всё есть, было и будет.

И главный вопрос единственный - уникально ли самосознание или тоже может воспроизводиться.. Если да, то ты тоже будешь перерождаться бесконечно со всеми своими воспоминаниями, чувствами и мыслями. Ницше от этих фактов поехал кукохой.

Теперь живите с этим. Чао. Али, привет.
2136 592952
>>92951
Им тут пиздейшенс бытия подъехал тотальный и великие люди позволили прикоснуться к тайнам мироздания на 1%.
А они все сраные магнитары и сраную луну обсуждают.
Смешно.
Чао.
2137 592953
>>92951
Ничего нового ты не сказал.
2138 592954
Вновь вброшу эту прекрасную книгу о происхождении жизни (а в начале и про происхождение солнечной системы) https://batrachos.com/sites/default/files/pictures/Books/Nikitin_2016_Proishozhdenie%20zhizni.pdf
2139 592957
как малая выживаемость особей может повлиять на численное увеличение выводка? люди ещё могут оценить и повлиять на это, но другим существам же похуй, откуда они получают обратную связь?
2140 592958
>>92957

> откуда они получают обратную связь


Те виды у которых остается мало приплода вымирают. Те которым повезло получить пачку мутаций на увеличение приплода выживают. Эт опро ситуацию когда изменения относительно медленные и постепенные. Резкое изменение условий (которое привело к серьезному падению выжываемости) это гарантирование вымирание.
2141 592959
>>92957

>как малая выживаемость особей может повлиять на численное увеличение выводка?


Иногда у таких живых представителей включается программа размножения, независимо от концентрации еды. То есть, несбалансированная программа, вынуждающая расширять ареал обитания. Потери при такой миграции и потери от недостатка ресурса еды\воды или энергии компенсируют экспоненциальное размножение такого вида и его расползание по территории, с захватом сырьевой и кормовой базы. Недостаточная выживаемость перекрывается избыточным размножением.
Если такая программа не включается, вид потихоньку приходит к самозацикливанию, и либо накапливает ген. мутации и вырождается, или замирает на границе окончания жизни вида. Чаще всего, любое внешнее воздействие на такой зацикленный вид в стагнации приводит к его уничтожению навсегда. Никаких полезных мутаций у таких организмов, если это позвоночные, не происходит.
Что включает такие программы в биомеханизмах - неясно.
2142 592960
>>92957

> но другим существам же похуй, откуда они получают обратную связь?


Становится меньше еды или ты становишься меньше едой
1721078813821big.jpg7 Кб, 150x141
2143 592961
Когда ученые создалут робото лоли богинь?
2144 592962
>>92900
Реагируйте, блять! Я че, зная своими наблюдениями делюсь?
2145 592963
>>92962

>зная


Зря*
фикс
2146 592971
Почему атомная энергетика дешёвая?
Ведь это тратить все деньги на безопасность
2147 592982
>>92971

> Почему атомная энергетика дешёвая?


Она не дешевая. Атомная электростанция обходится значительно дороже в строительстве, чем тепловая, и очень долго окупается.
2148 592983
>>92971

>Почему атомная энергетика дешёвая?


Потому что тебе пиздят.
Например, никто EROEI любых источников энергии считают через бочку нефти.
Вот только легко добываемой нефти на планете все меньше и меньше.
Никто никогда не считал сколько энергии ядерной станции тратится на само производство ядерной станции. И сколько она должна выработать, прежде чем одна станция за счет выработки построит другую в ситуации, когда нефти нет, но продукты её переработки для постройки необходимы. А ведь пластик и резина получается из нефти, например.
2149 592985
>>92983
Если ты так заботишься о пластике - сортируй свой, который ты выкидываешь тоннами за год. И откажись от тетра-пак упаковки.
Это позволит сэкономить тонны нефти. Правда, подорвет экономику, но это пох.
2150 592987
>>92985
Всем дебилам, которые сортируют пластик, я рекомендую не заниматься хуйней.
Потому что переработке подлежит с текущими технологиями только 10% максимум.
Какую у тебя экономику сэкономленная нефть подорвет, долбоеб малолетний, ты бредишь что ли?
Дешевый энергоресурс это наоборот буст экономики.
2151 592988
>>92951
Пидорашка оскотинилась и ёбнулась в лужу, вдруг тучи разошлись и на пидоранделя снизошло озарение.
2152 592989
>>92961
Дегенерат, нахуя ты пикчи в 7 килобайт прикрепляешь?
2153 592995
>>92542
Так у квантовой механики нет проблем со связыванием. Оно там работает автоматически из мат.аппарата. При этом информация не передается. Нахуя там поле причинности, оно добавит что? Какие феномены оно даст предсказать, которые кванты не предсказывают? Там уже буквально есть операторы, которые отвечают за запутывание, зачем что-то еще придумывать?
2154 592997
>>92995
Так и так, иди ты нахуй!
2155 592999
>>92989
Это скрытый тест на метаданные. На самом деле картинка весит 1 кб.
17211446611690.jpg13 Кб, 150x141
2156 593008
>>92999

>Это скрытый тест на метаданные. На самом деле картинка весит 1 кб.


Проебался ты, в скаченном виде она весит 16 кб.
2157 593011
>>92591

>почему он у нас есть


Нет никаких подобных эффектов.
2158 593014

>Домовые


>Астрология


>НЛО



Коротко о том как ложка одной тупой пизды может испортить бочку науки.
2159 593015
>>92951
Этот пчел - умный. Любое крайне невероятное событие в бесконечной вселенной 100% произойдет. Ставишь любую целью и достигаешь её. Вопрос только hardwork & laser-like concentration
2160 593018
>>93015

> Любое крайне невероятное событие в бесконечной вселенной 100% произойдет. Ставишь любую целью и достигаешь её.


Жаль что ты просто будешь рвать жопу в мастерстве сосания и римминга капиталисту, родишь-умрешь и цель увидишь только мономифом в кинотеатре. А так - все хорошо … бесконечность, любая цель, событие
2161 593021
Что за кал ебать семью нельзя а с негром можно, природа говно.
2162 593024
>>93015
Она не бесконечная, и уж тем более не вечная.
file8012b45af4original.jpg413 Кб, 800x576
2163 593025
>>92951

>уникально ли самосознание


Как оно может быть "уникальным", если просто конструкт. Математически даже не существует. Человека можно описать просто как функцию, вектор, систему, механизм, ничего при это не потеряя

По сути можно сказать, что прямо сейчас "ты " живешь еще и хомячком, и креветкой, и вьетнамским торговцем рыбой. Абстрактной разницы то нет, а физическая про колебания квантового поля без всяких "личнааааастей"

А если ты сделаешь клона с "твоим" нейроотпечатком, поставиш таймер на пробуждения через 5 минут и выстрелишь себе в голову. То проснувшись в новом теле даже не заметишь разницы, как просто после сна. Правда, придеться потом труп убирать...
2164 593029
>>93025

> То проснувшийся клон даже не заметит разницы, как просто после сна. Правда, придеться потом труп оригинала убирать...


Угу
8a28aa2d533d7827c7f7fef21dd22b52.gif487 Кб, 202x125
2165 593030
>>93029
«В качестве аналогии можно представить себе разумную амёбу с хорошей памятью. С течением времени амёба постоянно разделяется, и каждый раз получающиеся амёбы имеют те же воспоминания, что и родитель. Таким образом, у нашей амёбы нет линии жизни, а есть древо жизни. Вопрос идентичности или неидентичности двух амёб в более позднее время должен быть перефразирован. В любой момент мы можем рассмотреть двух из них, и у них будут общие воспоминания до определенной точки (общий родитель), после которой они разойдутся в соответствии с их отдельными жизнями после этой точки. Это становится просто вопросом терминологии, следует ли их считать одной и той же амёбой или нет, или следует ли зарезервировать фразу „амёба“ для всего ансамбля».

«Если бы мы взяли одну из этих разумных амёб, стёрли её прошлые воспоминания и ввели в бессознательное состояние на время деления, поместили две полученные амёбы в отдельные ёмкости и повторяли этот процесс для всех последующих поколений, ни одна из амёб не осознавала бы своего разделения. Через некоторое время у нас было бы большое количество индивидуумов, разделяющих друг с другом одни воспоминания и различающихся другими, каждый из которых совершенно не осознаёт своих „других я“ и пребывает в уверенности, что он является уникальной личностью. Было бы действительно трудно убедить такую амёбу в истинной ситуации, не столкнув её с её „другими я“».

пример с из теории Еверетта, но здесь частично подходит
22183715.jpg149 Кб, 1280x512
2166 593031
По уровню интеллекта синапсид был ближе к млекопитающим / рептилиям / амфибиям? Можно было бы приручить?
В целом, даже домашний лягух понимает, что рука человека означает кормёжку, а не нападение. Но современный лягух — итог сотен миллионов лет эволюции и должен выживать в сложном разностороннем мире с кучей экологических ниш, в отличии от условного горгонопса с первобытного болота.
2167 593032
>>93030

> Это становится просто вопросом терминологии


Удивительное рядом

> Было бы действительно трудно убедить такую амёбу в истинной ситуации, не столкнув её с её „другими я“».


И? Какие тут должны быть выводы? Трудно убедить - мы поняли. Дальше что?
dont drink and drive take lsd and teleport.gif757 Кб, 492x700
2168 593034
>>92951
Бесконечна вселенная весьма мала?

Представим бесклнечную вселенную как многомерную матрицу частиц, где каждая частица описывается каким-то числом. Для упрощения модели возьмём одномерный ряд рандомных натуральных чисел:
...1001000011101001010....

Одно и то же число, например 1 в разных местах, это не просто одинаковые явления, это одно и то же место, прокол в тоннеле реальности соединяет удалённые точки вселенной содержащей число 1 как одну и ту же сущность.
То же самое относится к одинаковым комбинациям чисел, например 100.

Будет ли такая вселенная конечная или бесконечной? Любые одинаковые состояния и их комбинации это тоже одно и тоже место.
dont drink and drive take lsd and teleport.jpg168 Кб, 492x700
2169 593035
20240719143651.jpg197 Кб, 3414x849
2170 593040
У меня не пилософский и нетеософский вопрос.
Как можно восстановить незапотевающее покрытие на пластике?
Пластик какой, не знаю. Поликарбонат вряд ли. Снаружи очки зеркальные. Изнутри есть какое-то покрытие, которое можно легко поцарапать.
Если его смочить, то оно долго не отпотевает. Если оставить сухим, то запотевает. Случайно протёр покрытие с одной стороны пальцем, а затем полотенцем, оно испортилось. Антифоги, мыло и пена для бритья не помогают исправить глянец.
Как в домашних условиях сделать плёнку которая уберёт дефект мутности?
Так понимаю, что желатин, КМЦ или агар-агар махом растворятся от воды. Что ещё можно сделать? Сделать плёнку на другом стекле и наклеить?
Форма поверхности не идеально горизонтальная.
2171 593042
>>93032

>Удивительное рядом


Было бы лучше получить более сформулированный вопрос

>"И что это значит для нас тобой?"


Попробую обьяснить более наглядно

Есть Вселенная, Квантовое Поле. Оно физично, как вода/струна/кабель/линейка домино в классической механике. Волны плотности же, которые по ним распространяются - абстракция, то есть информация.
Все что тебя окружает - не более чем квантовые волны, то есть абстракция, то есть информация. В физико-математичном понимании нет разницы между "оригиналом" и "копией". Главное, что информация цела/существует

Отрицание этой концепции продолжает тренд магического мышления с мистификацией, ритуализацией и персонофикацией окружающей действительности с духами камней

Наплевать, сколько тебе поменяли нейронов или атомов, если конфигурация мозга сохранена. Наплевать на "смерть", если конфигурация мозга и так меняется по жизни. Наплевать на персонализацию, если можно создать два разных единовременных воспоминания хотя бы на пару секунд прервав обмен импульсами полушарий мозга

Так мы из "закрытого индивидуализма" переходим к "пустому индивидуализму", а оттуда и "открытый индивидуализм" вытекает
2172 593047
>>93042

> В физико-математичном понимании нет разницы между "оригиналом" и "копией".


Конечно нет, ведь математика это абстрагирование от всех качеств, кроме количественной. В этом сама суть математики.

> Главное, что информация цела/существует


Для кого «главное»? Для застрелившегося и умершего оригинала информация не главное, оригинал все равно умрет.
Со стороны, с позиции стороннего наблюдателя типа да, клон неотличим от оригинала и если закрыть глаза на труп оригинала или сразу аннигилировать его, то ни чего не изменилось.
Но с позиции оригинала очень даже изменилось - он умер - получил феноменальный опыт смерти, организм прекратил поддерживать гомеостаз и начал разлагаться.

> Наплевать на "смерть", если конфигурация мозга и так меняется по жизни.


Не наплевать. Ты пытаешься отождествить смерть организма со смертью ощущения «самости», типа «самость» это иллюзия, феномен сознания, умирает и рождается каждое утро, час или мгновение, да «самость» иллюзия, но организм не иллюзия, он рождается (появляется) и умирает вполне очевидно и объективно.

> переходим к "пустому индивидуализму"


Буддизм с анатманом

> а оттуда и "открытый индивидуализм"


Адвайта и упанишады

Ты изобретаешь велосипед.

Ты либо сам запутался, либо специально сову натягиваешь. Когда тебе надо ты аппелируешь к организму (точная копия), а когда надо аппелируешь к феномену ощущения самости, перемешивая и отождествляя их.
Socialdino-with-hat.gif125 Кб, 2000x800
2173 593048
>>93047

>математика это абстрагирование от всего кроме, количества


>количества


Здесь уместно обратить внимание на причиность и эквивалентность. Наука про информацию же. На алгебре и геометрии изучаете алгоритмы и фигуры, чтобы потом узнать что человек это и есть алгоритм и фигура

>умершего оригинала


>оригинала


>умершего


Ясно, даже не вчитываемся в мою писанину. Для кого я распинался про информацию, нейроны и импульсы? Для кого я указывал на эквивалентность пустого и открытого индивидуализма?

Тебе наплевать, что там пишут, будем держаться за святые коровы. Камням молишся, вдруг помогут?

>организм не иллюзия


- По квантовой механике - "иллюзия".
- Даже забив на пункт 1, нам главное что сознание иллюзорно. А дальше работает принцип эквивалентности. Как прочертить границу тому, чего даже не существует? Да никак, это все условности. "Ты" - и ничто, и все нейроконструкции во Вселенной разом, потому что "ты" - фикция

Чем ты отличаешься от пика, дай угадаю, душой?

В экперименти с передачей информации между полушариями мозга мы делаем из одной единицы "сознания" две эквивалентные, а из них снова одну. И мы бы нарушали все законы причинности и сохранения, но поскольку мы работает не с чем-то физичным, то нет. "Сознание" можно умножать, делить, плюсовать и отнимать как угодно, сколько угодно, в каких-угодно пропорциях, потому что мы имеем дело с иллюзией

Мы опять пришли в "пустой индивидуализм". Который эквивалентет "открытому". Ведь какие могут быть границы хоть всех "я" во Вселенной, если "я" - фикция

>будизм


>индуизм


Ты еще палингенезию с инкарнацией сравни. Разница в материалистичном подходе. Нет никаких богов, душ, карм, брахм, свобод воли. Вот в чем разница

>ты запутался


Скорее у тебя нет необходимой физико-математико информационной базы, чтобы уверено со мной спорить. Либо просто магичское мышление давит

>самости


Так самость иллюзорна. Вот в чем "фишка". Пока мы переписывались, твоя самость "сдолха" невероятное количество раз. Но никто не переживает по этому поводу. Главное, что сохраняеться принцип передачи информации

И даже плевать "оригинал" или "копия" ты. Кстати, по классической механике твой "оригинал" остался в детстве, а по квантовой - ты перестаешь им быть стоит Земле чуточку пролететь по Квантовому Полю

Но ты дальше держись за святые одушевленные камни. Ведь душа и святость "оригинальности", чтобы, черт возьми, это не значило...
Socialdino-with-hat.gif125 Кб, 2000x800
2173 593048
>>93047

>математика это абстрагирование от всего кроме, количества


>количества


Здесь уместно обратить внимание на причиность и эквивалентность. Наука про информацию же. На алгебре и геометрии изучаете алгоритмы и фигуры, чтобы потом узнать что человек это и есть алгоритм и фигура

>умершего оригинала


>оригинала


>умершего


Ясно, даже не вчитываемся в мою писанину. Для кого я распинался про информацию, нейроны и импульсы? Для кого я указывал на эквивалентность пустого и открытого индивидуализма?

Тебе наплевать, что там пишут, будем держаться за святые коровы. Камням молишся, вдруг помогут?

>организм не иллюзия


- По квантовой механике - "иллюзия".
- Даже забив на пункт 1, нам главное что сознание иллюзорно. А дальше работает принцип эквивалентности. Как прочертить границу тому, чего даже не существует? Да никак, это все условности. "Ты" - и ничто, и все нейроконструкции во Вселенной разом, потому что "ты" - фикция

Чем ты отличаешься от пика, дай угадаю, душой?

В экперименти с передачей информации между полушариями мозга мы делаем из одной единицы "сознания" две эквивалентные, а из них снова одну. И мы бы нарушали все законы причинности и сохранения, но поскольку мы работает не с чем-то физичным, то нет. "Сознание" можно умножать, делить, плюсовать и отнимать как угодно, сколько угодно, в каких-угодно пропорциях, потому что мы имеем дело с иллюзией

Мы опять пришли в "пустой индивидуализм". Который эквивалентет "открытому". Ведь какие могут быть границы хоть всех "я" во Вселенной, если "я" - фикция

>будизм


>индуизм


Ты еще палингенезию с инкарнацией сравни. Разница в материалистичном подходе. Нет никаких богов, душ, карм, брахм, свобод воли. Вот в чем разница

>ты запутался


Скорее у тебя нет необходимой физико-математико информационной базы, чтобы уверено со мной спорить. Либо просто магичское мышление давит

>самости


Так самость иллюзорна. Вот в чем "фишка". Пока мы переписывались, твоя самость "сдолха" невероятное количество раз. Но никто не переживает по этому поводу. Главное, что сохраняеться принцип передачи информации

И даже плевать "оригинал" или "копия" ты. Кстати, по классической механике твой "оригинал" остался в детстве, а по квантовой - ты перестаешь им быть стоит Земле чуточку пролететь по Квантовому Полю

Но ты дальше держись за святые одушевленные камни. Ведь душа и святость "оригинальности", чтобы, черт возьми, это не значило...
2174 593049
>>93048

> Так самость иллюзорна. Вот в чем "фишка". Пока мы переписывались, твоя самость "сдолха" невероятное количество раз.


Да, самость иллюзорна, я и не спорил. Но организм не иллюзорен. Делая двойника и убивая оригинал ты не делаешь переноса самости (она иллюзия), а создаёшь копию организма и убиваешь оригинал.
Ты от объективного положения дел с организмом прикрываешься иллюзией самости (отождествляя самость с организмом), а про смерть оригинала организма умалчиваешь - не вижу, значит не существует.
intro-1606344204.jpg139 Кб, 1000x563
2175 593051
>>93049

>Но организм не иллюзорен


К классической механике?

>создаешь копию и убивешь оригинал


А может та же копия-информация просто продолжает перетикать по Вселенной, не нарушая принцип передачи информации?

Чел, ты сам уже давным-давно никакой не >ааааригинал, хыыыы"

>смерть оригинала


Да какая "смерть"?! Какого "оригинала"?! Черт возьми, ты бы сам задумался об значении своего лексикона. Организм, даже "умирая, гыыы" проложает существовать во Вселенной, он даже никуда не исчезает

Но нам не это сейчас важно. А понять, что у тебя вообще является тем самым святым нерушимым "оригиналом"

Тсунами на берегу китая от японского толчка - это оригинал или копия? Вода китайская, импульс японский. Задача, определите оригинал в этой ситуации и оцените градус его брахмы

- "Бля, а только что умер. Мой оригинал тупо исчез. Морти, ты понимаешь. Стою я значит такой, а тут бац - и Земля сдвинуласть. Моя квантовая волна в одной части квантового поля исчезла и появилась в другой. Морти, ты пониманиешь, я продолжаю умирать, прямо сейчас, пока мы говорим, я продолжаю умирать, Морти, помоги мне. Помоги мне Могти, я продолжаю умирать!"

- "Ох, блииин..."

- "Морти, я хочу жить! Спаси меня! Нах я жил 60 лет, чтобы так нелепо умирать?! Просто взял и испарился с точки квантового поля, за что?! Морти, сделай уже хоть что-нибудь!!!"
2176 593052
>>93051

> Организм, даже "умирая, гыыы" проложает существовать во Вселенной, он даже никуда не исчезает


Нет, организм это самоподдерживающаяся система. После отключения гомеостаза это уже не организм, а разлагающаяся плоть. Точно так же как автомобиль это детали соединённые в работающую систему, создающую целое с новыми свойствами (способность передвигаться и возить пассажиров), если свалить детали от автомобиля в кучу, автомобилем они не будут.

> Да какая "смерть"?! Какого "оригинала"?!


Ты шизик? Читай свои посты:

> А если ты сделаешь клона с "твоим" нейроотпечатком, поставиш таймер на пробуждения через 5 минут и выстрелишь себе в голову. То проснувшись в новом теле даже не заметишь разницы, как просто после сна.

Taun-WeFE.png337 Кб, 650x445
2177 593055
>>93052

>организм это самоподдерживающая система


А еще организм - это квантовая информация. К его распаду можно отнестись как к перезашифровке, а через несколько квадриллионов лет он снова перезашифруется в человека, или белку

Но нам даже не это важно. Организм будучи квантвой волной, то есть волной плотности квантового поля, перемещается по нему как абстракция, как перетекающая по Вселенной копия/информация

>ты шизик, читай свои посты


А может я лучше пока посты того детсадовца, который только-только вчера проходил "тела и предметы", почитаю?

Если сознания не существует, то мы приходим к пустому индивидуализму, но он эквивалентет открытому. Какая может быть буквальная граница "я", если этого самого "я" нет?

Для кого я специально не поленился про полушарии написать, шкил?

Хорошо, зайдем с другой стороны. Она несколько груба, но как по другому с павианом

У нас есть принцип причинности, эквивалентности и законы сохранения.
Некоторое по своему желанию могут считать, что они могут нарушаться. Однако, предположение смелое. Если его не делать, то единственная модель, где эти принципы сохраняются - вечная Вселенная или Мультивселенная. То есть получаеться, что целую вечность "ты", которого ты так любишь персонализировать, не существовал, а теперь вдруг засуществовал...

Или можно по другому. Чем организм на моем пике отличается от перетикающей молнии? А чем одна молния отличается от другой молнии?
2178 593056
>>93055

> То есть получаеться, что целую вечность "ты", которого ты так любишь персонализировать, не существовал, а теперь вдруг засуществовал...


Ты путаешься в Ты-организме, и Ты-самости. Да, Ты-организм не существовал и вдруг засуществовал, а потом исчезнет. А Ты-самость это иллюзия, ее нет. Вот в этом ты и плаваешь, подразумевая под Ты, то организм, то иллюзию самости.

> К его распаду можно отнестись как к перезашифровке, а через несколько квадриллионов лет он снова перезашифруется в человека, или белку


Понятно, шизик.
4469093-screen shot 2015-03-25 at 5.13.24 pm copy.jpg122 Кб, 610x640
2179 593057
>>93056

>ты


>самость


>Прочий бред детсадовца


Хорошо, опущусь на твой уровень, ближе к тебе, шкил

Чем копия отличается от оригинала? Душой, личностью, самостью, "тобой"? А это вообще что такое. Сможешь эти понятия в математике показать, мальчик?

>шизик


МАМ, ОН ОТРИЦАЕТ СВЯЩЕННОСТЬ ТЕЛА1 МАМ, ДЛЯ НЕГО ТЕЛО НЕ СВЯЩЕННО1
2180 593058
>>93057

> Чем копия отличается от оригинала?


Тем что это другой организм. Тем что оригинал умрет, будет иметь отсутствующий у копии опыт умирания и в принципе перестанет быть организмом, либо если не будут умерщвлять оригинал, то у копии и оригинала будет индивидуальный (разный) опыт, это будут разные организмы.
Шизик, с чего ты взял, что из-за копирования твоя иллюзорная самость куда-то (в копию) переместится? Или из-за копирования организмы «как-то» отождествяться в некую единую сущность? Этого не будет, появится новый организм (клон), а оригинал - умрет или останется собой. Ни куда, ни чего не перенесется.

> Сможешь эти понятия в математике показать, мальчик?


Шизик, математика это абстрагирование от качеств объектов, только количественное.
Galaxy.png592 Кб, 1000x625
2181 593059
>>93058

>тем что организм умрет


То есть квантовая информация просто поменяет конфигурацию...

>иметь опыт умирания


Прости, какой опыт? То есть типо сейчас некоего "опыта умирания" у тебя нет. Хоть твоя конфигурация постоянно меняется... Да и твоя конфигурация это просто квантовый отпечаток другой квантовой информации... которая постоянно переноситься по квантовому полю...

>будет разный опыт


Прямо как у тебя... и тебя же через любую единицу времени...

>шизик


Ох, как шаман-духовик меня оскорбил. Как теперь жить дальше...

>с чего ты взял, что из-за копирования твоя иллюзорная самость куда-то (в копию) переместится?


А что мне помешает, границы "я", которого нет?

>Или из-за копирования организмы «как-то» отождествяться в некую единую сущность?


Прямо как в опыте с двумя полушариями в мозге. Ты бы впомнил, если бы внимательно читал. Если бы ты был внимательным, любопытным и без лишнего магического сектанского фанатизма

>Этого не будет, появится новый организм (клон), а оригинал - умрет или останется собой. Ни куда, ни чего не перенесется.


Потому что ты так скозал, ясно

>шизик


Как же у тебя бомбит от того, что твой персонафикованный мистицизм кто-то посмел тронуть

>математика это абстрагирование от качеств объектов


То есть информация это не про качества обьектов? Форма и алгоритм это не качества обьекта? У тебя понимания математики профана

>только количественно


Это про причинность и эквивалентность. Садить, два

Кстати, смотри. Волны плотности рукавов галактики постоянно умирают от перехода звезд. В какой страшной Вселенной мы живем. И как теперь на улицу выходить?
Galaxy.png592 Кб, 1000x625
2181 593059
>>93058

>тем что организм умрет


То есть квантовая информация просто поменяет конфигурацию...

>иметь опыт умирания


Прости, какой опыт? То есть типо сейчас некоего "опыта умирания" у тебя нет. Хоть твоя конфигурация постоянно меняется... Да и твоя конфигурация это просто квантовый отпечаток другой квантовой информации... которая постоянно переноситься по квантовому полю...

>будет разный опыт


Прямо как у тебя... и тебя же через любую единицу времени...

>шизик


Ох, как шаман-духовик меня оскорбил. Как теперь жить дальше...

>с чего ты взял, что из-за копирования твоя иллюзорная самость куда-то (в копию) переместится?


А что мне помешает, границы "я", которого нет?

>Или из-за копирования организмы «как-то» отождествяться в некую единую сущность?


Прямо как в опыте с двумя полушариями в мозге. Ты бы впомнил, если бы внимательно читал. Если бы ты был внимательным, любопытным и без лишнего магического сектанского фанатизма

>Этого не будет, появится новый организм (клон), а оригинал - умрет или останется собой. Ни куда, ни чего не перенесется.


Потому что ты так скозал, ясно

>шизик


Как же у тебя бомбит от того, что твой персонафикованный мистицизм кто-то посмел тронуть

>математика это абстрагирование от качеств объектов


То есть информация это не про качества обьектов? Форма и алгоритм это не качества обьекта? У тебя понимания математики профана

>только количественно


Это про причинность и эквивалентность. Садить, два

Кстати, смотри. Волны плотности рукавов галактики постоянно умирают от перехода звезд. В какой страшной Вселенной мы живем. И как теперь на улицу выходить?
2182 593064
>>93059

>квантовая информация просто поменяет конфигурацию


Шапочку из фольги одень и таблетки не пропускай, у тебя все будет хорошо
2183 593068
>>93064

>РЯЯЯ, НАВУКИ ДОСТАЛИ, ОСТАВТЕ НАМ ВАСЯНАМ НАШУ ДУШУ


Держи в курсе, племеная пидорашка
2184 593073
17.08.2024 взрыв на АЭС Ростовская
22.09.2024 взрыв на АЭС Ленинградская
Какие последствия для РФ?
2185 593075
>>93064
Шапочка от фальГи не поможет от радиации, или поможет?
2186 593078
>>93074 (Del)
Для работающих с Ноосферой создали оченm секретный раздел /zog
2187 593080
Эксперименты с телепатией у животных показывают у них таки есть доступ к чужим мозгам (на уровне принятое хозяином решение повышает вероятность принятия определенных решений животным), можно подмешать сюда свидетельств существования сознания вне тела и слепить теорию что оно не привязано к одному мозгу.
Тогда, у любого сознания есть мозг с которым оно постоянно агрессивно взаимодействует. И коллекция мозгов с которыми оно взаимодействует менее активно и не всегда.
Доступную совокупность мозгов можно назвать ноосферой.
2188 593084
>>93080
Как вам сказать, в Ноосфере бессмертие, а в Вашей логике сугубо материализм.
Я скорее вижу так,Ноосфера- это база, склад, архив, оттуда Некто может использовать данные для творчества на уровне глобального.
Умершие люди что там, в виде фонтомов " живут" , в лучшей своей форме находятся, для того чтоб хранить данные индивида, и если чего распечатать ещё, или нераспечатать с этой копии, с фантома.
Но надо разбираться.
Там очень много вопросов.
Одно поняла, это есть. И реально получается тянуть инфу из общей базы.
испарение.jpg34 Кб, 1043x798
2189 593091
>>84346 (OP)
суп, памагице разобраться:
В закрытой ёмкости спирт, налили. До какого уровня он испарится, как посчитать примерно? Какая концентрация паров в свободной области будет?
1)Воздух в ёмкости
2) Вакуум
2190 593096
>>93091
В принципе это всё, что вам необходимо знать о том, на каком уровне находится развитие науки в российской федерации и на российских бордах.
2191 593097
>>93091
Если вакуум то жидкость становится газ, если просто воздух то не знаю очень сложная задача.
17110360005960.mp43 Мб, mp4,
1280x720, 0:20
2192 593098
>>93097

>Если вакуум то жидкость становится газ

изображение.png64 Кб, 1343x710
2193 593106
>>93091

>В закрытой ёмкости спирт, налили. До какого уровня он испарится, как посчитать примерно?


Он испарится до момента, когда давление паров спирта уравновесится с давлением паров спирта внутри жидкости. Равновесие сразу же завершит переход в пар спирта, до следующего притока тепловой энергии, которая позволит сместить равновесие и испарить большее количество спирта до нового равновесного состояния.
>>93091

>Какая концентрация паров в свободной области будет?


Исходя из таблицы по этому спирту, наверно. Я не химик, не помню уже, но логически попробуй вывести, там же стехиометрическая смесь будет, если ты допускаешь воздух.
>>93091

>2) Вакуум


Какой вакуум, ты совсем обезумел?
2194 593107
>>93096
Не смешно, это жеребенок спрашивает.
2195 593109
>>93107
Зачем вы таких тупых жеребят выращиваете на ферме у себя.
Жеребят прежде всего нужно учить задавать вопросы.
Если сами они ничем кроме тиктоков не интересуются.
Первый и самый главный вопрос в этом случай - какой спирт.
У спиртов между прочим классификация есть, которая определяет их свойства.
Второй вопрос - при какой температуре мы рассматриваем систему, это второй определяющий фактор, если давление постоянно.
Короче не пускайте таких жеребят в интернет, а то их обоссут все, кому не лень.
2196 593110
>>93106

>Какой вакуум, ты совсем обезумел?


Ничего сложного в этом нет.
2197 593116
>>93075
От радиации она и не должна помогать блядь.
Радиация это частицы с большой массой и высокой скоростью-энергией.
А шапочка помогает от ЭМИ.
мимо специалист по шапочкам
2198 593118
>>93116
Есть разные виды радиации, ионы фотоны нейтроны электроны, всё это. Я верю в шапочку, от чего-нибудь да защитит.
2199 593119
>>93109

>Зачем вы таких тупых жеребят выращиваете на ферме у себя.


На мямсо.

>Первый и самый главный вопрос в этом случай - какой спирт.


Какой пьет их батя - древесный.
Никто в РФ не спрашивает про ксилит\сорбит\глицерин\ментол, все спрашивают за СПИРТ.
Спроси любого дегена на улице - спирт бушь? И все сразу понимают, какой спирт, что ты предлагаешь, и чем это грозит.
А ты просто рафинированный какой-то, вафелька, поди...

>Второй вопрос - при какой температуре мы рассматриваем систему


Ёпта, при комнатной, ёпта! НОРМАЛЬНЫЕ УСЛОВИЯ для СПИРТА - комнатная температура. Ты точно рафинадик, сахарок мамкин...
2200 593120
>>93110
Я баюсь жидкаво ваккуумма
2201 593124
>>93120
Не будет никакого жидкого вакуума.
Технически просто создать емкость с вакуумом и поместить туда спирт.
2202 593128
Что будет, если человек переместиться вперед во времени и убьет себя же через 5 минут вперед по временной шкале. Потом он ждет 4 минуты 59 секунд и возвращается в тот момент, когда он должен убить себя же. Тот, который вернулся, умирает от рук себя из прошлого. Это замкнутый круг? Что в таком случае случится со вселенной, если не брать в расчет гипотезу о том, что при каждом действии вселенная разделяется?
2203 593130
В табаке реально много полония или это очередная хуита из разряда псевдонаучная страшилка, чтоб быдло бросало курить?
2204 593139
>>93057
>МАТЕМАТИКА ДАКААААЖЭТ! ДАКАААЖЭТ!
2205 593140
>>93073
параша всё.
2206 593141
>>93128
Путешествия во времени, очевидно, невозможны.
172167701666647594.jpg12 Кб, 274x383
2207 593142
>>93128
Почитай книжку, если осилишь английский.
image.png223 Кб, 555x356
Страх прожить обычную жизнь 2208 593143
Что делать, если у меня страх прожить обычную, заурядную жизнь? Может, есть какие-то методики принятия или что-то в этом роде. Я понимаю, что открыть что-то новое мне в вряд ли суждено, но и от мысли о своей ОБЫЧНОСТИ мне тоже как-то не по себе.
2209 593144
>>93143
Книгу напиши про это, обычный.
2210 593145
>>93144
>>93144
Про обычность? Обычная книга про обычного человека с обычной судьбой, что-ли?
2211 593146
>>93145
Я должен план рассказа тебе сформулировать? Это твоя жизнь, мне откуда знать, с чего ты решил, что она обычна. Вот об этом и распиши, докажи читателю, что твоя жизнь обычнее его.
Сидишь там в сытости и тепле, не в лохмотьях, почище бомжа, есть доступ ко всем знаниям и библиотекам человечества, можешь пообщаться с кем угодно, напечать рассказ. Ты ваще с чего взял, что свою судьбу знаешь, из "матрицы" выпрыгивал что ли? Иди взад есть свою слабосолёную форель.
2212 593147
>>93146
Блин, мне рыбки красной захотелось. Ладно, пойду докторской поем.
2213 593149
https://www.youtube.com/watch?v=Pqgo36d7Pu0

Оправдания, физики-ортодоксы?
2214 593154
>>93139

>этот подрыв мамкиного мистика

2215 593170
>>93149
Бля, я надеюсь, что это троллинг тупостью.
Потому что эксперимент поставлен просто очень хуево.
5366E9A4-8E3E-4D19-B4EA-6F7D580B9EB0.gif2,3 Мб, 359x360
2216 593175
Посмотрел кинофильм Интерстеллар, а потом прочел книгу Кипа Торна о фильме.

Торн пишет, что с пятимерном пространстве (балке), в котором оказался Купер время течет как обычно, в прошлое путешествовать нельзя

При этом Купер вернулся в прошлое чтобы передать данные дочери. Там была её комната во все моменты времени, он видел её временную трубу целиком

Я не понял, время в балке как у нас или по нему тает можно перемещаться??
2217 593176
>>93175
Киноляп книголяп, ничего необычного.
2218 593177
>>93176
Так я не догоняю, если старшая размерность есть, и она не свернутая, я смогу видеть мировые трубы всех тел? Если да, то почему так, ведь пятое измерение пространственное

Если нет, то из какого измерения я могу их (трубы) увидеть? Из второго временного?
2219 593178
>>93177
Я фильм не смотрел и книгу не читал, извини, ничего не могу сказать.
2220 593180
>>93178
Бля ну без фильма, согласно м-теории. Из старших размерностей мировые трубы видно или из неоткуда не видно
2221 593181
>>93180

>м-теории


А почему не ф-теории?

>Из старших размерностей мировые трубы видно или из неоткуда не видно


Может нити? Я просто не очень понимаю, про что ты пишешь.
2222 593182
>>93181
Можно и по ф-теории, если ты такой крутой
Мировая нить это для точки, а для зд тела труба
2223 593183
Переформулирую вопрос — из какого измерения можно увидеть пикрил? Из четвертого пространственного или из второго временного?
IMG6903.png114 Кб, 811x735
2224 593184
>>93183
Пикрил
2225 593185
>>93182
Был бы крутым, на двочах не сидел.

>Мировая нить это для точки, а для зд тела труба


Теперь понял. Но ты все равно ищешь смысл там, где его нет. Как я понял, после перемещения Купера (агента Купера), если следовать всей этой ебанине из книг и фильмов должен был случиться околовселеннский пиздец с нарушением всех законов логики и сценария, который бы потянул другие 9 (11) вселенных за собой и все бы схлопнулось а потом случился бы большой взрыв и все пошло по новой.

>>93184
Полагаю, из 10.
2226 593186
>>93185
10-е измерение является пространственным или временным?
2227 593187
>>93186
Мне кажется пространсвенным.
2228 593193
В общем, у меня чешется передний нижний зуб в основании. До этого полоскал рот ополаскивателем, но решил перестать на время. Читал, что может быть при простуде, у меня насморк, это считается за простуду?
2229 593204
А просто думать по квадратикам это ведь одна фраза можно научить комп? Исследует даже случайное ведь? (Если из простого строится сложное?)
2230 593216
Сап, двач. Я постоянно чувствую себя самым тупым в комнате, даже если нахожусь в комнате, полной пьяных, матерящихся людей за 40, чьи увлечения ограничиваются рыбалкой не говорю, что это плохо, сам рыбачил, очень расслабляет и футболом. Я все равно чувствую себя в ней самым тупым, самым глупым и самым никчемным. В чем проблема и как это можно исправить? Если будут вопросы про IQ, проходил только в интернете, а основывать что-то на таких данных, сами знаете, такое себе.
image.png228 Кб, 506x558
2231 593227
Сап, аноны. Представим, я открыл что-то или у меня появилась гипотеза. Что мне надо писать - эссе, научно-исследовательскую или что-то другое, и как сделать так, чтобы больше заинтересованных глаз это увидели?
2232 593229
>>93227
Рассказать врачу и начать пить таблетки
2233 593230
>>93229

РАССКАЗАЛ ДОКТОРУ
@
ТЕБЕ ОПРЕДЕЛЯЮТ ТАБЛЕТКИ ОТ ШИЗЫ И САЖАЮТ ПОД ДОМАШНИЙ АРЕСТ, ВЫПЛАЧИВАЯ ПЕНСИЮ
@
ЧЕРЕЗ ПАРУ НЕДЕЛЬ СЛУЧАЙНО НАТЫВАЕШЬСЯ НА ОКОЛОНАУЧНЫЙ САЙТ
@
ПОСЛЕ НЕДОЛГОГО СКРОЛА ОБНАРУЖИВАЕШЬ СТАТЬЮ С ТАКОЙ ЖЕ ТЕМОЙ, КОТОРУЮ ТЫ РАССКАЗЫВАЛ ДОКТОРУ
@
ОПИСАНИЕ ТАКОЕ ЖЕ, КАКОЕ ТЫ РАССКАЗЫВАЛ ДОКТОРУ
@
САМА СТАТЬЯ НАПИСАНА СЛОВАМИ, КОТОРЫЕ ТЫ РАССКАЗЫВАЛ ДОКТОРУ
@
В АВТОРАХ ЗНАЧИТСЯ ТВОЙ ДОКТОР
2234 593232
>>93227
Если ты открыл новое, что вряд ли, то знаешь и кому оно пригодися. Как сказать, не зная о чем речь, где обитает целевая аудитория твоего открытия?

>гипотеза


Обыватели понимают под этим чуть не любое высказывание. А научная гипотеза должна быть непротиворечивой остальной науке, для этого надо знать эту остальную науку, ее передний край, который может и в учебники еще не успел попасть. А знать это все можно только работая в этой среде, общаясь с людьми, которые пишут статьи и соотвественно знать где эти статьи публикуются.
2235 593233
>>93232
Если кратко, у меня вчера появилась мысль, сегодня я проверил эту "мысль" на трех людях, и она совпала полностью с моими ожиданиями.

Моя "мысль" - это психологический эффект, сродни эффекту зловещей долины или эффекту Тэтчер, который еще нигде не был упомянут (или, по крайней мере, я не нашел, хотя вбивал все вариации запросов на разных языках).

Что ты можешь посоветовать мне в такой ситуации?
2236 593237
>>90400
А Уран? Он же катится на боку по орбите
2237 593241
>>93230

> В АВТОРАХ ЗНАЧИТСЯ ТВОЙ ДОКТОР


Не сложно было догадаться про шизу, по этому посту можно еще точнее сказать - шиза паранойная
Да, да, за тобой следят, хотят украсть твои бесценные идеи, шиз
2238 593242
>>93241

>Не сложно было догадаться про шизу, по этому посту можно еще точнее сказать - шиза паранойная


>Да, да, за тобой следят, хотят украсть твои бесценные идеи, шиз


Ты шутки не воспринимаешь, что-ли? Сходи, подлечись, может, поможет.
2239 593248
Где можно выложить эссе?
2240 593249
>>93233
Советую запилить статью на википедии и потом везде ссылаться на нее. Будет такая же ненаучная пурга как указанные тобой эффекты, о которых знает полтора человека.
2241 593251
>>93249

>Советую запилить статью на википеди


Ты, видно, не знаешь, как Википедия работает.

>ненаучная пурга


Исследования, эксперименты и куча доказательств, конечно, игнорируются.
IMG0080.jpeg84 Кб, 755x504
2242 593252
Перечислите успехи теории струн (м теории, ф теории, похуй) за последние пять лет)))
2243 593253
>>93252
Какие могут быть успехи? Струны есть - есть, что еще надо?
2244 593254
>>93253
Доказательства, следствия, плоды
2245 593255
>>93254
Чтобы доказать струны, которые находятся где-то в десятом или двенадцатом измерении, нужно совершить невероятный прорыв, и доказательство струн, если они есть, произойдет минимум через пару сотен лет.
2246 593256
>>93255
Ок, а сам матан теории струн уже типа закончен? Все сошлось?
2247 593257
>>93256
Как бы тебе помягче сказать? Теория струн никому не всралась. Как и теория игр. Все по обсуждали, поговорили и забыли.
2248 593258
>>93257
Все научные прорывы в физике совершены 100 лет назад. С тех пор ничего нового, только развитие столетних идей.

Когда еще такое было??
Пиздец
IMG0613.jpeg315 Кб, 947x704
2249 593259
>>93252
Перечислите успехи теоретической физики за последние 5 лет)))
2250 593261
>>93258
Буквально всегда, даже открытия в начале науки были развитием тысячелетних идей.
image.png262 Кб, 1080x1080
2251 593266
Объясните. Какие у нас основания полагать, что всё вокруг состоит из одних и тех же элементарных частиц: протонов, нейтронов и электронов?
То есть допустим атом золота состоит из тех же протонов и нейтронов, что и атом кислорода, разница лишь в количестве. И если каким-нибудь высокотехнологичным, инопланетным образом попытаться слепить из атомов кислорода атомы золота, то из этого выйдет толк?
Откуда абсолютная уверенность, что это так и есть? Почему не может быть протонов и нейтронов золота, из которых состоит атом золота, и не может быть протонов и нейтронов кислорода, из которых состоит атом кислорода?
2252 593267
>>93216
Это вопрос психотерапевтии, а не науки. Скорее всего, у тебя есть некое глубинное, подсознательное убеждение, что ты тупой, возникло либо из-за одного серьёзного травматического события в прошлом, либо из-за череды таких событий. Или просто тебя упорно убеждали в том, что ты тупой.
Если верить книге, которой я читал, лечится это примерно так:
Исходя из этого убеждения у тебя произрастают некие поведенческие правила. Например "не говорить ничего, чтобы не показаться тупым" или "не браться за сложное дело, потому что я всё равно тупой, у меня не получится". После выявления правил нужно выявить, что в твоём понимании обязательно произойдёт, если ты нарушишь конкретное правило.
Чтобы изменить убеждение, нужно осознанно нарушать эти правила, которые из этого убеждения произрастают, и тщательно рефлексировать и записывать то, что произошло в результате этих нарушений. Вести статистику. Проверять, правда ли произойдёт то, чего ты и ожидал, или нет.
Скорее всего ты откроешь для себя, что в следствии нарушения правила события, которых ты так боялся всё таки происходят, но далеко не во всех случаях. И видя у себя на столе выборку из сотен ситуаций, когда ты нарушил эти правила, где написано, что по статистике твои опасения оказались напрасны в 60-80% случаев, ты как бы поймёшь, что ты был не прав, и ты не такой уж и тупой.(если быть точнее, ты поймёшь это в процессе ведения дневника и нарушения этих правил, а не когда в конце посмотришь на заполненный дневник, ну это так, если что).
Ну а если у тебя хуёвые результаты и опасения оказались напрасны лишь в 10-30% случаев, то тут либо смириться нужно что ты тупой, либо саморазвиваться.
2253 593268
>>84346 (OP)
Ща сидел смотрел на кастрюлю и возник вопрос. Уровень школы, не бейте.

Вот стоит кастрюля на идеально черном теле которое ее греет на плите в ней кура минус 17 градусов вода вокруг куры пусть ноль и вот вода набирает энергию двжения молекул что бы начать кипеть

Если смотреть по энергии, необходимой для поднятия на следующий градус на следующий гоадус на следующий градус то поднятие на каждый градус обходится одинаково или там есть изменения?
StandardModelofElementaryParticlesAnti.jpg197 Кб, 1070x665
2254 593271
>>93266
Коллайдеры для кого делали?
14718994433030-sci.jpg268 Кб, 705x1024
2255 593272
>>93106
спасибо
2256 593275
>>93251

>не знаешь, как Википедия работает


Даже не знаю, как двач работает, я не веб-разработчик. Наверно поэтому ты не понимаешь, что я пишу.
2257 593277
Струнщики, объясните на пальцах, чем отличаются друг от друга все пять мейнстримных теорий струн?
2258 593278
>>93275
Я не про техническую сторону вопроса говорил. Ты не можешь просто выложить статью без ссылок на проверенные источники (они снизу любой страницы), ведь дохуя модераторов и обычных неравнодушных пользователей это заметят и впоследствии твою статью удалят, а тебе дадут предупреждение.
Обновить тред
Двач.hk не отвечает.
Вы видите копию треда, сохраненную позавчера в 11:00.

Скачать тред: только с превью, с превью и прикрепленными файлами.
Второй вариант может долго скачиваться. Файлы будут только в живых или недавно утонувших тредах. Подробнее

Если вам полезен архив М.Двача, пожертвуйте на оплату сервера.
« /sci/В начало тредаВеб-версияНастройки
/a//b//mu//s//vg/Все доски